You are on page 1of 188

CBSE Term II

2022

Physics
Class
XI
CBSE Term II
2022

Physics
Class
XI
Complete Theory Covering NCERT

Case Based Questions

Short/Long Answer Type Questions

3 Practice Papers with Explanations

Author
Rajiv Pandey

ARIHANT PRAKASHAN (School Division Series)


CBSE Term II
2022

ARIHANT PRAKASHAN (School Division Series)

© Publisher
No part of this publication may be re-produced, stored in a retrieval system or by any means,
electronic, mechanical, photocopying, recording, scanning, web or otherwise without the written
permission of the publisher. Arihant has obtained all the information in this book from the sources
believed to be reliable and true. However, Arihant or its editors or authors or illustrators don’t take any
responsibility for the absolute accuracy of any information published and the damage or loss suffered
thereupon.
All disputes subject to Meerut (UP) jurisdiction only.

Administrative & Production Offices


Regd. Office
‘Ramchhaya’ 4577/15, Agarwal Road, Darya Ganj, New Delhi -110002
Tele: 011- 47630600, 43518550

Head Office
Kalindi, TP Nagar, Meerut (UP) - 250002, Tel: 0121-7156203, 7156204
Sales & Support Offices
Agra, Ahmedabad, Bengaluru, Bareilly, Chennai, Delhi, Guwahati,
Hyderabad, Jaipur, Jhansi, Kolkata, Lucknow, Nagpur & Pune.

ISBN : 978-93-25796-71-3
PO No : TXT-XX-XXXXXXX-X-XX
Published by Arihant Publications (India) Ltd.
For further information about the books published by Arihant, log on to
www.arihantbooks.com or e-mail at info@arihantbooks.com

Follow us on
CBSE Term II
2022

Contents
CHAPTER
Mechanical Properties of Solids -

CHAPTER
Mechanical Properties of Fluids -

CHAPTER
Thermal Properties of Matter -

CHAPTER
Thermodynamics -

CHAPTER
Kinetic Theory -

CHAPTER
Oscillations -

CHAPTER
Waves -

Practice Papers - -

Watch Free Learning Videos


Subscribe arihant Channel

þ Video Solutions of CBSE Sample Papers


þ Chapterwise Important MCQs
þ CBSE Updates
CBSE Term II
2022

Syllabus
CBSE Term II Class XI
Time: hrs Max Marks:
UNITS Periods Marks
VII Properties of Bulk Matter
Chapter : Mechanical Properties of Solids
Chapter : Mechanical Properties of Fluids
Chapter : Thermal Properties of Matter
VIII Thermodynamics
Chapter : Thermodynamics

IX Behaviour of Perfect Gases and Kinetic Theory of Gases


Chapter : Kinetic Theory
X Oscillations and Waves
Chapter : Oscillations
Chapter : Waves

Total

UNIT-VII Properties of Bulk Matter Periods


Chapter Mechanical Properties of Solids
Stress-strain relationship, Hooke s law, Young s modulus, bulk modulus

Chapter Mechanical Properties of Fluids


Pressure due to a fluid column; Pascal s law and its applications hydraulic lift and
hydraulic brakes , effect of gravity on fluid pressure.
Viscosity, Stokes law, terminal velocity, streamline and turbulent flow, critical velocity,
Bernoulli s theorem and its applications.
Surface energy and surface tension, angle of contact, excess of pressure across a
curved surface, application of surface tension ideas to drops, bubbles and capillary rise.
Chapter Thermal Properties of Matter
Heat, temperature, recapitulation only thermal expansion; thermal expansion of solids,
liquids and gases, anomalous expansion of water; specific heat capacity; CP, CV -
calorimetry; change of state - latent heat capacity.
CBSE Term II
2022

Heat transfer-conduction, convection and radiation recapitulation only , thermal


conductivity, qualitative ideas of Blackbody radiation, Wein s displacement Law,
Stefan s law, Greenhouse effect.

UNIT-VIII Thermodynamics Periods


Chapter- Thermodynamics
Thermal equilibrium and definition of temperature zeroth law of thermodynamics ,
heat, work and internal energy. First law of thermodynamics, isothermal and adiabatic
processes.
Second law of thermodynamics: reversible and irreversible processes

UNIT-IX Behaviour of Perfect Gases and Kinetic Theory of Gases Periods


Chapter- Kinetic Theory
Equation of state of a perfect gas, work done in compressing a gas.
Kinetic theory of gases - assumptions, concept of pressure. Kinetic interpretation of
temperature; rms speed of gas molecules; degrees of freedom, law of equi-partition of
energy statement only and application to specific heat capacities of gases; concept
of mean free path, Avogadro s number.

UNIT-X Oscillations and Waves Periods


Chapter- Oscillations
Periodic motion - time period, frequency, displacement as a function of time,
periodic functions.
Simple harmonic motion S.H.M and its equation; phase; oscillations of a loaded
spring- restoring force and force constant; energy in S.H.M. Kinetic and potential
energies; simple pendulum derivation of expression for its time period. Free, forced
and damped oscillations qualitative ideas only , resonance.
Chapter- Waves
Wave motion: Transverse and longitudinal waves, speed of travelling wave,
displacement relation for a progressive wave, principle of superposition of waves,
reflection of waves, standing waves in strings and organ pipes, Beats
CBSE Circular
Acad - 51/2021, 05 July 2021

Exam Scheme Term I & II

dsUnh;
z ek/;fed f'k{kk cksMZ
(f'k{kk ea=ky;] Hkkjr ljdkj ds v/khu ,d Lok;r laxBu)

Hkkjr CENTRAL BOARD OF SECONDARY EDUCATION


(An Autonomous Organisation under the Ministryof Education, Govt. of India)
dsUnh;
z ek/;fed f'k{kk cksMZ
(f'k{kk ea=ky;] Hkkjr ljdkj ds v/khu ,d Lok;r laxBu)
Hkkjr
CENTRAL BOARD OF SECONDARY EDUCATION
(An Autonomous Organisation under the Ministryof Education, Govt. of India)
dsUnh;
z ek/;fed f'k{kk cksMZ
(f'k{kk ea=ky;] Hkkjr ljdkj ds v/khu ,d Lok;r laxBu)

Hkkjr CENTRAL BOARD OF SECONDARY EDUCATION


(An Autonomous Organisation under the Ministryof Education, Govt. of India)

To cover this
situation, we
have given
both MCQs and
Subjective
Questions in
each Chapter.
dsUnh;
z ek/;fed f'k{kk cksMZ
(f'k{kk ea=ky;] Hkkjr ljdkj ds v/khu ,d Lok;r laxBu)

Hkkjr CENTRAL BOARD OF SECONDARY EDUCATION


(An Autonomous Organisation under the Ministryof Education, Govt. of India)
CBSE Term II Physics XI 1

CHAPTER 01

Mechanical
Properties of Solids

In this Chapter...
l Stress l Stress-Strain Curve
l Strain l Elastic Modulus
l Hooke‘s Law

Elasticity restoring force is equal and opposite to the external


The property of matter by virtue of which it regains its deforming force applied.
original shape and size, when the deforming forces have been Thus, quantitatively, stress can be given as
removed is called elasticity. External deforming force
Stress, S =
Plasticity Area of cross- section
The property of a body by virtue of which it does not regain Its SI unit is N/m 2 or pascal (Pa) and in CGS system unit is
its original shape and size even after the removal of
deforming force, is called plasticity. dyne/cm 2 . The dimensional formula of stress is [ML −1 T −2 ]
On the basis of applied forces on the body, the stress can be
Stress classified as

When a deforming force is applied on a body, it changes the 1. Normal Stress


configuration of the body by changing the normal positions of or Longitudinal Stress
the molecules or atoms of the body. As a result, an internal
It is defined as the restoring force per unit area, acts
restoring force comes into play which tends to bring the body perpendicular to the surface of the body. It is of two types
back to its initial configuration. which are given below
The internal restoring force acting per unit area of a (i) Tensile Stress
deformed body is called stress.
When two equal and opposite forces are applied at the ends
Restoring force (F ) of a circular rod to increase its length, then the restoring
i.e. Stress =
Area of cross -section ( A) force per unit area of cross-section is known as tensile stress.
F
If there is no permanent change in the configuration of the Tensile stress =
A
body, i.e. in the absence of plastic behaviour of the body, the
2 CBSE Term II Physics XI

In case of tensile stress, there is an increase in length of A


the body. Consider a rod of length L, if two equal forces F
are applied in the direction as shown in figure, then the
final length of the rod becomes L + ∆L. L

Thus, increment in the length of the rod is ∆L.


F F
∆x

L
θ
L L + ∆L
F
Deforming force on the surface of a body

F 3. Hydraulic or Bulk Stress


Tensile stress on a circular rod
If a body is subjected to a uniform force from all sides, then the
corresponding stress is called hydraulic stress or bulk stress.
(ii) Compressive Stress
There is a change in volume of the body but not change in
When two equal and opposite forces are applied at the geometrical shape.
ends of a rod to decrease its length or compress it, then F
the restoring force per unit area of cross-section of the rod Bulk stress =
is known as compressive stress. A
F In case of hydraulic stress, the force F is applied perpendicular
Compressive stress = to every point on the surface of body due to which the change in
A
volume ∆V of a body occurs.
In case of compressive stress, there is a decrease in the
length of a body. If a rod of length L, the two equal forces
F are applied in the direction as shown in figure, then the V0
V0
final the length of the rod becomes L − ∆L. Thus,
decrease in length of the rod is ∆L.
(Bodies outside the fluid)
F F
Original volume = V

Surface area = A F F
L L – ∆L F F
Decreased volume V F V F
F
Change in volume = ∆V
F F
Compressive stress on a circular rod F

Under tensile stress or compressive stress, the net force (Bodies immersed in a fluid)
acting on an object is zero but the object is deformed. Hydraulic stress on different surfaces

2. Tangential or Shearing Stress The hydraulic stress is also known as volumetric stress.
When a deforming force acts tangentially to the surface of
a body, it produces a change in the shape of the body Strain
without any change in volume. This tangential force When a deforming force acts on a body, the body undergoes a
applied per unit area of cross-section is known as change in its shape and size. The ratio of the change in
tangential stress. configuration of the body to the original configuration is called
F strain.
Tangential stress =
A Change in configuration
In case of tangential stress, the deforming force F is Strain =
Original configuration
applied on top surface of the cubical body in tangential
direction due to which the upper face is deformed by an Strain is the ratio of two like quantities, so it has no unit and
angle θ from its original position is shown in figure. dimension.
CBSE Term II Physics XI 3

The strain can be three types which are classifed as given (i) In the stress versus strain graph, the stress is found to
below be proportional to strain (% elongation) up to point A.
Thus, Hooke’s law is fully obeyed in this region, the
1. Longitudinal Strain
point A is known as point of proportional limit.
It is defined as the change in length per unit original length,
(ii) When stress is increased beyond A, then for small
when the body is deformed by external forces.
stress, there is a large strain in the wire upto point B.
Change in length ∆L
Longitudinal strain = = (iii) When the load is gradually removed between points
Original length L
O to B, the wire return to its original length.
2. Volumetric Strain The wire regains its original dimension only when
load applied is less than or equal to a certain limit.
It is defined as the change in volume per unit original This limit is called elastic limit. The point B on
volume, when the body is deformed by external forces. stress-strain curve is known as elastic limit or yield
Change in volume ∆V point.
Volumetric strain = =
Original volume V The material of the wire in the region OB shows the
elastic behaviour, hence known as elastic region.
3. Shear Strain
If the deforming forces produce a change in the shape of the (iv) If the stress or load increases beyond point B, the strain
body, then the strain is called shear strain. further increases. This increase in strain represented
∆L by BC part of the curve. Now, if the load is removed,
Shear strain, θ = tan θ = the wire does not regain its original length.
L
In other words, there is permanent strain and this
Hooke’s Law permanent strain in the wire is known as
permanent set.
It states that, ‘‘Within elastic limit, the stress developed is (v) Now, as the stress beyond C is increased, there is
directly proportional to the strain produced in a body.’’ large strain in the wire. The wire breaks at point D
i.e. Stress ∝ Strain which is also known as fracture point.
⇒ Stress = E × Strain The material of the wire from point C to point D
Stress shows the plastic behaviour or plastic deformation.
or E=
Strain The stress needed to cause the actual fracture of the
material is known as breaking stress or ultimate
where E is a constant and is known as modulus
of elasticity of the material of the body. tensile strength.
On the basis of elastic and plastic properties, materials can
Stress-Strain Curve be classified in two ways which are as given below
When a wire is stretched by a load, then a typical graph is (i) Ductile Materials The materials which have large
obtained as shown below plastic range of extension are called ductile materials.
Such materials undergo an irreversible increase in
Fracture length before snapping, so they can be drawn into
Elastic limit or
Yield point point thin wires, e.g. copper, silver, iron, aluminium, etc.
(ii) Brittle Materials The materials which have very
C D
B Plastic deformation small range of plastic extension are called brittle
A materials. Such materials break as soon as the stress is
Stress

Proportional limit
increased beyond the elastic limit, e.g. cast iron, glass,
Plastic behaviour
ceramics, etc.
Elastic behaviour
Permanent set
Malleability
O
0<1% O′ 30% When a solid is compressed, a stage is reached beyond
Strain
which it cannot regain its original shape after the deforming
A typical stress-strain curve
force is removed.
4 CBSE Term II Physics XI

This is the elastic limit point A′ for compression. The solid then F
behaves like a plastic body.
Compression
F V – ∆V F

Applied force
Crushing
point F
A′ Bulk modulus of elasticity
B′
Elastic The Bulk modulus of elasticity is given by
limit
Normal stress F /A
Load compression B= =
curve for a metal Volumetric strain ∆V/ V
−F V
∴ Bulk modulus, B =
The yield point B′ obtained under compression is called crushing A ∆V
point. After this stage, metals are said to be malleable. i.e. They
can be hammered or rolled into thin sheets. e.g. gold, silver, pV
⇒ B=−
lead, etc. ∆V
F
Elastomers where, p = is the normal pressure.
A
The materials which can be elastically stretched to large values of
strain are called elastomers. Compressibility
The reciprocal of the Bulk modulus of a material is
Elastic Modulus called its compressibility.
1 − ∆V
Young’s Modulus of Elasticity Compressibility, K = =
B pV
Within the elastic limit, the ratio of longitudinal stress to the
SI unit of compressibility = N −1 m2
longitudinal strain is called Young’s modulus of elasticity.
and CGS unit of compressibility = dyne−1 cm2 .
Longitudinal stress
i.e. Young’s modulus, Y =
Longitudinal strain The dimensional formula of compressibility is
[M −1 LT 2 ].
Tensile (or compressive) stress (σ )
Y= ∆L
Longitudinal strain ( ε)
Area = A F
Young’s modulus for a wire of length L is given by
θ
FL θ
Y=
A∆ L Fixed face

∆L
If L = 1 m, A = 1 m 2
A A′ D D′
F
and ∆L = 1 m, then Y = F
Thus, Young’s modulus of elasticity is equal to the force required L θ θ
to extend a wire of unit length and unit area of cross-section by a B C
F
unit amount.
According to diagram, by displacing its upper face
Bulk Modulus of Elasticity through distance AA′ = ∆L
Within the elastic limit, the ratio of normal stress to the Let AB = DC = L and ∠ ABA′ = θ,
volumetric strain is called bulk modulus of elasticity. In other F/A F
words, the ratio of hydraulic stress to the hydraulic strain is called η= =
bulk modulus. θ Aθ
Consider a body of volume V and surface area A. AA′ ∆L
Shear strain, θ ≈ tan θ = =
Suppose a force F acts uniformly over the whole surface of the AB L
body and it decreases the volume by ∆V as shown in figure. F L
Shear modulus, η = ⋅
A ∆L
CBSE Term II Physics XI 5

Factors Affecting Elasticity of Applications of Elastic Behaviour


Material of Solids
Factors affect the elasticity of a material which are as given Some applications of elastic behaviour of solids which are as
below given below
(i) Hammering and Rolling In both of these processes, (i) Any metallic part of a machinery is never subjected
the crystal grains are broken into small units and the to a stress beyond the elastic limit of the material.
elasticity of the material increases. (ii) The thickness of metallic ropes used in cranes to lift
(ii) Annealing This process results in the formation of and move heavy weights is decided on the basis
larger crystal grains and elasticity of the material of the elastic limit of the rope and the factor of safety.
decreases. (iii) In designing a bridge or beam that has to be
(iii) Presence of Impurities Depending on the nature of designed such that it can withstand the load of the
impurity, the elasticity of material can be increased or following traffic, the force of winds and its own
decreased. weight.
(iv) Temperature Elasticity of most of the materials When the beam is loaded in the middle with a load
decreases with increase in the temperature but w, then it gets depressed by an amount δ given by
elasticity of invor steel (alloy) does not change with the
change in temperature. l
b d
Elastic After Effect
When the deforming force is removed from the elastic bodies, δ
the bodies tend to return to their respective original state. It
has been found that, some bodies return to their original state
immediately, others take appreciably long time to do so. The w
delay in regaining the original position is known as elastic after
effect. wl 3
Depression, δ =
4 Ybd 3
Elastic Fatigue
where, Y = Young’s modulus of elasticity.
The loss in the elastic strength of a material caused due to
(iv) Maximum height of a mountain on earth (~10 km)
repeated alternating strains to which the material is subjected,
can be estimated from the elastic behaviour of earth.
is called elastic fatigue.
6 CBSE Term II Physics XI

Solved Examples
Example 1. Consider a solid cube which is subjected to Example 3. In a physics department, a Foucault
a pressure of 6 × 10 5 N/m 2 . Due to this pressure, pendulum consists of a 130 kg steel ball which
each side of the cube is shortened by 2%. Find out swings at the end of a 8.0 m long steel cable having
the volumetric strain of the cube. the diameter of 3.0 mm. If the ball was first hung
Sol. Let L be the initial length of the each side of the cube. from the cable, then determine how much did the
Volume, V = L × L × L = L3 cable stretch. (Take, Y = 20 × 10 8 N/m 2 )
= Initial volume (Vi say) Sol.Given, diameter, D = 3. 0 mm = 3.0 × 10 −3 m
If the each side of the cube is shortened by 2%, then final Length, L = 8. 0 m and mass, m = 130 kg
length of the cube = L − 2% of L D 3. 0 × 10 −3
Radius, r = = = 1. 5 × 10 −3 m
 2L   2  2 2
= L −  = L 1 − 
 100   100  The area of cross-section of the cable
 2 
3
 2 
3 A = πr 2 = π × (1. 5 × 10 −3 )2 = 7 . 065 × 10 −6 m 2
∴ Final volume, Vf = L3 1 −  = V 1 − 
 100   100  Thus, F = w = mg = 130 × 9. 8
3 F = 1274 N
 2 
Change in volume, ∆V = Vf − Vi = V 1 −  −V Stress F /A LF
 100  We know that, Y = = ⇒ ∆L =
Strain ∆L /L AY
 2 
3 
= V 1 −  − 1 8. 0 × 1274
Change in length, ∆L =
 100   7 . 065 × 10 −6 × 20 × 10 8
∆V  2 
3
 2 × 3 = 0.72 m = 720 mm
= 1 −  −1 ~− 1 − −1
V  100   100  Example 4. The ball of 200 g is attached to the end of
[Q (1 − x)n ~
− 1 − nx for x << 1 ] a string of an elastic material (say rubber) and
∆V having length and cross-sectional area of 51 cm
∴ Volumetric strain = = 1 − 0.06 − 1 = 0.06 and 22 mm 2 , respectively. Find the Young’s
V
modulus of this material if string is whirled round,
(take positive sign)
horizontally at a uniform speed of 50 rpm in a circle
Example 2. If a wire of length 4 m and cross-sectional of diameter 104 cm.
area of 2 m 2 is stretched by a force of 3 kN, then Sol. Given, mass of the ball, M = 200 g = 0.2 kg
determine the change in length due to this force. Area of cross-section, A = 22 mm 2 = 22 × 10 −6 m 2
(Take, Young’s modulus of material of wire D 104
= 110 × 10 9 N / m 2 ) Radius of the circle, r = = = 52 cm = 0. 52 m
2 2
Sol. Given, area of cross-section, A = 2 m 2 Length of the string, l = 51 cm = 0. 51 m
Force, F = 3 kN = 3 × 10 3 N Revolution per second, N = 50 × 60 rps = 3000 rps
Length, L = 4 m Certain petal force, F = mr ω 2 = 0.2 × 0. 52 × (2 π × N )2
Young’s modulus, Y = 110 × 10 9 N / m 2 F = 36. 95 × 10 6 N
FL Change in length, ∆ l
Applying Y =
A∆L = Radius of the circle − Length of the string
FL 3 × 10 3 × 4 = 0. 52 − 0. 51
⇒ ∆L = = ∆ l = 0. 01 m
AY 2 × 110 × 10 9
Young’s modulus of the material,
= 0.0545 × 10 −6 m
F l 36. 95 × 10 6 0. 51
∆L = 54.5 × 10 −3 mm Y= = × = 85.67 × 1012 Nm −2
A ∆l 22 × 10 −6 0. 01
CBSE Term II Physics XI 7

Example 5. What will be the decrease in volume of Sol. Given, modulus of rigidity, η = 2 × 10 7 dyne/cm2
3
100 cm of water under pressure of 100 atm, if the H H′ G G′
compressibility of water is 4 × 10 −5 per unit F

atmospheric pressure?
1 1 E
Sol. Bulk modulus, B = = E′ F F′
Compressibility K
8 cm θ C
1 D θ
=
4 × 10 −5
A B
= 0.25 × 10 5 atm
= 0.25 × 10 5 × 1.013 × 10 5 N / m 2 Side of the cube, l = 8 cm
= 2.533 × 10 N / m
9 2 Area, A = l 2 = 64 cm 2
Volume, V = 100 cm 3 =10 −4 m 3 Force or load, F = 300 kgf
= 300 × 1000 × 981 dyne
Pressure, p = 100 atm = 100 × 1.013 × 10 5 N / m 2
F
As, η= …(i)
= 1.013 × 10 7 N /m−2 Aθ
1 ∆V F
Now, apply =K = ⇒ θ=
B pV Aη
pV 1.013 × 10 7 × 10 −4 300 × 1000 × 981
∴ ∆V = = θ=
B 2.533 × 10 9 64 × 2 × 10 7
−6
∆V = 0.4 × 10 m 3 = 0.4 cm 3 ~− 0.23 rad
F l
Example 6. Consider an Indian rubber cube having As, η= …(ii)
modulus of rigidity of 2 × 107 dyne/cm 2 and of A∆l
side 8 cm. If one side of the rubber is fixed, while a ∆l
⇒ =θ [from Eqs. (i) and (ii)]
tangential force equal to the weight of 300 kg is l
applied to the opposite face, then find out the ⇒ ∆ l = l θ = 8 × 0.23
shearing strain produced and distance through ∆ l = 1.84 cm
which the strain side moves.
8 CBSE Term II Physics XI

Chapter
Practice
PART 1 7. A uniform cube is subjected to volume
compression. If each side is decreased by 1%, then
Objective Questions bulk strain is
(a) 0.01 (b) 0.06
(c) 0.02 (d) 0.03
l
Multiple Choice Questions
8. Three blocks are connected with wires A and B of
1. The property of a body, by virtue of which it tends same cross-section area x and Young’s modulus Y.
to regain its original size and shape of a body when All three blocks are of mass m each.
applied force is removed, is known as A B
m m
(a) fluidity (b) elasticity
(c) plasticity (d) rigidity
m
2. The maximum load a wire can withstand without
breaking, when its length is reduced to half of its With reference to the given situation, which of the
original length, will [NCERT Exemplar] following expression are correct?
(a) be double (b) be half 2
(c) be four times (d) remain same I. Tension in wire A = mg
3
3. A uniform bar of square cross-section is lying along 2
a frictionless horizontal surface. A horizontal force II. Tension in wire B = mg
3
is applied to pull it from one of its ends, then the 2 mg
(a) bar is under same stress throughout its length III. Stress in wire A =
(b) bar is not under any stress because force has been 3x
applied only at one end 2 mg
IV. Strain in wire B =
(c) bar simply moves without any stress in it 3xY
(d) stress developed gradually reduces to zero at the end (a) Both I and II (b) Both II and III
of the bar, where no force is applied (c) Both III and IV (d) Both II and IV
4. A spring is stretched by applying a load to its free 9. Stress-strain curves for the materials A and B are
end. The strain produced in the spring is [NCERT]
shown below
(a) volumetric (b) shear
(c) longitudinal and shear (d) longitudinal A
5. A wire is stretched to double its length. The strain
is B
Stress

(a) 2 (b) 1 (c) zero (d) 0.5


6. A wire of diameter 1 mm breaks under a tension of
1000 N. Another wire of same material as that of O
Strain
the first one, but of diameter 2 mm breaks under a
tension of Then,
(a) 500 N (b) 1000 N (a) A is brittle material (b) B is ductile material
(c) 10000 N (d) 4000 N (c) B is brittle material (d) Both (a) and (b)
CBSE Term II Physics XI 9

10. Which of the following statement (s) is/are correct 16. When a pressure of 100 atm is applied on a
regarding to elastomers? spherical ball of rubber, then its volume reduces to
I. They can be elastically stretched to a large value 0.01%. The bulk modulus of the material of the
of strain. rubber (in dyne cm −2 ) is
II. These materials do not obey Hooke’s law. (a)10 × 10 12 (b)100 × 10 12

III. Young’s modulus of elastomers is very large. (c)1 × 10 12 (d) 20 × 10 12


(a) Both I and II (b) Both II and III 17. Which of the following statement(s) is/are
(c) Both I and III (d) I, II and III incorrect?
11. In plotting stress versus strain curves for two (a) The bulk modulus for solid is much larger than for
materials P and Q, a student by mistake puts strain liquids.
on the Y-axis and stress on the X-axis as shown in (b) Gases are least compressible.
the figure. Then, which of the following (c) For a system in equilibrium, the value of bulk modulus
is always positive .
statement(s) is/are correct?
(d) The SI unit of bulk modulus is same as that of
pressure.
P 18. Over bridges are constructed with steel but not
Strain

Q
with aluminium because steel is
(a) more elastic than aluminium
(b) less elastic than aluminium
Stress
(a) P has more tensile strength than Q. (c) more plastic than aluminium
(d) less plastic than aluminium
(b) P is more ductile than Q.
(c) P is more brittle than Q 19. A metal bar is supported at two ends. If metal bar is
(d) The Young’s modulus of P is more than that of Q. loaded at centre with a heavy load, the depression
in bar at the centre is proportional to (Y = Young’s
12. On applying a stress of 20 × 10 8 Nm −2 , the length modulus of bar)
of a perfectly elastic wire is doubled. Its Young’s 1 1
modulus will be (a) 2
(b)
Y Y
(a) 40 × 10 8 Nm −2 (b) 20 × 10 8 Nm −2 (c) Y (d) Y 2
(c)10 × 10 8 Nm −2 (d) 5 × 10 8 Nm −2
20. If the load hanging from middle position of a metal
3 beam is increased to double, then depression in the
13. A wire of length 2 m is made from 10 cm of
copper. A force F is applied, so that its length bar at the centre is
increases by 2 mm. Another wire of length 8 m is (a) increased to four times
made from the same volume of copper. If the force (b) decreased to four times
F is applied to it, its length will increase by (c) increased to double
(a) 0.8 cm (b) 1.6 cm (c) 2.4 cm (d) 3.2 cm (d) decreased to half
14. In steel, the Young’s modulus and the strain at the l
Assertion-Reasoning MCQs
breaking point are 2 × 1011 Nm −2 and 0.15,
respectively. The stress at the breaking point for Direction (Q. Nos. 21-26) Each of these questions
steel is contains two statements Assertion (A) and Reason (R).
(a)1.33 × 1011 Nm −2 (b)1.33 × 1012 Nm −2 Each of these questions also has four alternative
choices, any one of which is the correct answer. You
(c) 7.5 × 10 − 13 Nm−2 (d) 3 × 1010 Nm −2
have to select one of the codes (a), (b), (c) and (d) given
15. A copper and a steel wire of the same diameter are below.
connected end-to-end. A deforming force F is (a) Both A and R are true and R is the correct
applied to this composite wire which causes a total explanation of A
elongation of 1 cm. The two wires will have [NCERT]
(b) Both A and R are true, but R is not the correct
(a) the same stress and strain
explanation of A
(b) different stress and strain
(c) A is true, but R is false
(c) the same strain but different stress
(d) the same stress but different strain (d) A is false and R is also false
10 CBSE Term II Physics XI

21. Assertion Spring balance shows incorrect readings (ii) What is the tensile stress on this plane?
after using it for a long time. (a) (F / A ) cos 2 θ (b) F / A
Reason Spring in the spring balance loses its (c) (F / A ) tan θ (d) (F / A ) sec2 θ
elastic strength over the period of time. (iii) What is the shearing stress on this plane?
22. Assertion When a solid sphere is placed in the F F
(a) sin 2 θ (b) cos2θ
fluid under high pressure, then it is compressed 2A A
uniformly on all sides. F F
(c) cos 2 θ (d)
Reason The force applied by fluids acts in 2A 4A 2
perpendicular direction at each point of surface. (iv) For what value of θ is the tensile stress maximum?
23. Assertion The strain produced by a hydraulic (a) 0° (b) 90° (c) 45° (d) 30°
pressure is volumetric in nature. (v) For what value of θ is the shearing stress
Reason It is a ratio of change in volume ∆V to the maximum?
original volume V. (a) 45° (b) 30 (c) 90° (d) 60°
24. Assertion Young’s modulus for a perfectly plastic
body is zero.
Reason For a perfectly plastic body, restoring force
PART 2
is zero.
25. Assertion Gases have large compressibility.
Subjective Questions
Reason Compressibility is defined as the fractional l
Short Answer (SA) Type Questions
change in volume per unit decrease in pressure.
1. Calculate the value of stress in a wire of steel
26. Assertion Maximum height of a mountain on earth having radius of 2 mm of 10 kN of force is applied
is ~10 km. on it.
Reason A mountain base is not under uniform 2. A steel cable with a radius of 1.5 cm supports a
compression and provides some shearing stress to chair lift at a ski area. If the maximum stress is not
rock under which it can flow. to exceed 10 8 N/m 2 , then what is the maximum
load the cable can support? [NCERT]
l
Case Based MCQs
3. A wire of length 2.5 m has a percentage strain of
27. Direction Read the following passage and answer 0.012% under a tensile force. Determine the
the questions that follows extension in the wire.
Restoring Force due to Stress 4. If the angle of shear is 30° for a cubical body and
When a bar of cross-section A is subjected to equal the change in length is 250 cm, then what must be
and opposite tensile forces at its ends, then a the volume of this cubical body?
restoring force equal to the applied force normal to
5. The ratio of stress/strain remains constant for a small
its cross-section comes into existence. This
deformation. What happens to this ratio, if
restoring force per unit area of cross-section is
deformation is made very large?
known as tensile stress. While when the deforming
force acts tangentially to the surface, then this 6. A wire is replaced by another wire of same length
tangential force applied per unit area of and material but of twice diameter.
cross-section is known as tangential stress. (i) What will be the effect on the increase in its
Consider a plane section of the bar, whose normal length under a given load?
makes an angle θ with the axis of the bar. (ii) What will be the effect on the maximum load
which it can bear?
θ
7. A wire of length L and radius r is clamped rigidly at
F F
one end. When the other end of the wire is pulled
(i) Which of the following property of the bar does not by a force f , its length increases by l. Another wire
change due to this force? of the same material of length 2L and radius 2r, is
(a) Area (b) Volume pulled by a force 2f. Find the increase in length of
(c) Shape (d) Size
this wire.
CBSE Term II Physics XI 11

8. Two wires made of same material are subjected to When a mass M is placed on the piston to compress
forces in the ratio 1 : 4. Their lengths are in the the liquid, find fractional change in the radius of
ratio 2 : 1 and diameters in the ratio 1 : 3. What is the sphere.
the ratio of their extensions? 15. The Mariana trench is located in the Pacific ocean
9. The stress-strain graphs for materials A and B are and at one place, it is nearly 11 km beneath the
shown in Figs. (a) and (b). surface of water. The water pressure at the bottom
of the trench is about 1.1 × 10 8 Pa.
A steel ball of initial volume 0.32 m 3 is dropped
D
D into the ocean and falls to the bottom of trench.
Stress

Stress

What is the change in the volume of the ball when


A B it reaches to the bottom, if the Bulk modulus of
steel is 1 . 6 × 1011 N/m 2 ? [NCERT]
Strain Strain 16. To what depth must a rubber ball be taken in deep
(a ) (b ) sea, so that its volume is decreased by 0.1%? (The
The graphs are drawn to the same scale. Bulk modulus of rubber is 9.8 × 10 8 N/m 2 and the
density of seawater is 10 3 kg/m 3 ) [NCERT Exemplar]
(i) Which of the materials has greater Young’s
modulus? 17. The maximum stress that can be applied to the
(ii) Which of the two is the stronger material? material of a wire used to suspend on elevator is
[NCERT] 1. 3 × 10 8 Nm −2 . If the mass of the elevator is 900 kg
and it moves up with an acceleration of
10. A wire elongates by l mm when a load w is hanged
2.2 ms −2 , then what is the minimum diameter of
from it. If the wire goes over a pulley and two
the wire?
weights w each are hung at the two ends, then what
will be the elongation (in mm) of the wire? 18. Two strips of metal are riveted together at their
ends by four rivets, each of diameter 6 mm. What is
11. The Young’s modulus for steel is much more than
the maximum tension that can be exerted by the
that for rubber. For the same longitudinal strain,
riveted strip, if the shearing stress on the rivet is
which one will have greater tensile stress?
[NCERT Exemplar] not to exceed 6. 9 × 107 Pa? Assume that, each rivet
is to carry one-quarter of the load. [NCERT]
12. Figure shows the strain-stress curve for a given
material. What are (i) Young's modulus and (ii) 19. Anvils made of single crystals of diamond, with the
approximate yield strength for this material? shape as shown in figure are used to investigate the
[NCERT] behaviour of materials under very high pressure.
Flat faces at the narrow end of the anvil have a
Stress (106 Nm–2)

300 diameter of 0.5 mm and the wide ends are


250 subjected to a compressional force of 50000 N.
200 What is the pressure at the tip of the anvil? [NCERT]
150
100
50
0
0.001 0.002 0.003 0.004 Diamond cones Metal gasket
Strain

13. Calculate the percentage increase in length of a


wire of diameter 2.5 mm stretched by a force of
100 kg weight. Young’s modulus of elasticity of wire 20. After a fall, a 95 kg rock climber finds himself
is 12.5 × 1011 dyne/sq cm. dangling from the end of a rope that had been 15 m
14. A solid sphere of radius R made of a material of long and 9.6 mm in diameter but has stretched by
bulk modulus B is surrounded by a liquid in a 2.8 cm. For the rope, calculate.
cylindrical container. A massless piston of area A (i) the strain, (ii) the stress and
floats on the surface of the liquid. (iii) the modulus of elasticity.
12 CBSE Term II Physics XI

l
Long Answer (LA) Type Questions 60 cm, respectively. Assuming the load distribution
to be uniform, calculate the compressional strain of
21. A steel wire of length 2 l and cross-sectional area A each column. (Take, Young’s modulus,
is stretched within elastic limit as shown in figure. Y = 2. 0 × 1011 Pa) [NCERT]
Calculate the strain and stress in the wire. 28. A slightly tappering wire of length l and end radii a
2l and b on both sides is subjected to the stretching
forces F on both sides as shown in figure. If Y is the
x Young’s modulus of the wire, then calculate the
extension produced in the wire.
T
m l

22. A steel wire of length 4.7 m and cross-sectional


area 3.0 × 10 −5 m 2 stretches by the same amount as F F
a copper wire of length 3.5 m and cross-sectional r1 = a
area 4.0 × 10 −5 m 2 under a given load. What is the r2 = b
ratio of the Young’s modulus of steel to that of
copper? [NCERT] 29. Two wires of diameter 0.25 cm, one made of steel
23. A uniform heavy rod of weight w, cross-sectional area and other made of brass are loaded as shown in
A and length l is hanging from a fixed support. figure below. The unloaded length of steel wire is
Young’s modulus of the material of the rod is Y. 1.5 m and that of brass wire is 1.0 m. Young’s
Neglecting the lateral contraction, find the modulus of steel is 2. 0 × 1011 Pa. Compute the
elongation produced in the rod. elongations of steel and brass wires.
24. A mild steel wire of length 1 m and cross-sectional (Take, 1 Pa = 1 N/m 2 ) [NCERT]
area 0. 5 × 10 −2 cm 2 is stretched, well within its
elastic limit, horizontally between two pillars. A
mass of 100 g is suspended from the mid-point of
the wire. Calculate the depression at the mid-point.
(Take, Young’s modulus for steel, Y= 2 × 1011 Pa) Element

[NCERT]
25. A rigid bar of mass 15 kg is supported 30. A rod of length 1.05 m having negligible mass is
symmetrically by three wires each 2 m long. Those
supported at its ends by two wires of steel (wire A)
at each end are of copper and the middle one is of
and aluminium (wire B) of equal lengths as shown
iron. Determine the ratio of their diameters, if each
in figure below. The cross-sectional areas of wires
wire have the same tension. Young’s modulus of
A and B are 1 mm 2 and 2 mm 2 , respectively.
elasticity for copper and steel are
Young’s modulus of elasticity for steel and
110 × 10 9 N / m 2 and 190 × 10 9 N / m2 , respectively.
aluminium are 2 × 1011 and 7 × 1010 N /m 2 ,
[NCERT]
respectively.
26. A 14.5 kg mass, fastened to one end of a steel wire
of unstretched length 1 m is whirled in a vertical
circle with an angular frequency of 2 rev/s at the
bottom of the circle. The cross-sectional area of the
A (Steel) (Aluminium) B
wire is 0.065 cm 2 . Calculate the elongation of the
wire, when the mass is at the lowest point of its x 1.05 m
path. [NCERT] y

27. Four identical hollow cylindrical columns of mild m


F1 F2
steel support a big structure of mass 50000 kg. The mg
inner and outer radii of each column are 30 cm and
CBSE Term II Physics XI 13

At what point along the rod should a mass m


l
Case Based Questions
be suspended in order to produce (i) equal 34. Direction Read the following passage and answer the
stresses and (ii) equal strains in both steel questions that follows
and aluminium wires. [NCERT]
Stress-Strain Curve
31. A rubber string 10 m long is suspended from
The stress-strain graph for a metal wire is shown in figure.
a rigid support at its one end. Calculate the Upto the point E, the wire returns to its original state O
extension in the string due to its own along the curve EPO, when it is gradually unloaded. Point
weight. The density of rubber is 1. 5 × 10 3 B corresponds to the fracture of the wire.
kg/m 3 and Young’s modulus for the rubber C
is 5 × 10 6 N / m 2 . EA
P
B
The breaking stress for a metal is
7.8 × 10 9 N / m 2 . Calculate the maximum

Stress
length of the wire made of this metal which
may be suspended without breaking. The
density of metal = 7.8 × 10 3 kg/ m 3 . O O′ Strain

32. What is the density of water at a depth, (i) Upto which point on the curve is Hooke’s law obeyed?
where pressure is 80. 0 atm? (Take, density at This point is sometimes called proportionality limit.
the surface is 1. 03 × 10 3 kg/ m 3 and (ii) Which point on the curve corresponds to elastic limit
compressibility of water is 45.8 × 10 −11 Pa −1 ) and yield point of the wire?
[NCERT] (iii) Indicate the elastic and plastic regions of the
33. Compute the Bulk modulus of water from stress-strain graph.
the following data; initial volume = 100.0 L, (iv) Describe what happens when the wire is loaded upto a
pressure increase = 100.0 atm (1 atm = 1.013 stress corresponding to the point A on the graph and
× 10 5 Pa), final volume = 100.5 L. Compare then unloaded gradually. In particular, explain the
the Bulk modulus of water with that of air dotted curve.
(at constant temperature). Explain in simple (v) How the graph from C to B is different from the rest?
terms, why the ratio is so large. Upto what stress can the wire be subjected without
[NCERT]
causing fracture?
Chapter Test
Short Answer Type Questions
Multiple Choice Questions
6. Find the change in volume which 1cc of water at the
1. If a force is applied to a plastic substance, then they surface will undergo, when it is taken to the bottom of the
have lake 100m deep. (Take, volume elasticity is 22000 atm)
I. no gross tendency to regain their original shape. (Ans. 4.5 × 10 −5 cc)
II. permanently deformed. 7. The star Sirius has a mass of 7 × 10 30 kg, its distance
III. tendency to regain their original shape. from the earth is 8 × 10 16 m and the mass of the earth is
IV. not permanently deformed. 6 × 10 24 kg. Calculate the cross-section of a steel cable
that can withstand the gravitational pull between the
Which of the following statement(s) is/are correct?
Sirius and the earth. (Take, G = 6.67 × 10 –11 Nm 2kg–2 and
(a) Only I (b) Both I and II
breaking stress = 10 10 Nm −2 ) (Ans. 44 m 2 )
(c) Only III (d) Both III and IV
2. In the given figure, if the dimension of the wire are the 8. A metal bar of length L and area of cross-section A, is
same and materials are different, Young’s modulus is rigidly clamped between two walls. The Young’s
more for modulus of its material is Y and the coefficient of linear
A expansion is α. The bar is heated, so that its
temperature is increased from 0 to θ°C. Find the force
Load

B exerted at the ends of the bar.


9. Assume that if the shear stress in steel exceeds about
4 × 10 8 N / m 2 , the steel reptures. Determine the
Extension shearing force necessary to (i) shear a steel bolt 1.00 cm
in diameter and (ii) punch a 1 cm diameter hole in a
(a) A (b) B steel plate 0.5 cm thick.
(c) Both (d) None of these (Ans. (i) 3. 14 × 10 4 N and (ii) 6.28 × 10 4 N)
3. Within the limit of elasticity, which of the following Long Answer Type Questions
graph obey Hooke’s law?
10. Four identical cylindrical columns of steel support a big
Extension
Extension

structure of mass 50000 kg. The inner and outer radii of


(a) (b) each column are 30 cm and 40 cm, respectively.
Assume the load distribution to be uniform, calculate
the compressional strain of each column. The Young’s
Load Load
modulus of steel is 2.0 × 10 11 Pa. . × 10 −6 )
(Ans. 28
Extension
Extension

11. Two different types of rubber are found to have the


(c) (d) stress-strain curves shown below in figure.

Load Load
Stress

Stress

4. A cube of aluminium of side 0.1 m is subjected to a


shearing force of 100 N. The top face of the cube is Rubber (A) Rubber (B)
displaced through 0.02 cm with respect to the bottom
face. The shearing strain would be O O
Strain Strain
(a) 0.02 (b) 0.1 (c) 0.005 (d) 0.002 (i) (ii)
5. One end of a uniform wire of length L and of weight w (i) In which significant ways, do these curves differ from
is attached rigidly to a point in the roof and a weight w 1 the stress-strain curve of a metal wire?
is suspended from its lower end. If S is the area of (ii) A heavy machine is to be installed in a factory. To
cross-section of the wire, the stress in the wire at a absorb vibrations of the machine, a block of rubber is
height 3L / 4 from its lower end is placed between the machinery and the floor. Which
w1 w1 + (w / 4 ) of the two rubbers A and B would you prefer to use
(a) (b)
S S for this purpose? Why?
w + (3 w / 4 ) w +w
(c) 1 (d) 1 (iii) Which of the two rubber materials would you choose
S S
for a car tyre?

Answers For Detailed Solutions


Multiple Choice Questions Scan the code
1. (b) 2. (a) 3. (c) 4. (d) 5. (c)
CBSE Term II Physics XI 15

EXPLANATIONS
m ⋅ (m + m) 2
PART 1 Tension in wire B = ⋅ g = mg …(i)
m + (m + m) 3
(b) The property of a body, by virtue of which it tends to
where, g is gravitational acceleration.
regain its original size and shape when the applied force
[Q Tension = Force = Mass × Acceleration]
is removed, is known as elasticity and the deformation
Force
caused is known as elastic deformation. Q Stress =
Area
(d) We know that,
Force (tension) 2 mg
Maximum force Stress in B = = …(ii)
Maximum stress = Cross -section area of wire B 3x
Area of cross - section
Area of cross-section remains same even, if the length of [using Eq. (i)]
Stress
the wire changes. Young’s modulus, Y =
Hence, maximum force will be same when length changes. Strain
Stress 2 mg
(d) When a horizontal force is applied on uniform bar to ∴ Strain in wire B = = [using Eq. (ii)]
Y 3 xY
pull it, then an acceleration is produced in the each
m⋅m mg
cross-section of rod. Hence, each section of rod Tension in wire A = g=
experiences a tension which is zero at other end. m + (m + m) 3
Therefore, stress in the rod developed gradually reduces mg
Similarly, stress in wire A =
to zero at the end of the bar, where no force is applied. 3x
(c) Consider the diagram, where a spring is stretched by So, statements II and IV are correct but I and III are
applying a load to its free end. Clearly, the length and incorrect.
shape of the spring changes. (c) B is brittle as there is no plastic region. However, A is
ductile as it has large plastic range of extension.
(a) Elastomers are those materials which can be
elastically stretched to a large value of strain. Elastic
Load
region for them is very large but they do not obey
Spring without load Hooke’s law.
The change in length corresponds to longitudinal strain Thus, Young’s modulus of elastomers are very small.
and change in shape corresponds to shearing strain. So, statements I and II are correct but III is incorrect.
(b) Initial length = L and final length = 2L (b) From given graph, P has more strain for same stress as
Initial length 2L − L on Q, so P is more ductile than Q.
Strain = = =1 Thus, the statement given in option (b) is correct, rest are
Original length L
incorrect.
(d) As wire are of some material, so stress produced is
(b) Given, stress, F = 20 × 10 8 Nm−2
same.
Stress
Q F ∝ A or F ∝ d 2 Young’s modulus =
Strain
Given, F1 = 1 × 10 −3 N, d 2 = 2 mm = 2 × 10 −3
As the length of wire gets doubled, therefore strain = 1.
d1 = 1 mm = 1 × 10 −3 m

F2  2 × 10 −3 
2
 Change in length 2L − L 
⇒ =  =4 Q Strain = = = 1
F1  1 × 10 −3   Original length L 

∴ F2 = F1 × 4 = 4000 N ∴ Y = Stress = 20 × 10 8 N m −2

(d) Volume of cube, V = L3 (d) Given, change in length of wire, l 1 = 2 mm,


∴ Percentage change in V = 3 × (Percentage change in L) Length of wire, L 1 = 2 m
= 3(1%) = 3% and length of another wire, L 2 = 8 m
∴ ∆V = 3% of V FL FL2
∆V 3 Change in length, l = =
⇒ Volumetric strain = = = 0.03 AY VY
V 100 where, Y is Young’s modulus.
(d) Let, cross-section area of wires A and B = x
∴ l ∝ L2
and Young’s modulus of wires = Y
(as V, Y and F are constants)
16 CBSE Term II Physics XI

l 2 L 2 
2
 8
2 (c) Since, depression δ in the bar at centre is directly
= =   = 16
l 1  L 1   2 proportional to load.
i.e. δ∝w
⇒ l 2 = 16 l 1 = 16 × 2 mm δ 2 w 2 2 w1
= = = 2 ∴ δ 2 = 2δ1
= 32 mm = 3.2 cm δ1 w1 w1
(d) Given, Young’s modulus, Y = 2 × 1011 Nm−2 , Hence, depression in the bar at the centre is increased to
Strain = 0.15 double.
Stress (a) Spring balance shows incorrect reading after using it
Q Y=
Strain for a long time as with time the spring in it loses its
∴ Stress = Y × Strain = 2 × 1011 × 0.15 = 0.3 × 1011 elastic strength. This phenomenon is knwon as elastic
fatigue.
= 3 × 1010 Nm−2
Therefore, both A and R are true and R is the correct
(d) Consider the diagram, where a deforming force F is explanation of A.
applied to the combination. (a) If a solid sphere placed in the fluid under high
Stress F/A pressure, then it is compressed uniformly on all sides.
For steel wire, Ysteel = =
Strain Strain The force applied by the fluids acts in perpendicular
direction at each point of the surface and the body is said
F to be under hydraulic compression.
Copper Steel This leads to decrease in its volume without any change
wire wire in its geometrical shape.
where, F is tension in each wire and A is cross-section Therefore, both A and R are true and R is the correct
area of each wires. explanation of A.
As F and A are same for both the wires, hence stress will (b) The strain produced by a hydraulic pressure is called
be same for both the wires. volumetric strain as pressure creates a normal force on
Stress Stress every point and is defined as the ratio of change in
(Strain) steel = , (Strain) copper = volume ∆V to the original volume V .
Ysteel Ycopper ∆V
i.e. Volume strain =
As, Ysteel ≠ Ycopper V
Hence, the two wires will have different strain. Therefore, both A and R are true but R is not the correct
(c) Given, 1 atm = 10 5 Nm −2 explanation of A.
∴ 100 atm = 10 7 Nm −2 and ∆V = 0.01% V Stress
(a) Young’s modulus of a material, Y =
Strain
∆V
∴ = 0.0001 Restoring force F
V ∴ Stress =
Area A
p 10 7
B= = = 1 × 1011 Nm −2 As, restoring force is zero for a plastic body.
∆V / V 0. 0001 ∴ Y=0
= 1 × 1012 dyne/cm 2 Therefore, both A and R are true and R is the correct
explanation of A.
(b) Statement given in option (b) is incorrect and it can be
corrected as (a) Molecules in gases are very poorly attracted by their
Gases are about a million times compressible than solids, neighbouring molecules.
so solids are least compressible. Since, compressibility is defined as the fractional change in
Rest statements are correct. volume per unit increase or decrease in pressure.
(a) A bridge has to be designed such that it can withstand K = 1 / B = − (1 / ∆p ) × ( ∆V / V )
the load of flowing traffic, the force of winds and its own where, B is bulk modulus and ∆p change in pressure.
weight. Since, steel is more elastic than aluminium. So, it So, in gases, fractional change in volume with per unit
can withstand the load of traffic. Thus, over bridges are increase or decrease in pressure is not very prominent.
constructed with steel but not with aluminium. Thus, they have large compressibility.
(b) A beam of length l, breadth b and depth d when Therefore, both A and R are true and R is the correct
loaded at the centre by a load w depresses by an amount explanation of A.
given by
(a) As a mountain base is not under uniform compression
wl 3 1
δ= 3
, i.e. δ ∝ and this provides some shearing stress to the rocks under
4 bd Y Y which they can flow.
CBSE Term II Physics XI 17

Thus, mathematically, it has been calculated that under Given, original length, L = 2.5 m
the elastic limit, maximum height of a mountain is ∆L 0. 012
~10 km. Strain = = 0. 012% =
L 100
Therefore, both A and R are true and R is the correct ∆L = Strain × L
explanation of A. 0.012
(i) (b) This force produces a change in shape of the or ∆L = Extension = ×L
100
body. Hence, volume remains constant.
0.012 × 2.5
(ii) (a) The resolved part of F along the normal is the = = 3 × 10 −4 m
100
tensile force on this plane and the resolved part
parallel to the plane is the shearing force on the = 0.3 mm
plane. Given, angle of shear, θ = 30°
F cos θ and change in length, ∆L = 250 cm = 2.5 m
F sin θ
∆L 2.5
θ ∴ Shear strain, tan θ = ⇒ tan 30 ° =
L L
F F
2.5 2.5
L= = = 4.332 m
Q Area of plane section = A sec θ tan 30 ° 0.577
Force F cos θ F Volume, V = L3 = 81.309 m 3
Tensile stress = = = cos 2 θ
Area A sec θ A When the deformation is sufficient enough such that it
 1  exceeds the elastic limit, the strain increases more rapidly
Q secθ = 
 cosθ  than stress. Hence, ratio of stress/strain decreases.
Force F sin θ Young’s modulus of wire,
(iii) (a) Shearing stress = = Mgl 4Mgl
Area A sec θ Y= =
πr 2 ∆l πD 2 ∆l
F F
= sin θ cos θ = sin 2 θ 4Mgl
A 2A (i) Elongation, ∆l =
πD 2 Y
(iv) (a) Tensile stress will be maximum, when cos 2 θ is
1
maximum, i.e. cos θ = 1 or θ = 0 °. or ∆l ∝ 2
D
(v) (a) Given, shearing stress will be maximum, when
sin2θ is maximum, i.e. sin2θ = 1 or 2 θ = 90 ° or So, if diameter is increased to two times, then the
θ = 45 °. elongation or increase in length will become
one-fourth.
πD 2 Y∆l
PART 2 (ii) Also, load, Mg =
4l
⇒ Mg ∝ D 2

Given, force, F = 10 kN = 1 × 10 4 N So, when diameter changes by a factor of two times,


Radius, r = 2 mm = 2 × 10 −3 m the wire can bear four times the original load.
Area, A = πr 2 = π × (2 × 10 −3 )2 = 12.56 × 10 −6 m 2 The situation is as shown in the figure
Force 1 × 10 4 N
Stress = = r
Area 12.56 ×10 −6 m 2
= 0.0796 × 1010 = 7.96 × 10 8 N/m 2
L
Given, radius of steel cable, r =1.5 cm = 1.5 × 10 −2 m
Maximum stress = 10 8 N/m 2
Area of cross-section of steel cable, A = πr 2 f
= 3.14 × (1.5 × 10 −2 )2 m 2 F L
Now, Young’s modulus, Y = ×
= 3.14 × 2.25 × 10 −4 m 2 A l
Maximum force F L
Maximum stress = For first wire, Y= × ...(i)
Area of cross - section πr 2
l
⇒ Maximum force = Maximum stress × Area of 2F 2L F L
For second wire, Y = × = × ...(ii)
cross-section π(2 r )2
l′ πr 2
l′
= 10 × (3.14 × 2.25 × 10 −4 ) N
8
From Eqs. (i) and (ii), we get
= 7.065 × 10 N4 F L F L
× = ×
= 7.1 × 10 4 N πr 2 l πr 2 l ′
18 CBSE Term II Physics XI

∴ l = l′ (ii) Yield strength of the given material


(Q Both wires are of same material, hence Young’s = Maximum stress
modulus will be same) = 300 × 10 6 N/m 2
According to Hooke’s law,
= 3 × 10 8 N/m 2
F l Fl
Modulus of elasticity, E = × or ∆l = Given, diameter = 2. 5 mm = 0.25 cm or r = 0.125 cm
πr 2
∆ l πr 2E
22
or ∆l ∝ 2
Fl
(QE is same for two wires) ∴ A = πr 2 = × ( 0.125 )2 sq. cm
7
r
2 F = 100 kg = 100 × 1000 g
∆l 1 F1 l 1 r22 1 2  3  9
∴ = × × = × ×  = = 100 × 1000 × 980 dyne
∆l 2 F2 l 2 r12 4 1  1  2
Y = 12. 5 × 1011 dyne/sq. cm
So, ∆l 1 : ∆l 2 = 9 : 2. F×l
As, Y=
Hence, the ratio of their extensions is 9 : 2. A × ∆l
(i) In the two graphs, the slope of graph in Fig. (a) is ∆l F
∴ =
greater than the slope of graph in Fig. (b), so l AY
material A has greater Young’s modulus.
Hence, % increase in length
(ii) Material A is stronger than material B because it can ∆l F
withstand more load without breaking. For material = × 100 = × 100
l AY
A, the break even point (D) is higher.
(100 × 1000 × 980 ) × 7 × 100
According to Hooke’s law, =
w L wL 22 × ( 0.125 )2 × 12. 5 × 1011
Modulus of elasticity, E = × ⇒ l =
A l AE = 0.1812 %
where, L = original length of the wire When mass M is placed on the piston, the excess
and A = cross-sectional area of the wire. pressure, p = Mg / A. As this pressure is equally
wL applicable from all the directions on the sphere, hence
∴ Elongation, ∆l = ...(i) there will be decrease in volume due to decrease in
E
radius of sphere.
On either side of the wire, tension is w and length is l /2. 4
wL /2 wL l Volume of the sphere, V = π R 3 .
∆l = = = [from Eq. (i)] 3
AE 2 AE 2
Differentiating it, we get
l l
∴ Total elongation in the wire = + = l 4
2 2 ∆V = π( 3 R 2 ) ∆R
3
Stress
Young’s modulus, Y = = 4 πR 2 ∆R
Longitudinal strain
∆V 4 πR 2 ∆R 3 ∆R
For same longitudinal strain, Y ∝ stress ∴ = =
V 4 R
Ysteel (stress)steel πR3
∴ = …(i) 3
Yrubber (stress)rubber
p Mg 3 ∆R
But Ysteel > Yrubber We know that, B = =
∆V / V A R
Ysteel
∴ >1 ∆R Mg
Yrubber or =
R 3BA
Therefore, from Eq. (i), we get
Depth, h = 11 km = 11 × 10 3 m
(stress)steel
> 1 or (stress)steel > (stress)rubber Pressure at the bottom of the trench, p
(stress)rubber
= 1.1 × 10 8 Pa
Therefore, steel will have greater tensile stress than
rubber. Initial volume of the ball, V = 0.32 m 3
(i) Young’s modulus of the given material, Y Bulk modulus of steel, B = 1.6 × 1011 N/m 2
= Slope of strain-stress curve p pV
We know that, B = =
150 × 10 6 ( ∆V / V ) ∆V
Y=
0. 002 pV 1.1 × 10 8 × 0.32
∆V = =
= 75 × 10 9 N/m 2 B 1.6 × 1011
= 7.5 × 1010 N/m 2 ∆V = 2.2 × 10 −4 m 3
CBSE Term II Physics XI 19

Force
Bulk modulus of rubber, B = 9.8 × 10 8 N/m 2 Pressure at the tip of the anvil, p =
Area
Density of seawater, ρ = 10 3 kg/m 3 F 50000
∴ p= = = 2.5 × 1011 Pa
Percentage decrease in volume, πr 2
3.14 × (2.5 × 10 −4 )2
 ∆V  ∆V 0.1 ∆V 1 Given, L = 1500 cm is the unstretched length of the rope,
 × 100 = 0.1 or = or =
 V  V 100 V 1000 and ∆L = 2.8 cm is the amount of length stretches.
∆L 2.8 cm
Let the rubber ball be taken up to depth h. (i) Strain = = = 1. 9 × 10 −3
Q Change in pressure, p = h ρg L 1500 cm
p h ρg Stress = Force/ Area
∴ Bulk modulus, B = =
( ∆V / V ) ( ∆V / V ) Force, F = force of gravity on the rock climber
1 = mg = 95 × 9.8 N
9.8 × 10 8 ×
B × ( ∆V / V ) 1000  D  9. 6
2

2
or h = = = 100 m Area, A = πr 2 = π ×   = π ×  × 10 −3 m
ρg 10 3 × 9.8 2  2 
Given,, m = 900 kg, a = 2.2 ms −2 95 × 9. 8
(ii) Stress = • 1.29 × 10 7 N/m 2
and maximum stress = 1. 3 × 10 Nm
8 −2 π × ( 4. 8 )2 × 10 −6
As the elevator moves up, the tension in the wire, (iii) Modulus of elasticity,
F = mg + ma = m ( g + a ) = 900 × ( 9. 8 + 2.2 ) = 10800 N Stress 1.3 × 10 7
E= = = 6.84 ×10 9 N/m 2
F F Strain 1.9 × 10 −3
Stress in the wire = =
A πr 2 Total length, L = 2 l .
Clearly, when the stress is maximum, r is minimum. Increase in length of the wire, when it is stretched from
F its mid-point is as shown in the figure.
∴ Maximum stress =
πrmin
2
l O l
A B
F 2T1 cos θ
or 2
rmin = x
π × Maximum stress √l2 + x2 θ θ
10800 T1
= = 0.2645 × 10 −4 m C T
3.14 × 1. 3 × 10 8 T
rmin = 0. 5142 × 10 −2 m
M Mg
or
Minimum diameter From Pythagoras theorem, BC2 = l 2 + x2
= 2 rmin = 2 × 0. 5142 × 10 −2 BC = l 2 + x2
= 1. 0284 × 10 −2 m Similarly, AC = l 2 + x2
Given, diameter of each rivet, D = 6 mm Change in length of the wire,
D ∆L = ( AC + CB ) − AB
∴ Radius, r = = 3 mm = 3 × 10 −3 m
2 = ( l 2 + x2 + l 2 + x2 ) − 2 l
Maximum shearing stress on each rivet = 6.9 × 10 7 Pa 1/ 2
 x2 
Let w be the maximum load that can be subjected to the = 2 ( l 2 + x2 )1 / 2 − 2 l = 2 l 1 + 2  − 2 l ...(i)
riveted strip, as each rivet carry one-quarter of the load.  l 
w Since x << l , so using binomial expansion, we have
Therefore, load on each rivet =
4 1/ 2
Maximum shearing force  x2   x2 
Maximum shearing stress = 1 + 2  = 1 + 2 
Area  l   2l 
w/4
∴ 6.9 × 10 7 = (neglecting terms containing higher powers of x)
πr 2
 x2  x2
or w = 6.9 × 10 7 × 4 πr 2 ∴ ∆ L = 2 l 1 + 2  − 2 l =
−3 2
 2l  l
or w = 6.9 × 10 × 4 × 3.14 × (3 × 10 )
7
∆L x2 x2
= 6.9 × 4 × 3.14 × 9 × 10 = 7.8 × 10 3 N Hence, strain = = = 2
L l × 2 l 2l
Given, compressional force, F = 50000 N
T = 2 T1 cosθ
Diameter, D = 0.5 mm = 5 × 10 −4 m
Q
Mg
D ∴ T1 = (Q T = Mg )
∴ Radius, r = = 2.5 × 10 −4 m 2 cosθ
2
20 CBSE Term II Physics XI

x
Putting cosθ = Elongation of the element
l + x2
2
Stress
1/ 2 = Original length ×
Mg Mgl  x2  Y
T1 = ( l 2 + x2 ) = 1 + 2  F/A w
2x 2x  l  = dx × = ( l − x) dx
Y l Ay
Mgl  x2 
= 1 + 2  [using (1 + x)x = 1 + xx] Total elongation produced in the rod
2x  2l  l
w  x2 
l
w
Q x << l ∴
x 2
→ 0 Thus, 1 +
x 2
=1
= ∫
l AY 0
( l − x) dx =  lx − 
l Ay  2 0
2l 2 2l 2
w  2 l2 wl
∴ T1 =
Mgl = l −  =
2x l Ay  2  2 Ay

Stress in the wire =


T1 Mgl
= Given, length, l = 1 m
A 2 xA Area of cross-section, A = 0.5 × 10 −2 cm 2
Given, for steel wire, length, l 1 = 4.7 m = 0. 5 × 10 −6 m 2
−5
Area of cross-section, A1 = 3.0 × 10 m 2
Mass, m = 100 g = 0.1 kg
and for copper wire, length, l 2 = 3.5 m ∴ Load, w = mg = 0.1 × 9.8 N = 0. 98 N
Area of cross-section, A 2 = 4.0 × 10 −5 m 2 Young’s modulus for steel, Y = 2 × 1011 Pa
Let F be the given load under which steel and copper A 0.5 × 10 −6
Area, A = πr 2 or r 2 = =
wires be stretched by the same amount ∆l. π π
F/A F×l Depression in a wire, when a load is suspended at its
Young’s modulus, Y = =
∆l / l A × ∆l centre,
F × l1 w l3 0. 98 × (1 )3
For steel, Ys = …(i) δ= =
A1 × ∆l 12πr 4 Y  0. 5 × 10 −6 
2

12 π ×   × 2 × 10
11
F × l2  π 
For copper, Yc = …(ii)
A 2 × ∆l 0. 98 × π
δ= = 5.12 m
Dividing Eq. (i) by Eq. (ii), we get 12 × 0.25 × 2 × 10 −1
Ys F × l1 A × ∆l Young’s modulus of copper, Y1 = 110 × 10 9 N/m 2
= × 2
Yc A1 × ∆l F × l2
Young’s modulus of steel, Y2 = 190 × 10 9 N/m 2
l A 4.7 4.0 × 10 −5
= 1 × 2 = × Let d1 and d 2 be the diameters of copper and steel wires.
l 2 A1 3.5 3.0 × 10 −5 Since, tension in each wire is same, therefore each wire
Ys 18.8 has same extension. As each wire is of same length, hence
= = 1.79= 1.8
Yc 10.5 each wire has same strain.
Stress F/A
The ratio of the Young’s modulus of steel to that of Young’s modulus, Y = =
copper is 1.8. Strain Strain
F 4F
As shown in figure, consider a small element of thickness or Y= =
dx at distance x from the fixed support. Force acting on  πd 
2
πd × Strain
2
  × Strain
the element dx,  4 
F = Weight of length ( l − x) of the rod 1 1
w Q Y∝ ⇒ d2 ∝
= ( l − x) d2 Y
l 2
d1 Y2
∴ =
d 22 Y1
x d1 Y2 190 × 10 9
or = =
d2 Y1 110 × 10 9
19
dx = = 1.73 = 1.31
11
l–x
∴ d1 : d 2 = 1.31 : 1
Hence, the ratio of their diameters is 1.31 : 1.
CBSE Term II Physics XI 21

Given, mass, m = 14 .5 kg =
F/A
=
F
Length of wire, l = 1 m Y AY
Angular frequency, ν = 2 rev/s (Q Young’s modulus, Y = 2 × 1011 Pa)
Angular velocity, ω = 2 πν 122500
=
= 2 π × 2 = 4π rad/s (3.14 × 0.27) × 2 × 1011
= 0.722 × 10 −6 = 7 .22 × 10 −7
If R be the rate of change of radius per length
O r − r1 b − a
T
R= 2 = …(i)
l l
Consider an element of thickness dx at a distance of l
from narrow end of the wire.
mg

Area of cross-section of wire, A = 0.065 cm 2 r1


y r2
−6
= 6.5 × 10 m 2

Young’s modulus for steel, Y = 2 × 10 N/m .


11 2
dx
l
At lowest point of the vertical circle,
Radius of element y = r1 + Rx = a + Rx …(ii)
T − mg = m lω 2
From Young’s modulus, the extension in the element,
or T = mg + mlω 2 Fdx Fdx
dl = =
= (14.5 × 9.8) + 14.5 × 1 ×( 4 π )2 AY ( πy 2 )Y
= 14.5(9.8 + 16 π 2 ) Integrating both sides, we get
l Fdx
= 14.5(9.8 + 16 × 9.87) (Q π = 9.87 )
2
∫ dl = ∫0 πYy 2
= 14.5 × 167.72 N = 2431.94 N
Stress ( T / A ) Tl As, y = a + Rx [using Eq. (ii)]
Young’s modulus, Y = = =
Strain ∆l / l A ⋅ ∆l ⇒ dy = Rdx
T⋅l 2431. 94 × 1 F a + Rl dy
πY ∫a
∴ ∆l = = ⇒ l=
A ⋅ Y 6. 5 × 10 −6 × 2 × 1011 Ry 2
a + Rl
= 1.87 × 10 −3 m = 1.87 mm F  −1 
=  
Given, total mass supported by cylindrical columns, πYR  y  a
m = 50000 kg F 1 1 
= −
Q Total weight supported by cylindrical columns πYR  a ( a + Rl )
= mg = 50000 × 9. 8 = 490000 N
F  a + Rl − a 
∴ Load acting on each cylindrical support, =
πYR  a ( a + Rl ) 
mg 490000
F= = N Fl
4 4 = [using Eq. (i)]
= 122500 N   b − a 
πYa a +   l
Inner radius of each column, r1 = 30 cm = 0. 3 m   l  
Outer radius of each column, r2 = 60 cm = 0. 6 m Fl
=
∴ Area of cross-section of each cylindrical column, πabY
A = π r22 − π r12 = π ( r22 − r12 ) Given, diameter of wires, 2r = 0.25 cm
= 3.14 [(0.6)2 − (0.3)2 ] ∴ r = 0.125 cm = 1.25 × 10 −3 m
For steel wire
= 3.14 × 0.27 m 2
Load, F1 = ( 4 + 6 ) kg-f = 10 × 9.8 N = 98 N
Compressional stress
Young’s modulus, Y = Length of steel wire, l 1 = 1.5 m
Compressional strain
Young’s modulus, Y1 = 2. 0 × 1011 Pa
Compressional stress
⇒ Compressional strain = F × l1
Young’s modulus Young’s modulus, Y = 1
A1 × ∆l 1
22 CBSE Term II Physics XI

F1 × l 1 F ×l F1
∴ Change in length, ∆ l 1 = = 12 1 For steel wire A, (Strain) steel =
A1 × Y1 πr1 × Y1 A1 Y1
98 × 1.5 For aluminium wire B,
=
3.14 × (1.25 × 10 −3 )2 × 2.0 × 1011 (Strain) aluminium =
F2
= 1.5 × 10 m −4 A 2 Y2
For brass wire, When equal strains are produced in both wires,
Load, F2 = 6 kg-f = 6 × 9. 8 N = 58. 8 N then
F1 F2
Length of brass wire, l 2 = 1. 0 m =
A1 Y1 A 2 Y2
Young’s modulus, Y2 = 0. 91 × 1011 Pa
F1 AY
F ×l or = 1 1 …(iii)
Change in length, ∆ l 2 = 22 2 F2 A 2 Y2
πr2 × Y2
∴ From Eqs. (ii) and (iii), we get
58.8 × 1.0 (1. 05 − x) A1 Y1
= =
3.14 × (1.25 × 10 −3 )2 × 0.91 × 1011 x A 2 Y2
= 1.3 × 10 −4 m 1 × 10 −6 2 × 1011
Let the length of wires A and B is equal to L and their = −6
×
2 × 10 7 × 1010
area of cross-section be A1 and A 2 , respectively.
(1. 05 − x) 10
Given, A1 = 1 mm 2 = 1 × 10 −6 m 2 =
x 7
A 2 = 2 mm 2 = 2 × 10 −6 m 2 10 x = 7 . 35 − 7 x
Ysteel = 2 × 1011 N/m 2 7 . 35
⇒ 17 x = 7 . 35 or x =
YAl = 7.0 × 1010 N/m 2 17
Let F1 and F2 be the tensions in the two wires, x = 0.43 m
respectively. ∴ The mass m must be suspended at a distance
(i) When equal stresses are produced, then 0.43 m from the steel with A.
F1 F2 Given, l = 10 m, ρ = 1. 5 × 10 3 kg / m 3
=
A1 A 2 Y = 5 × 10 6 N / m 2
F1 A1 1 × 10 −6 Fl
⇒ = = We know that, Y =
F2 A 2 2 × 10 −6 A ∆l
F1 1
⇒ = …(i)
F2 2
Let mass m be suspended at distance x from steel Mg
wire A. dy
Taking moment of forces about the point of
y
suspension of mass from the rod, we get
F1 × x = F2 × ( 1. 05 − x)
F1 ( 1. 05 − x) Efficient force = Mg
or = …(ii)
F2 x Consider a small length dy at a distance y from free end.
From Eqs. (i) and (ii), we get The length above this, ( l − y ) will experience a force of
1 (1. 05 − x) M
= Fdy = gdy
2 x l
x = 2.10 − 2 x ∴ Extension, dl =
Fl
or 3 x = 2.10 AY
or x = 0.70 m (l − y) M Mg
⇒ dl = ⋅ gdy = ( l − y )dy
∴ The mass m must be suspended at a distance AY l l AY
0.70 m from steel wire A. Net extension due to its own weight = ∫ dl
Stress
(ii) Young’s modulus, Y = l
Mg  y2 
l
Strain Mg Mg l
∴ Strain =
Stress F / A
=
= ∫
A Yl 0
( l − y )dy = ly −  =
l AY  2  0 2 AY
Y Y
CBSE Term II Physics XI 23

Mg l Mg l 2 ρgl 2 1.03 × 10 3
Net extension = = = ⇒ = 1 − 3.712 × 10 −3
2 AY 2 YV 2Y ρ′
Extension of rubber string 1.03 × 10 3
or ρ′ = = 1.034 × 10 3 kg/m 3
1. 5 × 10 3 × 10 × 10 2 1 − 0.003712
= = 0.15 m
2 × 5 × 10 6 Given, initial volume, V1 = 100. 0 L
Breaking stress for a metal = 7 . 8 × 10 9 N / m 2 Final volume, V2 = 100.5 L
Density = 7 . 8 × 10 3 kg / m 3 ∴ Increase in volume, ∆V = V2 − V1
Force Mg Mgl Mgl = 100. 5 − 100. 0 = 0. 5 L
Stress = = = = = ρ gl
Area A Al Volume = 0. 5 × 10 −3 m 3 (Q 1 L = 10−3 m 3 )
If ρgl > Breaking stress, the wire will break. Increase in pressure,
7 . 8 × 10 9 7 . 8 × 10 9 ∆p = 100.0 atm
∴ l≤ ,l≤
ρg 7 . 8 × 10 3 × 10 = 100. 0 × 1. 013 × 10 5 Pa
i.e. l ≤ 10 5 m (Q 1 atm = 1.013 × 10 5 Pa)
Maximum length of wire = 10 5 m = 1. 013 × 10 7 Pa
Bulk modulus of water,
Density of water at the surface, ρ0 = 1. 03 × 10 3 kg/m 3
∆p
Pressure, p = 80. 0 atm = 80. 0 × 1. 013 × 10 5 Pa Bw =
( ∆V / V )
(Q 1 atm = 1.013 × 10 5 Pa) ∆pV 1. 013 × 10 7 × 100 × 10 −3
1  = =
Compressibility of water   = 45. 8 × 10 −11 Pa −1 ∆V 0. 5 × 10 −3
 B
10.13
= × 10 9
Let V and V′ be the volumes of certain mass of water at 5
the surface and at a given depth. The density of water at
= 2.026 × 10 9 Pa
the given depth be ρ′.
m Bulk modulus of air, B a = 1.0 × 10 5 Pa
Volume of water at the surface, V =
ρ Bulk modulus of water (B w ) 2.026 × 10 9
∴ =
m Bulk modulus of air (B a ) 1.0 × 10 5
At the given depth, V ′ =
ρ′ = 2.026 × 10 4
1 1  This ratio is too large as gases are more compressible
∴ Change in volume, ∆V = V − V ′ = m  −  than those of liquids. In liquids, interatomic forces are
 ρ ρ′ 
more strong than that for gases.
∆V 1 1  ρ (i) Upto point P, stress ∝ strain. So, Hooke’s law is
Volumetric strain = = m −  ×
V  ρ ρ′  m obeyed upto this point.
 ρ (ii) Point E corresponds to elastic limit and yield point
= 1 − 
 ρ′  of wire, as it returns to original state at O along EPO,
when gradually unloaded.
1
Compressibility = (iii) The graph from O to E shows elastic region and
Bulk modulus (B )
from E to B shows plastic region.
1 ∆V
= = (iv) Upto point P, stress is proportional to strain. From P
∆p ∆pV to E, strain increases more than stress. Here,
( ∆V / V ) Hooke’s law is not obeyed. When wire is unloaded
 ρ 1 at point A beyond E, it does not retrace the curve
45.8 × 10 −11 = 1 −  × along AEPO but follows the dotted curve along
 ρ′  80 × 1.013 × 10 5
AEPO. So, a strain OO′ is left for zero stress on wire.
1.03 × 10 3 (v) Between points C and B, the wire virtually flows
⇒ 45.8 × 10 −11 × 80 × 1.013 × 10 5 = 1 −
ρ′ out, i.e. the strain increases even when the wire is
being unloaded. Fracture takes place at point B. The
1.03 × 10 3
⇒ 3.712 × 10 −3 = 1 − stress can be applied to the value corresponding to
ρ′ point C without causing fracture.
24 CBSE Term II Physics XI

CHAPTER 02

Mechanical
Properties of Fluids
In this Chapter...
l Thrust l Flow of Fluids
l Pressure l Bernoulli’s Theorem
l Pascal’s Law l Viscosity
l Archimedes’ Principle l Critical Velocity
l Surface Tension l Stokes’ Law
l Surface Energy l Terminal Velocity
l Angle of Contact l Reynold’s Number
l Capillarity

If the total force F acts normally over a flat area A, then


Thrust
F
The molecules of a fluid kept in a container are in random Pressure, p =
motion due to their thermal velocities. So, they constantly A
collide with the walls of the container and rebounding from The unit of pressure is dyne / cm 2 in CGS system and N /m 2
them, suffering a change in momentum normal to the or pascal in SI system and dimensional formula of pressure
colliding walls due to which some normal change in is [ML −1 T −2 ] .
momentum is transferred to the walls.
Other units of pressure 1 Pa (or 1 N / m 2 ) = 10 dyne / cm 2
1 atm = 1.013 × 10 5 Pa
If the force is not distributed uniformly over the given
surface, then pressure will be different at different points. If a
force ∆F acts normally on a small area ∆A surrounding a
Liquid in a container is exerting thrust
given point, then pressure at that point will be
The normal change in momentum transferred to the walls of ∆F dF
the container per unit time by the molecules of the fluid is p = lim =
∆A → 0 ∆ A dA
called thrust of the fluid on the container.
Pressure is a scalar quantity because liquid pressure at a
Pressure particular point in liquid has same magnitude in all
directions. This shows that a definite direction is not
The pressure of liquid at a point is the thrust (or normal associated with liquid pressure.
force) exerted by the liquid at rest per unit area around that
point.
CBSE Term II Physics XI 25

Density and Relative Density It is equivalent to one atmosphere (1 atm).


Density of a substance is defined as mass per unit volume of the
substance. It is denoted by ρ. If M be the mass of a substance of
volume V, then density of that substance is given by
h
M
Density, ρ =
V
The SI unit of density is kg / m 3 and dimensional formula pa
is [ML − 3 ].
Relative density (or specific gravity) of substance is defined as the B A
ratio of the density of that substance to the density of water at 4 ° C,
Density of substance
i.e. Relative density =
Density of water at 4 ° C Mercury barometer

Atmospheric pressure is also measured in torr a unit


It has no unit and no dimensions. It is a positive scalar quantity.
named after Torricelli.
The density of water at 4° C is maximum and equal
1 torr = 1 mm of Hg = 133 Pa
to1000 kg/ m 3 .
1 atm = 1.013 bar = 760 torr
Variation of Pressure with Depth Open Tube Manometer
The pressure p at depth h below the surface of a liquid open to the It is used to measure pressure difference. It consists
atmosphere is greater than atmospheric pressure by an amount ρgh. of a U-shape tube containing a liquid having low
Pressure, p = p a + hρg density such as oil to measure the small pressure
difference and mercury as the high density liquid to
This excess of pressure at depth h in liquid p − p a = hρg, i.e. called measure the large pressure differences.
gauge pressure.
pa
Hydrostatic Paradox
The pressure exerted by a liquid depends only on the height of fluid
column and is independent of the shape of the containing vessel, Liquid
known as hydrostatic paradox. h
p
Atmospheric Pressure A B
The gaseous envelope surrounding the earth is called earth’s Vessel
atmosphere. The pressure exerted by the atmosphere is called
atmospheric pressure. The force exerted by air column of air on a
unit area on the earth’s surface is equal to the atmospheric pressure.
Open tube manometer
It is denoted by p a .
The value of atmospheric pressure on the surface of the earth at sea
level called 1 atmosphere (1 atm), it is nearly about 1.013 × 10 5 N / m 2 .
Pascal’s Law
The earth’s atmosphere exerts a huge pressure, which can be
This law states that, whenever external pressure is
demonstrated by the following methods are given below
applied on any part of a fluid contained in a vessel at
Mercury Barometer rest, it is transmitted undiminished and equally in all
It is an instrument to measure atmospheric pressure. It is known as directions.
barometer. In this, a long glass tube closed at one end and filled with
mercury is inverted into a trough of mercury.
We find that, mercury column in the tube has height of about 76 cm
above the mercury level in the trough.
26 CBSE Term II Physics XI

Applications of Pascal’s Law The weight of the body acts at its centre of gravity. But the
Some applications of Pascal’s law are given below buoyant force acts at the centre of buoyancy which is the
centre of gravity of the liquid displaced by the body, when
(i) Hydraulic Lift It is used to lift heavy loads and it is a
immersed in the liquid.
force multiplier.
f Load
Archimedes’ Principle
F
The Archimedes’ principle gives the magnitude of buoyant
a force on a body. It states that,
C1 C2
A ‘‘When a body is immersed wholly or partially in a liquid at
rest, it experiences an upthrust.’’
E The upthrust is equal to the weight of the liquid displaced by
Liquid
the immersed part of the body and its upthrust acts through
the centre of gravity of the displaced liquid.
Hydraulic lift
The following three cases are possible, when the body is
According to Pascal’s law, pressure p applied on immersed in the fluid
smaller piston C1 is transmitted to the larger piston of (i) If the weight of the body is greater than the upward
cross -sectional area A, then upward force on larger force or upthrust by the fluid acting upwards, then
piston C 2 is the body sinks.
A (ii) If the weight of the body is equal to the upthrust or
f F=
a the weight of the body is just balanced by upthrust,
(ii) Hydraulic Brakes The working of the hydraulic then the body floats fully immersed.
brakes is also based on the Pascal’s law. (iii) If the weight of the body is less than the upward
It consists of a tube T containing brake oil. One end of force, then the body floats partly immersed.
the tube is connected to the wheel cylinder having If the total volume of the body is VS and a part VP of it is
two pistons P1 and P2 . The pistons P1 and P2 are submerged in the fluid, then at equilibrium,
connected to the brake shoes S1 and S 2 respectively
and the other end of tube is connected to master Weight of the body = Weight of fluid displaced
cylinder fitted with the piston P. VS ρS g = VP ρ l g …(i)
where, ρS and ρ l are the densities of the body and fluid
Lever system
To other respectively.
wheels Tube T P ρS VP
= [from Eq. (i)]
ρl VS
P1 P2
Wheel Brake
Master cylinder pedal
cylinder
Surface Tension
Brake shoes It is defined as the force acting per unit length of an
imaginary line drawn on the liquid surface, the direction of
S1 S2
force being perpendicular to this line and tangential to the
liquid surface. It is denoted by S and it is a scalar quantity.
Force
Constructional details of hydraulic brakes Surface tension, S =
Length
The piston P is connected to the brake pedal through
the lever system. The area of cross-section of the SI unit of surface tension = N/ m and CGS unit of surface
tension = dyne/ cm.
wheel cylinder is greater than that of master cylinder.
Dimensional formula of surface tension is [ML 0 T − 2 ].
The arrangement is shown in above figure.
Here, a small force applied to the pedal exerts a much Factors Affecting Surface Tension
larger force on the wheel drums, which enables the
driver to keep the vehicle under control. There are some factors affecting surface tension which are as
given below
Buoyancy (i) Temperature The surface tension of liquid decreases
The upward force acting on the body immersed in a fluid is with rise in temperature and vice-versa.
called upward thrust or buoyant force and the phenomenon The surface tension of a liquid becomes zero at a
is called buoyancy. particular temperature, called critical temperature of
that liquid.
CBSE Term II Physics XI 27

S t = S 0 (1 − αt ) where, S sa = surface tension between solid and air,


where, S t = surface tension at t ° C, S la = surface tension between liquid and air
S 0 = surface tension at 0° C and S sl = surface tension between solid and liquid.
and α = the temperature sufficient of surface tension. The following cases arise for the given figure
(ii) Addition of Impurities The surface tension of liquids (i) If the surface tension at the solid-liquid S sl interface is
changes appreciably with addition of impurities. greater than the surface tension at the liquid-air S la
interface, i.e. S sl > S la , then cos θ is negative and
Applications of Surface Tension θ > 90° (the angle of contact is obtuse angle).
Some applicaitons of surface tension are given below (ii) If the surface tension at the solid-liquid S sl interface is
(i) Rain drops and drops of mercury placed on glass plate less than the surface tension at the liquid-air S la
are spherical. interface, i.e. S sl < S la , then cos θ is positive and
(ii) Hair of shaving brush/painting brush, when dipped in θ < 90 ° (the angle of contact is acute angle).
water spread out, but as soon as, it is taken out, its
Excess Pressure Inside a Liquid Drop
hair stick together.
(iii) A greased needle placed gently on the free surface of Suppose a spherical liquid drop of radius R and S be the
water in a beaker does not sink. surface tension of liquid. Due to its spherical shape, there is
an excess pressure p inside the drop over that on outside.
(iv) Oil drop spreads on cold water but does not change
This excess pressure acts normally outwards. Due to this
shape on hot water.
pressure, radius increases from R to R + dR , then extra
surface energy can be determined.
Surface Energy
It may be defined as the amount of work done in increasing
the area of the liquid surface by unity. Thus,
R dR
Work done in increasing the surface area
Surface energy = pi p0
Increase in surface area
p=pi – p0
The SI unit of surface energy is J/m 2 .
The value of surface energy of liquid is numerically equal to Excess pressure inside a liquid drop
the value of surface tension.
Excess pressure inside the drop, p = p i − p 0
Angle of Contact where, p i = total pressure inside the liquid drop
and p 0 = atmospheric pressure.
The angle between tangent to the liquid surface at the point
of contact and the solid surface inside the liquid is called as Initial surface area of the liquid = 4 πR 2
angle of contact. It is denoted by θ. Final surface area of the liquid drop
The value of angle of contact depends on the following = 4 π (R + dR ) 2
factors
Increase in the surface area of liquid drop
(i) Nature of the solid and liquid in contact.
(ii) Cleanliness of the surface in contact. = 4 πR 2 + 8 πR dR − 4 πR 2
(iii) Medium above the free surface of the liquid. = 8πRdR
(iv) Temperature of the liquid. External work done in increasing the surface area of the drop
For the molecule O to be in equilibrium, angle of contact is = Increase in surface area × Surface tension
given by = (8πRdR) × S ...(i)
Sla But work done
Sla
= excess pressure × area × change in radius
θ θ = p × 4 πR 2 × dR ...(ii)
Ssa O Ssl Ssa Ssl
(a) (b) From Eqs. (i) and (ii), we get
Different shapes of water drops with interfacial tensions p × 4 π R 2 × dR = 8 πRdRS
S sa − S sl 2S
cos θ = Excess pressure, p =
S la R
28 CBSE Term II Physics XI

So, pressure difference in a drop bubble, This phenomenon of rise or fall of a liquid in the capillary is
2S called capillarity.
pi − p0 =
R
Excess Pressure Inside a Soap Bubble
From the above case, increase in surface area = 8πRdR
But a soap bubble has two free surfaces.
Water Mercury
Capillarity

Some examples are given below


pi p0 (i) We use towels for drying our skin.
(ii) In trees sap rises due to vessels. It is similar to capillary
action.
Excess pressure inside a soap bubble Capillary Rise
So, effective increase in surface area of the soap bubble One application of the pressure difference across a curved
= 2 × 8πR dR = 16πR dR ...(i) surface is the water rises up in a narrow tube (capillary) in
External work done in increasing the surface area of the soap spite of gravity. Consider a capillary of radius R is inserted
bubble = Increase in surface energy into a vessel containing water.
= Increase in surface area × Surface tension pa r θ
R
= 16πR dRS A p0 h θ

But, work done = force × change in radius B


S
where, force = excess pressure ( p ) × area ( 4 πr 2 )
So, work done = p × 4 πR 2 × dR ...(ii)
(a) (b)
From Eqs. (i) and (ii), we get
Capillary rise
p × 4 πR 2 × dR = 16 πRdRS
The surface of water in the capillary becomes concave. It
4S
p= means that, there must be a pressure difference between the
R two sides of the meniscus.
Pressure difference inside a soap bubble, So, ( p a − p 0 ) = (2S/ r )
4S = 2S/(R sec θ)
pi − p0 =
R
= (2S / R ) cos θ …(i)
Excess pressure inside an air bubble in a liquid is similar to a
liquid drop in air, it has only one free spherical surface. where, r = radius of curvature of the concave meniscus.
Hence, excess pressure is given by Now, consider two points A and B. According to Pascal’s law,
2S they must be at the same pressure,
p= p 0 + hρg = p A = p B = p a
R
Note When an air bubble of radius R lies at a depth h below the free So, p a = p 0 + hρg
surface of a liquid of density ρ and surface tension S, the excess p a − p 0 = ρgh
pressure inside the bubble will be …(ii)
2S
p= + hρ g
R (p a = atmospheric
pressure)
Capillarity From Eqs. (i) and (ii), we get
2S
If a capillary tube of glass is dipped in liquid like water, the ρgh = cos θ
R
liquid rises in the tube, but when the capillary tube is dipped
in a liquid like mercury, the level of liquid falls in the tube. Hence, height of rise of liquid in capillary is given by
2S cos θ
h=
ρRg
CBSE Term II Physics XI 29

irrotational flow remains constant at every cross-section


Flow of Fluids throughout the liquid flow.’’
(i) Streamline Flow Mathematically, it can be expressed as
1
The flow of a liquid in which each particle of the liquid p + ρv 2 + ρgh = constant
passing through a point travels along the same path and with 2
the same velocity as the preceding particle passing through p v 2 constant
or +h+ = = new constant
the same point, is called streamline flow. ρg 2g ρg
This flow has following properties are given below p
Here, is called pressure head, h is called gravitational
(a) In streamline flow, no two streamlines can cross each ρg 2
other. v
head and is called velocity head.
(b) The greater is the crowding of streamline at a place 2g
greater is the velocity of the liquid particles at that
Bernoulli’s Equation for the Fluid at Rest
place and vice-versa.
When a fluid at rest, i.e. the velocity is zero everywhere, then
(ii) Laminar Flow the Bernoulli’s equation becomes
If the liquid flows over a horizontal surface in the form of p 1 − p 2 = ρg ( h 2 − h 1 )
layers of different velocities, then the flow of liquid is called
laminar flow. Limitations of Bernoulli’s Theorem
Some limitations of Bernoulli’s theorem are given below
(i) Bernoulli’s equation ideally applies to fluids with zero
viscosity or non-viscous fluids.
(ii) The fluids must be incompressible, as the elastic
energy of the fluid is also not taken into consideration.
Laminar flow of liquid (iii) Bernoulli’s equation is applicable only to streamline
flow of a fluid. It is not valid for non-steady or
(iii) Turbulent Flow turbulent flow.
The flow of fluid in which velocity of all particles crossing a
given point is not same and the motion of the fluid becomes Applications of Bernoulli’s Theorem
irregular or disordered is called turbulent flow as shown in Some applications of Bernoulli’s theorem are given below
figure.
(i) Speed of Efflux (Torricelli’s Law)
According to Torricelli’s, velocity of efflux, i.e. the velocity
with which the liquid flows out of an orifice (i.e. a narrow
hole) is equal to that which a freely falling body would
Pillar
acquire in falling through a vertical distance equal to the
depth of orifice below the free surface of liquid.
A closed tank containing a liquid of density ρ with a small
Turbulent flow of liquid hole on its side at a depth h.
2 p2 = p A2
Equation of Continuity
It states that, during the streamline flow of the non-viscous h
A1
and incompressible fluid through a pipe of varying
1
cross-section, the product of area of cross-section and the y2 v1
normal fluid velocity ( Av ) remains constant throughout the pa

flow. y1

Bernoulli’s Theorem Fluid flow from an orifice


It is based on the law of conservation of energy and applied
to ideal fluids. It states that Velocity of the liquid falling from orifice,
‘‘the sum of pressure energy per unit volume, kinetic energy 2 (p − p a )
per unit volume and potential energy per unit volume of an v 1 = 2 gh +
ρ
incompressible, non-viscous fluid in a streamlined
30 CBSE Term II Physics XI

When the tank is open to the atmosphere, (iv) Blood Flow and Heart Attack
v 1 = 2 gh According to Bernoulli’s principle, the pressure inside artery
becomes low and the artery may collapse due to external
Distance at which the stream strikes the floor, pressure. The activity of heart is further increased in order to
x = 2 hy 1 force the blood through that artery. As the blood rushes
through that artery, the internal pressure once again drops due
(ii) Venturimeter to same reason. This will be leading to a repeat collapse. This
It is a device used to measure the flow speed of incompressible phenomenon is called vascular flutter which can be heard on
fluid through a pipe. It is also called flow meter or venturi a stethoscope. This may result in a heart attack.
tube.
(v) Dynamic Lift
It consists of a horizontal tube having wider opening of
Dynamic lift is the force that acts on a body by virtue of its
cross-section a 1 and a narrow neck of cross-section a 2 . These
motion through a fluid. It is responsible for the curved path
two regions of the horizontal tube are connected to a
of a spinning ball and the lift of an aircraft wing.
manometer, containing a liquid of density ρ m .
l
Ball Moving without Spin When the velocity of the air
A v1 above the ball is same as below the ball at the
v2 B
corresponding points resulting in zero pressure difference.
a1
The air therefore, exerts no upward or downward force on
the ball.
a2
l
Ball Moving with Spin The speed of air above the ball with
1
h respect to it is greater than below the ball. Hence, the
Liquid of density ρm
pressure below the ball is greater than that above the ball.
The force acts on the ball which makes it follow a curved
path.
Venturimeter The difference in lateral pressure, which causes a spinning
− 1/ 2
ball to take a curved path which is curved towards the
2 hρ m g  a 12  greater pressure side, is called magnus effect.
Velocity of flow in wider region, v 1 =  2 − 1
ρ  a2  l
Aerofoil Lift of an Aircraft Wing Aerofoil is a solid object
shaped to provide an upward dynamic lift as it moves
The volume of the liquid flowing per second through the
horizontally through air. This upward force makes
wider tube is
aeroplane fly. The cross-section of the wing of an aeroplane
2 (p1 − p 2 )
V = a1 a 2 looks like an aerofoil.
ρ ( a 12 − a 22 )

(iii) Atomizer or Sprayer Viscosity


The forward stroke of the piston produces a stream of air When a solid body slides over another solid body, the force
past the end of the tube. The air flowing past the open end of of friction opposes the relative motion of the solid bodies. In
the tube reduces the pressure on the liquid. So, the the same way, when a layer of fluid slides over another layer
atmospheric pressure acting on the surface of liquid forces, of the same fluid, a force of friction comes into play which is
the liquid into the tube D. As a result, the liquid rises up in called viscous force. This force opposes the relative motion
the vertical tube A. When it collides with the high speed air of the different layers of a fluid.
in tube B, it breaks up into fine spray. So, the tendency of fluids to oppose the relative motion of its
layers is called viscosity of fluid. The backward dragging
C p2 B force called viscous drag or viscous force.
Spray
In a pipe or a tube, the velocity of the liquid is maximum
Rubber balloon
along the axis of the tube and decreases gradually, as it moves
D
towards the walls, where it becomes zero.
A Stem

p1 Liquid Coefficient of Viscosity


Container It is defined as the ratio of shearing stress to the strain rate.
Fl F / A
η= =
Atomizer or sprayer vA dv
dx
CBSE Term II Physics XI 31

Dimensional formula of η is [ML −1 T −1 ]. (ii) The flow of liquids of lower viscosity and higher density
through broad pipes tends to become turbulent because
In CGS system, the unit of ηis
in that case the critical velocity will be very small.
1 poise = 1 dyne-s /cm 2 .
The SI unit of ηis
Stokes’ Law
1 poiseuille or 1 decapoise = 10 poise
The backward dragging force F acting on a small spherical
The coefficient of viscosity is a scalar quantity. body of radius r, moving through a fluid of coefficient of
η liquid
Note Relative viscosity of liquid = viscosity η, with velocity v is given by
η water
F = 6πηrv
Relative viscosity of bloods remains constant between 0° C
and 37°C. This is called Stokes’ law of viscosity.

Effect of Temperature on the Viscosity


Terminal Velocity
The viscosity of liquids decreases with increase in
temperature and increases with decrease in temperature. The maximum constant velocity acquired by a body while
falling through a viscous fluid is called its terminal velocity.
1
i.e. η∝ 2 r 2 (ρ − σ ) g
T v= ⋅
9 η
On the other hand, the viscosity of gases increases with the
increase in temperature and vice-versa. where, r = radius of the spherical body,
v = terminal velocity
i.e. η∝ T
and η = coefficient of viscosity of fluid.

Critical Velocity Reynold’s Number


The critical velocity of a liquid is that limiting value of its It is a dimensionless parameter whose value decides the nature
velocity of flow upto which the flow is streamlined and of flow of a liquid through a pipe, i.e. whether a flow will be
above which the flow becomes turbulent. steady or turbulent. It is given by
ρvD
Mathematically, it can be expressed as Reynold’s number, R e =
kη η
Critical velocity, v c =
ρr where, ρ = density of the liquid,
where, η = coefficient of viscosity of the liquid, v = velocity of the liquid,
η = coefficient of viscosity of the liquid
ρ = density of the liquid
and D = diameter of the pipe.
and r = radius of the tube.
(i) If R e lies between 0 and 2000, then liquid flow is
For the flow to the streamline, value of v c should be as
streamline or laminar.
large as possible. For this, ηshould be large, ρ and r should
be small. So, we conclude that (ii) If R e > 3000, then liquid flow is turbulent.
(iii) If R e lies between 2000 and 3000, then flow of liquid is
(i) The flow of liquids of higher viscosity and lower unstable, it may change from laminar to turbulent and
density through narrow pipes tends to be vice-versa.
streamlined.
32 CBSE Term II Physics XI

Solved Examples
Example 1. An open container has dimensions of relative to the smaller piston of diameter 10 cm. The
4.0 m × 5.0 m and height of 3.0 m. mass on the smaller piston is 20 kg. What is the force
exerted on the load by the larger piston? The density
(i) Find the weight of the air in the container at
of oil in the press is 750 kgm −3 .
20 °C.
(Take, g = 9 .8 ms −2 )
(ii) What is the weight of an equal volume of water?
Also, find pressure at the base of container due
to this weight of water. 1.5 m
(iii) What is the total downward force on the base of 20 kg
the container due to air pressure of 1.0 atm?
(Take, the densities of air and water as 1.2 kg/m 3 and
10 3 kg/m 3 , respectively)
Sol. Given, density of air, ρair = 1.2 kg/ m 3 force
Sol. We know that, pressure =
area
and density of water, ρwater = 10 3 kg /m3
20 × 9. 8
Pressure on the smaller piston = Nm−2
(i) The volume of air in the container, Vwater π × (5 × 10 −2 )2
= (4.0)(5.0)(3.0) = 60 m3 F
Pressure on the larger piston = N m −2
∴ The mass of the air, m air = ρair ⋅ Vair −2 2
π × (17 . 5 × 10 )
= (1.2) (60) = 72 kg The difference between the two pressures = hρg
The weight of the air,
where, h = 1.5 m and ρ = 750 kgm −3
w air = m air g = (72) (9.8) = 705.6 N
Thus,1. 5 × 750 × 9. 8
(ii) The weight of water, 20 × 9. 8 F
w water = m water g = ρwater ⋅ Vwater ⋅ g = − which gives,
π × ( 5 × 10 −2 )2 π × (17 .5 × 10 −2 )2
= (10 3 )( 60 )( 9. 8 )
F = 1. 3 × 10 3 N
= 5.9 × 10 5 N
Pressure at the base due to this weight, Example 4. Two pistons of a hydraulic machine have
F ( = w ) 5.9 × 10 5 diameters 20 cm and 2 cm. Find the force exerted
p= = = 2.95 × 10 4 Nm −2 on the larger piston when 50 kg-wt is placed on the
A 4× 5
smaller piston. When the smaller piston moves in
(iii) The downward force on the base through 5 cm, by what distance, the other piston
= air pressure × surface area moves out?
= ( 1.013 × 10 5 ) (4.0 × 5.0) ~ − 2.0 × 10 6 N Sol. For smaller piston, area, A 1 = π × (1 )2
Example 2. Two liquids of densities ρ and 3ρ having For larger piston, area, A 2 = π × (10 )2
volumes 3 V and V are mixed together. Find density ∴ Force exerted on the larger piston,
of the mixture. A2
F2 = × F1
Sol. Given, density of first liquid, ρ1 = ρ A1
Density of second liquid, ρ2 = 3ρ π(10) 2
= × 50 × 9.8
Volume of first liquid, V1 = 3 V π(1) 2
Volume of second liquid, V2 = V = 100 × 50 × 9.8
∴ Density of the mixture,
− 5 × 10 4 N
~
ρ V + ρ2 V2
ρm = 1 1 This is the force exerted on the larger piston. The liquids
V1 + V2
are considered incompressible. Therefore, volume
ρ × 3 V + 3 ρ × V 6ρ  3  covered by movement of smaller piston inwards equal to
= = = ρ
3V + V 4  2 the outward movement of larger piston.
Example 3. Figure shows a hydraulic press with the ∴ L 1A 1 = L 2A 2
larger piston of diameter 35 cm at a height of 1.5 m
CBSE Term II Physics XI 33

 A1  2 × 24
⇒ L2 =   L1 =
A 2 1.06 × 10 3 × (2 2 − 1)
π(1 cm)2 = 0.123 ms −1
= × 5 cm
π(10 cm)2 Example 7. If the water emerge from an orifice in a
1 tank in which the gauge pressure is 4 × 10 5 Nm −2
= × 5 cm
100 before the flow starts, then what will be the
= 0.05 cm velocity of the water emerging out? (Take, density
So, the distance moved out by the larger piston is of water is 1000 kg m −3 )
0.05 cm. Sol. Given, p = 4 × 10 5 N m −2 and ρ = 1000 kg m −3 ,
Example 5. Water is flowing through a horizontal tube g = 10 m s −2
of non-uniform cross-section. At a place, the radius p 4 × 10 5
of the tube is 1.0 cm and the velocity of water is Apply, p = hρg ⇒ h = =
ρg 1000 × 10
2 ms −1 . What will be the velocity of water, where
the radius of the pipe is 2.0 cm? 2 × 10 × 4 × 10 5
Velocity of efflux, v = 2 gh =
Sol. Using equation of continuity, 1000 × 10
A  = 800 = 28.28 ms −1
A1 v1 = A 2 v 2 or v 2 =  1  v1
 A2  Example 8. 0.04 cm liquid column balances the excess
 πr 2 
2 pressure inside a soap bubble of radius 6 mm.
r 
or v 2 =  12  v1 =  1  v1 Evaluate density of the liquid. (Take, surface
 π r2   r2 
tension of soap solution = 0.03 Nm −1 )
Given, r1 = 1 cm = 1 × 10 −2 m, r2 = 2 cm = 2 × 10 −2 m, Sol. The excess pressure inside a soap bubble,
v1 = 2ms−1 ∆p = 4 T / R
4 × 0.03 Nm −1
Substituting the above values, we get = = 20 Nm −2
2 6 × 10 −3 m
 1.0 × 10 −2  −1
v2 =   (2 ) or v 2 = 0.5 ms The pressure due to 0.04 cm of the liquid column,
 2.0 × 10 −2 
∆p = hρg = (0.04 × 10 −2 m)ρ (10 ms −2 )
Example 6. The flow of blood in a large artery of an Thus, 20 Nm −2 = (0.04 × 10 −2 m) ρ(10 ms −2 )
anesthetised dog is diverted through a
∴Density of the liquid, ρ = 5 × 10 3 kgm −3
venturimeter. The wider part of the meter has a
cross-sectional area equal to that of the artery, Example 9. A glass tube of radius 0.4 mm is dipped
A = 16 mm 2 . The narrower part has an area vertically in water. Find upto what height, the
a = 8mm 2 . The pressure drop in the artery is 24 Pa. water will rise in the capillary. If the tube is
What is the speed of the blood in the artery? inclined at an angle of 60° with the vertical, how
Sol. Bernoulli’s equation for the horizontal flow of blood is much length of the capillary is occupied by water?
1 1
p1 + ρv12 = p 2 + ρv 22
(Take, surface tension of water = 7.0 × 10 −2 Nm −1
2 2 and density of water = 10 3 kgm −3 )
By equation of continuity, Sol. For glass-water, angle of contact, θ = 0 °.
Av1 = av 2 or v 2 = Av1 / a Now, height of water in capillary,
1 ρ A 2 v12 1 2 2 T cos θ (2)(7.0 × 10 −2 ) cos 0 °
∴ p1 − p 2 = − ρv1 h= =
2 a2 2 rρg (0.4 × 10 −3 )(10 3 )(9.8)
1  A2  = 3.57 × 10 −2 m = 3.57 cm
= ρv12  2 − 1
2 a  Length of capillary occupied after tilting,
Here, p1 − p 2 = 24 Pa h 3.57
l= =
cos 60 ° 1 / 2
ρ (blood) = 1.06 × 10 3 kg m −3 , A / a = 16 / 8 = 2
= 7.14 cm
2 ( p1 − p 2 )
∴ v1 =
 A2 
ρ 2 − 1
a 
34 CBSE Term II Physics XI

Chapter
Practice
PART 1 It is observed that, the meniscus is convex. The
liquid in the trough is [NCERT Exemplar]

Objective Questions (a) water


(c) mercury
(b) ethylalcohol
(d) methyliodide
l
Multiple Choice Questions 6. A capillary tube A is dipped in water. Another
identical tube B is dipped in soap-water solution.
1. Pressure at a point inside a liquid does not depend Which of the following shows the relative nature of
on the the liquid columns in the two tubes?
(a) depth of the point below the surface of the liquid
A B A B
(b) nature of the liquid
(c) acceleration due to gravity at that point (a) (b)
(d) total weight of fluid in the beaker
2. Three liquids of densities d, 2 d and 3d are mixed in
A B A B
equal proportion of weights. If density of water is d,
then the specific gravity of the mixture is (c) (d)
11 18 13 23
(a) (b) (c) (d)
7 11 9 18
3. The force required to separate two glass plates of
10 −2 m 2 with a film of water 0.05 mm thick between
7. Which of the following diagrams does not represent
a streamline flow? [NCERT Exemplar]
them, is (surface tension of water is 70 × 10 −3 Nm −1 )
(a) 28 N (b)14 N (c) 50 N (d) 38 N
4. In figure given below, pressure inside a spherical
drop is more than pressure outside (S = surface
(a) (b)
tension and r = radius of bubble).
p0

pi
r
(c) (d)
The extra surface energy, if radius of bubble is
increased by ∆r is 8. Two water pipes of diameters 2 cm and 4 cm are
(a) 4πr ∆r S (b) 8πr ∆r S connected with the main supply line. The velocity
(c) 2πr ∆r S (d) 10πr ∆r S of flow of water in the pipe of 2 cm diameter is
5. The angle of contact at the interface of water-glass (a) 4 times that in the other pipe
is 0°, ethyl alcohol-glass is 0°, mercury-glass is 140° (b) (1/4) times that in the other pipe
and methyliodide-glass is 30°. A glass capillary is (c) 2 times that in the other pipe
put in a trough containing one of these four liquids. (d) (1/2) times that in the other pipe
CBSE Term II Physics XI 35

9. I. Bernoulli’s equation ideally applies to


non-viscous fluids.
II. Fluid must be compressible in Bernoulli’s (a) v (b) v
theorem.
III. Bernoulli’s equation does not hold for non-steady
flow. t t

Which of the following statement(s) is/are correct?


(a) Both I and II
(b) Only I (c) v (d) v
(c) Both I and III
(d) Only II
t t
10. Which amongst the following statement(s) is/are
incorrect? l
Assertion-Reasoning MCQs
(a) A jet of air striking a plate placed perpendicular to it Direction (Q. Nos. 16-20) Each of these questions
is an example of turbulent flow.
contains two statements Assertion (A) and Reason (R).
(b) The carburetor of automobile has a venturi channel
(nozzle) through which air flows with a high speed. Each of these questions also has four alternative
(c) Ball moving without spinning inside a fluid choices, any one of which is the correct answer. You
experiences a net upward force. have to select one of the codes (a), (b), (c) and (d) given
(d) None of the above below.
11. A cylinder of height 20 m is completely filled with (a) Both A and R are true and R is the correct
water. The velocity of efflux of water (in ms −1 ) explanation of A
through a small hole on the side wall of the (b) Both A and R are true, but R is not the correct
cylinder near its bottom is explanation of A
(a) 10 (b) 20 (c) A is true, but R is false
(c) 25.5 (d) 5 (d) A is false and R is also false
12. We have three beakers A , B and C containing three 16. Assertion In steady flow, the velocity of each a
different liquids. They are stirred vigorously and passing fluid particle remains constant in time.
placed on a table, then liquid which is Reason Each particle follows a smooth path and the
(a) most viscous comes to rest at the earliest paths of the particle do not cross each other.
(b) most viscous comes to rest at the last
(c) most viscous slows down earliest but comes to rest at 17. Assertion The shape of an automobile is so
the last designed that its front resembles the streamline
(d) All of them come to rest at the same time pattern of the fluid through which it moves.
13. The coefficient of viscosity for hot air is Reason The resistance offered by the fluid is not
(a) greater than the coefficient of viscosity for cold air maximum.
(b) smaller than the coefficient of viscosity for cold air 18. Assertion The machine parts are jammed in
(c) same as the coefficient of viscosity for cold air winter.
(d) increases or decreases depending on the external
pressure Reason The viscosity of the lubricants used in the
machine increases at low temperature.
14. As the temperature of water increases, its viscosity
(a) remains unchanged 19. Assertion No net force acts on a body falling in a
(b) decreases liquid with a velocity equal to the terminal velocity.
(c) increases Reason The weight of the body is balanced by the
(d) increases or decreases depending on the external upward buoyant force.
pressure
20. Assertion The surface of water in the capillary
15. A tall cylinder is filled with viscous oil. A round tube is concave.
pebble is dropped from the top with zero initial
velocity. From the plot shown in figure, indicate the Reason The pressure difference between two sides
2S
one that represents the velocity ( v) of the pebble as of the top surface of water is cos θ.
a function of time ( t ). a
36 CBSE Term II Physics XI

l
Case Based MCQs
21. Direction Read the following passage and answer
PART 2
the questions that follows
Fluid Dynamics
Subjective Questions
Consider the flow at two regions 1 (i.e. BC) and 2 l
Short Answer (SA) Type Questions
(i.e.DE). Consider the fluid initially lying between B
and D. In an infinitesimally time interval ∆t, this fluid 1. If the required pressure in the tyre of a car is
would have moved. Suppose v1 is the speed at B and 199 kPa, then what is the
v 2 at D, then fluid initially at B has moved a distance (i) gauge pressure and
v1 ∆t to C ( v1 ∆t is small enough to assume constant (ii) absolute pressure?
cross-section along BC).
In the same interval ∆t, the fluid initially at D
2. Two vessels have the same base area but different
shapes. The first vessel takes twice the volume of
moves to E, a distance equal to v 2 ∆t. Pressures p1
water than the second vessel required to fill upto a
and p 2 act as shown on the plane faces of areas A1
particular common height. Is the force exerted by
and A 2 binding the two regions as shown in figure
the water on the base of the vessel the same in the
D E two cases? If so, why do the vessels filled with
A2
p2 water to that same height give different readings on
C v2,∆t a weighing scale? [NCERT]
B
A1
p1 3. A vertical off-shore structure is built to withstand a
v 1,∆
t h2 maximum stress of 10 9 Pa. Is the structure suitable
h1 for putting up on top of an oil well in the ocean?
Take, the depth of the ocean to be roughly 3 km
and ignore ocean currents. [NCERT]
(i) The work done on the fluid at BC is
(a) p1A1∆t (b) p1v1∆t (c) p1∆V (d) A1∆V 4. A balloon with hydrogen in it rises up but a balloon
with air comes down, why?
(ii) The work done on the fluid at DE is
(a) p 2∆V (b) − p 2∆V (c) p1A 2∆t (d) p 2v 2∆t 5. Fig.(a) shows a thin liquid film supporting a small
weight = 4. 5 × 10 − 2 N. What is the weight
(iii) Total work done on the fluid is
p1 − p 2 supported by a film of the same liquid at the same
(a) ∆V (b) p 2∆V temperature in Figs. (b) and (c)? Explain your
2
answer. [NCERT]
(c) p1∆V (d) ( p1 − p 2 ) ∆V
(iv) The change in its kinetic energy is
1 1
(a) ∆V ( v 22 − v12 ) (b) ρ∆V ( v 22 − v12 )
2 2
1
(c) ρ∆V ( v 22 − v12 ) (d) ρ∆V ( v 22 + v12 )
2
(v) Expression of Bernoulli’s equation is
1 40 cm 40 cm 40 cm
(a) p + ρv 2 = constant
2 (a) (b) (c)
1
(b) p + ρv 2 + ρgh = constant 6. The surface tension and vapour pressure of water
2 at 20° C is 7.28 × 10 − 2 N/ m and 2.33 × 10 3 Pa,
1
(c) p + ρv 2 + ρgh = constant respectively. What is the radius of the smallest
2 spherical water droplet which can form without
1 2
(d) ρv + ρgh = constant evaporating at 20° C ? [NCERT]
2
CBSE Term II Physics XI 37

7. A liquid drop breaks into 27 small drops. If surface 16. In a test experiment on a model aeroplane in a
tension of the liquid is S, then find the energy wind tunnel, the flow of speeds on the upper
released. and lower surfaces of the wings are 70 m/s and
8. The sap in trees, which consists mainly of water in 63 m/s, respectively. What is the lift on the
summer, rises in a system of capillaries of radius wings, if its area is 2.5 m 2 ? (Take, density of air =
r = 2 . 5 × 10 − 5 m. The surface tension of sap is 1.3 kg / m 3 ) [NCERT]
S = 7 . 28 × 10 − 2 N/m and angle of contact is 0°. Does 17. A venturimeter is connected to two points in the
surface tension alone account for the supply of water mains, where its radii are 20cm and 10cm,
to the top of all trees? [NCERT] respectively and the levels of water column in the
9. In deriving Bernoulli’s equation, we equated the tubes differ by 10 cm. How much water flows
work done on the fluid in the tube to its change in through the pipe per minute?
the potential and kinetic energies. 18. What is the largest average velocity
(i) of blood
(i) How does the pressure change as the fluid moves flow in an artery of radius 2 × 10 − 3 m, if the flow
along the tube, if dissipative forces are present? must remain laminar?
(ii) Do the dissipative forces become more important (ii) What is the corresponding flow rate? (Take,
as the fluid velocity increases? Discuss viscosity of blood = 2.084 × 10 − 3 Pa -s) [NCERT]
qualitatively. [NCERT] 19. The flow rate of water from a tap of diameter
10. The relative velocity between two parallel layers of 1.25 cm is 0.48 L/min. The coefficient of viscosity
water is 8 cms −1 and the perpendicular distance of water is 10 −3 Pa - s. After sometime, the flow
between them is 0.1 cm. Calculate the velocity rate is increased to 3 L/min. Characterise the
gradient. flow for both the flow rates. [NCERT]

11. The terminal velocity of a copper ball of radius 20. A tank with a square base of area 1.0 m 2 is
2.0 mm falling through a tank of oil at 20° C is divided by a vertical partition in the middle. The
6 . 5 cms − 1 . Compute the viscosity of the oil at 20° C. bottom of the partition has a small hinged door of
(Take, density of oil is 1.5 × 10 3 kg m − 3 and density of area 20 cm 2 . The tank is filled with water in one
copper is 8.9 × 10 3 kg m− 3 ) compartment and an acid (of relative density 1.7)
in the other, both to a height of 4.0 m. Compute
12. What should be the average velocity of water in a the force necessary to keep the door close.
tube of radius 0.005 m, so that the flow is just
turbulent? The viscosity of water is 0.001 Pa-s.
13. Explain, why 4m Water Acid
(i) The blood pressure in humans is greater at the feet
than the brain.
(ii) Atmospheric pressure at a height of about 6 km Door
decreases to nearly half of its value at the sea level
though the height of the atmosphere is more than l
Long Answer (LA) Type Questions
100 km.
21. If a number of little droplets of water, each of
(iii) Hydrostatic pressure is a scalar quantity even radius r, coalesce to form a single drop of radius
though pressure is force divided by area. [NCERT] R and the energy released is converted into
14. A U-shaped wire is dipped in a soap solution and kinetic energy, then find out the velocity acquired
removed. The thin soap film formed between the by the bigger drop.
wire and a light slider supports a weight of 22. What is the excess pressure inside a bubble of
1. 5 × 10 − 2 N (which includes the small weight of the soap solution of radius 5.00 mm? Given that, the
slider). The length of the slider is 30 cm. What is the surface tension of soap solution at the
surface tension of the film? [NCERT] temperature 20° C is 2.50 × 10 − 2 N / m. If an air
15. If a drop of liquid breaks into smaller droplets, it bubble of the same dimension were formed at a
results in lowering of temperature of the droplets. depth of 40.0 cm inside a container containing
Let a drop of radius R, break into N small droplets the soap solution (of relative density 1.20), what
each of radius r. Estimate the lowering in would be the pressure inside the bubble?
temperature. [NCERT] (Take, 1 atm pressure is 1.01 × 10 5 Pa) [NCERT]
38 CBSE Term II Physics XI

23. Explain, why 4 × 10 − 3 m 2 and 8 × 10 − 3 m 2 . The velocity of the


(i) The angle of contact of mercury with glass is liquid at point P is 1 ms − 1 . Find the work done per
obtuse, while that of water with glass is acute. unit volume by the pressure and the gravity forces
as the fluid flows from point P to Q.
(ii) Water on a clean glass surface tends to spread
out while mercury on the same surface tends to 27. If a sphere of radius r falls under gravity through a
form drops. (Put differently, water wets the glass liquid of viscosity η, its average acceleration is half
while mercury does not.) that of in starting of the motion. Then, show that
the time taken by it to attain the terminal velocity is
(iii) Surface tension of a liquid is independent of the
independent of the liquid density.
area of the surface.
(iv) Water with detergents dissolved in it should l
Case Based Questions
have small angles of contact.
28. Direction Read the following passage and answer
(v) A drop of liquid under no external forces is
the questions that follows
always spherical in shape. [NCERT]
Measuring the Surface Tension of a Liquid
24. Two narrow bores of diameter 3.0 mm and 6.0 mm
The surface tension of liquid can be measured
are joined together to form a U-tube open at both
experimentally as shown in figure. A flat vertical
ends. If the U-tube contains water, what is the
glass plate, below which a vessel filled with some
difference in its levels in the two limbs of the tube?
liquid is kept. The plate is balanced by weights on
Surface tension of water at the temperature of the
the other side. The vessel is raised slightly until the
experiment is 7. 3 × 10 − 2 N/ m . (Take, the angle of
liquid touches the glass plate and pulls it down
contact to be zero, density of water to be
because of the force of surface tension. Weights are
1. 0 × 10 3 kg /m 3 and g = 9.8 m/s 2 ) [NCERT] added till the plate just detaches from water.
25. Explain, why
Glass plate
(i) To keep a piece of paper horizontal, you should
blow over, not under it.
l
(ii) When we try to close a water tap with our w
fingers, fast jets of water gush through the
opening between our fingers.
Measuring the surface tension
(iii) The size of the needle of a syringe controls flow
rate better than the thumb pressure exerted by a (i) If a wet piece of wood burns, then water droplets
doctor while administering an injection. appear on the other end, why?
(iv) A fluid flowing out of a small hole in a vessel
(ii) Why surface tension of all lubricating oils and
results in a backward thrust on the vessel.
paints is kept low?
(v) A spinning cricket ball in air does not follow a
parabolic trajectory. [NCERT] (iii) It becomes easier to spray the water in which some
soap is dissolved, explain it.
26. A non-viscous liquid of constant density
1000 kgm − 3 flows in a streamline motion along a (iv) Find the work done required to make a soap bubble
tube of variable cross-section. The tube is a kept of radius 0.02 m. (Take, surface tension of soap
inclined in the vertical plane. The area of = 0.03 N/m)
cross-section of the tube at two points P and Q at (v) Why does a stream of water from a faucet become
heights of 2 m and 5 m are respectively, narrow, as it falls?
Chapter Test
Multiple Choice Questions Short Answer Type Questions
1. A hydraulic lift has 2 limbs of areas A and 2A. Force F is 6. A 50 kg girl wearing high heels shoes balances on a
applied over limb of area A to lift a heavy car. If distance single heel. The heel is circular with a diameter
moved by piston P1 is x, then distance moved by piston P2 1.0 cm. What is the pressure exerted on the
is horizontal floor? [NCERT]
P1 P2 (Ans. 6.24 × 10 6 Pa)
7. Torricelli’s barometer used mercury. Pascal duplicated
it using French wine of density 948 kg / m 3.
Determine the height of the wine column for normal
atmospheric pressure. [NCERT]
The pressure exerted by a liquid column, p = hρg
Mercury where, h = height of liquid column,
A 2A ρ = density of liquid
(a) x (b) 2 x
and g = acceleration due to gravity.
x (Ans. 10.5 m)
(c) (d) 4x
2 8. Two soap bubbles of radii 6 cm and 8 cm coalesce to
2. A liquid is kept in a bowl opened to atmosphere. form a single bubble. Find the radius of the new
bubble. (Ans. 10 cm)
A
9. Figs. (a) and (b) refer to the steady flow of a
B
non-viscous liquid. Which of two figures are incorrect
and why?
Consider two consecutive layers A and B as shown above,
which of the following statement(s) is/are correct?
(a) Total energy of surface A = Total energy of surface B.
(b) Number of molecules in surface A > Number of molecules in
surface B.
(c) Energy of a molecule of layer B is nearly half of energy of (a) (b)
layer A.
(d) Net force on a molecule of surface B is zero. Long Answer Type Questions
3. The excess pressure inside an air bubble of radius r just 10. Show that, if n equal rain droplets falling through air
below the surface of water is p1 . The excess pressure with equal steady velocity of 10 cms − 1 coalesce, the
inside a drop of the same radius just outside the surface resultant drop attains a new terminal velocity of
is p2. If T is surface tension, then 10 n 2/ 3cms − 1 .
(a) p1 = 2p2 (b) p1 = p2
11. A liquid is kept in cylindrical vessel which is rotated
(c) p2 = 2p1 (d) p2 = 0 , p1 ≠ 0
along its axis. The liquid rises at the sides. If the radius
4. An ideal fluid flows through a pipe of circular of vessel is 0.05 m and the speed of rotation is 2 rev/s,
cross-section made of two sections with diameters 2.5 cm find the difference in height of the liquid at the centre
and 3.75 cm. The ratio of the velocities in the two pipes is of the vessel and its sides. (Ans. 0. 02 m)
(a) 9 : 4 (b) 3 : 2
12. (i) Pressure decreases as one ascends the
(c) 3 : 2 (d) 2 : 3 atmosphere. If the density of air is ρ, what is the
5. A ball is moving without spinning in a straight line change in pressure dp over a differential height dh?
through a fluid is as shown in figure (ii) Considering the pressure p to be proportional to
A the density, find the pressure p at a height h, if
the pressure on the surface of the earth is p0.
(iii) If p0 = 1.03 × 105 Nm −2, ρ0 = 1.29 kg m −3 and
g = 9.8 ms −2, at what height will be pressure drop
B to (1/10) the value at the surface of the earth?
If pA and pB are pressure values at A and B, then (iv) This model of the atmosphere works for relatively
(a) pA < pB (b) pB < pA small distances. Identify the underlying
(c) pA × pB = 1 (d) pA / pB = 1 assumption that limits the model.
[Ans. (iii) 18 × 10 −3 m]

Answers For Detailed Solutions


Multiple Choice Questions Scan the code
1. (c) 2. (d) 3. (a) 4. (a) 5. (d)
40 CBSE Term II Physics XI

EXPLANATIONS
PART 1 7. (d) In a streamline flow at any given point, the velocity of
each passing fluid particles remains constant. If we
1. (d) Pressure at a point inside a liquid at h depth is given consider a cross-sectional area, then a point on the area
by cannot have different velocities at the same time, hence
F two streamlines of flow cannot cross each other.
p= = ρgh
A 8. (a) From equation of continuity, Av = constant
∴ It does not depend on the weight of fluid. d A = 2 cm and dB = 4 cm
2. (b) Given, w1 = w 2 = w 3 ∴ rA = 1 cm and rB = 2 cm
⇒ m1 = m 2 = m 3 = m (say) vA A π( rB )2  2 
2

m m m ∴ = B = =   ⇒ v A = 4 vB
Then, V1 = , V2 = , V3 = vB A A π( rA )2  1 
d 2d 3d
9. (c) Statements I and III are correct but statement II is
Mass 3m 18
∴ d mix = = = d incorrect and it can be corrected as,
Volume V1 + V2 + V3 11
A restriction on application of Bernoulli’s theorem is that
d 18 the fluids must be incompressible, as the elastic energy of
So, specific gravity of mixture = mix =
d water 11 the fluid is also not taken into consideration.
3. (a) Given, area, A = 10 −2 m 2 10. (c) Statement given in option (c) is incorrect and it can be
corrected as,
Thickness, t = 0 . 05 mm = 0. 05 × 10 −3 m
The streamlines around a non-spinning ball moving
Surface tension, S = 70 × 10 −3 Nm −1 relative to a fluid is as shown below
For both surfaces,
Total area, A' = 2 × A = 2 × 10 −2
Also, A' = Length (L ) × Thickness ( t )
A' 2 × 10 2
⇒ Length, L = = = 400 m
t 0. 05 × 10 −3
F
∴ Surface tension, S =
L
⇒ F = S × L = 70 × 10 −3 × 400 = 28 N From the symmetry of streamlines, it is clear that the
4. (b) Suppose a spherical drop of radius r is in equilibrium. velocity of fluid above and below the ball at
corresponding points is the same resulting in zero
If its radius increases by ∆r , the extra surface energy is
pressure difference. The fluid therefore, exerts no
|4 π ( r + ∆r )2 − 4 πr 2| S = 8 πr∆r S upward or downward force on the ball.
5. (c) According to the question, the observed meniscus is of Rest statements are correct.
convex figure shape, which is only possible when angle of 11. (b) Given, height, h = 20 m
contact is obtuse. Hence, the combination will be of
mercury-glass (140°). As we know, velocity of efflux,
v = 2 gh
Convex = 2 × 10 × 20
140° = 20 m/s
12. (a) More viscous the liquid, more will be the opposition
between different layers. So, most viscous liquid will
Mercury
come to rest at the earliest.
13. (a) For gases, viscosity increases with temperature.
6. (d) The height of liquid column in a capillary tube is Therefore, viscosity of hot air is greater than viscosity of
given by cold air.
2S cos θ 14. (b) With rise in temperature, the molecules of water
h= ⇒ h ∝S
ρ Rg moves faster and so its viscosity decreases.
Since on mixing soap in water, its surface tension 15. (c) When the pebble is falling through the viscous oil, the
decreases, so the height of soap-water solution is less viscous force,
than that due to water only as shown in option (d). F = 6πηr v
CBSE Term II Physics XI 41

As the force is variable, hence acceleration is also Therefore, both A and R are true and R is the correct
variable, so v-t graph will not be straight line.First explanation of A.
velocity increases and then becomes constant known as 21. (i) (c) The work done on the fluid at left end BC,
terminal velocity. So, graph (c) is correct option. W1 = p1A1 ( v1∆t ) = p1∆V (Q V = A × x
16. (a) The flow of the fluid is said to be steady, if at any )
given point, the velocity of each passing fluid particle (ii) (b) The work done on the fluid at DE,
remains constant in time.
W2 = − p 2A 2 ( v 2 ∆t ) = − p 2∆V
Every other particle which passes the second point
behaves exactly, as the previous particle that has just (as same volume passes through it)
passed that point. (iii) (d) The total work done on the fluid,
This is because, each particle follows a smooth path and W = W1 − W2 = ( p1 − p 2 ) ∆V
the paths of the particles do not cross each other. (iv) (b) Some part of work goes into changing the kinetic
Therefore, both A and R are true and R is the correct energy of the fluid and other goes into changing the
explanation of A. gravitational potential energy. If the density of the
17. (b) When a body moves through a fluid, its motion is fluid is ρ and ∆m = ρA 1v1∆t = ρ∆V is the mass
opposed by the force of fluid friction called resistance of passing through the pipe in time ∆t, the change in
fluid. It acts normal to the surface and increases with its kinetic energy,
1
increasing speed of body. ∆K = ρ∆V ( v 22 − v12 )
It is due to this reason, the shape of an automobile is, so 2
designed that it resembles the streamline pattern of the (v) (b) We can apply the work-energy theorem to this
fluid through which it moves, so that air friction is volume of the fluid and this yields
minimum. 1 
( p1 − p 2 ) ∆V =   ρ ∆V ( v 22 − v12 ) + ρg∆V ( h 2 − h1)
Also, the resistance offered by the fluid is not maximum.  2
Therefore, both A and R are true but R is not the correct We now divide each term by ∆V to obtain
explanation of A.
1 
18. (a) The machine parts are jammed in winter because the ( p1 − p 2 ) =   ρ ( v 22 − v12 ) + ρg ( h 2 − h1)
 2
viscosity of the lubricants used in the machine increases
at low temperature. We can rearrange the above terms to obtain
Therefore, both A and R are true and R is the correct 1  1 
p1 +   ρv12 + ρgh1 = p 2 +   ρv 22 + ρgh 2
explanation of A.  2  2
19. (c) The weight of the body is balanced by two upward This is Bernoulli’s equation. Since, 1 and 2 refer to
forces; namely the buoyant force and viscous force. any two locations along the pipeline, we may write
No net force acts on a body falling in a liquid with a the expression in general as
velocity equal to the terminal velocity, because this force 1
p + ρv 2 + ρgh = constant
(viscous) is balanced by the weight of body. 2
Therefore, A is true but R is false.
20. (a) The pressure difference between the two sides of the PART 2
top surface of a liquid in a capillary tube is given by
2S 2S 2S 1. (i) Gauge pressure, p g = 199 kPa
pi − p0 = = = cos θ
r a secθ a (ii) Absolute pressure,
where, θ = angle of contact. p = p a + p g = 101 kPa + 199 kPa = 300 kPa
In case of water is taken in the capillary tube, the contact where, p a = atmospheric pressure.
angle between water and glass is acute. Thus, the 2. The pressure exerted by a liquid column depends upon
pressure of water inside the tube, just at the meniscus its height. The height of water in both vessels are same,
(air-water interface) is less than the atmospheric pressure. therefore the pressure on the base of each vessel will be
So, the surface of water in the capillary is concave as same.
shown below
θ r

θ
The area of the base of each vessel is also same and hence
force exerted by the water on the base of the vessels will
also be same.
42 CBSE Term II Physics XI

Liquids also apply force on the walls of the vessel. As 8. Given, radius, r = 2. 5 × 10 − 5 m
walls of the vessels are not perpendicular to the base, Surface tension, S = 7 .28 × 10 − 2N / m
therefore force exerted on the walls by the liquid has a Angle of contact, θ = 0 °,
non-zero vertical component, which is more in first Density of water, ρ = 10 3 kg/m3
vessel. Therefore, the two vessels filled with water to The maximum height which sap can rise in trees through
the same height give different readings on a weighing capillarity action is given by
scale.
2S cos θ
3. Given, depth of ocean, h = 3 km = 3000 m h=
r ρg
Density of water, ρ = 10 3 kg / m3
2 × 7 .28 × 10 − 2 × cos 0 °
Pressure exerted by water column, h= = 0 . 59 m
2. 5 × 10 − 5 × 1 × 10 3 × 9. 8
p = hρg = 3000 × 10 3 × 9. 8
But the height of many trees are more than 0.59 m,
= 29.4 × 10 6Pa therefore the rise of sap in all trees is not possible
= 2. 94 × 10 7 Pa through capillarity action alone.
Maximum stress which can be withstand by the vertical 9. (i) If dissipative forces are present, then a part of
off-shore structure = 10 9 Pa pressure energy is utilised in overcoming these
As, 10 9Pa > 2.9 × 10 7Pa forces. Due to which, the pressure decreases as the
Therefore, the vertical structure is suitable for putting up fluid moves along the tube.
on top of an oil well in the ocean. (ii) Yes, the dissipative forces become more important as
4. The density of hydrogen is less than air, so the buoyant the fluid velocity increases.
force on the balloon will be more than its weight in case dv
The viscous drag is given by F = − ηA
of the hydrogen. Hence, in this case, the balloon rises up. dx
In case of air, the weight of balloon is more than the As the velocity of fluid increases, the velocity
buoyant force acting on it, so balloon will come down. gradient increases and hence, viscous drag
5. As liquid is same, temperature is same and the length of increases, i.e. dissipative force also increases.
the film supporting the weight is also same. Therefore, in 10. Given, relative velocity between two layers, dv = 8 cms −1
Figs. (b) and (c), the film will support same weight, i.e.
Perpendicular distance between layer, dx = 0.1 cm
4. 5 × 10 − 2N.
dv 8
6. Given, surface tension of water, S = 7 .28 × 10 − 2 N/ m The velocity gradient is given by = = 80 s −1
dx 0.1
Vapour pressure, p = 2. 33 × 10 3 Pa
11. Given, v t = 6.5 × 10 − 2 m / s,
The drop will evaporate, if the water pressure is greater
than the vapour pressure. r = 2 × 10 − 3 m, g = 9.8 m / s 2,
Let a water droplet of radius R can be formed without ρ = 8.9 × 10 3 kg / m 3 and σ = 1.5 × 10 3 kg / m 3
evaporating. ρ − σ = 8.9 × 10 3 − 1.5 × 10 3
∴ Vapour pressure = Excess pressure in drop = 7 . 4 × 10 3 kg / m 3
2S As we know, terminal velocity,
∴ p=
R 2 r 2 (ρ − σ )
vt = g
2S 2 × 7 .28 × 10 − 2 9 η
or R= = ≈ 6.25 × 10 − 5 m
p 2. 33 × 10 3 2 (2 × 10 − 3 )2 × 9.8
⇒ η= × × 7.4 × 10 3
7. Let the radius of larger drop = R 9 (6.5 × 10 − 2 )
and radius of each small drop = r ⇒ η = 9.9 × 10 − 1kg / ms
Volume of 27 small drops = Volume of the large drop 12. Given, r = 0.005 m, diameter, D = 2 r = 0.010 m,
4 4
27 × πr 3 = πR 3 η = 0.001 Pa - s and ρ = 1000 kgm− 3
3 3 For flow to be just turbulent, Re = 3000
So, r = R/3 R η 3000 × 0.001
∴ v= e = = 0.3 ms− 1
Surface area of large drop = 4 πR 2 ρD 1000 × 0.010
Surface area of 27 small drops = 27 × 4 πr 2 13. (i) The pressure of liquid column is given by p = hρg,
2
R where h is depth, ρ is density and g is acceleration
= 27 × 4 π   = 12 πR 2
 3 due to gravity.
Therefore, pressure of liquid column increases with
∴ Increase in surface area = 12 πR 2 − 4 πR 2 = 8 πR 2
depth. The height of blood column in human body is
Increase in energy = Increase in surface area more at feet than at the brain. Therefore, the blood
× Surface tension pressure in humans is greater at the feet than the
= 8 πR × S
2
brain.
CBSE Term II Physics XI 43

(ii) The density of air is maximum near the surface of 16. Let the lower and upper surfaces of the wings of the
the earth and decreases rapidly with height. At a aeroplane be has difference in height h and speeds of air
height of 6 km, the density of air decreases to nearly on the upper and lower surfaces of the wings be v1 and v 2.
half of its value at the sea level. Beyond 6 km height, Speed of air on the upper surface of the wings,
the density of air decreases very slowly with height. v1 = 70 m/s
Hence, the atmospheric pressure at a height of
about 6 km decreases to nearly half of its value at the Speed of air on the lower surface of the wings,
sea level. v 2 = 63 m/s
(iii) When force is applied on a liquid, the pressure is Density of the air, ρ = 1.3 kg / m 3
transmitted equally in all directions inside the Area, A = 2.5 m 2
liquid. Therefore, hydrostatic pressure has no fixed According to Bernoulli’s theorem,
direction and hence it is a scalar quantity. 1 1
p1 + ρv12 + ρgh = p 2 + ρv 22 + ρgh
14. Given, length of the slider, l = 30 cm 2 2
As a soap film has two free surfaces, therefore total length 1
of the film to be supported, or p 2 − p1 = ρ ( v12 − v 22 )
2
l ′ = 2 l = 2 × 30 = 60 cm = 0. 60 m ∴ Lifting force acting on the wings,
Let S be the surface tension of the soap solution. F = ( p 2 − p1) × A
Total force on the slider due to surface tension, 1
F = S × 2l = ρ ( v12 − v 22 ) × A
2
F = S × 0. 60 N …(i) (Q Force = Pressure × Area )
Weight supported by the slider, w = 1. 5 × 10 −2N 1
= × 1.3 × [(70)2 − (63)2 ] × 2.5
In equilibrium, 2
Force on the slider due to surface tension = Weight 1
= × 1.3 [ 4900 − 3969] × 2.5
supported by the slider 2
∴ F=w 1
= × 1.3 × 931 × 2.5
S × 0. 60 = 1 . 5 × 10 − 2 2
1 . 5 × 10 − 2 = 1.51 × 10 3N
or S= = 2 . 5 × 10 −2 N/ m
0. 60 17. As we know that,
15. When a big drop of radius R, break into N droplets each The volume of water flowing per second,
of radius r, the volume remains constant. 2 hg
∴ Volume of big drop = N × Volume of small drop Q V = A 1A 2
A 12− A 22
4 4
πR 3 = N × πr 3 Given, r1 = 20 cm, A1 = πr12 = π(20)2 cm 2,
3 3
R3 r2 = 10 cm, A 2 = πr22 = π(10 )2 cm 2,
or R = Nr or N = 3
3 3

r g = 9. 8 m/s 2 = 980 cm/s 2


Now, change in surface area = 4 πR 2 − N 4 πr 2 2 × 10 × 980
= 4 π (R 2 − Nr 2 ) ∴ V = π 2(20 )2 ⋅ (10 )2 cc/s
π ((20)4 − (10)4
2
Energy released = S × ∆ A = S × 4 π(R 2 − Nr 2 )
Due to releasing of this energy, the temperature is 175.93 × 10 3
= cc / s
lowered. 15
If ρ is the density, s is specific heat of liquid and its 175.93 × 10 3
temperature is lowered by ∆θ, then = × 60 cc / min
15
Energy released = ms ∆θ = 2728.7 L/min
4  18. Given, radius of artery, r = 2 × 10 − 3 m
S × 4 π (R 2 − Nr 2 ) =  π × R 3 × ρ s ∆θ
3 
∴ Diameter of artery, D = 2 r = 4 × 10 − 3 m
S × 4 π (R 2 − Nr 2 ) 3S R 2 Nr 2  Density of whole blood, ρ = 1.06 × 10 3 kg / m 3
∆θ = = R 3 − R 3 
4
πR 3ρ × s ρs   Coefficient of viscosity of blood, η = 2.084 × 10 3Pa -s
3 For laminar flow, maximum value of Reynold’s number,
3S  1 (R 3 / r 3 ) × r 2   R3 R e = 2000
= − QN = 3 
ρs R R3 
  r  R η 2000 × 2.084 × 10 −3
(i) Critical velocity, v c = e =
3S  1 1  ρD 1.06 × 10 3 × 4 × 10 − 3
∆θ = −
ρs R r  = 0. 98 m /s
44 CBSE Term II Physics XI

(ii) Flow rate of blood = Volume of blood flowing R3  1 1 


per second = 4π  − R 2  = 4 πR 3  − 
 r   r R
= Av c
22 The energy released, (Q nr 3 = R 3)
= × (2 × 10 −3 )2 × 0. 98
7 E = Surface tension × Decrease in surface area
= 1.23 × 10 −5 m 3/s 1 1 
= 4 πSR 3  − 
19. Let the speed of the flow be v.  r R
Given, diameter of tap, d = 1.25 cm The mass of bigger drop, M =
4 4
πR 3 × 1 = πR 3
Volume of water flowing out per second, 3 3
πd 2 4Q 4  1 1 
Q=v× ⇒ v= 2 ∴ E = πSR 3 ⋅ 3  − 
4 dπ 3  r R
4ρQ
Reynold’s number, Re = 1 1   4 3
π dη = 3 SM  −  QM = πR 
 r R  3 
Q = 0.48 L / min = 8 ×10 −3L /s = 8 × 10 − 6 m 3 / s
QKE of bigger drop = Energy released
4 × 10 3 × 8 × 10 − 6
Re = 1 1 1 
3.14 × 1.25 × 10 − 2 × 10 − 3 Mv 2 = 3SM  − 
2  r R
Re = 815 (i.e. below 1000, the flow is steady)
After sometime, when R − r 
∴ v = 6S  
Q = 3 L / min = 5 × 10 − 5 m 3 / s  Rr 
4 × 10 3 × 5 × 10 −5 22. Given, surface tension of soap solution,
Re = = 5095
3.14 × 1.25 × 10 −2 × 10 −3 S = 2. 50 × 10 − 2N / m
∴ The flow will be turbulent. Density of soap solution, ρ = 1.2 × 10 3 kg / m 3
20. Given, height of water and acid, h = 4.0 m Radius of soap bubble, r = 5. 00 mm = 5. 0 × 10 − 3 m
Density of water, ρw = 1 × 10 3 kg / m 3 Radius of air bubble, R = 5 × 10 −3 m
Density of acid, ρa = 1.7 × 10 3 kg / m 3 Atmospheric pressure, p 0 = 1. 01 × 10 5 Pa
Pressure exerted by water at the door, Excess pressure inside the soap bubble
p1 = hρw g = 4 × 1 × 10 3 × 9.8 4S 4 × 2. 50 × 10 − 2
= = = 20 Pa
= 39.2 × 10 3Pa r 5. 0 × 10 − 3
Pressure exerted by acid at the door, Excess pressure inside the air bubble
p 2 = hρa g = 4 × 1.7 × 10 3 × 9.8 2S 2 × 2. 50 × 10 − 2
= = = 10 Pa
= 66.64 × 10 3Pa R 5. 0 × 10 − 3
∴ Difference in pressure, ∆ p = p 2 − p1 ∴ Pressure inside the air bubble = Atmospheric pressure
= 66.64 × 10 3 − 39.2 × 10 3 + Pressure due to 40 cm of soap solution column +
= 27.44 × 10 3Pa Excess pressure inside the bubble
Area of the door, A = 20 cm 2 = 20 × 10 − 4 m 2 = (1. 01 × 10 5 ) + ( 0. 40 × 1.2 × 10 3 × 9. 8 ) + 10
 Force  = (1. 01 × 10 5 ) + 4.704 × 10 3 + 10
Force acting on the door = ∆ p × A Q Pressure =  = 1. 01 × 10 5 + 0. 04704 × 10 5 + 0. 00010 × 10 5
 Area 
= 1. 05714 × 10 5 Pa = 1. 06 × 10 5 Pa
= 27.44 × 10 3 × 20 × 10 − 4
23. (i) When a small quantity of a liquid is poured on a
= 54.88 N ≈ 55 N
solid, three types of interfaces namely liquid-air,
Therefore, 55 N force is necessary to keep the door close. solid-air and solid-liquid are occured. The surface
21. Let n be the number of small droplets which coalesce to tension corresponding to these three interfaces are
form single drop, then SLA , SSA and SSL , respectively. If θ is the angle of
Volume of n small droplets = Volume of single drop contact between solid and liquid, then
4 4
or n × πr 3 = πR 3 or nr 3 = R 3 SLA
3 3 SLA
Decrease in surface area
θ SSA
= n × 4 πr 2 − 4 πR 2 SSL SSA
θ
 nr 3  Solid (glass)
SSL SSL cos θ
= 4 π ( nr 2 − R 2 ) = 4 π  − R 2
 r 
SSL + SLA cos θ = SSA
CBSE Term II Physics XI 45

⇒ SLA cos θ = SSA − SSL 2S cos θ


and h2 =
S − SSL r2ρg
cos θ = SA …(i)
SLA ∴ Difference in levels of water rises in both tubes,
For mercury and glass SSA < SSL , therefore from 2S cos θ  1 1
Eq. (i); ∆ h = h1 − h 2 =  − 
ρg  r1 r2 
cos θ is negative and therefore θ > 90 °, i.e. obtuse.
2 × 7 . 3 × 10 − 2 × cos 0 °
For water and glass SSA > SSL , therefore from Eq. (i), =
cos θ is positive and therefore θ < 90 °, i.e. acute. 1. 0 × 10 3 × 9. 8
(ii) For equilibrium of a liquid drop on a solid surface,  1 1 
1. 5 × 10 − 3 − 3. 0 × 10 − 3 
SSL + SLA cos θ = SSA
In case of mercury and glass SSL > SSA , therefore for 14. 6 2 − 1 
= × 10 − 2 
equilibrium cos θ should be negative, i.e. θ should 9. 8  3 
be obtuse. 14. 6
In order to achieve this obtuse value of angle of = × 10 − 2
9. 8 × 3
contact, the mercury tends to form a drop. In case of
water and glass SSA > SSL , therefore for equilibrium, = 0. 497 × 10 − 2 m
cos θ should be positive, i.e. θ should be acute. = 4. 97 × 10 − 3 m
In order to achieve this value of angle of contact, the = 4. 9 mm
water tends to spread. 25. (i) When we blow over a piece of paper, the velocity of
(iii) Surface tension of a liquid is defined as the force air above the paper increases. So, in accordance with
 1 
acting per unit length on an imaginary line drawn Bernoulli's theorem  p + ρv 2 = constant  , the
tangentially to the liquid surface at rest. Therefore,  2 
surface tension is independent of the area of the pressure of air decreases above the paper. Due to
liquid surface. the pressure difference of air between below and
(iv) The rise of liquid in a capillary tube is given by above the paper, a lifting force acts on paper and
2S cos θ hence it remains horizontal.
h= (ii) According to equation of continuity, for steady flow
r ρg
of liquid, the product of area of cross-section of the
i.e. h ∝ cos θ tube and velocity of liquid remains constant at each
The cloth has narrow spaces in the form of fine point, i.e. A 1 v1 = A 2v 2 .
capillaries. If angle of contact θ is small, then the When we try to close a water tap with our fingers,
value of cos θ will be large and hence detergent will the area of cross-section of the outlet of water jet is
rise more in fine capillaries in the cloth. Now, the reduced due to the restriction provided by the
detergent solution will penetrate more in clothes fingers and therefore the velocity of water increases
and remove stains and dust from the clothes. and fast jets of water gush through the openings
(v) In the absence of external forces, the size of a liquid between our fingers.
drop is decided only by the force due to surface (iii) According to Bernoulli’s theorem,
tension. Due to surface tension, a liquid drop tends 1
to acquire minimum surface area. As, surface area is p + ρv 2 = constant
2
minimum for a sphere in every possible solid shapes. In this relation, pressure p occurs with one power
Therefore, under no external force, a liquid is always while velocity v occurs with two powers. Therefore,
spherical in shape. the influence of velocity is higher than pressure. The
24. Given, surface tension of water, S = 7 . 3 × 10 − 2 N / m size of the needle controls the velocity of flow and
Density of water, ρ = 1. 0 × 10 3 kg / m3 the thumb pressure controls pressure.
Acceleration due to gravity, g = 9. 8 m / s 2 Therefore, size of the needle of a syringe controls
Angle of contact, θ = 0 ° flow rate better than thumb pressure.
Diameter of one side, 2 r1 = 3. 0 mm (iv) A fluid flowing out of a small hole in a vessel have a
∴ r1 = 1. 5 mm = 1. 5 × 10 − 3 m large velocity and therefore a large momentum. As
Diameter of other side, 2 r2 = 6. 0 mm no external force is acting, therefore according to
r2 = 3. 0 mm = 3. 0 × 10 − 3 m law of conservation of momentum, equal momentum
in opposite direction and hence a backward velocity
Height of water column rises in first and second tubes,
is attained by the vessel. Therefore, a backward
2S cos θ  dp 
h1 = thrust F =
r1ρg  acts on the vessel.
 dt 
46 CBSE Term II Physics XI

(v) A spinning cricket ball in air does not follow a Work done per unit volume by the gravitational force
parabolic trajectory due to Magnus effect. = ρg ( h1 − h 2 )
Let a spinning cricket ball is moving forward with a = 1000 × 10 (2 − 5 ) = − 3 × 10 4 J/ m3
velocity v and spinning clockwise with velocity u. As 27. Let the density of sphere’s material is ρ and that of liquid
ball moves forward, it leaves a lower pressure region is σ, when the sphere just enters in the liquid.
behind it.
v Downward force on the sphere, F = Weight of the sphere
u – Weight of the fluid displaced by it
4 4 4
F = πr 3 ⋅ ρg − πr 3 ⋅ σg = πr 3(ρ − σ )g
v 3 3 3
Q Mass = Volume × Density
4
u M = πr 3ρ
v 3
∴ Acceleration of the sphere at this instant,
To fill this region, air moves backward with velocity 4 3
v. The layers of air in contact with the ball spin with πr (ρ − σ )g
F 3  σ
ball with velocity u. Therefore, the resultant velocity a= = = 1 −  g
m 4 3
πr ρ  ρ
of air above the ball is ( v − u ) and below the ball is
( v + u ). 3
When the sphere approaches to terminal velocity, its
According to Bernoulli’s theorem,
acceleration becomes zero.
1 a+0
p + ρv 2 = constant ∴ Average acceleration of the sphere =
2 2
Therefore, pressure below the ball becomes lower  σ
1 −  g
than above the ball. Due to this pressure difference,  ρ  σ g
a force acts on ball in downward direction. = = 1 − 
2  ρ 2
Therefore, the ball follows a curved path inspite of a
parabolic trajectory. If time t taken by the sphere to attain the terminal velocity
26. Given, ρ = 1000 kg/m 3 , v1= 1 m/s, A1 = 4 × 10−3 m2, v, then
A 2 = 8 × 10 −3 m 2, h1 = 2 m, h 2 = 5 m 2 r2
Terminal velocity, v = (ρ − σ )g
Apply Bernoulli’s theorem, 9 η
1 1 Q The sphere falls from rest,
p1 + ρv12 + g ρh1 = p 2 + ρv 22 + g ρh 2
2 2 ∴ u=0
1 Using v = u + at
( p1 − p 2 ) = ρ ( v 2 − v1 ) + ρg ( h 2 − h1)
2 2

2 2 r2  σ g
Putting values (ρ − σ )g = 0 + 1 −  ⋅ t
where, ( p1 − p 2 ) = work done by pressure per unit 9 η  ρ 2
volume, 4 r 2ρ
 W  1 ∴ t=
i.e.   = ρ ( v 2 − v1 ) + ρg ( h 2 − h1)
2 2
…(i) 9 η
 Volume  p 2
Thus, t is independent of the liquid density (σ ).
Q
28. (i) When a piece of the wood burns, then steam formed
and water appear in the form of drops due to surface
tension on the other end.
5m (ii) Surface tension of all lubricating oils and paints is kept
P low because they can spread over large area easily.
(iii) When some soap is dissolved in water, the surface
2m
tension of water decreases. Thus, the less energy
required to spray water.
From equation of continuity, A1v1 = A 2v 2
(iv) Given, S = 0.03 N/m
A v 4 × 10 − 3 × 1
⇒ v2 = 1 1 = = 0.5 m / s Work done = Surface area × Surface tension
A2 8 × 10 − 3
= 2 × 4 πr 2 × S = 2 × 4 × 3.14 ×(0.02)2 × 0.03
Putting the values in Eq. (i), we get = 3 × 10 − 4 J
 W  1 (v) This is due to the surface tension. The molecules of
  = × 1000 [0.25 − 1] + 1000 × 10 (5 − 2)
 Volume  p 2 water come close to each other due to surface
= − 375 + 30000 = 29625 J / m 3 tension and surface area tends to be minimum
making the steam narrow.
CBSE Term II Physics XI 47

CHAPTER 03

Thermal
Properties of Matter

In this Chapter...
l Ideal Gas Equation and l Heat Transfer
Absolute Temperature
l Newton’s Law of Cooling
l Thermal Expansion
l Greenhouse Effect
l Heat Capacity
l Change of State

On the Celsius scale, there are 100 equal intervals between two
Temperature reference points (i.e. ice and steam point) while on the Fahrenheit
It is the measure of degree of hotness or coldness of a scale, they are 180.
body. The measurement of temperature of a body is a
relative measure. Different Scales to Measure the Temperature
Freezing or ice point Boiling or steam point
Consider, there are two bodies with temperatures T1 Scale Unit
(Lower fixed point) (Upper fixed point)
and T2 , where T1 > T2 , then the body with T1 is called
hotter one with respect to another one which is Celsius Degree centigrade 0°C 100°C
known as colder body. scale (°C)
Fahrenheit Degree Fahrenheit 32°F 212°F
Heat scale (°F)
It is the form of energy which flows from hotter body Reaumur Degree Reaumur 0° R 80°R
to colder body by virtue of temperature difference. scale (°R)
The amount of heat is measured in joule (SI unit). Kelvin Kelvin (K) 273.15 K 373.15 K
Another widely used unit for the heat is calorie (in scale
CGS), where 1 joule equals
4.2 calorie (cal). Relation among the Temperatures Measured by
Different Scales
Measurement of Temperature The temperature measured by different scales is given as
The measurement of temperature is done by some C − 0 F − 32 R − 0 K − 273.15
specified scales called thermometers. = = =
100 180 80 100
The ice and steam point of water are 32°F and 212°F
Here, C, F, R and K are the readings of different scales.
on the Fahrenheit scale and 0°C and 100°C on Celsius
scale.
48 CBSE Term II Physics XI

If a rod having initial length l 1 at temperature T1 and


Ideal Gas Equation and Absolute final increased length l 2 at increased temperature T2 ,
Temperature then fractional change in its length is given by
At low densities, the gases exhibit same behaviour of ∆l ( l 2 − l 1 )
expansion. There are three characteristics of a gas which = = α l ( T2 − T1 ) = α l ∆T
l1 l1
describe its behaviour, these are pressure ( p ), volume ( V) and
temperature ( T ).
l1 ∆l
The relation among the characteristics of a gas is given by
(i) Boyle’s Law According to this law,
1 Linear expansion
p∝ (at constant temperature)
V Here, ∆l ∝ l 1 and ∆l ∝ ∆T , therefore ∆l ∝ l 1 ∆T
i. e. pV = constant ⇒ Expansion in length, ∆l = α l l 1 ∆T
(ii) Charles’ Law According to this law, The quantity α l is known as coefficient of linear
V∝T (at constant pressure) expansion and its unit is ° C −1 or K −1 .
V (ii) Area Expansion The expansion in the area of a
i. e. = constant
T surface due to increase in its temperature is called
area expansion.
Ideal Gas Equation
On combining these two laws (above mentioned), we get A1
pV ∆A
= constant (for a given quantity of gas) Area expansion
T
This relation is known as ideal gas law. It can be written in If a plate having initial area A1 at temperature T1 and
more general forms for a given amount of gas as final area A 2 at temperature T2 , then fractional
change in its area is given by
pV = µRT
∆A ( A 2 − A 1 )
where, µ is number of moles of a gas and R is known as = = α A ( T2 − T1 ) = α A ∆T
A1 A1
universal gas constant valued 8.31 J mol −1 K −1 . This
Here, ∆A ∝ A1 and ∆A ∝ ∆T
equation is known as ideal gas equation.
∴ ∆A ∝ A 1 ∆T
Absolute Zero Temperature ⇒ Expansion in area, ∆A = α A A1 ∆T
Theoretically, there is no limit for maximum temperature but where, α A is known as coefficient of area expansion.
there is a sharp point for minimum temperature that no body (iii) Volume Expansion The expansion in the volume of
can have the temperature lower than this minimum value of an object due to increase in its temperature is known
temperature which is known as absolute zero temperature. as volume expansion.
Absolute Zero Temperature
from p-T Graph
V1
The absolute zero temperature is regarded as 0 K or
∆V
− 273.15 ° C. This implies 273.15 K = 0 ° C.
Volume expansion
The temperature measured in Kelvin scale is known as
absolute temperature and the scale itself is known as absolute The fractional change in volume of an object is given
scale. by
∆V ( V2 − V1 )
= = α V ( T2 − T1 ) = α V ∆T
Thermal Expansion V1 V1
The phenomenon of change in dimensions of an object due to Here, ∆V ∝ V1 and ∆V ∝ ∆T
heat supplied is known as thermal expansion. ∴ ∆V ∝ V1 ∆T
There are three types of thermal expansion ⇒ Expansion in volume, ∆V = α V V1 ∆T
(i) Linear Expansion The expansion in length of a body where, α V is known as coefficient of volume expansion.
due to increase in its temperature is called the linear Note The dimensions of all types of coefficients of expansion is [ K −1]
expansion. and SI unit is per kelvin, i.e. K −1.
CBSE Term II Physics XI 49

Relation among the Coefficients It is defined as amount of heat needed to change the
of Expansion temperature by unity, i.e. 1° C, it is denoted by S and having
SI unit JK −1.
The coefficients of expansion are in ratio
αl : αA : αV =1 : 2 : 3 ∆Q
Heat capacity, S =
∆T
Anomalous Expansion of Water where, ∆Q = heat absorbed or rejected by body
Water shows an anomalous behaviour, it contracts on heating and
between the temperature 0°C to 4°C. When water is cooled
∆T = change in temperature.
below the room temperature (i. e. normal temperature), the
volume of given amount of water decreases. Until its Dimensional formula of heat capacity = [ML 2 T −2K −1 ]
temperature reaches to 4°C and below 4°C, the volume Note Mass of water having the same heat capacity as a given body is
increases (and hence density decreases). So, it is clear that called water equivalent of the body. The unit of water equivalent
water has maximum density (and hence minimum volume) is gram.
at 4°C.
The figure shows the variation of volume of 1 kg of water Specific Heat Capacity
with temperature (°C). The amount of heat needed to raise the temperature of unit
V mass of a substance by unity is known as the specific heat
1.04343
capacity or specific heat. It is denoted by s.
S ∆Q
s= =
Volume of 1 kg

m m∆T
(×10–3 m3)
water

where, m = mass of given substance.


The SI unit of specific heat capacity is Jkg −1K −1 .
1.00013 Note Water has the highest specific heat capacity (
1.00000 4.18 × 103 J kg −1 ° C−1) compared to other substances. For this
T
0 5 10 100 reason, water is used as a coolant in automobile radiators as well
Temperature (°C) as a heater in hot water bags.
Abnormal behaviour of water
Molar Specific Heat Capacity
Comparison of Expansions in
The amount of heat needed to raise the temperature of one
Solids, Liquids and Gases mole of a substance (gas) by unity is known as the molar heat
The expansion in solids and liquids is rather small as capacity of that substance. It is denoted by C. Its SI unit is
compared to the gases at ordinary temperature. The Jmol −1K −1 .
coefficient of volume expansion for the liquids is relatively S ∆Q
independent of the temperature. C= =
µ µ∆T
where, µ = number of moles of substance (gas).
Thermal Strain and Thermal Stress
When a metal rod whose ends are rigidly fixed so as to Relation between Specific Heat and
prevent the rod from expansion or contraction, it undergoes a Molar Specific Heat Capacity
change in temperature, this way thermal strains and thermal m
As, number of moles, µ =
stresses are developed in the rod. M
∆l where, m = mass of the substance
Thermal (compressive) strain = = α ∆T and M = molecular mass,
l
m = µM
and Thermal stress = Yα∆T 1  ∆Q  ∆Q 
C=   =M  
Here, α = linear expansion coefficient of the material of rod 
µ ∆T   m∆T 
and Y = Young’s modulus of the material of given rod. ∆Q
But = s ⇒ C = Ms
m∆T
Heat Capacity where, s = specific heat capacity,
To change the temperature of substance, a given quantity of M = molecular mass of the substance
heat is absorbed or rejected by it, which is characterised by a and C = molar specific heat capacity.
quantity is known as heat capacity.
50 CBSE Term II Physics XI

Types of Molar Specific Heat Capacity Effect of Pressure on the Boiling Point of a Liquid The
boiling point of a liquid increases with the increase in
There are two types of molar specific heat capacity as given below pressure. The boiling point of water is 100° C at 1 atm
(i) Molar Specific Heat Capacity at Constant Pressure It is pressure and it is 128°C at 2 atm pressure.
molar heat capacity of a gas at constant pressure, i. e. the
amount of heat required to raise the temperature of 1 mole Triple Point
of a gas by unity at constant pressure and is denoted by The temperature and pressure at which all three
Cp . phases of a substance co-exist simultaneously is
(ii) Molar Specific Heat Capacity at Constant Volume It is known as the triple point of the substance, e.g. the
molar heat capacity of a gas at constant volume, i. e. the triple point for water is represented by temperature
amount of heat required to raise the temperature of 1 mole 273.16 K and pressure 6.11 × 10 −3 Pa.
of gas through 1° C at constant volume and denoted by C V . The figure below shows the phase diagram with triple
point for water.
Calorimetry
p
It is the branch of science that deals with the measurement of (atm)
heat. Calorimeter is a device used for measuring the quantities of A
heat.
According to the principle of calorimetry, 218 C
Solid
Heat lost by hotter body = Heat gained by colder body Liquid Vaporisation
Fusion
If there are two bodies of masses m1 and m 2 and having values of curve curve
specific heats s 1 and s 2 respectively, then for temperature 0.006
O Vapour
difference ∆T. B
373.13
–220 0.01 T (°C)
⇒ m1 s 1 ∆T = m 2 s 2 ∆T Sublimation
curve

Change of State Phase diagram for water

The process of converting one state of a substance into another state The three curves represent the states in which solid and
is known as change of state of a substance or matter. vapour phases, solid and liquid phases and liquid and
For a constant heat supply, the change of state takes place with no vapour phases co-exist.
variation in temperature of the system.
Latent Heat
Terms Related to Change of State The amount of heat transferred per unit mass during the
There are some important terms related to change of state as change of phase of a substance without any change in its
given below temperature is called latent heat of the substance for
particular change.
(i) Melting and Melting Point The process of change of
state from solid to liquid is called melting. The Latent heat is denoted by L and having SI unit J kg −1 .
temperature at which solid starts to liquify is known as the i. e. Latent heat, L = Q/ m
melting point of that solid. There are two types of latent heat of materials
(ii) Fusion and Freezing Point The process of change of (i) Latent Heat of Fusion or Melting It is latent
state from liquid to solid is called fusion. The temperature heat for solid-liquid state change. It is denoted by
at which liquid starts to freeze is known as the freezing L f and is given by
point of the liquid. Q
Latent heat of fusion, L f =
(iii) Vaporisation and Boiling Point The process of change of m
state from liquid to vapour (or gas) is called vaporisation.
Its SI unit is Jkg −1 .
The temperature at which the liquid and the vapour states
of the substance co-exist is called the boiling point of (ii) Latent Heat of Vaporisation It is in latent heat
the liquid. for liquid-gas state change. It is denoted by L V
(iv) Sublimation The process of change of state directly from and is given by
solid to vapour (or gas) is known as sublimation. There is Q
Latent heat of vaporisation, L V =
no matter of liquid state of substance formed in the m
process. The reverse process of sublimation is not
possible, e.g. camphor, nepthalene balls, etc. Its SI unit is Jkg −1 .
CBSE Term II Physics XI 51

Variation of Temperature l
The equivalent thermal conductivity of series connection
A plot of temperature versus heat for a fixed quantity of of rods (as thermal conductor) is given by
water is shown below 2K 1K 2
K eq =
K1 + K 2
Boiling 22.6×105 J/kg
point (540 kcal/kg) l
The equivalent thermal conductivity of parallel
Temperature (°C)

100 Gas phase connection of rods is given by


3.33×103 J/kg (steam)
(80 kcal/kg) K +K2
Melting K eq = 1
Liquid phase 2
point (water)
0
Solid phase Phase change Thermal Current and Thermal Resistance
(ice) The rate of flow of heat is known as heat current. It is
ic denoted by H.
Heat (energy)
Temperature versus heat for water at 1 atm pressure The SI unit of thermal current is J/s or watt (W) and its
dimensions is [ML 2 T −3 ].
Mechanical Equivalent of Heat
∆Q ( T − T2 ) T1 − T2
The temperature of a body may also be increased by doing H= = KA 1 =
mechanical work on the system. The mechanical equivalent ∆t L L 
 
of heat gives how many joules of mechanical work is needed  KA
to raise the temperature of 1 g of water by 1°C. ∆T T − T L
Thermal resistance, R = = 1 2
=
It is denoted by J expressed in J/cal. H H KA
The SI unit of thermal resistance (R) is K-s/J or K/W and its
Heat Transfer dimension is [M −1L −2 T 3K].
Heat is the form of energy which can flow from one body to
another due to their temperature difference in the form of (ii) Convection
radiations, molecular vibrations, molecular displacement, Convection is the process in which heat is transferred from
etc. These processes of heat flow are collectively known as one point to another by the actual motion of matter from a region
heat transfer. of high temperature to a region of lower temperature. This
process of heat transfer takes place only in fluids.
There are three modes of heat transfer
There are two types of convections
(i) Conduction (i) Forced Convection This convection is the process in
The transfer of heat taking place due to molecular vibrations which heat is transferred from one place to other by
(i. e. molecular collisions) is known as heat conduction. In actual transfer of heated material (or molecules). If heated
this process, there is no mass movement of matter. material is forced to move say by a blower or a pump, the
Generally in solids, heat is transferred by the process of process of heat transfer is called forced convection.
conduction. (ii) Natural or Free Convection In the process of
convection, if the heated material moves due to
Thermal Conductivity
difference in density. This process of heat transfer is
The ability of material to conduct the heat through it
called natural or free convection.
is known as thermal conductivity. Thus, heat conduction is
Some natural examples of convection are given below
defined as the time rate of heat flow in a material for a given
temperature difference. Land and Sea Breezes
∆Q KA ( T1 − T2 ) ∆T During the day, the land heats up faster than the sea. This
Rate of heat transfer = = = KA creates a breeze from the sea to the land which is called a sea
∆t L L
breeze.
Q
Q H= Day
t
∆T
∴ Heat transfer, Q = KA ⋅t Air current
L (sea breeze)

Here, K is known as coefficient of thermal conductivity of


material of rod. The SI unit of K is Js −1 m −1K −1 or
Wm −1K −1 and dimensions of K is [MLT −3K −1 ].
Land warmer than water
52 CBSE Term II Physics XI

During the night, the opposite happens, creating a land The ratio of emissive power ( e ) of a body to the emissive power
breeze. (E ) of a perfect black body at the same temperature is called
Night emissivity. It is denoted by ε.
e
Thus, Emissivity, ε =
Air current E
(land breeze)
Stefan-Boltzmann Law
This states that the total energy emitted per second by a unit
area of a perfect black body is proportional to the fourth power
Water warmer than land of its absolute temperature.
i.e. E ∝ T4
Formation of Trade Winds
As equatorial and polar regions of the earth receive i.e. Total energy, E = σ T 4
unequal solar heat. At the equator, air near the earth’s where, σ is a universal constant called Stefan-Boltzmann
surface is hot while the air in upper atmosphere of the constant, σ = 5.67 × 10 −8 Wm −2 K −4 .
pole’s is cool.
If the body is not a perfect black body and has emissivity ε,
Hence, the steady wind blowing from North-East to then above relations get modified as
equator, near the surface of earth is called trade wind. It is
E = εσT 4
an example of natural convection.
where, ε = emissivity of that body.
(iii) Radiation If T0 be the surrounding temperature,
It is a mode of heat transfer from one place to another then H = e σA ( T 4 − T04 )
without heating the intervening medium. The heat is
transferred by the mean of thermal radiations, radiant Wien’s Displacement Law
energy or simply radiation. This states that the wavelength ( λ m ) corresponding to which
Thermal Radiation the energy emitted by a black body is maximum and is
inversely proportional to its absolute temperature ( T ).
The electromagnetic radiation emitted by a body, by virtue
1
of its temperature like the radiation by a red hot iron or Thus, λm ∝ or λ m T = b
light form filament lamp is called thermal radiation. T

Black Body Radiation where, b = Wien’s constant = 2.9 × 10 −3 mK.


A body that absorbs all the radiations falling on it is
known as a black body. It emits the radiations at the fastest Newton’s Law of Cooling
rate. The radiations emitted by a black body is known as This law states that the rate of cooling of a body is directly
black body radiation. The black body is also called the ideal proportional to the temperature difference between the body
radiator. and its surroundings, provided the temperature difference is
Absorptive and Emissive small .
dQ
Powers and Emissivity Rate of loss of heat, − = k ( T − T0 )
dt
The ratio of the amount of thermal radiation absorbed by a
body in a given time to the total amount of thermal Greenhouse Effect
radiations incident on the body in the same time is known
as absorptance (a) or absorptive power of the body. The gas such as carbon dioxide is known as the greenhouse gas
as it is transparent to the solar radiations but have the ability to
Energy absorbed
⇒ a= absorb infrared waves. These are also known as the long waves.
Energy incident
The emissive power of a body at a given temperature and The carbon dioxide concentration in the atmosphere has
for a given wavelength λ is defined as the amount of increased which may be attributed to the increase in the
radiant energy per unit time per unit surface area of the temperature of atmosphere. This effect is known as the
body within a unit wavelength range around the greenhouse effect. It melts the ice caps on the mountains and
wavelength λ. increases the sea level.
CBSE Term II Physics XI 53

Solved Examples
∆l
Example 1. An arbitrary scale has the ice point at −20° ⇒ 12 × 10 −6 = …(i)
l×t
and the steam point at 180°. When the
∆l b
thermometer reads 5°, then find the reading of Similarly , ∆l b = l b α t 2 ⇒ α b =
centigrade thermometer. lb × t2
∆l
Sol. In order to convert the temperature from one scale to ⇒ 18 ×10 −6 = …(ii)
C− 0 t − ( −20 ) (l − 5) × t
another, =
100 − 0 180 − ( −20 ) Dividing Eq. (i) by Eq . (ii), we get
C 5 + 20 12 × 10 −6 ∆l / l × t 2 l−5
Here, t = 5 ° ⇒ = = ⇒ =
100 200 18 × 10 −6 ∆l /( l − 5 ) × t 3 l
∴ Reading of centigrade thermometer, C = 12. 5 °C ⇒ 2 l = 3 l − 15 ⇒ l =15 cm
Example 2. The following observations were recorded So, l s = 15 cm and l b = l − 5 =15 − 5 = 10 cm
on a platinum resistance thermometer. Resistance
Example 4. The volume of mercury in the bulb of a
at melting point of ice = 3.70 Ω, resistance at
thermometer is 10 −6 m 3 . The area of cross-section
boiling point of water at normal pressure = 4.71 Ω
of the capillary tube is 2 × 10 −7 m 2 . If the
and resistance at t ° C = 5.29 Ω. Calculate
temperature is raised by 100° C, then find the
(i) temperature coefficient of resistance of platinum increase in the length of the mercury column.
(ii) and value of temperature t. (Take, γ Hg = 18 × 10 −5 / ° C )
Sol. Given, resistance at melting point of ice, R 0 = 3.70 Ω
Sol. From volume expansion relation, ∆V = V × γ ∆T …(i)
Resistance at boiling point of water at normal pressure,
where, γ = 18 × 10 −5 /°C, initial volume, V = 10 −6 m 3
R100 = 4.71 Ω
Resistance at t ° C, R t = 5.29 Ω and increase in temperature, ∆T = 100°C
(i) According to the formula, temperature coefficient of Putting these values in Eq. (i), we get
resistance is given by ∆V = 10 −6 × 18 × 10 −5 × 10 2 = 18 × 10 −9 m3
R − R 0 4.71 − 3.70 1.01 Now, ∆V = A × ∆l ⇒ 18 × 10 −9 = 2 × 10 −7 × ∆l
α = 100 = = = 2.73 × 10 −3 /° C
R 0 × 100 3.70 × 100 370
or 9 × 10 −2 = ∆l or ∆l = 9 cm
(ii) According to the formula, for temperature t, we
have Example 5. An aluminium cylinder 10 cm long, with a
R − R0 cross-sectional area of 20 cm 2 , is to be used as a
t = 100 ° C × t
R100 − R 0 spacer between two steel walls. At 17.2°C, it just
5.29 − 3.70 1. 59
slips in between the walls. When it warms to
= 100 ° C × = 100 ° C × = 157.4 ° C 22.3°C, calculate the stress in the cylinder and the
4.71 − 3.70 1. 01
total force, it exerts on each wall. Assuming that,
Example 3. The length of a steel rod is 5 cm longer the walls are perfectly rigid and a constant distance
than that of a brass rod. If this difference in their apart. (For aluminium, α = 2. 4 × 10 −5 K −1 )
lengths is to remain same at all temperatures, then [NCERT Exemplar]
find the length of brass rod. (Coefficients of linear Sol. As we know, for aluminium, Y = 7 . 0 × 1010 Pa
expansion for steel and brass are 12 × 10 −6 / °C and and α = 2 . 4 × 10 −5 K −1
18 × 10 −6 / °C, respectively)
Change in temperature, ∆T = T2 − T1 = (22. 3 − 17 . 2 )° C
Sol. Given, ∆l s − ∆l b = ∆l = 5.1 ° C
Let l s = l cm ⇒ l b = ( l − 5 ) cm Area, A = 20 cm 2 = 20 × 10 −4 m 2
(Qsteel rod is 5 cm longer than that of a brass rod)
αs = 12 ×10 −6 / °C and α b =18 × 10 −6 / ° C
As, ∆l s = l s αs t 1 A= 20 cm2
∆l s
∴ αs =
ls × t 1 10 cm
54 CBSE Term II Physics XI

∴ Stress =
F
= Yα∆T Latent heat, L V = 539 cal g −1
A Specific heat, cw = 1 cal g −1 ° C−1
= (0.70 × 1011 Pa)(2.4 × 10 −5 K −1 )(5.1 K)
Let Q be the heat required to convert 200 g of water at
= 1.68 × 5.1 × 10 6 = 8.6 × 10 6 Pa 40°C into 100°C, then Q = mc∆T
The total force F is the product of cross-sectional area and = (200) (1.0) (100 − 40) = 12000 cal
the stress.
Now, suppose m 0 mass of steam converts into water to
F = Stress × Area liberate this much amount of heat, then
= (8.6 × 10 6 Pa) (20 × 10 −4 m 2 ) = 1.7 × 10 4 N Q 12000
m0 = = = 22.26 g
Example 6. The coefficient of apparent expansion of a L 539
liquid when determined using two different vessels Since, it is less than 30 g, the temperature of the mixture
A and B are γ 1 and γ 2 , respectively. If the is 100°C.
coefficient of linear expansion of vessel A is α. Find Mass of steam in the mixture
the coefficient of linear expansion of the vessel B. = m1 − m 0 = 30 – 22.26 = 7.74 g
Sol. From thermal expansion in liquid, γ real = γ app + γ vessel and mass of water in the mixture = m + m 0
Given, γ vessel = 3 α = 200 + 22.26 = 222.26 g
For vessel A , γ real = γ 1 + 3α A
Example 9. Three identical rods have been joined at a
For vessel B , γ real = γ 2 + 3α B junction to make it a Y shape structure. If two free
γ1 − γ 2 ends are maintained at 45°C and the other end is at
Hence, γ 1 + 3α A = γ 2 + 3α B ⇒ α B = + αA
3 0°C, then what is the junction temperature T?
Example 7. A 10 kW drilling machine is used to drill a 45°C
bore in a small aluminium block of mass 8.0 kg.
How much is the rise in temperature of the block 0°C
T
in 2.5 min? Assuming 50% of power is used up in
heating the machine itself or lost to the
surroundings. Specific heat of aluminium 45°C
= 0.91 Jg − 1 ° C − 1 . Sol. According to the question,
45°C
Sol. Given, power, P = 10 kW = 10 × 10 3 W H2
Time, t = 2.5 min = 2.5 × 60 s H
0°C
As power is defined as rate at which energy is consumed, T
E
i.e. P = H1
t 45°C
∴ Total energy used, Rods are in steady state, so H = H1 + H 2 .
E = Pt = 10 × 10 3 × 2.5 × 60 = 1.5 × 10 6 J KA( T − 0 ) KA( 45 − T ) ( 45 − T )
⇒ = + KA
Energy absorbed by aluminium block, L L L
Q = 50% of the total energy = 50% of ⇒ T = 45 − T + 45 − T ⇒ 3 T = 90 ⇒ T = 30 ° C
1.5 × 10 6 = 0.75 × 10 6 J
Example 10. A body cools in 10 min from 60°C to 40°C.
Also, m = 8.0 kg = 8.0 × 10 3 g, c = 0.91 Jg − 1 ° C− 1 What will be its temperature after next
As, Q = mc∆T 10 min? The temperature of the surroundings is 10°C.
Q 0.75 × 10 6 Sol. According to Newton’s law of cooling,
∴ ∆T = = = 103.02 ° C
mc 8.0 × 10 3 × 0.91  T1 − T2   T1 + T2  
  = k   − T0 
Example 8. In a container of negligible mass, 30 g of  t    2  
steam at 100°C is added to 200 g of water that has a 60 − 40  60 + 40 
temperature of 40°C. If no heat is lost to the For the given conditions, =k − 10 …(i)
10  2 
surroundings, what is the final temperature of the
system? Also, find masses of water and steam in Let T be the temperature after next 10 min and now take
equilibrium. T1 = 40 ° C , then
(Take, L V = 539 cal g −1 and 40 − T  40 + T 
=k − 10 …(ii)
c water = 1 cal g −1 ° C −1 ) 10  2 
Sol. Given, mass, m = 200 g, m1 = 30 g Solving Eqs. (i) and (ii), we get
Temperature, T1 = 100 ° C, T2 = 40 ° C T = 28 ° C
CBSE Term II Physics XI 55

Chapter
Practice
PART 1 6. A bar of iron is 10 cm at 20°C. At 19°C, it will be
(Take, α of iron = 11 × 10 − 6 °C −1 )
Objective Questions (a) 11 × 10 − 6 cm, longer (b) 11 × 10 − 5 cm, shorter
(c) 11 × 10 − 6 cm, shorter (d) 11 × 10 − 5 cm, longer
l
Multiple Choice Questions 7. The radius of a metal sphere at room temperature T
1. Heat is associated with [NCERT Exemplar] is R and the coefficient of linear expansion of the
(a) kinetic energy of random motion of molecules metal is α. The sphere heated a little by a
(b) kinetic energy of orderly motion of molecules temperature ∆T, so that its new temperature is
(c) total kinetic energy of random and orderly motion of T + ∆T. The increase in the volume of the sphere is
molecules approximately [NCERT Exemplar]
(d) kinetic energy of random motion in some cases and (a) 2π Rα ∆T (b) π R 2α ∆T
kinetic energy of orderly motion in other (c) 4 π R 3α ∆T / 3 (d) 4 π R 3α ∆T
2. A glass of ice-cold water left on a table on a hot 8. Coefficient of volumetric expansion α V is not a
summer day eventually warms up, whereas a cup of constant and it depends on temperature. Variation
hot tea on the same table cools down because of α V with temperature for metals is
(a) its surrounding media are different
αV αV
(b) the direction of heat flow depends on the surrounding
temperature with respect to the object
(c) heating or cooling does not depend on surrounding (a) (b)
temperature
(d) Both (a) and (b) T T

3. The common physical property which is to be used αV αV


as the basis for constructing thermometer is the
variation of the
(c) (d)
(a) volume of a liquid with temperature
T T
(b) pressure of a gas with temperature
(c) resistance of a wire with temperature
9. Variation of the density of water with respect to
(d) All of the above
temperature from 0°C to 10°C is correctly
4. On a hilly region, water boils at 95° C. The represented by
temperature expressed in Fahrenheit is
(a) 100°F (b) 20.3°F
Density

Density

(c) 150°F (d) 203°F (a) (b)


5. A uniform metallic rod rotates about its 0 10°C 0 4°C 5°C
perpendicular bisector with constant angular Temperature Temperature
speed. If it is heated uniformly to raise its
temperature slightly, its speed of rotation
Density

Density

(a) increases [NCERT Exemplar] (c) (d)


(b) decreases
(c) remains same 0 10°C 0 4°C 10°C
Temperature Temperature
(d) increases because its moment of inertia increases
56 CBSE Term II Physics XI

10. An aluminium sphere is dipped into water. Which of 17. The amount of heat that a body can absorb by
the following is true? [NCERT Exemplar] radiation depends on
(a) Buoyancy will be less in water at 0°C than that in water (a) colour and temperature both of body
at 4°C (b) colour of body only
(b) Buoyancy will be more in water at 0°C than that in (c) temperature of body only
water at 4°C (d) density of body only
(c) Buoyancy in water at 0°C will be same as that in water
at 4°C 18. Due to the change in main voltage, the temperature
(d) Buoyancy may be more or less in water at 4 o C of an electric bulb rises from 3000K to 4000K.
depending on the radius of the sphere What is the percentage rise in electric power
consumed?
11. A normal diet furnishes 200 kcal to a 60 kg person in (a) 216 (b) 100
a day. If this energy was used to heat the person
(c) 150 (d) 178
with no losses to the surroundings, how much would
the person’s temperature increases? 19. According to Newton’s law of cooling, the rate of
(The specific heat of the human body is loss of heat depends on the
0.83 cal g −1 ° C −1 ). (a) sum of temperature of the body and its surroundings
(b) difference in temperature of the body and its
(a) 8.2°C (b) 4.01°C (c) 6.0°C (d) 5.03°C surroundings
12. Time taken to heat water upto a temperature of (c) product of temperature of the body and its
40° C (from room temperature) is t 1 and time taken surroundings
to heat mustard oil (of same mass and at room (d) ratio of temperature of the body and its surroundings
temperature) upto a temperature of 40° C is t 2 , then
(given mustard oil has smaller heat capacity)
l
Assertion-Reasoning MCQs
(a) t 1 = t 2 Direction (Q. Nos. 20-26) Each of these questions
(b) t 1 > t 2 contains two statements Assertion (A) and Reason (R).
Each of these questions also has four alternative
(c) t 2 > t 1
choices, any one of which is the correct answer. You
(d) t 1 and t 2 both are less than 10 min have to select one of the codes (a), (b), (c) and (d) given
13. When water boils or freezes, during these processes below.
its temperature (a) Both A and R are true and R is the correct
(a) increases explanation of A
(b) decreases (b) Both A and R are true, but R is not the correct
(c) does not change explanation of A
(d) sometimes increase and sometimes deceases
(c) A is true, but R is false
14. At atmospheric pressure, water boils at 100°C. If (d) A is false and R is also false
pressure is reduced, then it
(a) still boils at same temperature 20. Assertion A hotter body has more heat content
(b) now boils at a lower temperature than a colder body.
(c) now boils at a higher temperature Reason Temperature is the measure of degree of
(d) does not boil at all hotness of a body.
15. The latent heat of vaporisation of a substance is always 21. Assertion If equal amount of heat is added to
(a) greater than its latent heat of fusion equal masses of different substances, the resulting
(b) greater than its latent heat of sublimation change in temperature is also equal.
(c) equals to its latent heat of sublimation Reason Every substance requires same value of
(d) less than its latent heat of fusion heat to change its temperature per unit mass, per
16. Two rods of same length and material transfer a degree centigrade (or per kelvin).
given amount of heat in 12 s, when they are joined 22. Assertion Water kept in an open vessel will
end-to-end (i.e. in series). But when they are joined quickly evaporate on the surface of the moon.
in parallel, they will transfer same heat under same
temperature difference across their ends in Reason The temperature at the surface of the
(a) 24 s (b) 3 s (c) 38 s (d) 1.5 s
moon is much higher than boiling point of
water.
CBSE Term II Physics XI 57

23. Assertion When a rod is heated freely, it expands III. Radiation is a mode of heat transfer from one
and thermal strain set up in rod due to heating. place to another without heating the intervening
Reason Strain is a change in length per unit medium. The heat is transferred by the mean of
original volume. thermal radiations, radiant energy or simply
radiation.
24. Assertion The triple point of water is a standard (i) On a cold morning, a metal surface will feel colder
fixed point in modern thermometry.
to touch than a wooden surface because metal has
Reason Melting point of ice and the boiling point (a) high specific heat
of water changes due to change in atmospheric (b) high thermal conductivity
pressure. (c) low specific heat
25. Assertion The radiation from the sun’s surface (d) low thermal conductivity
varies as the fourth power of its absolute (ii) Woolen clothes keep the body warm, because wool
temperature. (a) is a bad conductor
Reason Sun is not a black body. (b) increases the temperature of body
26. Assertion The rate of loss of heat of a body at (c) decreases the temperature of body
300 K is R. At 900 K, the rate of loss becomes (d) All of the above
81 R. (iii) Heat travels through vacuum by
Reason This is as per Newton’s law of cooling. (a) conduction (b) convection
(c) radiation (d) Both (a) and (b)
l
Case Based MCQs (iv) Ratio of the amount of heat radiation, transmitted
27. Direction Read the following passage and answer through the body to the amount of heat radiation,
the questions that follows incident on it, is known as
(a) conductance (b) inductance
Heat Transfer
(c) transmittance (d) absorptance
Heat is the form of energy which can flow from one
(v) The unit of thermal conductivity is
body to another due to their temperature difference
(a) Wm −1K −1 (b) JK −1 (c) WmK (d) JK
in the form of radiations, molecular vibrations,
molecular displacement, etc. These processes of heat
flow are collectively known as heat transfer.
The processes of heat transfer are shown in figure PART 2
Molecular
displacement
Molecular vibrations
(conduction) Subjective Questions
(convection)
l
Short Answer (SA) Type Questions
1. Find out the temperature which has same
numerical value on celsius and fahrenheit scale.
Radiations 2. In what ways are the gas thermometers superior to
mercury thermometers?
Fire
(source) 3. There is a slight temperature difference between
Modes of heat transfer the water fall at the top and the bottom. Why?
4. A steel tape 1m long is correctly calibrated for a
There are three modes of heat transfer namely
temperature of 27.0°C. The length of a steel rod
I. Conduction is the made of transfer of heat taking measured by this tape is found to be 63.0 cm on a
place due to molecular vibrations (i. e. molecular hot day, when the temperature is 45.0°C.
collisions).
What is the actual length of the steel rod on that
II. Convection is the process in which heat is day? What is the length of the same steel rod on a
transferred from one point to another by the actual day, when the temperature is 27.0°C? Coefficient of
motion of matter from a region of high temperature
linear expansion of steel = 1.20 × 10 −5 ° C −1 ?
to a region of lower temperature. [NCERT]
58 CBSE Term II Physics XI

5. The difference between length of a certain brass 14. A piece of paper wrapped tightly on a wooden rod
rod and that of a steel rod is claimed to be constant is observed to get burnt quickly when held over a
at all temperatures. Is this possible? flame as compared to a similar piece of paper when
6. There are two spheres of same radius and material wrapped on a brass rod. Explain why?
at same temperature but one being solid while the 15. In a coal fire, the pockets formed by coals appear
other hollow. Which sphere will expand more, if brighter than the coals themselves. Is the
(i) they are heated to the same temperature and temperature of such a pocket higher than the
(ii) same amount of heat is given to each of them? surface temperature of a glowing coal?
7. The coefficient of volume expansion of glycerine is 16. Why rooms are provided with the ventilators near
49 × 10 −5 ° C −1 . What is the fractional change in its the roof ?
density for a 30°C rise in temperature? [NCERT] 17. The earth constantly receives heat radiation from
8. What kind of thermal conductivity and specific heat the sun and gets warmed up. Why does the earth
requirements would you specify for cooking not get as hot as the sun?
utensils? 18. If a drop of water falls on a very hot iron, it does
9. Two thermos flasks are of the same height and same not evaporate for a long time. Give reason.
capacity. One has a circular cross-section while the 19. Why it is much hotter above a fire than by its side?
other has a square cross-section. Which of the two 20. How does tea in a thermoflask remain hot for a long
is better? time?
10. The coolant used in a nuclear reactor should have 21. Two bodies of specific heats C 1 and C 2 having
high specific heat. Why? same heat capacities are combined to form a single
11. Given below are observations on molar specific composite body. What is the specific heat of the
heats at room temperature of some common gases. composite body?
[NCERT]
22. A Celsius and Fahrenheit thermometer are put in
Molar specific heat ) an hot bath. The reading of Fahrenheit
Gas (CV ) (cal mol − 1 K − 1 ) thermometer is (3/2) times the reading of Celsius
thermometer. What is the temperature of bath on
Hydrogen 4.87 Celsius, Fahrenheit and Kelvin’s scales.
Nitrogen 4.97
23. The triple points of neon and carbon dioxide are
Oxygen 5.02 24.57 K and 216.55 K, respectively. Express these
Nitric oxide 4.99 temperatures on the celsius and fahrenheit scales.
[NCERT]
Carbon monoxide 5.01
Chlorine 6.17
24. Two ideal gas thermometers A and B use oxygen
and hydrogen, respectively. The following
The measured molar specific heats of these gases observations are made
are markedly different from those for monoatomic Pressure Pressure
Temperature
gases. (Typically, molar specific heat of a thermometer A thermometer B
monoatomic gas is 2.92 cal/mol K). Explain this Triple point of 1.250 × 105 Pa 0.200 × 105 Pa
difference. What can you infer from the somewhat water
larger (than the rest) value for chlorine? Normal melting 1.797 × 105 Pa 0.287 × 105 Pa
12. On a hot day, a car is left in sunlight with all the point of sulphur
windows closed. After sometime, it is found that (i) What is the absolute temperature of normal
the inside of the car is considerably hotter than the melting point of sulphur as read by
air outside. Explain, why? thermometers A and B?
13. Two vessels of different materials are identical in (ii) What do you think is the reason for slightly
size and wall thickness. They are filled with equal different answers from A and B? [NCERT]
quantities of ice at 0° C .
25. A metallic ball has a radius of 9.0 cm at 0°C.
If the ice melts completely in 10 and 25 min Calculate the change in its volume, when it is
respectively, compare the coefficients of thermal heated to 90°C. Given that, coefficient of linear
conductivity of the materials of the vessels. expansion of metal of ball is 1.2 × 10 −5 K −1 .
CBSE Term II Physics XI 59

26. A steel wire of 2.0 mm 2 cross-section is held l


Long Answer (LA) Type Questions
straight (but under no tension) by attaching it
firmly to two points a distance 1.50 m apart at 33. The electrical resistance in Ohms of a certain
30°C. If the temperature now decreases to 5°C and thermometer varies with temperature according to
if the two points remain fixed, what will be the the approximate law R = R 0 [1 + 5 × 10 −3 ( T − T0 )]
tension in the wire? The resistance is 101.6 Ω at the triple point of
(Take, Young’s modulus of steel = 2 × 1011 Nm −2 and water and 165.5 Ω at the normal melting point of
coefficient of thermal expansion of steel lead (600.5 K). What is the temperature, when the
α = 1.1 × 10 −5/ °C.) resistance is 123.4 Ω? [NCERT]

27. The brass scale of a barometer gives correct 34. A brass rod of length 50 cm and diameter 3.0 mm is
reading at 0°C. Coefficient of linear expansion of joined to a steel rod of the same length and diameter.
brass is 2. 0 × 10 −5 /°C. The barometer reads What is the change in length of the combined rod at
75.00 cm at 27°C. What is the true atmospheric 250°C, if the original length are at 40.0°C? Is there
pressure at 27°C? a ‘thermal stress’ developed at the junction? The
ends of the rod are free to expand. Coefficient of
28. A brass boiler has a base area of 0.15 m 2 and linear expansion of brass = 2.0 × 10 −5 °C −1 and that
thickness 1.0 cm. It boils water at the rate of of steel = 1.2 × 10 −5 °C −1 . [NCERT]
6.0 kg min − 1 , when placed on a gas stove.
35. A hole is drilled in a copper sheet. The diameter of
Estimate the temperature of the part of the flame in the hole is 4.24 cm at 27°C. What is the change in
contact with the boiler. Thermal conductivity of the diameter of the hole, when the sheet is heated
brass = 109 Js − 1 m − 1 ° C − 1 and heat of to 227°C? Coefficient of linear expansion of copper
vaporisation of water = 2256 Jg −1 . [NCERT] is 1.70 × 10 −5 /°C. [NCERT]
29. Explain the following 36. Show that the coefficient of volume expansion for a
(i) Hot tea cools rapidly, when poured into the solid substance is three times its coefficient of
saucer from the cup. linear expansion.
(ii) Temperature of a hot liquid falls rapidly in the 37. Answer the following .
beginning but slowly afterwards. (i) The triple point of water is a standard fixed point
(iii) A hot liquid cools faster, if outer surface of the in modern thermometry. Why? What is wrong in
container is blackened. taking the melting point of ice and the boiling
30. A copper block of mass 2.5 kg is heated in a furnace point of water as standard fixed points (as was
to a temperature of 500° C and then placed on a originally done in the Celsius scale)?
large ice block. What is the maximum amount of (ii) There were two fixed points in the original
ice that can melt? (Take, specific heat of copper celsius scale as mentioned above which were
= 0.39 Jg − 1 ° C − 1 , and heat of fusion of water assigned the number 0°C and 100°C,
= 335 Jg − 1 .) [NCERT] respectively. On the absolute scale, one of the
31. A thermocol cubical ice box of side 30 cm has a fixed points is the triple point of water, which on
thickness of 5.0 cm. If 4.0 kg of ice are put in the the Kelvin absolute scale is assigned the number
box, estimate the amount of ice remaining after 6 h. 273.16 K. What is the other fixed point on this
The outside temperature is 45° C and coefficient of (Kelvin) scale?
thermal conductivity of thermocol (iii) The absolute temperature (Kelvin scale) T is
= 0.01 Js − 1 m − 1 ° C − 1 . Given, heat of fusion of related to the temperature TC on the Celsius
water = 335 × 10 3 J kg − 1 scale by Tc = T − 273.15. Why do we have 273.15
[NCERT]
in this relation and not 273.16?
32. Two rods of the same area of cross-section, but of (iv) What is the temperature of the triple point of
lengths l 1 and l 2 and conductivities K 1 and K 2 are water on an absolute scale, whose unit interval
joined in series. Show that the combination is size is equal to that of the Fahrenheit scale?
equivalent of a material of conductivity [NCERT]
l1 + l 2
K= 38. A child running a temperature of 101°F is given an
 l1   l 2  antipyretic (i. e. a medicine that lowers fever) which
  + 
 K1   K 2  causes an increase in the rate of evaporation of
60 CBSE Term II Physics XI

sweat from his body. If the fever is brought down to 98 l


Case Based Questions
°F in 20 min. What is the average rate of extra
evaporation caused by the drug? Assume, the
43. Direction Read the following passage and
answer the questions that follows
evaporation mechanism to be the only way by which
heat is lost. The mass of the child is 30 kg. The specific Triple Point
heat of human body is approximately the same as that The temperature of a substance remains
of water and latent heat of evaporation of water at that constant during its phase change (or change of
temperature is about 580 cal g − 1 . [NCERT] state).
39. Explain, why The graph between temperature and
pressure of substance can be plotted which
(i) a body with large reflectivity is a poor emitter. is called phase diagram or p-T diagram.
(ii) a brass tumbler feels much colder than a wooden
tray on a chilly day. The diagram (phase diagram of water and CO 2 )
shows p-T plane divided into three regions,
(iii) an optical pyrometer (for measuring high temperature) i. e. solid region, liquid region and vapour
calibrated for an ideal black body radiation gives too
low a value for the temperature of a red hot iron piece region which are separated by sublimation
in the open, but gives a correct value for the curve, fusion curve and vaporisation curve.
temperature when the same piece is in the furnace. These three curves represent the states in
(iv) the earth without its atmosphere would be which solid and vapour phases, solid and liquid
inhospitably cold. phases and liquid and vapour phases co-exist.
(v) heating systems based on circulation of steam are The temperature and pressure at which all
more efficient in warming a building than those three phases of a substance co-exist
based on circulation of hot water. [NCERT] simultaneously is known as the triple point
40. Distinguish between conduction, convection and of the substance.
radiation. The p-T phase diagram of carbon dioxide as
41. An iron bar having length L 1 = 0.1 m, area of shown in the figure
cross-section 0.02 m 2 thermal conductivity II
I
K i = 79 Wm −1 K −1 and brass bar having length 73.0
L 2 = 0.1 m area of cross-section, A 2 = 0.02 m 2 and
50.0
thermal conductivity K 2 = 109 Wm −1 K −1 are soldered Liquid
p(atm)

end to end as shown in figure. Solid

The terminal ends of two rods are maintained at 373 K O Vapour


5.11
and 273 K, respectively. Find the expression and
III
compute 1
(i) the temperature of the junction of two bars. 0 –78.5°– 56.6° 20° 31.1°
(ii) equivalent thermal conductivity of composite bar T(°C)→
and
(i) At what temperature and pressure can the
(iii) the heat current through the composite bar
solid, liquid and vapour phases of CO 2
T0 co-exist in equilibrium?
T1 = 373 K T2 = 273 K (ii) What is the effect of decrease of pressure on
Iron Brass the fusion and boiling point of CO 2 ?
(iii) What are the critical temperature and
42. A body cools from 80°C to 50°C in 5 min. Calculate the pressure for CO 2 ? What is their significance?
time it takes to cool from 60°C to 30°C, the (iv) Is CO 2 solid, liquid or gas at (a) − 70° C
temperature of the surrounding is 20°C. [NCERT] under 1 atm, (b) − 60° C under 10 atm and (c)
15° C under 56 atm?
Chapter Test
Multiple Choice Questions 7. What should be the length of steel and copper rods at
1. Temperature of atmosphere in Kashmir falls below 0°C, so that the length of the steel rod is 5 cm longer
than the copper rod at any temperature? (Take, linear
− 10 ° C in winter. Due to this, water animal and plant life
of Dal-lake expansion coefficient of steel = 12 × 10 − 5 / ° C and for
copper= 1.6 × 10 − 5 / ° C) (Ans. 20 cm and 15 cm)
(a) is destroyed in winters
(b) frozen in winter and regenerated in summers 8. A metal ball having a diameter of 0.4 m is heated from
(c) survives as only top layer of lake in frozen 273K to 360 K. If the coefficient of areal expansion of
(d) None of the above the material of the ball is 0.000034 K −1 , then determine
the increase in surface area of the ball.
2. When temperature of water is raised from 0°C to 4°C, it (Ans. 1.486 × 10 −3 m 2 )
(a) expands
(b) contracts 9. A steel rod is 50 m long and has a cross- sectional area
(c) expands upto 2°C and then contracts upto 4°C 250 cm 2 . What is the force exerted by the rod, when
(d) contracts upto 2°C and then expands upto 4°C heated from 5°C to 25°C? (Take, α s = 11 × 10 − 6 / ° C and
Ys or α V = 2 × 10 11 / ° C) (Ans. 11 × 105 N)
3. A bimetallic strip is made of aluminium and steel
(α Al > α steel ). On heating, the strip will Long Answer Type Questions
[NCERT Exemplar] 10. A specific book describes a new temperature scale
(a) remain straight called Z, in which boiling and freezing points of water
(b) get twisted are referred as 65°Z and − 15 ° Z, respectively.
(c) will bend with aluminium on concave side (i) To what temperature on Fahrenheit scale would a
(d) will bend with steel on concave side temperature − 95 ° Z correspond?
4. The temperature of two bodies A and B are respectively (ii) What temperature change on the Z scale would
(Take, 727° C and 327° C). The ratio HA : HB of the rates correspond to a change of 40° on Celsius scale?
of heat radiated by them is [Ans. (a) − 148 ° F and (b) 17°Z]
(a) 727 : 327 (b) 5 : 3 11. A box having total surface area 0.05 m 2 and of 6 mm
(c) 25 : 9 (d) 625 : 81 thick side walls is filled with melting ice and kept in a
5. The rate of cooling due to conduction, convection and room. Calculate the thermal conductivity of the box
radiation combined, is proportional to the difference in material, if 0.5 kg of ice melts in 1 h. The room
temperature, for temperature = 40°C and latent heat of fusion of ice
(a) large temperature differences = 3.33 × 105 J kg− 1 .
(b) small temperature differences (Ans. 0.138 Js −1 m −1 °C −1 )
(c) any temperature difference 12. Calculate the heat required to convert 3 kg of ice
(d) None of the above at − 12 ° C kept in a calorimeter to steam at 100° C at
atmospheric pressure.
Short Answer Type Questions
(Take, specific heat capacity of ice is 2100 Jkg−1K −1 ,
6. The density of mercury is 13.6 × 10 3 kg m − 3 at 0°C and
specific heat capacity of water is 4186 Jkg−1K −1 , latent
its coefficient of volume expansion is 1.82 × 10 − 4 K − 1 .
heat of fusion of ice is 3.35 × 105 Jkg−1 and latent heat of
Find the density at 50°C. (Ans. 13.48 g cm −3 )
steam is 2.256 × 10 6 Jkg−1 ) (Ans. 9. 1 × 10 6 J)

Answers For Detailed Solutions


Multiple Choice Questions Scan the code
1. (c) 2. (b) 3. (d) 4. (d) 5. (b)
62 CBSE Term II Physics XI

EXPLANATIONS
PART 1 ⇒ L 2 = 9.99989
Length is shorter by = 10 − 9. 99989 = 0. 00011
1. (a) We know that, as temperature increases vibration of = 11 × 10 −5 cm
molecules about their mean position increases, hence
kinetic energy associated with random motion of these 7. (d) Let the radius of the sphere is R. As, the temperature
molecules increases. Thereby leading to production of increases, radius of the sphere increases as shown in
heat. figure
2. (d) In given case, the ice-cold water having lower
temperature than the surrounding, when left on a table
on a hot summer day takes heat from surrounding and R
warms up, whereas a cup of tea having higher
temperature than surrounding cools down by releasing
dV
heat to surrounding. Therefore, surrounding media are
different for both.
4
Thus, both the statements given in options (a) and (b) are Original volume, V = πR 3
correct. 3
Coefficient of linear expansion = α
3. (d) Any physical property which varies linearly with
temperature can be used in constructing thermometers. ∴ Coefficient of volume expansion = 3α
4. (d) It is given that on a hilly region, water boils at 95° C. 1 dV
∴ = 3α
∴ Temperature in centigrade, C = 95 ° C. V dT
So, the temperature in Fahrenheit can be calculated. ⇒ dV = 3 Vαdt ~
− 4 πR 3α∆T
F − 32 C = Increase in the volume
By using relation = , we get
180 100 8. (c) α V is a characteristic of the substance but is not
F − 32 95 strictly a constant. It depends in general on temperature
= (Q C = 95 ° C)
9 5 as shown in Fig. (c). It is seen that, α V becomes constant
⇒ F − 32 = 9 × 19 only at a higher temperature. Hence, option (c) is correct.
⇒ F − 32 = 171 9. (d) Water contracts when it is heated from 0°C to 4°C.
⇒ F = 171 + 32 Thus, its density increases and volume decreases.
Density of water is maximum at 4°C and hence volume is
= 203 ° F
minimum. When the water is further heated, it expands
5. (b) As the rod is heated, it expands. As, no external torque and density decreases.
is acting on the system, so angular momentum should be
So, the graph given in option (d) shows the correct
conserved.
variation of density of water.
10. (a) Let volume of the sphere is V and ρ is the density of
water, then we can write buoyant force
Rod F = V ρ g (g = acceleration due to gravity)
L = Angular momentum ⇒ F ∝ρ (Q V and g are almost constants)
= Iω = constant ⇒ I1 ω1 = I 2ω 2 F 4 ° C ρ4 ° C
⇒ = >1 (Q ρ4 °C > ρ0 °C )
Due to expansion of the rod, I 2 > I1 F 0 ° C ρ0 ° C
ω I ⇒ F4 ° C > F0 ° C
⇒ 2 = 1 < 1 ⇒ ω 2 < ω1
ω1 I 2 Hence, buoyancy will be less in water at 0°C than that in
So, angular velocity (speed of rotation) decreases. water at 4o C.
6. (b) According to linear expansion, we get 11. (b) Given, m = 60 kg = 60 × 10 3 g, c = 0.83 cal g −1 ° C−1 ,

L = L 0 (1 + α∆θ ) Q = 200 kcal = 2 × 10 5 cal


L 1 1 + α ( ∆θ1 ) Amount of heat required for a person,
=
L 2 1 + α ( ∆θ 2 ) Q = mc∆T
Q 2 × 10 5
10 1 + 11 × 10 −6 × 20 ⇒ ∆T = = = 4.016 ° C
⇒ = mc 60 × 10 3 × 0.83
L 2 1 + 11 × 10 −6 × 19
CBSE Term II Physics XI 63

∆Q As we know, percentage rise in power


12. (b) The heat capacity of a substance is S = .
∆T P − P1 256 − 81
= 2 × 100 = × 100
The heat capacity of mustard oil is less than that of water P1 81
for same mass. So, same temperature rise ( ∆T = 40 ° C), 175
the quantity of heat ∆Q would be less than that is = × 100 = 216%
required by the same amount of water. 81
Hence, the time taken by water ( t 1 ) to heat upto 40°C 19. (b) According to Newton’s law of cooling, the rate of loss
will be higher than that of mustard oil (t 2 ), i.e. t 1 > t 2 . of heat is directly proportional to the difference in
temperature of the body and its surroundings.
13. (c) When water boils or freezes, its temperature does not
change during these processes. Heat here is absorbed or 20. (d) Temperature is the measure of degree of hotness or
liberated as latent heat. coldness of a body. It is not true that a hotter body has
more heat content than a colder body.
14. (b) When pressure is increased, boiling point is elevated.
i.e. At higher pressure, water boils at temperature greater Therefore, A is false and R is also false.
than 100°C. Similarly, at reduced pressure, water boils at 21. (d) If equal amount of heat is added to equal masses of
a lower temperature. different substances, then the resulting temperature
15. (a) The energy required to completely separate the changes will not be the same.
molecules, moving from liquid to gas is much greater Because every substance has a unique value for the
than the energy required to separate from solid to liquid. amount of heat absorbed or rejected to change the
Hence, the latent heat of vaporisation is greater than the temperature of unit mass of it by one unit.
latent heat of fusion. Therefore, A is false and R is also false.
16. (b) When two rods of same length are joined in parallel, 22. (c) As there is no atmosphere on the moon, so water kept
the time required for heat transfer, in an open vessel quickly evaporates or boils due to
∆Q ( ∆x ) reduced pressure, which causes reduction in boiling
∆t = point.
KA( ∆T )
Therefore, A is true but R is false.
Two rods of same length, which are joined in series, when
connect in parallel combination, the area of heat 23. (d) In freely heated rod, there is no thermal stress and no
conduction becomes twice, i.e. A → 2 times and length thermal strain, there is thermal expansion only.
1 When a rod (whose ends are fixed) is heated, thermal
becomes half, i.e. ∆x → times as shown in figure
2 expansion of rod is prevented but this in turn, develops a
A
compressive strain due to external forces provided by the
A
A
rigid support at the ends.
∆x ∆x The thermal stress, so set up is given by
2∆x ∆x
∆F  ∆l 
Stress = = Y 
In series In parallel
A  l
∆t 1 ∆x1 A where, Y is the Young’s modulus of the rod.
∴ = × 2
∆t 2 A1 ∆x 2 ∆ l lα l ∆ T
Also, thermal strain = = = α l ∆T
∆x1 A 2 l l
= ×
∆x2 A1 which means that strain is a change in length per unit
original length.
=2×2= 4
1 1 Therefore, A is false and R is also false.
⇒ ∆t 2 = ∆t 1 = × 12 = 3 s 24. (a) In modern thermometry, the triple point of water is
4 4
taken as a standard fixed point because its value does not
17. (a) The thermal radiation that falls on a body is partly change under any condition.
reflected and partly absorbed. The amount of heat that a
body can absorb, by radiation depends on the colour of Melting point of ice and the boiling point of water
the body and temperature of the body. changes due to change in atmospheric pressure. Due to
this factor, callibration of a thermometer is affected.
18. (a) Electric power consumed in first case,
Therefore, both A and R are true and R is the correct
P1 = σT14 = σ ( 3000 )4 …(i) explanation of A.
Electric power consumed in second case, 25. (c) At a high temperature of 6000 K, the sun acts like a
P2 = σT24 = σ ( 4000 )4 …(ii) black body emitting complete radiation.
On dividing Eq. (ii) by Eq. (i), we get It follows from Stefan’s law that E ∝ T 4 , i.e. the radiation
from the sun’s surface varies as the fourth power of its
P2 ( 4000 )4 256
= = absolute temperature.
P1 ( 3000 )4 81 Therefore, A is true but R is false.
64 CBSE Term II Physics XI

26. (c) Stefan’s law is applied here not the Newton’s law of 5. Yes, it is possible to describe the difference of length to
cooling. remains constant. So, the change in length of each rod
According to Stefan’s law, must be equal at all temperature. Let L b & L s be the
E 2  T2 
4
 900 
4
E2 length of the brass & the steel rod and α b & αs be the
=  =  = 81 ⇒ = 81 coefficients of linear expansion of the two metals. Let
E1  T1   300  R
there is change in temperature be ∆T, then
Therefore, E 2 = 81 R α b L b ∆ T = αs L s ∆ T
Therefore, A is true but R is false. or α b L b = αs L s
27. (i) (b) Due to higher thermal conductivity of metal than ⇒ L b / L s = αs / α b
wood, heat begins to flow readily from our body to Hence, the lengths of the rods must be in the inverse
the metal surface and so we feel colder. ratio of the coefficient of linear expansion of their
(ii) (a) Woolen fibres enclose a large amount of air in materials.
them. Both wool and air are bad conductors of heat 6. (i) As thermal expansion of isotropic solids is similar to
and do not allow loss of heat from our body due to true photographic enlargement, the expansion of a
conduction. cavity is same, as if it were a solid body of the same
(iii) (c) Heat travels through vacuum due to radiation. material, i.e. ∆V = γ V ∆T. As here V, γ and ∆T are
(iv) (c) The ratio of amount of heat transmitted through same for both solid and hollow spheres, so the
an object to the amount of heat incident on it is expansions of both will be equal.
called transmittance. (ii) If same amount of heat is given to the two spheres,
then due to lesser mass, rise in temperature of
(v) (a) The SI unit of thermal conductivity K is hollow sphere will be more (as ∆T = Q / Mc) and
Wm −1K −1 . hence the expansion will be more as ∆V = γ V ∆T.
7. Let M be the mass of glycerine, ρ0 its density at 0°C and
PART 2 ρt its density at t°C.
M M

1. Let θ be the same numerical value of temperature on the Vt − V0 ρt ρ0
both scales. Then, γ= =
TC TF − 32 V0 ∆T (M / ρ0 ) ∆T
∴ = 1 1
5 9 –
θ θ − 32 ρ ρ0 ρ − ρt
⇒ = (Qθ ° C = θ ° F = θ ) γ= t = 0
5 9 (1 / ρ0 ) ∆T ρ0 ∆ T
⇒ 9θ = 5θ − 160 ∴ Fractional change in density,
⇒ −4θ = 160 ρ0 − ρt
= γ ∆T
∴ θ = −40 ° ρ0
θ = −40 ° C = −40 ° F = 49 × 10 −5 × 30 = 0.0147
2. A gas thermometer is more superior to a mercury 8. A cooking utensil should have (i) high conductivity, so
thermometer, as its working is independent of the nature that it can conduct heat through itself and transfer it to
of gas (working substance) used. As the variation of the contents quickly (ii) and low specific heat, so that it
pressure (or volume) with temperature is uniform, the immediately attains the temperature of the source.
range, in which temperature can be measured with a gas
9. As both flasks have same height and capacity, the area of
thermometer is quite large. Further, a gas thermometer is
the cylindrical wall will be less than that of the square
more sensitive than mercury thermometer.
wall. Hence, the thermos flask of circular cross-section
3. The potential energy of water at the top of the fall gets will transmit less heat as compared to the thermos flask of
converted into heat (kinetic energy) at the bottom of the square cross-section and it will be better.
fall. When water hits the ground, a part of its kinetic
10. The purpose of a coolant is to absorb maximum heat with
energy gets converted into heat which increases its
least rise in its own temperature. This is possible only, if
temperature slightly.
specific heat is high because Q = mc ∆T. For a given
4. Given, t 1 = 27 ° C, l 1 = 63 cm, value of m and Q, the rise in temperature ∆T will be
t 2 = 45 ° C, α = 1.20 × 10 −5 ° C−1 small, if c is large. This will prevent different parts of the
Length of the rod on the hot day is nuclear reactor from getting too hot.
l 2 = l 1 [1 + α ( t 2 − t 1 )] 11. A monoatomic gas has three degrees of freedom, while a
= 63 [1 + 1.20 × 10 −5 (45 − 27)] = 63.0136 cm diatomic gas possesses five degrees of freedom.
Therefore, molar specific heat of a diatomic gas (at
As the steel tape has been calibrated for a temperature of
constant volume).
27°C, so length of the steel rod at 27°C = 63 cm.
CBSE Term II Physics XI 65

f 5 5 8.31 evaporate. But if the drop of water falls on iron which is


CV = R= R= × = 5 cal mol − 1K − 1
2 2 2 4.2 not very hot, then it comes in direct contact with iron and
In the given table, all the gases are diatomic gases and for evaporates immediately.
all of them (except chlorine), the value of CV is about 19. Heat carried away from a fire sideways mainly by
radiation. Above the fire, heat is carried by both radiation
5 cal mol − 1K − 1 .
and convection of air but convection carries much more
The slightly higher value of CV for chlorine is due to the heat than radiation. So, it is much hotter above a fire than
fact that even at room temperature, a chlorine gas by its sides.
molecule possesses the vibrational mode of motion also.
20. The air between the two walls of the thermoflask is
12. Glass transmits about 50% of heat radiation coming from evacuated. This prevents heat loss due to conduction and
a hot source like the sun but does not allow the radiation convection.
from moderately hot bodies to pass through it. Due to
The loss of heat due to radiation is minimised by
this, when a car is left in the sun, heat radiation from the
silvering the inside surface of the double wall. As, the
sun gets into the car but as the temperature inside the car
loss of heat due to the three processes is minimised and
is moderate, they do not pass back through its windows.
the tea remains hot for a long time.
Hence, inside of the car becomes considerably hotter.
21. As the heat capacities are equal, so m1 C1 = m 2 C2 .
13. Let K 1 and K 2 be the coefficients of thermal conductivity
of the materials and t 1 and t 2 be the times in which ice Let C be the specific heat of the composite body, then
melts in the two vessels. ( m1 + m 2 ) C = m1 C1 + m 2 C2
As the same quantity of ice melts in the two vessels, = m1 C1 + m1 C1 = 2 m1 C1
the quantity of heat flowed into the vessels must be 2 m1 C1 2 m1 C1 2 C1 C2
same. or C= = =
m1 + m 2 m + m C1 C1 + C2
K A ( T1 − T2 ) t 1 K 2 A ( T1 − T2 ) t 2 1 1
Q Q= 1 = C2
x x
22. Let the reading on Celsius scale be θ.
⇒ K 1t 1 = K 2t 2
3
K1 t 2 25 min Reading on TF = θ
∴ = = = 5 :2 2
K 2 t1 10 min
θ T − 32
14. Brass is a good conductor of heat. It quickly conducts
As, = F
100 180
away the heat, so the paper does not reaches its ignition 3
point easily. On the other hand, wood is a bad conductor θ − 32
θ 2
of heat and is unable to conduct away the heat. So, the =
5 9
paper quickly reaches its ignition point and is charged.
On solving , we get
15. The temperature of pockets formed by coals are not
appreciably different from the surface temperature of θ = −106. 67 °C
glowing coals. Temperature on Kelvin’s scale,
However, the pockets formed by coals act as cavities. The TK = −106. 67 + 273.15 = 166. 48 K
radiations from these cavities are black body radiations
TF = 160. 005 ° F
and so have maximum intensity. Hence, the pockets
appear brighter than the glowing coals. 23. For neon triple point, T = 24.57 K
16. It is done so to remove the harmful impure air and to ∴ TC = T (K) − 273.15
replace it by the cool fresh air. The air we breath out is = 24.57 − 273.15 = − 248.58 ° C
warm and so it is lighter. It rises upwards and can go out 9 9
TF = TC + 32 = × ( − 248.58) + 32 = − 415.44 ° F
through the ventilator provided near the roof. 5 5
The cold fresh air from outside enters the room through For carbon dioxide, triple point, T = 216.55 K
the doors and windows. Thus, the convection current is ∴ TC = T(K) − 273.15
set up in the air.
= 216. 55 − 273.15 = − 56. 6 ° C
17. Because the earth is located at a very large distance from
9 9
the sun, hence it receives only a small fraction of the heat TF = TC + 32 = × ( − 56. 6 ) + 32 = − 69.88 ° F
radiation emitted by the sun. Further, due to loss of heat 5 5
from the surface of the earth due to convection and 24. (i) (a) For pressure thermometer A,
radiation also, the earth does not become as hot as the Ttr = 273 K, p tr = 1.250 × 10 5 Pa,
sun.
p = 1.797 × 10 5 Pa
18. When a drop of water falls on a very hot iron, it gets
insulated from the iron by a layer of poor conducting Normal freezing point of sulphur,
water vapour. As the heat is conducted very slowly p
T= × Ttr
through this layer, it takes quite long for the drop to p tr
66 CBSE Term II Physics XI

1.797 × 10 5 × 273 Heat used to boil water,


= = 392.028 K
1.250 × 10 5 Q = mL = 6000 g × 2256 Jg − 1 = 6000 × 2256 J
(b) For pressure thermometer B, 109 × 0.15 × (T1 − 100) × 60
∴ = 6000 × 2256
Ttr = 273 K , p tr = 0.200 × 10 5 Pa, 0.01
6000 × 2256 × 0.01
p = 0.287 × 10 5 Pa or T1 − 100 = = 138 ° C
109 × 0.15 × 60
0.287 × 10 5 × 273 or T1 = 138 + 100 = 238 ° C
∴ T= = 391.75 K
0.200 × 10 5 29. (i) As surface area increases on pouring hot tea in
(ii) The slight difference is due to the fact that, oxygen saucer from the cup and the rate of loss of heat is
and hydrogen do not behave strictly as ideal gases. directly proportional to surface area of the radiating
25. As radius of ball, r0 = 9.0 cm = 0.090 m at 0°C, hence its surface, so the tea will cool faster in the saucer.
4 4 (ii) Temperature of a hot liquid falls exponentially in
volume, V0 = πr03 = × 3.14 × (0.090)3 accordance with Newton’s law of cooling. In other
3 3
words, rate of cooling is directly proportional to the
= 3.05 × 10 −3 m 3
temperature difference between hot liquid and the
Again as, α = 1.2 × 10 −5 K −1 , surroundings. It is due to this reason that, a hot
∴ γ = 3α = 3 × 1.2 × 10 −5 = 3.6 × 10 −5 K −1 liquid cools rapidly in the beginning but slowly
afterwards.
Moreover rise in temperature,
(iii) When outer surface of container is blackened, the
∆T = 90 ° C − 0 ° C = 90 ° C = 90 K
surface becomes good emitter of heat and so the hot
∴ Increase in volume, ∆V = V γ ∆T liquid in it cools faster.
= 3.05 × 10 −3 × 3.6 × 10 −5 × 90 30. Mass of copper block , M = 2.5 kg = 2.5 × 10 3 g
= 9.88 × 10 −6 m 3
Specific heat of copper, c = 0.39 J g − 1 ° C− 1
= 9.88 cm 3
Fall in temperature, ∆T = 500 − 0 = 500 ° C
26. Given, cross-section area, A = 2.0 mm2 = 2 × 10 −6 m 2
Heat lost by copper block = mc ∆T
Change in temperature, ∆T = 30 − 5 = 25 ° C = 2.5 × 10 3 ×0.39 × 500 J
−2
Young’s modulus of steel wire, Y = 2 × 10 Nm 11
Let mass of ice melted = M gram
and coefficient of linear expansion of steel, Heat of fusion of ice, L = 335 Jg −1
α = 1.1 × 10 −5 / °C. Heat gained by ice = ML = M × 335 J
∴ Tension developed in the rod, Q Heat gained = Heat lost
F = YA α∆T ∴ M × 335 = 2.5 × 10 3 × 0.39 × 500
= 2 × 1011 × 2 × 10 −6 × 1.1 × 10 −5 × 25 2 . 5 × 10 3 × 0. 39 × 500
or M=
F = 110 N 335
27. As the brass scale of a barometer gives correct reading at = 1455.2 g = 1.455 kg
T1 = 0 ° C, hence at temperature T2 = 27 ° C, the scale will 31. Here, A = 6 × side 2 = 6 × 30 × 30
expand and will not give correct reading. = 5400 cm 2 = 0.54 m 2
In such a case, true value
x = 5 cm = 0.05 m, t = 6 h = 6 × 3600 s
= observed scale reading × (1 + α∆T )]
T1 − T2 = 45 − 0 = 45 ° C,
∴ True pressure = 75.00 cm × [1 + 2.0 × 10 −5 × (27 − 0)]
K = 0.01 Js −1 m −1 ° C−1
= 75 × (1 + 2.0 × 10 −5 × 27)
L = 335 × 10 3 J kg − 1
= 75.00 (1 + 54 × 10 −5 ) cm
= 75.04 cm Total heat entering the box through all the six faces,
28. Here, A = 0.15 m 2 , x = 1.0 cm = 0.01 m, KA ( T1 − T2 ) t
Q=
K = 109 Js − 1 m − 1 ° C− 1 , L = 2256 Jg − 1 , x
0.01 × 0.54 ×45 × 6 × 3600
T2 = 100 ° C, t = 1 min = 60 s = = 104976 J
0.05
Let T1 be the temperature of the part of the flame in Let m kg of ice melt due to this heat. Then,
contact with boiler, then amount of heat that flows into Q = mL
water in 1 min. Q 104976 J
KA ( T1 − T2 ) t 109 × 0.15 × (T1 −100 ) × 60 or m= = = 0.313 kg
Q= = J L 336 × 10 3 J kg − 1
x 0. 01
Mass of water boiled per min = 6 kg = 6000 g Mass of ice left after six hours = 4 − 0.313 = 3.687 kg
CBSE Term II Physics XI 67

32. It is given that conductivities K 1 and K 2 are in series, so 35. Given, diameter of the hole, d1 = 4.24 cm
rate of flow of heat energy is same. But the sum of the Initial temperature, T1 = 27 + 273 = 300 K
difference in temperature is the difference across their Final temperature, T2 = 227 + 273 = 500 K
free ends.
Q Coefficient of linear expansion, α = 1.70 × 10 −5 / ° C
θ1 K1 K2 θ1 Coefficient of superficial expansion,
β = 2α = 3.40 × 10 −5 / ° C
l1 l2
πd12
Initial area of hole at 27°C, A1 = πr 2 =
∴ (θ1 − θ ) + (θ − θ 2 ) = (θ1 − θ 2 ) 4
θ l1 θ l2 θ (l + l ) π
i.e. ⋅ + = ⋅ 1 2 = (4.24)2 = 4.494 π cm 2
t K 1 A t K 2 A t K eq A 4
l1 l l +l Area of hole at 227°C, A 2 = A1 (1 + β ∆t )
⇒ + 2 = 1 2
K1 K 2 K eq = 4. 494 π [1 + 3. 40 × 10 −5 × (227 − 27 )]
l1 + l 2 = 4. 494 π (1 + 3. 40 × 10 −5 × 200 )
∴ K eq =
 l1 l  = 4.495 π × 1.0068 = 4. 525 π cm 2
 + 2
K1 K 2  πd 22
If diameter of hole becomes d 2 at 227°C, then A 2 =
33. When T = 273 K,R = 101.6 Ω 4
∴ 101. 6 = R 0 [1 + 5 × 10 −3 (273 − T0 )] …(i) πd 22
or 4. 525 π = ⇒ d 22 = 4.525 × 4
Given, T = 600. 5 K , R = 165. 5 Ω 4
∴ 165. 5 = R 0 [1 + 5 × 10 −3 ( 600. 5 − T0 )] …(ii) ⇒ d 2 = 4.2544 cm
∴ Change in diameter, ∆d = d 2 − d1 = 4.2544 − 4.24
Dividing Eq. (ii) by Eq. (i), we get
= 0.0144 cm
165.5 1 + 5 ×10 −3 (600.5 − T0 )
= = 1.44 × 10 −2 cm
101.0 1 + 5 × 10 −5 (273 − T0 )
36. Consider a solid in the form of a rectangular
On solving, we get T0 = − 49. 3 K parallelopiped of sides a , b and c respectively, so that its
Substituting in Eq. (i), we get volume V = abc.
101. 6 = R 0 [1 + 5 × 10 −3 (273 + 49. 3 )] If the solid is heated, so that its temperature rises by ∆T,
then increase in its sides will be
101.6
or R0 = = 38. 9 Ω ∆a = a ⋅ α ⋅ ∆T , ∆b = b ⋅ α ⋅ ∆T and ∆c = c ⋅α ⋅ ∆T
1 + 5 × 10 −3 × 322.3
or a ′ = a + ∆a = a (1 + α ⋅ ∆T )
For R = 123. 4 Ω , we have
b ′ = b + ∆b = b (1 + α ⋅ ∆T )
123. 4 = 38. 9 [1 + 5 × 10 −3 ( T + 49. 3 )] and c′ = c + ∆c = c(1 + a ⋅ ∆T )
On solving, we get T = 384.8 K Q New volume, V ′ = V + ∆V = a ′b ′c′
34. For brass rod, = abc(1 + α ⋅ ∆T )3
l = 50 cm, t 1 = 40 ° C , t 2 = 250 ° C, ∴ Increase in volume,
α = 2.0 × 10 −5 ° C−1 ∆V = V ′ − V = [ abc(1 + α ⋅ ∆T )3 − abc]
Change in length of brass rod is ∴ Coefficient of volume expansion,
∆l = αl ( t 2 − t 1 ) ∆V abc(1 + α ⋅ ∆T )3 − abc
γ= =
= 2.0 × 10 −5 × 50 × (250 − 40) = 0.21 cm V ⋅ ∆T abc ⋅ ∆T
For steel rod, l = 50 cm, t 1 = 40 ° C, t 2 = 250 ° C, (1 + α ⋅ ∆T ) − 1
3
∴ γ=
α = 1.2 × 10 −5 ° C−1 ∆T
Change in length of steel rod is (1 + 3α ⋅ ∆T + 3α 2 ⋅ ∆T 2 + α 3 ⋅ ∆T 3 ) − 1
=
∆l ′ = αl ( t 2 − t 1 ) ∆T
= 1.2 × 10 −5 × 50 × (250 − 40) = 0.13 cm = 3α + 3α 2 ∆T + α 3 ⋅ ∆T 2 .
Change in length of the combined rod at 250°C However, as α has an extremely small value for
= ∆l + ∆l ′ = 0.21 + 0.13 = 0.34 cm solids, hence terms containing higher powers of α may be
As the rods expand freely, so no thermal stress is neglected. Therefore, we obtain the relation γ = 3α.
developed at the junction. i.e. Coefficient of volume expansion of a solid is three
times of its coefficient of linear expansion.
68 CBSE Term II Physics XI

37. (i) The melting point of ice as well as the boiling point When the body is placed in the open at temperature
of water change with change in pressure. The T0 , the heat radiated/second/unit area,
presence of impurities also changes the melting and E ′ = σ ( T4 − T04 ).
boiling points. However, the triple point of water has
Clearly, E ′ < E. So, the optical pyrometer gives
a unique temperature and is independent of external
lower value for the temperature in the open.
factors.
(iv) Heat radiated out by the earth is reflected back by
(ii) The other fixed point on Kelvin scale is absolute
the atmosphere. In the absence of atmosphere, at
zero, which is the temperature at which the volume night all heat would escape from the earth’s surface
and pressure of any gas become zero. and thereby the earth’s surface would be
(iii) As the triple point of water on Celsius is 0.01°C (and inhospitably cold. Also, atmosphere helps in
not 0°C) and on Kelvin scale 273.16 and the size of maintaining the temperature through convection
degree on the two scales is same, so current.
Tc − 0. 01 = T − 273.16 (v) Though steam and boiling water are at the same
∴ Tc = T − 273.15 temperature but each unit mass of steam contains a
(iv) One degree on Fahrenheit scale larger amount of additional heat called the latent
180 9 heat. e.g. Each gram of steam has 540 cal of more
= = divisions on Celsius scale. heat than each gram of boiling water. Hence, steam
100 5
But one Celsius scale division is equal to one loses more heat than boiling water.
division on Kelvin scale. 40.
∴ Triple point on Kelvin scale (whose size of a Conduction Convection Radiation
degree is equal to that of the Fahrenheit scale) It is the transfer It is the transfer It is the transfer
= 273.16 × 9 / 5 = 491.69 of heat by direct of heat by the of heat by
physical contact. motion of a fluid. electromagnetic
38. Mass of child, M = 30 kg = 30 × 10 3 g
waves.
5 5
Fall in temperature, ∆T = 101 − 98 = 3 ° F = 3 × = ° C It is due to It is due to It occurs from all
9 3 temperature difference in bodies at
Specific heat of human body, difference. Heat density. Heat temperatures
c = specific heat of water = 1 cal g − 1 ° C− 1 flows from high flows from low above 0 K.
temperature density region to
Heat lost by child in the form of evaporation of sweat,
region to low high density
5
Q = Mc ∆T = 30 × 10 3 × 1 × temperature region.
3 region.
= 50000 cal It occurs in solids It occurs in fluids It can take place
If M ′ gram of sweat evaporates from the body of the through by actual flow of at large distances
child, then heat gained by sweat, molecular matter. and does not heat
collisions, without the intervening
Q = M ′L = M ′ × 580 cal [QL = 580 calg − 1 ]
actual flow of medium.
Q Heat gained = Heat lost matter.
50000 It is a slow It is also a slow It propagates at
M ′ × 580 = 50000 ⇒ M ′ = = 86.2 g
580 process. process. the speed of light.
Time taken by sweat to evaporate = 20 min It does not obey It does not obey It obeys the laws
86.2 the laws of the laws of of reflection and
∴ Rate of evaporation of sweat = = 4.31 g min− 1 reflection and reflection and refraction.
20
refraction. refraction.
39. (i) A body with large reflectivity is a poor absorber of
heat. According to Kirchhoff ’s law, a poor absorber 41. Given, L 1 = L 2 = L = 0.1 m
of heat is a poor emitter. Hence, a body with large A1 = A 2 = A = 0.02 m2
reflectivity is a poor emitter.
K 1 = 79 Wm −1K −1 , K 2 = 109 Wm −1K −1
(ii) Brass is a good conductor of heat. When a brass
tumbler is touched, heat quickly flows from human T1 = 373 K and T2 = 273 K
body to tumbler. Consequently, the tumbler appears At steady state, heat transferred from each section of
colder. Wood is a bad conductor, so, heat does not thermal conductor is same
flow from the human body to the tray in this case. i.e. H1 = H 2 = H
Thus, it appears comparatively hotter. K 1 A1 ( T1 − T0 ) K 2 A 2 ( T0 − T2 )
(iii) Let T be the temperature of the hot iron in the ⇒ = …(i)
L1 L2
furnace.
Heat radiated per second per unit area, E = σT 4 For A1 = A 2 = A and L 1 = L 2 = L , Eq. (i) becomes
CBSE Term II Physics XI 69

K 1 ( T1 − T0 ) = K 2 ( T0 − T2 ) heat is proportional to the temperature difference


K T + K 2 T2 between the body and the surroundings.
⇒ T0 = 1 1 T − T2
K1 + K 2 mc 1 = K ( T − T0 )
t
Therefore, heat current through each bar
T + T2
K A ( T1 − T0 ) K 2 A ( T0 − T2 ) where, T = 1 = average of the initial and final
H= 1 = 2
L L temperatures of the body and T0 is the temperature
A( T1 − T2 ) of the surroundings.
=
 1 1  Given, T1 = 80 ° C, T2 = 50 °C, T0 = 20 ° C,
L + 
K1 K 2  t = 5 min = 300 s
K 1K 2 A ( T1 − T2 ) T + T2 80 + 50
= …(ii) T= 1 = = 65 ° C
L (K 1 + K 2 ) 2 2
80 − 50
Now, heat current through composite bar of length ∴ mc = K ( 65 − 20 ) …(i)
L 1 + L 2 = 2L and equivalent thermal conductivity K ′, 300
can be given by If the liquid takes t seconds to cool from 60°C to
K ′ A ( T1 − T2 ) 30°C, then
H′ = =H …(iii) 60 + 30
2L T= = 45 ° C
From Eqs. (ii) and (iii), we get 2
60 − 30
2K 1K 2 ∴ mc = K ( 45 − 20 ) …(ii)
⇒ K′ = t
K1 + K 2
Dividing Eq. (i) by Eq. (ii), we get
(i) So, the temperature of the junction of two bars is
30 t 45
K T + K 2 T2 × =
T0 = 1 1 300 30 25
K1 + K 2
45
79Wm −1K −1 × 373 K + 109Wm −1K −1 × 273 K or t= × 300 = 540 s = 9 min
= 25
79 Wm −1K −1 + 109Wm −1K −1 43. (i) The solid, liquid and vapour phases of CO2 coexist
= 315 K in equilibrium at its triple point O for which
(ii) Equivalent thermal conductivity p tr = 5.11 atm and Ttr = − 56.6 ° C
2K 1K 2 (ii) The vaporisation curve I and fusion curve II show
K′ =
K1 + K 2 that, both the boiling point and fusion point of CO2
decrease with decrease of pressure.
2 × 79 Wm −1K −1 × 109 Wm −1K −1
= (iii) For CO2 , p c = 73.0 atm and Tc = 31.1 ° C.
79 Wm −1K −1 + 109 Wm −1K −1 Above its critical temperature, CO2 gas cannot be
= 91.6 Wm −1K −1 liquefied, however large pressure may be applied.
(iii) Heat current through the composite bar (iv) (a) − 70 ° C under 1 atm, this point lies in vapour
K ′ A ( T1 − T2 ) region.
H′ = H =
2L Therefore, at −70 ° C under 1 atm, CO2 is vapour.
91.6 Wm −1K −1 × 0.02 m 2 × (373 − 273) K (b) − 60 ° C under 10 atm. This point lies in solid
= region, therefore CO2 is solid at − 60 ° C under
2 × 0.1 m
10 atm.
= 916.1 W
(c) 15°C under 56 atm. This point lies in liquid
42. According to Newton’s law of cooling, when the region, therefore CO2 is liquid at 15°C under
temperature difference is not large, rate of loss of 56 atm.
70 CBSE Term II Physics XI

CHAPTER 04

Thermodynamics

In this Chapter...
l Thermodynamic System, l Thermodynamic State
Surroundings and Boundary Variables and Equation of State
l Zeroth Law of l Thermodynamic Processes
thermodynamics
l Second Law of Thermodynamics
l Heat, Internal Energy and
Work l Reversible and Irreversible
Processes
l First Law of Thermodynamics
l Carnot Engine

thermodynamic system occupies, through which energy


Thermodynamic System, exchange can or cannot be possible and work could be done on
Surroundings and Boundary the system and by the system.

Thermodynamic system is an assembly of an extremely large Classification


number of particles (atoms or molecules), so that the A thermodynamic system may be
assembly has a certain value of pressure, volume and
temperature. (i) Open System It exchanges both energy and matter
Everything outside the system which has a direct effect on with the surroundings.
the system is called its surroundings. All space in universe (ii) Closed System It exchanges only energy (not matter)
outside the system is surroundings, e.g. environment. with its surroundings.
A system is separated from its surroundings by a boundary. (iii) Isolated System It exchanges neither energy nor
matter with its surroundings.
System
boundary
Thermodynamic Equilibrium
A thermodynamic system is said to be in thermodynamic
Thermodynamic
system
equilibrium, when macroscopic variables (like pressure,
Surroundings volume, temperature, mass, composition etc.) that
characterise the system do not change with time.

A thermodynamic system having boundary and surroundings Thermal Equilibrium


A system boundary is a real or imaginary two-dimensional Two systems are said to be in thermal equilibrium with each
closed surface that encloses the volume or region that a other, if they are at the same temperature.
CBSE Term II Physics XI 71

Zeroth Law of Thermodynamics Heat


This law was formulated by RH Fowler in 1931. The zeroth It is the energy that is exchanged between a system and its
law of thermodynamics states that, environment because of the temperature difference between
them. SI unit of heat is joule.
‘‘If two systems A and B are separately in thermal equilibrium
with a third system C, then A and B are in thermal Change in heat, DQ = ms ( Tf - Ti )
equilibrium with each other.’’ where, m = mass of body, Ti = initial temperature
Tf = final temperature
These conditions are shown in Figs. (a) and (b).
and s = specific heat of material.
C C Work
By work, we mean work done by the system or on the system.
Adiabatic
wall Suppose, a gas is confined in a cylinder with a movable
A B A B
Diathermic piston, if p is pressure on the piston and A is area of piston,
wall then force exerted by the gas on the piston of cylinder,
Adiabatic wall Diathermic wall F = pA
(a) (b) When the piston is pushed outward an infinitesimal distance
dx, then the work done by the gas,
Concept of Temperature dW = Fdx = pA dx = p dV (QAdx = dV)
Zeroth law of thermodynamics implies that, temperature is a Piston
physical quantity which has the same value for all systems
which are in thermal equilibrium with each other.
Area, A dx
Hence, zeroth law says that if systems A and B are separately
in thermal equilibrium with C, then
TA = TC and TB = TC F
Gas
Þ TA = TB
So, systems A and B will also be in thermal equilibrium with and for a finite volume change from Vi to Vf ,
each other. Vf Vf
Work done, W = ò dW = ò pdV
Vi Vi

Heat, Internal Energy Here, p could be variable or constant.


and Work p
i
Internal energy of a system is defined as the total energy
possessed by the system due to molecular motion and
molecular configuration. It is represented by U. The energy f
due to molecular motion is called internal kinetic energy V
UK . The energy due to molecular configuration is called O Vi Vf
internal potential energy U P , then p-V diagram

Internal energy, U = U K + U P The indicator diagram represents the variation of pressure (p)
of the gas with the volume (V).
Internal Energy of an Ideal Gas On indicator diagram, the area bounded by p-Vgraph
Internal energy of an ideal gas is the sum of kinetic energies represents the work done
associated with various random motions (i.e. translational, Work = ò
Vf
pdV = area under p- V graph
rotational and vibrational) of its molecules. Thus, the internal Vi
energy of an ideal gas depends on its temperature. If system expands, Vf > Vi , i. e. W = positive .
Internal Energy of a Real Gas If system contracts, Vf < Vi , i. e. W = negative .
The internal energy of a real gas is the sum of internal kinetic Ways to Change Internal Energy of a System
energy and internal potential energy of the molecules of the
gas. It would obviously depend on both the temperature and Four ways to change the internal energy of a system which
volume of the gas. are as given below
(i) By doing work on the system (DW = - ve).
72 CBSE Term II Physics XI

(ii) If system is doing some work (DW = + ve). Extensive State Variables
(iii) If some heat energy is given to the system (DQ = +ve ).
Extensive state variables indicate the size of the system.
(iv) If some heat energy is taken out from the system
The variable whose values get halved on division are
(DQ = - ve).
extensive. e.g. Internal energy (U ), volume ( V), total mass
(M ) are extensive variables.
First Law of Thermodynamics
First law of thermodynamics is a statement of conservation of Intensive State Variables
energy applied to any system in which energy transfer from or to These are state variables that do not depend on the size
the surroundings is taken into account. of the system. The variables that remain unchanged on
division are intensive.
It states that,]
e.g. Pressure ( p ), temperature ( T ) and density (r ) are
‘‘Heat given to a system is either used in doing external work or it intensive variables.
increases the internal energy of the system or both.’’
i.e. Heat supplied, DQ = DU + DW Thermodynamic Processes
where, When state of a system changes or the state variables
DQ = heat supplied to the system by the surroundings, changes with time, then this process is known as
DW = work done by the system on the surroundings, thermodynamic process.
DU = change in internal energy of the system Quasi-Static Process
and DU depends only on the initial and final states. Quasi-static process is a hypothetical concept.
Practically, processes that are sufficiently slow are
Relation between Cp and CV considered as quasi-static.
(Mayer’s Formula)
Some Important Processes
For an ideal gas, the relation between specific heat capacity at
constant pressure C p and specific heat capacity of constant Some important processes which are as given below
volume C V is C p - C V = R (i) An isothermal process occurs at constant
temperature.
where, R is the universal gas constant. e.g. Freezing of water at 0°C to form ice at 0°C.
This relation is known as Mayer’s formula. (ii) An isobaric process occurs at constant pressure.
The ratio of C p and C V is notified by g. e.g. Boiling of water in an open container.
g = Cp / CV (iii) An isochoric process is one in which volume is
It is also known as heat capacity ratio, then kept constant, meaning that the work done by the
R R system will be zero.
CV = and C p = g e.g. Heat given to a system with fixed walls.
g -1 g -1
(iv) An adiabatic process does not allow transfer of
heat by or to the system.
Thermodynamic State Variables e.g. Rapid compression, like filling of a cycle tube
by a hand pump.
and Equation of State (v) Cyclic and non-cyclic processes In cyclic
Every equilibrium state of a thermodynamic system is process, initial and final states are same, while in
completely described by specific values of some macroscopic non-cyclic process, they are different.
variables and these are called state variables.
e.g. Pressure, volume, temperature and mass. Work Done in an Isothermal Process
The various state variables are not necessarily independent. The For an isothermal process, the equation of state is given
equation which represents the relationship between the state by
variables of a system is called its equation of state. pV = constant
e.g. For an ideal gas, the equation of state is the ideal gas Work done in an isothermal process = Area under p-V
equation pV = mRT graph
For a fixed amount of gas given, there are thus only two æV ö
independent variables, say p and V or T and V. The Wiso = 2.303 mRT log ç f ÷
pressure-volume curve for a fixed temperature is called an è Vi ø
isotherm.
CBSE Term II Physics XI 73

There are two cases for finding the nature of the work Thermodynamic Processes
done. Constant
Process Path Heat/Work/Internal energy
Case I If Vf > Vi , then W > 0. Quantity
Thus, it is an isothermal expansion which Isobaric p a DQ = nC p DT, DW = p DV,
means the gas absorbs heat. DU = nC V DT
Case II If Vf < Vi , then W < 0. Isochoric V d DQ = DU = nC V DT, DW = 0
Thus, it is an isothermal compression, work is Isothermal T b DQ = DW = nRT ln Vf / Vi
done on the gas by the environment and heat is DU = 0
released. Adiabatic pV g c DQ = 0, DW = - DU

Work Done in an Adiabatic Process


In an adiabatic process, pV g = constant Second Law of Thermodynamics
mR ( Ti - Tf ) The principle that put a restriction on the first law of
So, work done in an adiabatic process, W = thermodynamics is known as the second law of thermodynamics.
g -1
The second law of thermodynamics gives a fundamental
Work Done in an Isochoric Process limitation to the efficiency of a heat engine and the coefficient of
p performance of a refrigerator. Two statements of Kelvin-Planck
In case of an isochoric process, = constant and Clausius put restrictions on the possibility of perfect heat
T engine and perfect refrigerator, respectively.
Work done, DW = 0
Kelvin-Planck’s Statement
Work Done in an Isobaric Process No process is possible whose sole result is the absorption of heat
In this process, p = constant from a reservoir and the complete conversion of the heat into
work.
Work done, W = mR ( Tf - Ti )
Clausius Statement
Work Done in a Cyclic Process No process is possible whose sole result is the transfer of heat
The work done in a cyclic process is equal to the area from a colder object to a hotter object.
enclosed by the loop.
l
If the cycle is clockwise, work done is positive and if Reversible Process
the cycle is anti-clockwise, the work done is negative. A process which could be reserved in such a way that the system
l
A process in which the system do not return to its initial and its surrounding returns exactly to their initial states with no
state after undergoing a series of change is known as other changes in the universe is known as reversible process.
non-cyclic process. The conditions for reversibility which are as given below
(i) There must be complete absence of dissipative forces such
p-V Diagram of Thermodynamics as friction, viscosity, electric resistance, etc.
Processes (ii) The direct and reverse processes must take place
p-V diagram to represent various thermodynamic infinitely slowly.
processes are given below (iii) The temperature of the system must not differ
appreciably from its surroundings.
T1
T2
T3
p a Isobaric Irreversible Process
d b Any process which is not reversible exactly is an irreversible
c process.
Isochoric
tic Isotherms Irreversibility arises mainly from two causes which are as given
aba
Adi below
Isotherms
(i) Many processes (like diffusion of gases, decay of organic
Isotherms matter) take the system to non-equilibrium states.
O V (ii) Many processes involve the dissipative forces like
p-V diagram to represent various thermodynamic processes
friction, viscosity, inelasticity, etc.
74 CBSE Term II Physics XI

= Q1
Carnot Engine æV ö
= RT1 log e ç 2 ÷ = Area ABGEA
The reversible engine which operates between two è V1 ø
temperatures of source ( T1 ) and sink ( T2 ) is known as Carnot
heat engine. The designed engine is a theoretical engine Second Stroke (Adiabatic Expansion) (Curve BC)
which is free from all the defects of a practical engine. The cylinder is then placed on the non-conducting stand and
the gas is allowed to expand adiabatically till the temperature
Cylinder with
conducting base
falls from T1 to T2 .
R
W2 = ( T1 - T2 ) = Area BCHGB
( g - 1)

Third Stroke (Isothermal Compression) (Curve CD)


Working Ideal
The cylinder is placed on the sink and the gas is compressed
substance
at constant temperature T2 .
Source Insulated Sink
T1 stand T2 Work done, W3 = heat released by the system
Carnot engine V3
= Q 2 = RT2 log e = Area CDFHC
V4
The main parts of Carnot engine are as given below
Cylinder The cylinder has conducting base and insulating Fourth Stroke (Adiabatic Compression) (Curve DA)
walls. It consists an ideal gas as a working substance. Finally, the cylinder is again placed on non-conducting stand
Insulating and frictionless piston is attached with it. and the compression is continued, so that gas returns to its
Source It is a hot reservoir at a temperature T1 with initial stage.
conducting walls. It has infinite thermal capacity. Any amount R
W4 = ( T1 - T2 ) = Area ADFEA
of heat can be taken from it without changing the temperature. g -1
Sink It is a cold reservoir at temperature T2 . It has infinite
Efficiency of Carnot Cycle (Engine)
thermal capacity, so any amount of heat can be rejected to it
without changing the temperature. The efficiency of engine is defined as the ratio of work done
to the heat supplied. i. e.
Insulating Stand The base of the cylinder could be placed
on the insulating stand, to isolate it completely from the Work done Wnet T
h= = =1 - 2
surroundings. Heat input Q1 T1

Carnot Cycle The efficiency of the Carnot engine depends only on the
temperatures of source and sink.
As the engine works, the working substance of the engine
undergoes a cycle known as Carnot cycle. The efficiency of Carnot engine will be 100%, if the
temperature of the sink is 0 K. As practically, we cannot
The Carnot cycle consists of the following four strokes
p attain a sink at 0 K, so it is not possible to have 100%
( p1, V1, T1) efficiency.
A Q1 ( p2, V2, T1)
B
Isotherma Carnot Theorems
compression

l T1
expansion
According to Carnot theorem,
Adiabatic

Ad ansio
exp

W = Q1 – Q2
(i) A heat engine working between the two given
iab
atic n

temperatures T1 of hot reservoir, i.e. source and T2 of


D cold reservoir, i.e. sink cannot have efficiency more
( p4, V4, T2) Isothermal than that of the Carnot engine.
compress Q2 C ( p3, V3, T2) T3
ion (ii) The efficiency of the Carnot engine is independent of
O E F G H T the nature of working substance.
Various processes in Carnot cycle
Coefficient of Performance of Carnot Engine
First Stroke (Isothermal Expansion) (Curve AE) For Carnot refrigerator,
The cylinder containing one mole of an ideal gas as working T2
substance allowed to expand slowly at the constant Coefficient of performance, b =
T1 - T2
temperature T1 by putting it on the source.
where, T1 = temperature of surrounding
Work done, W1 = heat absorbed by the system
and T2 = temperature of cold body.
CBSE Term II Physics XI 75

Solved Examples
Example 1. The quantities in the following table Example 4. If 70 cal of heat is required to raise the
represent four different paths for same initial and temperature of 2 mol of an ideal gas at constant
final states. Find the values of a, b, c, d, e, f and g. pressure from 30°C to 35°C, calculate
S. No. Q(J) W (J) DU (J) (i) work done by the gas and
Path I – 80 -120 d (ii) increase in internal energy of gas.
Path II 90 c e
(Take, R = 2 cal/mol-K)
Path III a 40 f Sol. (i) In isobaric process, p = constant
b and DW = work done = pDV
Path IV - 40 g
= nR ( DT ) = 2 ´ 2 ´ ( 35 - 30 ) = 20 cal
Sol. For path I, DQ = DW + DU Þ DU = DQ - DW (ii) DU = DQ - DW (from first law)
= - 80 - ( - 120 ) = 40 J = 70 - 20 = 50 cal
As, DU = a state function, so d = e = f = g = 40 J. DQ DQ
Now for path II, DQ = DW + DU Example 5. Find the ratio of and in an isobaric
DU DW
90 = c + 40 Þ c = 50 J Cp
Similarly, a = 80 J, b = 0 process. The ratio of molar specific heats = g.
CV
Example 2. Arrange four paths shown in ascending
order on the basis of (i) change in internal energy, Sol. In an isobaric process, p = constant
\ C = Cp
(ii) work done by the gas and (iii) magnitude of
energy transferred or heat. DQ nCp DT Cp
and = = =g
p DU nCV DT CV
a
DQ DQ
1
b also =
c DW DQ - DU
2 nCp DT Cp g
d = = =
nCp DT - nCV DT Cp - CV g - 1
V
O Example 6. A gaseous mixture enclosed in a vessel
Sol. (i) As, DU depends on initial and final states but not on æ 5ö
path , so for all a, b, c and d paths, DU is same. consists of 1g mole of a gas A ç g = ÷ and some
è 3ø
(ii) Work done is given by area under curve and above
æ 7ö
volume axis so in ascending order, work done is amount of gas B ç g = ÷ at a temperature T. The
d , c , b, a. è 5ø
(iii) By first law, DQ = DU + DW and DU is same for all, gases A and B do not react with each other and are
so DQ is greatest for which DW is greatest. So, assumed to be ideal.
ascending order for DQ is d , c , b, a. Find number of gram moles of the gas B, if g for the
Example 3. Three moles of an ideal gas kept at constant æ 19 ö
gaseous mixture is ç ÷ .
temperature of 300 K are compressed from a è 13 ø
volume of 4 L to 1 L. Calculate the work done in Sol. For an ideal gas,
the process. (Take, R = 8.31 J/mol-K) Cp
= g and Cp - CV = R
Sol. Given, m = 3, T = 300 K, Vi = 4 L, Vf = 1 L, CV
R = 8. 31 J/mol-K So, combining above equations we get
Work done in isothermal process is given by R
CV =
V g -1
W = 2 . 303 mRT log f
Vi R 3
( CV )A = = R
1 5
= 2.303 ´ 3 ´ 8.31 ´ 300 log = -1.037 ´ 10 4 J -1 2
4 2
76 CBSE Term II Physics XI

and ( CV )B =
5
R = 3 ´ 8.31 ´ 300 ´ log e 5
2 = 12.03 kJ
R 13 As for isothermal process,
and ( CV )mix = = R
19
-1 6 DT = 0
13 Þ DU = 0
Now, from conservation of energy and by first law, DQ = DW = 12.03 kJ
U mix = U A + U B For isochoric process BC,
or DU mix = DU A + DU B V = constant Þ DW = 0
Þ (m A + m B ) ( CV )mix DT = m A ( CV )A DT + m B( CV )B DT DU = mCV DT = 3 CV DT
m ( C ) +m B( CV )B p p
Þ ( CV )mix = A V A Now, for process BC, B = C
mA + mB TB TC
3 5 p/ 5 p
1 ´ R + mB ´ R Þ =
13 2 2 300 TC
Þ R=
6 1 + mB Þ TC = 1500 K
Þ 13 + 13 m B = 9 + 15 m B Þ DTBC = TC - TB = 1500 - 300 = 1200 K
Þ m B = 2 g- mol So, DU = mCV DT = 3 ´ CV ´ 1200 = 3. 6 CV kJ
Example 7. 3 moles of an ideal gas at 300 K is \ ( DQ)isochoric = 3. 6 CV kJ
isothermally expanded to five times its initial Now, given that, DQprocess ABC = 83.14 kJ
volume and heated at this constant volume, so that Þ DQABC = DQisothermal + DQisochoric
the pressure is raised to its initial value before Þ 83.14 = 12.03+ 3.6 CV Þ CV = 19.75 J/mol-K
expansion.
and Cp = CV + R = 19.75+ 8.31
In the whole process, heat supplied to the gas is
83.14 kJ. Calculate the ratio of C p / C V for the gas. = 28.05 J/mol -K
Cp 28. 05
(Take, log e 5 = 1. 61 and R = 8.31 J mol –1 K –1 ) \ g= = = 1. 42
CV 19.75
Sol. Process is shown in the indicator diagram Example 8. A scientist claims to have developed 60%
p efficient engine, while working between 27°C and
A 327°C. Does he claim right?
p C
Sol. Given, sink temperature, T2 = 273 + 27 = 300 K
(low temperature happens to be sink temperature)
and source temperature, T1 = 327 + 273 = 600 K
p/5 B
So, within this temperature difference, the maximum
V T 300
O V 5V
possible efficiency, hmax = 1 - 2 = 1 - = 0.5 = 50%
T1 600
For isothermal process AB,
But the scientist is claiming more efficiency. So, as per
DW = mRT log e ( Vf / Vi ) Carnot theorem, his claim is not correct.
CBSE Term II Physics XI 77

Chapter
Practice
PART 1 Which amongst the given statements is (are)
correct?
Objective Questions (a) Only I
(c) Both I and II
(b) Only II
(d) Neither I nor II
l
Multiple Choice Questions 4. Choose the correct option.
(a) Zeroth law gives the concept of temperature
1. If a system is in thermodynamic equilibrium with (b) Temperature measures the hotness of the body
its surroundings, it means (c) Heat flows from higher temperature to lower
(a) temperature of system and surroundings must be temperature until thermal equilibrium is attained
same (d) All of the above
(b) pressure, volume and temperature of system and
surroundings must be same 5. Heat capacity of a substance depends on
(c) pressure, volume and temperature of system and (a) the mass of the substance
surroundings must be different (b) the temperature of the substance
(d) None of the above (c) Both (a) and (b)
(d) Neither (a) nor (b)
2. If two systems are in thermal equilibrium with each
other, it means their 6. An ideal gas is having molar specific heat capacity
3
(a) masses are equal, temperatures may be unequal at constant volume equal to R, then the molar
(b) temperatures are equal 2
(c) masses and temperatures are equal
specific heat capacities at constant pressure will be
1 5 7 9
(d) None of the above (a) R (b) R (c) R (d) R
2 2 2 2
3. A gas is enclosed in a box with a partition as shown
in the Fig. (a). 7. An ideal gas undergoes cyclic process ABCDA as
shown in given p-V diagram. The amount of work
done by the gas is
p
D
2p0 C
(a)

p0 B
A

V
V0 3V0
(b)
(a) 6 p 0V0 (b) −2 p 0V0 (c) +2 p 0V0 (d) +4 p 0V0
If this partition in the box is suddenly removed [as
shown in Fig. (b)] leading to free expansion of the 8. According to second law of thermodynamics,
gas, then (a) a heat engine cannot have efficiency equal to 1
(b) a refrigerator (or heat pump) could have infinite value
I. it is not an equilibrium state.
of coefficient of performance
II. state variables can be used to describe the (c) a heat engine can convert heat fully in to work
situation. (d) heat can flow from cold to hot body
78 CBSE Term II Physics XI

9. Reversibility is not possible because 16. Assertion In adiabatic process, work is


(a) resistive forces are present everywhere independent of the path.
(b) every process around us is Quasi-static
Reason In adiabatic process, work done is equal to
(c) gases are viscous
negative of change in internal energy.
(d) gases have density
10. Carnot engine is a 17. Assertion First law of thermodynamics is based on
energy conservation.
(a) irreversible (b) petrol engine
(c) reversible (d) diesel engine Reason Second law of thermodynamics put
limitations on first law.
11. Which condition is true for an ideal Carnot engine
which have 100% efficiency? 18. Assertion Rotating blade in a liquid stops after a
(a) Temperature of source T1 = 0K certain time is a irreversible process.
(b) Temperature of sink, T 2 = 0K Reason Irreversibility arises as the system attains a
(c) Temperature of source, T1 = 1. 0 K non-equilibrium state.
(d) None of the above 19. Assertion Reversible Carnot engine with one
12. According to Carnot, which type of engine working particular substance cannot be more efficient than
between two temperatures T1 and T2 have the one using another substance.
maximum efficiency? Reason Efficiency of Carnot engine = T2 /T1 .
(a) Reversible engine
(b) Irreversible engine
l
Case Based MCQs
(c) External combustion engine
20. Direction Read the following passage and answer
(d) Diesel engine
the questions that follows
13. The efficiency of a Carnot’s engine working Equilibrium State
between steam point and ice point is
An equilibrium state of a gas is completely
(a) 26. 81% (b) 29%
specified by the values of pressure, volume,
(c) 30% (d)10%
temperature and mass (and composition, if there is
a mixture of gases). A gas allowed to expand freely
l
Assertion-Reasoning MCQs
against vacuum is not an equilibrium state. During
Direction (Q. Nos. 14-19) Each of these questions the rapid expansion, pressure of the gas may not be
contains two statements Assertion (A) and Reason (R). uniform throughout. Similarly, a mixture of gases
Each of these questions also has four alternative undergoing an explosive chemical reaction is not an
choices, any one of which is the correct answer. You equilibrium state, again its temperature and
have to select one of the codes (a), (b), (c) and (d) given pressure are not uniform. Here, a system is as
below. shown below
(a) Both A and R are true and R is the correct Diathermic walls
explanation of A and piston

(b) Both A and R are true, but R is not the correct


explanation of A T
T Surroundings
(c) A is true, but R is false p

(d) A is false and R is also false


An ideal gas initially at thermodynamic
14. Assertion Specific heat capacity and molar specific and mechanical equilibrium
heat capacity both have same units.
Reason Specific heat capacity and molar specific (i) If the external pressure is reduced suddenly, then
heat capacity both depend on mass. (a) the gas will expand suddenly which has
non-equilibrium intermediate stages
15. Assertion In isothermal process, whole of the heat (b) the piston will accelerate outwards
energy supplied to the body is converted into (c) Both (a) and (b)
internal energy. (d) Neither (a) nor (b)
Reason According to the first law of (ii) If a finite temperature difference exists between
thermodynamics, ∆Q = ∆U − ∆W. system and surroundings,
CBSE Term II Physics XI 79

(a) there will be a rapid exchange of heat between system 5. The initial state of a certain gas is (p i , Vi , Ti ). It
and surroundings with non-equilibrium intermediate
stages undergoes expansion till its volume becomes Vf .
(b) after sometime the temperature and pressure of Consider the following two cases
surroundings and system will become same (i) The expansion takes place at constant temperature.
(c) Both (a) and (b)
(ii) The expansion takes place at constant pressure.
(d) Neither (a) nor (b)
Plot the p-V diagram for each case. In which of the
(iii) Choose the correct option. two cases, is the work done by the gas more?
(a) Thermodynamics is a microscopic science [NCERT Exemplar]
(b) Thermodynamics results based on molecular
description of the molecules of the gas 6. A cylinder with a movable piston contains 3 moles
(c) p , V and T are state variables of hydrogen at standard temperature and pressure.
(d) Q is a state variable The walls of the cylinder are made of a heat
insulator and the piston is insulated by having a
(iv) The internal energy of the given system could be
pile of sand on it. By what factor, does the
changed
pressure of the gas increase if the gas is
(a) by extracting heat from the system
compressed to half its original volume? [NCERT]
(b) by doing work done on the system
(c) if system do some work on the surroundings 7. State the process in which air pressure in a car tyre
(d) All of the above increase during driving. Explain, why.
(v) A mixture of gases undergoing explosive chemical 8. Two samples of a gas initially at the same
reaction, temperature and pressure are compressed from a
(a) is in equilibrium state during explosion volume V to a volume V/2. One sample is
(b) have constant temperature and pressure values during compressed isothermally and the other
explosion adiabatically. In which sample is the pressure
(c) finally the gas will attained equilibrium state with its greater? Justify.
surroundings
(d) Both (b) and (c) 9. A system goes from P to Q by two different paths in
the p-V diagram as shown in figure. Heat given to
the system in path 1 is 1000 J. The work done by
PART 2 the system along path 1 is more than path 2 by 100 J.
What is the heat exchanged by the system in
Subjective Questions path 2?
p
[NCERT Exemplar]

1 Q
l
Short Answer (SA) Type Questions 2
P
1. Two bodies at different temperatures T1 and T2 are V
O
brought in contact. Under what condition, they
settle to mean temperature (after they attain 10. Consider a Carnot cycle operating between
equilibrium) ? [NCERT Exemplar] T1 = 500 K and T2 = 300 K producing 1 kJ of
2. A geyser heats water flowing at the rate of mechanical work per cycle. Find the heat
3.0 L/min from 27°C to 77°C. If the geyser operates transferred to the engine by the reservoirs.
[NCERT Exemplar]
on a gas burner, what is the rate of consumption of
the fuel if its heat of combustion is 4.0 × 10 4 J/g?. 11. A Carnot engine is operating between 600 K and
[NCERT] 200 K. Consider that, the actual energy produced is
2 kJ per kilocalorie of heat absorbed. Compare the
3. When ice melts, then change in internal energy is real efficiency with the efficiency of Carnot engine.
greater than the heat supplied, why?
12. A Carnot engine takes in a thousand kilocalories of
4. What amount of heat must be supplied to heat from a reservoir at 827°C and exhausts it to a
−2
2.0 × 10 kg of nitrogen (at room temperature) to sink at 27°C.
raise its temperature by 45°C at constant pressure?
(i) How much work does it perform?
(Molecular mass of N 2 = 28, R = 8.3 J mol −1 K −1 )
[NCERT] (ii) What is the efficiency of the engine?
80 CBSE Term II Physics XI

13. (i) Why does a gas have two molar specific heats? 17. Draw indicator diagram for an adiabatic process.
(ii) Which one is greater and why? Find the slope of an adiabat curve at any point.
(iii) What is the difference between the two molar 18. Explain, why.
specific heats and what is their ratio? (i) 500 J of work is done on a gas to reduce its
14. Consider one mole of perfect gas in a cylinder of volume by compression adiabatically.
unit cross-section with a piston attached What is the change in internal energy of the gas?
(as shown in figure). A spring (spring constant k) is (ii) The coolant in a chemical or a nuclear plant, i.e.
attached (unstretched length L) to the piston and the liquid used to prevent the different parts of a
to the bottom of the cylinder. Initially, the spring plant from getting too hot should have high
is unstretched and the gas is in equilibrium. specific heat.
Atmospheric (iii) The climate of a harbour town is more temperate
pressure = pa
than that of a town in a desert at the same
latitude. [NCERT]
19. In changing the state of a gas adiabatically from an
R
equilibrium state A to another equilibrium state B,
an amount of work equal to 22.3 J is done on the
system. If the gas is taken from state A to B via a
process in which the net heat absorbed by the
A certain amount of heat Q is supplied to the gas system is 9.35 cal, how much is the net work done
causing an increase of volume from V0 to V1 . by the system in the latter case? (Take, 1 cal
= 4.19 J). [NCERT]
(i) What is the initial pressure of the system?
(ii) What is the final pressure of the system? 5 
20. A monoatomic ideal gas  γ =  initially at 17°C is
(iii) Using the first law of thermodynamics, write  3
down a relation between Q , p a , V, V0 and k. suddenly compressed to one-eight of its original
volume.
15. Consider a p-V diagram in which the path
followed by one mole of perfect gas in a Find the final temperature after compression.
cylindrical container is shown. 21. A thermodynamic system is taken from an original
p state D to an intermediate state E by the linear
1( p1 , V1 ,T1 ) process shown in figure.
pV1/2= constant
D
600
Pressure, p (N/m2)

2( p2 , V2 ,T2 )

O V 300 F E
V1 V2

(i) Find the work done, when the gas is taken from
state 1 to state 2. 2.0 5.0
Volume, V (m3)
(ii) What is the ratio of temperatures T1 / T2 , if
V2 = 2 V1 ? Its volume is then reduced to the original value from
(iii) Given that, the internal energy for one mole of E to F by an isobaric process. Calculate the total work
gas at temperature T is (3/2) RT, find the heat done by the gas from D to E to F. [NCERT]
supplied to the gas when it is taken from states 22. Find out whether these phenomena are reversible or
1 to 2, with V2 = 2 V1 . [NCERT Exemplar]
not.
16. Two identical samples of a gas are at a pressure p1 (i) Waterfall and (ii) Rusting of iron.
and volume V1 . The two samples are allowed to
23. An ideal gas heat engine operates in Carnot cycle
expand, so as to acquire a volume V2 . But one
between 227°C and 127°C. It absorbs 6 × 10 4 cal of
sample expands isothermally while the other
heat at higher temperature. Calculate the amount of
expands adiabatically. In which case, more work is
heat converted into work.
done and why?
CBSE Term II Physics XI 81

24. An ideal Carnot engine takes heat from a source at case, gas is expanded isothermally while in second
317°C, does some external work and delivers the case, gas is expanded adiabatically.
remaining energy to a heat sink at 117°C. If 500 kcal (i) In which case, is the pressure is greater and why?
of heat is taken from the source, how much (ii) Which work done is more and why?
(i) work is done and
30. A cycle followed by a machine (made of one mole of
(ii) heat is delivered to the sink? perfect gas in a cylinder with a piston) is shown in
25. Temperatures of the hot and cold reservoirs of a figure
p
Carnot engine is raised by equal amounts. How the
B C
efficiency of the Carnot engine affected?
l
Long Answer (LA) Type Questions A D
26. A cycle followed by an engine (made of one mole of O V
an ideal gas in a cylinder with a piston) is shown in VA =VB VC =VD
figure. Find heat exchanged by the engine with the
surroundings for each section of the cycle. A to B = volume constant,
[C V = ( 3/ 2 )R ] B to C = adiabatic,
p
B
C to D = volume constant
C
and D to A = adiabatic,
D (VC = VD = 2 VA = 2 VB )
A
(i) In which part of the cycle, heat is supplied to the
O V machine from outside?
(i) AB = constant volume (ii) In which part of the cycle, heat is being given to
the surrounding by the machine?
(ii) BC = constant pressure
(iii) What is the work done by the machine in one
(iii) CD = adiabatic
cycle? Write your answer in term of p A , pB , VA .
(iv) DA = constant pressure [NCERT Exemplar]
(iv) What is the efficiency of the machine?
27. In the given p-V diagrams, 5 3
p
(Take, γ = for the gas and C V = R for one mole)
3 2
I [NCERT Exemplar]
II 31. What is a cyclic process? What is change in
III
internal energy of the system in a cyclic process?
IV
V In changing the state of a gas adiabatically from an
O equilibrium states A to B, an amount of 40.5 J of
Find which curve represents work is done on the system. If the gas is taken from
(i) isothermal process, states A to B via a process in which net heat
(ii) adiabatic process for absorbed by the system is 12.6 cal. How much is
the net work done by the system in the later case?
(a) monoatomic, (Take, 1 cal = 4.19 J) [NCERT]
(b) diatomic and
(c) triatomic gas. [NCERT Exemplar]
32. Figure shows a process PQRP performed on an
ideal gas. Find the net heat given to the system
28. Consider that an ideal gas (n moles) is expanding in during the process.
a process given by p = f ( V ), which passes through a V
point ( V0 , p 0 ). Show that the gas is absorbing heat
at ( p 0 , V0 ), if the slope of the curve p = f ( V ) is V2 R
larger than the slope of the adiabat passing through
( p 0 , V0 ). V1 P Q

29. A gas is expanded twice in a way, so that the T


volume becomes twice the initial value. In first T1 T2
82 CBSE Term II Physics XI

33. Two cylinders A and B of equal capacity are (i) When the outputs of the two engines are equal.
connected to each other via a stopcock. A contains a (ii) When the efficiencies of the two engines are
gas at standard temperature and pressure and B is equal.
completely evacuated. The entire system in
thermally insulated. The stopcock is suddenly l
Case Based Questions
opened.
39. Direction Read the following passage and answer
Answer the following. the questions that follows
(i) What is final pressure of the gas in A and B? Carnot Engine
(ii) What is the change in internal energy of the gas? Carnot engine is an ideal heat engine proposed by
(iii) What is the change in temperature of the gas? Sadi Carnot in 1824. The reversible engine which
(iv) Do the intermediate states of the system (before operates between two temperatures of source ( T1 )
settling to the final equilibrirum state) lie on its and sink ( T2 ) is known as Carnot heat engine.
p-V-T surface? [NCERT]
The designed engine is a theoretical engine which
34. Explain with the suitable example that a reversible is free from all the defects of a practical engine.
process must be carried slowly and a fast process is This engine cannot be realised in actual practice,
necessarily irreversible. however this can be taken as a standard against
35. The efficiency of a Carnot engine is 1 /2 . If the sink which the performance of an actual engine can be
judged.
temperature is reduced by 100°C, then engine Cylinder with
efficiency becomes 2 / 3. Find conducting base
(i) sink temperature,
(ii) source temperature and
(iii) explain why a Carnot engine cannot have 100%
efficiency. Working Ideal
substance
36. Explain the construction and various operations for
Carnot heat engine working between two Source Insulated Sink
T1 stand T2
temperatures.
37. A Carnot cycle is performed by 1 mole of air Carnot engine
( γ = 1. 4) initially at 327 °C. Each stage represents a The coefficient of performance is defined as the
compression or expansion in the ratio of 1 : 6.. ratio of the amount of heat removed (Q 2 ) per cycle
Calculate to the mechanical work done on it. It is denoted
(i) the lowest temperature, by β.
(ii) network done during each cycle and (i) A Carnot refrigerator has to transfer an average of
263 J of heat per second from temperature −10° C
(iii) efficiency of the engine. to 25° C. Calculate the average power consumed,
(Take, R = 8.31 J/mol-K) assuming no energy losses in the process.
38. Two Carnot engines A and B are operated in series. (ii) A Carnot engine with ideal gas is used for
The first one A receives heat at 800 K and rejects to freezing water, which is at 0°C. The engine is
a reservoir at temperature T K . The second engine B operated by a 600 W electric motor having an
receives the heat rejected by the first engine and in efficiency of 50%. Find time to freeze 25 kg of
turn rejects to a heat reservoir at 300 K. Calculate the water. (Take, 25°C and 0°C, the source & sink
temperature T K for the following cases temperatures and latent heat of ice
= 333 × 10 3 J kg − 1 )
Chapter Test a
11. Find molar specific heat for the process p =
T
for a

Multiple Choice Questions  3 


monoatomic gas.  Take, a = constant, C V = R
 2 
1. According to the first law of thermodynamics,
(a) any process that involves energy conservation is possible in
[Ans. (7 /2) R]
nature 12. 1 g of water at 100°C is converted into steam of the
(b) ∆Q = ∆U + ∆W same temperature. If the volume of steam is 1551 cm 3 ,
(c) Both (a) and (b) find out the change in internal energy of the water.
(d) Neither (a) nor (b) Latent heat of steam, L = 2256 × 10 3 J/kg. Consider atm
2. Mechanical equivalent of heat is equal to the amount pressure. (Ans. 2099 J)
of 13. 2 g-mole ideal gas is expanded isothermally at 27°C. Its
(a) work done to produce 1 cal heat volume increases three times of initial volume. Find the
(b) a conversion factor between calorie and joule work done and heat absorbed by the gas. (Take,
(c) Both (a) and (b) R = 8.3 J mol−1K −1 ) . × 10 3 cal)
(Ans. 5.48 × 10 3 J and 131
(d) Neither (a) nor (b)
Long Answer Type Questions
3. Choose the state variable from the given options.
(a) Heat (b) Work
14. What is Carnot engine? On what factors, does it
(c) Internal energy (d) All of these
depend? Also, define the efficiency of a Carnot engine.

4. A gas is expanded isothermally from volume V1 to V2 at 15. The figure shows a p-V graph of the thermodynamic
behaviour of an ideal gas. Find out from this graph
a constant temperature T, the work done by the gas in (i) work done by the gas in the process A → B, B → C ,
this expansion is C → D and D → A and (ii) work done by the gas in
V1 V1
(a) µRT ln (b) µRT × complete cycle A → B → C → D → A.
V2 V2
V V
(c) µRT ln 2 (d) µRT × 2
V1 V1 14
A B
p(105 Nm–2)

5. For Carnot engine, which process should be chosen to 12


take the working substance from T1 to T2 or vice-versa? 10
8
(a) Isochoric (b) Isothermal
6
(c) Adiabatic (d) Isobaric
4
Short Answer Type Questions 2 D C
A′ B′
6. Which one of a solid, a liquid or a gas of same mass 1.0 2.0 3.0 4.0 5.0 6.0 7.0
and at the same temperature has greatest internal V(L)
energy? Give reason.
[Ans. (i) 7200 J, − 1200 J and (ii) 6000 J]
7. Why water at the base of a waterfall is slightly warmer
16. Find the work done during the perfectly circular cyclic
than the top?
process as shown in the figure.
8. Two blocks of same metal having masses 5 g and 10 g p
collide against a target with same velocity. If total
energy is used in heating the balls, which will attain p1
higher temperature?
9. Find work done during the cyclic process shown in
indicator diagram. p2
10. Define reversible process. Write two characteristics of V
reversible process. V1 V2

Answers
Multiple Choice Questions For Detailed Solutions
1. (c) 2. (c) 3. (c) 4. (c) 5. (c) Scan the code
84 CBSE Term II Physics XI

EXPLANATIONS
1. (a) If a system is in thermodynamic equilibrium with its 8. (a) Second law puts limitation on the efficiency of a heat
surroundings, it means that its state variables will not engine and on the coefficient of performance of a
change with time, which is only possible when refrigerator. Heat engine cannot have efficiency equal
temperature of system and surroundings is same. to 1 and a refrigerator cannot have infinite value of
2. (b) If two systems are in thermal equilibrium with each coefficient of performance.
other, it means their temperatures must be same. Masses 9. (a) Reversibility is not possible because there are
may be equal or unequal. resistive forces present everywhere in the environment.
3. (a) A gas allowed to expand freely against vacuum is not Also, reversible process are slow process and take infinite
an equilibrium state. As during the rapid expansion, time to complete.
pressure of the gas may not be uniform throughout. 10. (c) Carnot cycle starts from p1 , V1 , T1 and reverses back to
Also, thermodynamic state variables describe equilibrium p1, V1, T1 after doing work (given by area enclosed as
states of the system. Since, the given system is not in shown below).
equilibrium state, thus cannot be described by state Carnot engine is a reversible engine working between
variables. two temperatures.
So, statement I is correct but II is incorrect. T2
4. (d) Heat flow from higher temperature to lower 11. (b) The efficiency of Carnot engine is given by η = 1 −
T1
temperature until thermal equilibrium is attained. So,
heat flows from body A to body B because temperature of where, T1 = temperature of source and T2 = temperature
body A is higher. At thermal equilibrium, TA ′ = TB ′ of sink.
Therefore, the efficiency η will be100%, when
temperature of sink, T2 = 0. Here, all the temperatures
C are in kelvin, so correct option is (b).
12. (a) For reversible engine, according to Carnot, efficiency is
maximum.
Adiabatic wall 13. (a) Given, T1 = (100 + 273 ) = 373 K (steam point)
From zeroth law, when A is in thermal equilibrium with T2 = (0 + 273) = 273 K (ice point)
C and B is in thermal equilibrium with C, then A is in T2 273 100
thermal equilibrium with B. Efficiency, η = 1 − =1− =
Thus, this law gives the concept of temperature T1 373 373
100
and temperature of a body is that parameter which %η= × 100 = 26.81 %
determines the degree of hotness or coldness of a body. 373
5. (c) Heat capacity depends on the mass of the substance ∆Q
14. (d) Specific heat capacity, s = , where m is mass of
and its temperature. m ∆T
Heat consumed by given mass substance. Its SI unit is Jkg −1K −1.
Heat capacity, S =
Temperature raised ∆Q
Molar specific heat capacity, C = , where µ is
If given mass is increased, then heat capacity increases. µ ∆T
3 number of moles. Its SI unit is Jmol −1K −1.
6. (b) Given, CV = R Both of these are constants characteristics of a substance
2
3 5 and are independent of its mass.
Since, Cp − CV = R ⇒ Cp = CV + R = R + R = R Therefore, A is false and R is also false.
2 2
So, molar specific heat capacity at constant pressure is 15. (d) As there is no change in internal energy of the system
5 during an isothermal change. So, the energy taken by the
R. gas is utilised by doing work against external pressure.
2
7. (b) Consider the p-V diagram given in the question. According to first law of thermodynamics, ∆Q = ∆U + ∆W
Work done in the process ABCD As, ∆U = 0 ⇒ ∆Q = ∆W
= Area of rectangle ABCDA Hence, whole heat energy supplied to the body in an
= ( AB ) × BC = ( 3 V0 − V0 ) × (2 p 0 − p 0 ) isothermal process is converted into work done.
= 2 V0 × p 0 = 2 p 0 V0 Therefore, A is false and R is also false.
As the process is going anti-clockwise, hence there is a 16. (a) In adiabatic process, heat transfer to the
net compression in the gas. thermodynamic system is zero, i.e. ∆Q = 0.
So, work done by the gas = − 2 p 0 V0. Hence, by first law of thermodynamics,
CBSE Term II Physics XI 85

Q = 0 = ∆U + ∆W (iii) (c) Only option (c) is correct as p , V and T are state


∴ ∆W = − ∆U variables. Thermodynamics is a macroscopic science
So, work done is equal to negative of change in internal and result are based on macroscopic or measurable
energy. variables.
Since, the internal energy is a function of thermodynamic (iv) (d) Internal energy is a state variable. Heat and work
state of the system, which does not depend on the path, are not state variables. Heat and work are related to
therefore work done is also independent of path in change in the state but not to actual state of system.
adiabatic process. So, by changing heat and work, we can change the
Therefore, both A and R are true and R is the correct internal energy of the system.
explanation of A. (v) (c) A mixture of gases undergoing an explosive
17. (b) The first law of thermodynamics is the general law of chemical reaction is not in equilibrium because
conservation of energy applied to any system in which during explosion temperature rises abruptly. When
energy transfer from or to the surrounding (through heat explosions are over with time, it attains equilibrium
and work) is taken into account. with surroundings.
The second law of thermodynamics disallows some
process consistent with the first law of thermodynamics.
It states that,
PART 2
No process is possible whose sole result is the absorption 1. Let m1 and m 2 are masses of bodies with specific heats s1
of heat from a reservoir and complete conversion of heat and s 2, then if their temperature after they are in thermal
into work (Kelvin-Planck statement). equilibrium is T.
No process is possible whose sole result is the transfer of
T1 T2
heat from a colder object to a hotter object (Clausius
statement).
m1 ,s1 m2 ,s2
Therefore, both A and R are true but R is not the correct
explanation of A. Then, if T1> T > T2 and assuming no heat loss.
18. (b) Irreversibility mainly arises due to Heat lost by hot body = Heat gained by cold body
(i) dissipative factor like friction, viscosity, etc m1s1( T1 − T ) = m 2s 2 ( T − T2 )
(ii) and as many processes takes the system to m1s1T1 + m 2s 2T2
⇒ = T (equilibrium temperature)
non-equilibrium state. m1s1 + m 2s 2
So, when the blade is rotated in a liquid, then due to the So for, bodies to settle down to mean temperature,
viscosity, the system becomes irreversible as it stops after
a certain time. m1 = m 2 and s1 = s 2
It means bodies will have same specific heat and have
Thus, it is a irreversible process. equal masses.
Therefore, both A and R are true but R is not the correct T +T
explanation of A. Then, T= 1 2 (mean
2
T
19. (c) Efficiency of Carnot engine, η = 1 − 2 temperature)
T1 2. Given, volume of water heated = 3.0 L/min
where, T2 = sink temperature
Mass of water heated, m = 3000 g/min
and T1 = source temperature.
Rise in temperature, ∆T = 77 − 27 = 50 ° C
This expression of η does not include working substance
information, so η is independent of working substance. Specific heat of water, C = 4.2 J g −1 ° C−1
∴ Reversible Carnot engine with one particular substance Amount of heat used, ∆Q = m C ∆T = 3000 × 4.2 × 50
cannot be more efficient than the one using another = 63 × 10 4 J / min
substance because it does not matter what substance is Heat of combustion = 4 . 0 × 10 4 J/g
being used. 63 × 10 4
Rate of combustion of fuel =
Therefore, A is true but R is false. 4 . 0 × 10 4
20. (i) (c) Free expansion of gas is the example of = 15.75 g / min
non-equilibrium states. When outside pressure is
3. When ice melts, volume of water formed is less than that
less, piston accelerates outward.
of ice. So, surroundings (environment) does work on the
(ii) (c) Finite temperature difference between system system (ice) and by first law,
and surroundings causes rapid exchange of heat
between them with non-equilibrium intermediate ∆Q = ∆W + ∆U ⇒ ∆U = ∆Q − ∆W
stages. After sometimes, equilibrium is attained. (∆W = negative as work is done on the system)
⇒ ∆U > ∆Q
86 CBSE Term II Physics XI

4. Given, mass of gas, m = 2 . 0 × 10 −2 kg = 20 g 9. For path 1, heat given, Q1 = + 1000 J


Rise in temperature, ∆T = 45 ° C Work done = W1 (Let)
Molecular mass, M = 28 For path 2, work done, W2 = ( W1 − 100 )J
m 20
Number of moles, n = = = 0.714 As, change in internal energy between two states for
M 28 different path is same.
As nitrogen is a diatomic gas, molar specific heat at ∴ ∆U = Q1 − W1 = Q2 − W2
constant pressure,
7 7 ⇒ 1000 − W1 = Q2 − ( W1 − 100 )
Cp = R = × 8.3 J mol −1K −1 ⇒ Q2 = 1000 − 100 = 900 J
2 2
Q2 T2 3
As ∆Q = n Cp ∆T 10. As we know, = =
Q1 T1 5
7
∴ ∆Q = 0.714 × × 8.3 × 45 = 933.4 J Q T
2 Q 1 − 2 =1 − 2
5. Q1 T1
Q1 − Q2 500 − 300
( p,i Vi ) Case (ii)
⇒ = (Q Q1 − Q2 = W )
pi ( pi , Vf ) Q1 500
W 2
⇒ =
( pf , Vf ) Q1 5
p Case (i)
5
∴ Q1 = 10 3 × = 2500 J
2
O V Vf
11. Given, T1 = 600 K, T2 = 200 K
The p-V diagram for each case is shown in the figure. T − T2 600 − 200
In case (i), process is isothermal, therefore p i Vi = p f Vf . Efficiency of Carnot engine, η = 1 =
T1 600
In case (ii), pressure is constant, work done = area under
400 2
the p-V curve, so work done is more when the gas = = = 66%
expands at constant pressure. 600 3
6. As no heat is allowed to be exchanged, the process is Energy output
Real efficiency =
adiabatic. γ Energy input
p 2  V1 
∴ p 2V2γ = p1V1γ or =  2
p1  V2  = = 0. 47 = 47 %
1 × 4.2
1.4
1 p 2  V1  Real efficiency 47
As V2 = V1 or = = 2 1.4 = 2. 64 ∴ = = 0.71
2 p1 (1 / 2 ) V1 Carnot engine efficiency 66
7. During driving, due to friction between the tyres of car 12. Given, T1 = 827 ° C = 827 + 273 = 1100 K
and road, the temperature of the tyre and hence that of T2 = 27 ° C = 27 + 273 = 300 K
air inside it, increases. Since the volume of air in the tyre Q1 = 1000 kcal = 1 × 10 6 cal
is constant, pressure of the air increases due to increase Q T
in temperature ( pV = nRT ). (i) As, 2 = 2
Q1 T1
8. The p-V diagram for two samples is given below
T 300
p ⇒ Q2 = 2 × Q1 = × 1 × 10 6 = 0.273 × 10 6 cal
T1 1100
C
∴ Work done by engine,
B W = Q1 − Q2 = 1 × 10 6 − 0.273 × 10 6
A
= 0.727 × 10 6 J
= 7 .27 × 10 5 J
T 300
V/2 V V (ii) Efficiency of engine, η = 1 − 2 = 1 − = 72 .7%
T1 1100
As, an adiabatic curve is steeper than an isotherm curve, 13. (i) When the gas is heated, there occurs an appreciable
so the adiabatic compression curve AC lies above the change in its volume. So, energy is required for
isothermal compression curve AB. expansion and we have two specific heats one at
Clearly from the graph, the value of p at point C is constant pressure and one at constant volume. While
in case of solids or liquids, expansion is negligible,
greater than that at point B. Hence, pressure of the
so they have only one specific heat at constant
sample compressed adiabatically will be greater.
volume.
CBSE Term II Physics XI 87

(ii) When a gas is heated at constant volume, all the V2 V2 dV  µRT 


heat supplied is used to increase the temperature or
= ∫
V1
pdV = µRT ∫V1 V
Q pV = µRT ⇒ p =
 V 

internal energy of the gas.
= µRT ln( V )VV12 = µRT ln( V2 − V1)
While when a gas is heated at constant pressure, it
V 
expands. Thus, the heat supplied is used in two ways Wiso = µRT ln  2 
(a) Partly to increase the internal energy.  V1 
(b) Partly to do work against external pressure. Also, work done in an adiabatic process,
V2 V2 K
So, additional heat is utilised here. Therefore, the W = ∫ pdV = ∫ dV
molar specific heat at constant pressure is greater V1 V1 V γ

than that at constant volume i.e. Cp > CV . Q pV γ = K (constant) …(i)


(iii) The difference between the two molar specific heats ∴ p1V1γ = p 2V2γ = K
is equal to the amount of heat equivalent to the work From Eq. (i), we get
performed by the gas during expansion at constant V2
pressure. V− γ + 1 K  1 1 
p =K =  − 
Using their ratio, we can determine the degree of − γ + 1 V 1 − γ  V2γ − 1 V1γ − 1
1
freedom or atomicity of any gas.
1  p 3V2γ p1V1γ  p1V1 − p 3V2
C 2 Thus, W =  −  =
γ = p =1 + 1 − γ  V2γ − 1 V1γ − 1  γ −1
CV f
Work done is the area under the curve of p-V graph. The
14. (i) Initial pressure of system, p i = p a
k p-V graphs for isothermal and adiabatic process can be
(ii) Final pressure of system, p f = p a + ( V − V0 )
A drawn as
p
(iii) According to first law of thermodynamics,
∆Q = ∆U + ∆W p3

where, ∆U = CV ( T − T0 ) Adiabatic
1 Isothermal
and ∆W = p a ( V − V0 ) + k ( V − V0 )2 p2
2
p1
1
∴ ∆Q = CV ( T − T0 ) + p a ( V − V0 ) + k ( V − V0 )2 V
2 O V2 V1
where, T0 = p a V0 / R, Clearly, the area under the isothermal expansion is more
and T = [ p a + ( k / A ) × ( V − V0 )] V / R than that under adiabatic expansion and hence the work
15. (i) Work done by the gas (let pV1/ 2 = A ) done also.
V2
V2 V2 dV  V 17. The indicator diagram represents the variation of
∆W = ∫ pdV = A ∫ =A  pressure p of the gas with the volume V.
1 / 2  V1
V1 V1
V
On a p-V diagram, the process is represented by an
= 2 A ( V2 − V1 ) = 2 p1V11/ 2[ V21/ 2 − V11/ 2 ] adiabatic curve as shown in figure
A p
(ii) Since, T = pV / nR = ⋅ V i
nR
T2 V2
Thus, = = 2 An adiabat
T1 V1
T1 1
∴ =
T2 2 f

(iii) Then, the change in internal energy, O Vi Vf


V
3 3
∆U = U 2 − U 1 = R ( T2 − T1) = RT1( 2 − 1 ) An adiabatic expansion process
2 2
∆W = 2 A V1( 2 − 1 ) = 2RT1( 2 − 1 ) A curve representing an adiabatic process is called an
According to first law of thermodynamics, adiabat. If we differentiate pV γ = constant, we get
∆Q = ∆U + ∆W dp V γ + p γ V γ − 1 dV = 0
⇒ ∆Q = (7 / 2 )RT1( 2 − 1 ) dp  p
⇒ =− γ 
16. Work done in an isothermal process dV  V
= Area under p-V graph
88 CBSE Term II Physics XI

 p 22. (i) Waterfall The falling of water cannot be reversible


So, slope of adiabatic curve = − γ   , which is γ times
 V process. During the water fall, its potential energy
more than slope of an isotherm. convert into kinetic energy of the water.
18. (i) Q Process is adiabatic On striking the ground, some part of potential
energy converts into heat and sound. In nature, it is
∴ ∆Q = 0
not possible to convert the heat and sound along
Work done on the gas, ∆W = −500J with kinetic energy too and potential energy so that
According to the first law of thermodynamics. the water will rise back. So, waterfall is not a
∆Q = ∆U + ∆W ⇒ ∆U = − ∆W = 500J reversible process.
(ii) This is because, heat absorbed by a substance (ii) Rusting of iron In rusting of iron, the iron become
(coolant) is directly proportional to the specific heat oxidised with the oxygen of the air as it is a chemical
of the substance. reaction, it cannot be reversed.
(iii) This is because in a harbour town, the relative 23. Given, T1 = 227 ° C = 227 + 273 = 500 K
humidity is more than in a desert town. Hence, the T2 = 127 ° C = 127 + 273 = 400 K
climate of a harbour town is without extremes of hot
Q1 = 6 × 10 4 cal
and cold.
T 400
19. Given, work done, W = − 22.3 J The efficiency of a Carnot engine, η = 1 − 2 = 1 −
T1 500
Work done is taken negative as work is done on the system.
work output W
In an adiabatic change, ∆Q = 0 Also, efficiency, η = =
heat input 6 × 10 4
Using first law of thermodynamics,
1
∆U = ∆Q − W = 0 − ( − 22. 3 ) = 22. 3 J ⇒ W = η × 6 × 10 4 = × 6 × 10 4 = 1.2 × 10 4 cal
For another process between states A and B, 5
Heat absorbed, ∆Q = + 9.35 cal 24. Given, T1 = 317 ° C = 317 + 273 = 590 K
= + (9.35 × 4.19) J = + 39.18 J T2 = 117 ° C = 117 + 273 = 390K
Change in internal energy between two states via Q1 = 500 kcal = 5 × 10 5 cal
different paths are equal. (i) Efficiency of Carnot engine,
∴ ∆U = 22.3 J T 390 20
η=1 − 2 =1 − =
∴ From first law of thermodynamics, T1 590 59
∆U = ∆Q − W Also, η=
W
or W = ∆Q − ∆U = 39.18 − 22. 3 = 16.88 J ≈ 16.9 J Q1
20. Suddenly compressed implies no time for heat exchange 20
⇒ W = η × Q1 = × 5 × 10 5 = 1.7 × 10 5 J
with the surrounding, so it can be treated as an adiabatic 59
process. (ii) As, work done, W = Q1 − Q2
In an adiabatic process, relation between volume and ∴ Heat delivered to the sink,
temperature,
Q2 = Q1 − W = 5 × 10 5 − 1.7 × 10 5 = 3. 3 × 10 5 cal
TV γ − 1 = constant ⇒ TV 1 1
γ −1
= T2V2γ −1
25. Let the initial temperatures of hot and cold reservoirs
5
Given, T1 = 17 ° C = 17 + 273 = 290 K , γ = were T1 and T2. The efficiency of the Carnot engine is
3 given by
V
V1 = V and V2 = T − T2
8 η= 1 …(i)
5
−1
T1
5
−1  V 3
⇒ 290( V ) 3
= T2   As given the temperature of both the reservoirs is raised
 8 by equal amount t, so T1′ = T1 + t and T2′ = T2 + t . The
⇒ T2 = 290( 8 )2/ 3 = 1160 K final efficiency of the Carnot engine,
= 1160 − 273 = 887 ° C T ′ − T2′ ( T1 + t ) − ( T2 + t ) T1 − T2
η′ = 1 = = …(ii)
21. From figure, change in pressure, T1′ ( T1 + t ) ( T1 + t )
dp = DF = 600 − 300 = 300 Nm−2 Dividing Eq. (ii) by Eq. (i), we have
Change in volume, dV = EF = 5 − 2 = 3 m 3
 T1 − T2 
Work done by the gas from D to E to F = Area ∆DEF  
η′  T1 + t  T1
1 = =
W = × EF × DF η  T1 − T2  T1 + t
2  
1  T1 
= × 300 × 3 = 450 J
2 As η′ < η , i.e. the efficiency of Carnot engine decreases.
CBSE Term II Physics XI 89

26. (i) In figure, portion AB of the cycle involved increases Since, T = (1 / nR ) pV = (1 / nR ) V f ( V )


in pressure/temperature of gas at constant volume. ∴ dT = (1 / nR ) [ f ( V ) + V f ′( V )] dV
Therefore, the system gains heat from the dQ C
surroundings. Thus, = V [ f ( V0 ) + V0f ′( V0 )] + f ( V0 )
dV V = V0 R
QAB = U AB (Q W = 0 )
 1  V f ′( V0 )
3
= R( TB − TA ) = + 1 f ( V0 ) + 0
2 γ − 1  γ −1
3 γ V0
= ( pBVB − p A VA ) (Q ∆U = nCV ∆T ) = p0 + f ′( V0 )
2 γ −1 γ −1
3 Heat is absorbed when dQ / dV > 0 when gas expands,
= ( pB − p A ) VA (Q VB = VA )
2 that i.e. when γ p 0 + V0f ′( V0 ) > 0
(ii) In the portion BC, the gas increases in volume at f ′( V0 ) > − γ p 0 / V0
constant pressure. Heat required for this is gained 29. The figure shown below is the p-V diagram for the gas in
from surroundings, two cases. p
QBC = U BC + WBC A
3
= ( p CVC − pBVB ) + pB( VC − VB )
2 Isothermic
5 Adiabatic
= pB( VC − VB ) [Q p C = pB ] pC C
2
pB B
5
QBC = pB( VC − VA ) E D
V
2 O VO V
(iii) As CD represents an adiabatic change, therefore As an adiabatic curve is steeper than an isothermic curve,
QCD = 0. so adiabatic expansion curve AB lies below the isothermal
(iv) DA involves compression of gas from VD to VA at expansion curve AC.
constant pressure p A , so work is done on the gas (i) pB and p C are the final pressures for adiabatic and
(negative). isothermal expansions, respectively. Clearly, p C > pB.
∆QDA = ∆U DA + ∆WDA Hence, the final pressure is greater for the
3 isothermal expansion.
= R ( TA − TD ) + p A ( VA − VD )
2 (ii) Work done in an adiabatic expansion = Area ABDE
3 5 Work done in an isothermal expansion = Area
= p A ( VA − VD ) + p A ( p A − VD ) = p A ( VA − VD ) ACDE
2 2
27. For isothermal process, pV = constant As, area ACDE > Area ABDE
For adiabatic process, pV γ = constant So, more work is done in the isothermal expansion.
5 30. (i) A to B because TB > TA , as p ∝ T (QV constant)
For monoatomic gas, γ = = 1. 66
3 (ii) C to D because TC > TD , as p ∝ T (QV constant)
7
For diatomic gas, γ = = 1. 4
B

5
(iii) WAB = ∫A
pdV = 0 and WCD = 0 (Q V constant)
8 VC
For triatomic gas, γ = = 1. 33 C C dV  V− γ + 1 
6 Similarly, WBC = ∫
B
pdV = K ∫
B Vγ
= K  − γ + 1
  VB
and slope increases with γ, so
For I, γ = 1 (Q pV γ = K )
For II, γ = 1.33 1
= ( p CVC − pBVB )
For III, γ = 1.4 1−γ
For IV, γ = 1.66 1
Similarly, WDA = ( p A VA − pD VD )
So, graph I is isothermal, 1−γ
Graph II is triatomic adiabatic, γ
V 
Graph III is diatomic adiabatic, Now, p C = pB  B  = 2 − γ pB (Q VC = 2 VB)
 VC 
Graph IV is monoatomic adiabatic.
28. Slope of p = f ( V ), curve at ( V0 , p 0 ) = f ( V0 ) Similarly, pD = p A 2 − γ
Total work done = WBC + WDA
Slope of adiabat at ( V0 , p 0 ) = K ( − γ ) V0− 1− γ = − γ p 0 / V0
1
Now, heat absorbed in the process p = f ( V ) = [ pBVB(2 − γ + 1 − 1 ) − p A VA (2 − γ + 1 − 1 )]
1−γ
dQ = dU + dW = nCV dT + pdV
90 CBSE Term II Physics XI

=
1
(21− γ − 1 ) ( pB − p A ) VA (Q VA = VB ) and ∆U 3 = nCV ( T2 − T1)
1−γ ∴ Q3 = ∆U 3 − W3 = nCV ( T2 − T1) + p( V1 − V2 )
3  1 
2/ 3 So, net heat, Q = Q1 + Q2 + Q3
=  ( pB − p A ) VA
1 −   V 
2    2 = nCV ( T2 − T1) + nRT2 ln  2 
 V1 
(iv) Heat supplied during process A to B,
+ nCV ( T2 − T1) + p( V1 − V2 )
dQAB = dU AB
3 3 V 
QAB = nR( TB − TA ) = ( pB − p A ) VA = 2 nCV ( T2 − T1) + nRT2 ln  2  + p( V1 − V2 )
2 2  V1 
Net work done  1  
2/ 3
33. (i) Let capacity of each cylinder be V and atmospheric
Efficiency = = 1 −    pressure be p.
Heat supplied   2 
 p1 = p
31. A cyclic process restores the system back to its initial Initial volume of gas = Volume of the cylinder A
state after completion of the cycle.
∴ V1 = V
As internal energy is a state function, so its value at initial When stopcock is opened, then volume available for
point is same as that at final point (initial state is same as gas becomes 2V.
that of final state).
∴ V2 = 2 V
p
Final pressure p 2 = ?
As system is thermally insulated, therefore there is
B ∆Q = 12.6 cal no change in temperature during the process and
hence it is an isothermal process.
A For an isothermal process (according to Boyle’s law),
Adiabatic ∆W = – 40.5 J p1V1 = p 2V2
V V V
O or p 2 = p1 1 = p ×
V2 2V
For adiabatic process A → B, p 1
= = atm = 0.5 atm
∆Q = 0, so by first law of thermodynamics, 2 2
∆Q = ∆W + ∆U (ii) Change in internal energy, ∆U = 0, as no work is
done on or by the gas.
or ∆U = − ∆W = − ( − 40. 5 ) = 40. 5 J
(iii) Change in temperature of the gas is zero, as gas does
For another process, A → B,
no work in expansion.
∆Q = + 12.6 cal (iv) No, because free expansion of gas is rapid and cannot
= 12.6 × 4.19 J = 52.8 J be controlled. The intermediate states are
and by first law of thermodynamics, non-equilibrium states and do not satisfy the gas
equation. Therefore, the intermediate state of the
∆Q = ∆W + ∆U
gas does not be on the p-V-T surface.
⇒ ∆W = ∆Q − ∆U
34. A reversible process must pass through equilibrium
⇒ ∆W = 52.8 − 40.5 = 12.3 J states which are very close to each other, so that when
(as ∆U is a state function, ∆UAB is process is reversed, it passes back through these
same for this process also) equilibrium states.
32. For path PQ, Then, it is again decompressed or it passes through same
∆V = 0 equilibrium states, system can be restored to its initial
⇒ W1 = 0 state without any change in surroundings.
and ∆U 1 = nCV ( T2 − T1) e.g. If a gas is compressed as shown
∴ Q1 = ∆U 1 = nCV ( T2 − T1)
V
For path QR, W2 = − RT2 ln 2
V1
and ∆U 2 = 0, as temperature is constant.
V 
∴ Q2 = − W2 = nRT2 ln  2 
 V1 
V1
For path RP , W3 = − ∫ V2
pdV = − p( V1 − V2 )
CBSE Term II Physics XI 91

But a reversible process can proceeds without reaching Source It is a hot reservoir at a temperature T1 with
equilibrium in intermediate states. conducting walls. It has infinite thermal capacity. Any
amount of heat can be taken from it without changing the
temperature.
Sink It is a cold reservoir at temperature T2. It has
infinite thermal capacity, so any amount of heat can be
Expansion Compression rejected to it without changing the temperature.
T2 Working Substance We use an ideal gas as a working
35. (i) Efficiency, η = 1 − substance in the cylinder.
T1
Insulating Stand The base of the cylinder could be
where, T2 = sink temperature placed on the insulating stand, to isolate it completely
and T1 = source temperature. from the surroundings.
T2 1
1− = …(i) Carnot Cycle As the engine works, the working
T1 2 substance of the engine undergoes a cycle known as
 T − 100  2 Carnot cycle.
1− 2  = …(ii) The Carnot cycle consists of the following four strokes as
 T1  3
given below
T2 1
From Eq. (i), = …(iii) First Stroke (Isothermal Expansion)(Curve AB)
T1 2 The cylinder containing one mole of an ideal gas as
and Eq. (ii) working substance allowed to expand slowly at the
T2 − 100 1 constant temperature T1 by putting it on the source.
= …(iv) p
T1 3 ( p1, V1, T1)
A Q1 ( p2, V2, T1)
Dividing Eq. (iii) by Eq. (iv), we get B
T2 3 Isotherma
compression

T1
= expansion
l
T2 − 100 2
Adiabatic

Ad ansio
exp
W = Q1 – Q2

ia b
⇒ T2 = 300 K

a tic n
(ii) Substituting in Eq. (i), T1 = 600 K
T D
(iii) As efficiency, η2 = 1 − 2 ( p4, V4, T2) Isothermal
T1 compress Q2 C ( p3, V3, T2) T3
ion
T O
∴ It equals to 1 only, when 2 = 0. E F G H T
T1 Various processes in Carnot cycle
or T2 = 0 K Work done = Heat absorbed by the system
But absolute zero is not possible. V2 V 
36. The reversible engine which operates between two W1 = Q1 = ∫ pdV = RT1 log e  2  = Area ABGEA
V1  V1 
temperatures of source ( T1) and sink ( T2 ) is known as
Carnot heat engine. Second Stroke (Adiabatic Expansion) (Curve BC)
Cylinder with
The cylinder is then placed on the non-conducting stand
conducting base
and the gas is allowed to expand adiabatically till the
temperature falls from T1 to T2.
V3 R
W2 = ∫ pdV = ( T1 − T2 )
V2 (γ − 1)
Working Ideal
substance
= Area BCHGB
Third Stroke (Isothermal Compression) (Curve CD)
Source Insulated Sink
T1 stand T2 The cylinder is placed on the sink and the gas is
Carnot engine
compressed at constant temperature T2.
Work done = Heat released by the system
The main parts of Carnot engine are as given below V4 V
W3 = Q2 = − ∫ pdV = − RT2 log e 4
Cylinder The cylinder has conducting base and V3 V3
insulating walls. It consists an ideal gas as a working
V3
substance. Insulating and frictionless piston is attached = RT2 log e = Area CDFHC
with it as shown in the figure. V4
92 CBSE Term II Physics XI

Fourth Stroke (Adiabatic Compression) (Curve DA) Since, the engine B absorbs the heat rejected by the engine
Finally, the cylinder is again placed on non-conducting A, so Q1′ = Q2
stand and the compression is continued so that gas  300 
∴ WB = 1 −  Q2
returns to its initial stage.  T 
V1 R
W4 = − ∫ pdV = − ( T2 − T1) (i) When outputs of the two engines are equal,
V4 γ −1
WA = WB
R
= ( T1 − T2 ) = Area ADFEA  T   300 
γ −1 or 1 −  Q1 = 1 −  Q2
 800   T 
37. Given, T1 = 327 ° C = 327 + 273 = 600K
 T   300  Q2
R = 8. 31 J mol −1K −1 or 1 −  = 1 − 
 800   T  Q1
γ = 1. 4
 300  T
Ratio of compression or expansion,
V1 1
= = 1 − 
 T  800
V2 6
(i) For adiabatic process, TV γ −1
= T2V2γ − 1 On solving, we get T = 550 K.
1 1

V
γ −1 (ii) When the efficiencies are equal, ηA = ηB
⇒ T2 = T1 1  T 300
 V2  or 1− =1 −
1.4 − 1
800 T
1  or T = 24 × 10 4
2
= 600 ×  
 6 ∴ T = 489.9 K
39. (i) Given, Q2 = 263 J/s,
0 .4
1 
= 600 ×  
 6 T2 = −10 ° C = −10 + 273 = 263 K
= 600 × 0. 488 = 293 K and T1 = 25 ° C = 25 + 273 = 298 K,
Q T2
= 20 ° C β= 2 =
(ii) Net work done during each cycle, W T1 − T2
V V ⇒ Average power,
W = nRT 1 log 2 − nRT2 log 2
V1 V1 Q ( T − T2 ) 263 (298 − 263 )
P= 2 1 =
V T2 263
= nR × ( T1 − T2 ) × log 2
V1 = 35 J / s = 35W
= 2. 303 × 8. 31 × ( 600 − 293 ) × log 6 (ii) Given, T1 = 273 K,
= 2. 303 × 8. 31 × 307 × 0.7782 = 4572 . 2 J T2 = 25 + 273 = 298 K
(iii) Efficiency of engine, L = 333 × 10 3 J kg −1
T 293 Efficiency of electric motor = 50%
η=1 − 2 =1 − = 0. 512 or 51.2%
T1 600 The used power of the engine
= 50% of 500 W = 250 W
38. For engine A T1 = 800 K, T2 = T K
Coefficient of performance,
T2 T
Efficiency, ηA = 1 − =1 − Q T2
T1 800 β= 2 =
W T1 − T2
Q2 T2 T
Also, = = Heat extracted from water in unit time,
Q1 T1 800
T2
 Q2  Q2 = ×W
Work output, WA = Q1 − Q2 = ηA × Q1 Q ηA = 1 −  T1 − T2
 Q1  273
= × 250 = 2730. 0 Js −1
 T  298 − 273
or WA = 1 −  Q1
 800  Total heat extracted from 25 kg water to freeze it
For engine B, T1′ = T K, T2′ = 300 K into ice,
T′ 300 Q = mL = 25 × 333 × 103 J
Efficiency, ηB = 1 − 2 = 1 − Total time taken in freezing water into ice
T1′ T
Q 25 × 333 × 10 3
 300  t= = = 3049. 45 s
Work output, WB = Q1′ − Q′2 = ηB × Q1′ = 1 −  Q1′ Q2 2730
 T 
CBSE Term II Physics XI 93

CHAPTER 05

Kinetic Theory

In this Chapter...
l Molecular Nature of Matter l Law of Equipartition of Energy
l Behaviour of Gases l Specific Heat Capacity
l Laws for an Ideal Gas l Mean Free Path
l Kinetic Theory of an Ideal Gas l Brownian Motion
l Degree of Freedom

K = constant that varies with volume or with number of


Molecular Nature of Matter atoms or molecules (N) of gas in the given sample and
Dalton proposed the atomic theory to explain the laws of k B = Boltzmann constant =1.38 ´ 10 -23 J / K .
definite and multiple proportions obeyed by elements when
they combine into compounds. l
According to Avogadro’s hypothesis, equal volumes of all
As number of atoms combine to form a molecule so Dalton’s gases under identical conditions of pressure and
atomic theory is also referred as molecular theory of matter. temperature will contain equal number of molecules. It is
Atoms of gases are much free to move and can travel long represented by NA .
distances without colliding. Due to this property of gases, Avogadro’s number, N A = 6.023 ´ 10 23 .
they get dispersed away if they are not enclosed. l
The perfect gas equation can be written as,
In solids and liquids, the closeness makes the interatomic
pV = mRT
force important. The atoms attract when they are at few
angstroms distance but repel when they come closer. M N
where, m = number of moles = = ,
In dynamic equilibrium, molecules collide with each other M0 NA
and change their speeds during collision. R = NA k B is universal constant
= 8 . 314 J mol -1 K -1
Behaviour of Gases and M = mass of the gases.
l
Gases at low pressure and high temperature much above rRT
l
In terms of density, perfect gas equation is p =
that at which they liquify (or solidify) follow a relation M0
pV = KT = Nk BT
where, r = mass density of the gas
where, p = pressure, V = volume, T = absolute temperature and M 0 = molar mass of the gas (mass of one mole of
(i. e. in Kelvin scale), the gas).
94 CBSE Term II Physics XI

Laws for an Ideal Gas Assumptions of Kinetic Theory of Gases


Assumptions of kinetic theory of gases are given below
Boyle’s Law l
A given amount of gas consists of a very large number of
It states that for a given mass of a gas at constant
molecules (of the order of Avogadro’s number 10 23 ) and all
temperature, the volume of that mass of gas is inversely
proportional to its pressure. molecules are identical in all respect.
1 l
The molecules of a gas are in a state of incessant random
i.e. Vµ motion in all directions with different speeds.
p
l
The size of a molecule is much smaller than the average
or pV = constant
separation between the molecules.
Charles’ Law l
There is no intermolecular forces between molecules of gas
It states that for a given mass of an ideal gas at constant except during collision.
pressure, volume of a gas is directly proportional to its l
The collision between molecules among themselves or
absolute temperature. between molecules and walls are perfectly elastic.
V
i. e. V µ T or = constant l
The duration of collision between two molecules is
T
negligible as compared to time interval of two successive
Dalton’s Law of Partial Pressure collisions, i.e. collisions are instantaneous.
It states that, the total pressure of a mixture of l
The density and the distribution of molecules is uniform
non-interacting ideal gases is equal to the sum of partial throughout the gas.
pressures exerted by individual gases in the mixture.
i. e. p = p1 + p 2 + L Pressure of an Ideal Gas
m RT m 2RT Þ Pressure exerted by an ideal gas,
= 1 + +L
V V 1 æN ö 2 1 2
p= ç ÷ mv = nm v
Graham’s Law of Diffusion 3 è Vø 3
It states that, the rate of diffusion of a gas is inversely where, n = number density (i. e. number of molecules per
proportional to the square root of its density. unit volume) of gas,
1 m = mass of one molecule
i.e. rµ and v 2 = mean square speed.
r
It is also called kinetic gas equation.
Work Done on Compressing a Gas Relation between pressure exerted by an ideal gas and its
Work done by a gas which is compressing from state 1 to 1
density p = rv 2
state n is given by 3
n
W = - ò p dV Kinetic Interpretation of Temperature
1
( p V - p 1 V1 ) The average kinetic energy of a molecule is directly
=- 2 2
1-n proportional to the absolute temperature of the gas. It is
mR ( T2 - T1 ) independent of pressure, volume and the nature of the ideal
=- gas.
1-n 1
Average kinetic energy, m v 2 µ T
Similar, expression can be obtained for work done during 2
expansion of gas, 1
If T = 0, then mv 2 = 0 but m ¹ 0, so v 2 = 0.
n p V - p 1 V1 2
W = ò p dV = 2 2 Thus, absolute zero of temperature may be defined as that
1 n -1 temperature at which the mean square speed of the gas
mR ( T2 - T1 ) molecules reduces to zero.
=
n -1
Root Mean Square Speed
Kinetic Theory of an Ideal Gas The square root of the mean square speed v 2 is known as root
mean square speed.
It is based on the molecular picture of matter. It correlates
1 3
the macroscopic properties (e.g. pressure and temperature) of From kinetic interpretation, m v 2 = k B T
gases to microscopic properties (e.g. speed and kinetic 2 2
energy) of gas molecules. 3k B T
So, v2 =
m
CBSE Term II Physics XI 95

1
Taking square root on both sides, we get Rotational energy of the molecules = (I 1 w12 + I 2 w22 )
3k BT 2
v2 = Total energy of a diatomic gas molecule is the sum of
m
translational energy E t and rotational energy E r .
3k B T
Root mean square speed, v rms = æ1 1 1 ö æ1 1 ö
m E = E t + E r = ç mv 2x + mv 2y + mv 2z ÷ + ç I 1 w12 + I 2 w22 ÷
è2 2 2 ø è2 2 ø
where, m = mass of one molecule,
Molecules like CO at moderate temperatures contributes a
k B = Boltzmann constant vibrational energy E v to total energy,
and T = absolute temperature. 2
1 æ dy ö 1
Ev = mç ÷ + k y 2
Maxwell’s Speed Distribution 2 è dt ø 2
In a given sample of gas, all molecules don’t move with same where, k = force constant of the oscillator
speed. They move randomly in different directions, but the and y = vibrational coordinate.
distribution of velocities among the molecules remain fixed.
\ Total energy, E = E t + E v + E r
Most Probable Speed The speed possessed by the maximum
number of molecules in a gas at a given temperature. Each translational and rotational degrees of freedom of a
1
2k B T molecule contributes to k B T to the energy, while each
v mp = 2
m 1
vibrational frequency contributes to 2 ´ k B T = k B T,
Mean or Average Speed The arithmetic mean of the speed of 2
the molecules of a gas at a given temperature. Since, vibrational has both kinetic and potential energy
8k B T modes.
v av =
pm
Specific Heat Capacity
Degree of Freedom Monoatomic Gases
The total number of coordinates or independent quantities In monoatomic gases, a molecule has three translational
required to describe completely the position and degrees of freedom.
configuration of a dynamical system is known as number of
degrees of freedom of the system. It is represented by f and Total internal energy of one mole of monoatomic gas,
expressed as 3
U = RT
f = 3N - K 2
where, N is the number of particles in a system and K is The specific heat of gas at constant volume,
number of independent relations between the particles. 3
CV = R
(i) In case of monoatomic gas, 2
f =3 and the molar specific heat at constant pressure,
(ii) In case of diatomic gas, 5
\ Cp = R
f =5 2
Cp 5
(iii) In case of triatomic gases, \Ratio of specific heat, g = = = 1. 67
f = 7 for linear molecules CV 3
and f = 6 for non-linear molecules.
Diatomic Gases
Law of Equipartition of Energy In case of diatomic gases, if the vibrational mode is not
considered, then it has five degrees of freedom (three
It states that, ‘‘For a dynamic system in thermal equilibrium, translational and two rotational) at room temperature.
the total energy is distributed equally amongst all the degree Total internal energy of one mole of such type of gas,
of freedom and the energy associated with each molecule per
5
degree of freedom is 1 k B T.’’ U = RT
2
2
1 The molar specific heat C V is given by
The kinetic energy of single molecule, E t = m( v 2x + v 2y + v 2z ) 5
2 CV = R
3 2
Translational energy of the molecules, E t = k B T
2 and the molar specific heat C p is given by
96 CBSE Term II Physics XI

7 Specific Heat Capacity of Solids


Cp = R
2
Total energy of one mole of solid, U = 3RT
\Ratio of specific heat capacities, DQ DU
Cp 7 So, molar specific heat of solid, C = = = 3R
g= = = 1.40 DT DT
CV 5 = 24. 93 J mol -1K -1
Triatomic Gas Specific Heat Capacity of Water
In case of such kind of gases (linear triatomic gas), there are Total energy of 1 mole of water, U = 9RT
seven degrees of freedom.
\ Molar specific heat of water,
The total internal energy of one mole of such gases,
D Q DU
7 C= = = 9R = 74.79 J mol -1 K -1
U = RT DT DT
2
7 9
C V = R and C p = R
2 2 Mean Free Path
\ Ratio of specific heat capacities, The average distance travelled by a molecule between two
Cp 9 successive collisions is known as the mean free path of the
g= = = 1 . 28 molecule.
CV 7
Average time between two successive collisions,
Polyatomic Gas 1
t=
A polyatomic gas molecule in general has three translational, n pd 2 v
three rotational degrees of freedom and with certain number 1
(let f ) of vibrational modes. Mean free path, l = …(i)
2 npd 2
The total internal energy of one mole of such gases,
where, T = temperature of gas (in kelvin)
U = ( 3 + f ) RT and d = molecules diameter.
The molar specific heat capacities are The mean free path of a given gas is (i) directly proportional
CV = ( 3 + f ) R to its absolute temperature and (ii) inversely proportional to
k BT
and Cp = ( 4 + f ) R its pressure, i.e. l = .
Cp ( 4 + f ) 2 pd 2 p
\ Ratio of specific heat capacities, g = =
CV ( 3 + f )
Brownian Motion
Determination of g from It is the zig-zag motion of the particles of microscope of size
the Degrees of Freedom suspended in water, air or some other fluid. This motion can
A polyatomic gas molecule has f degrees of freedom, then be explained on the basis of kinetic theory.
f æf ö The Brownian motion depends upon the following factors
C V = R and C p = ç + 1 ÷ R
2 è2 ø (i) Size of the suspended particle
Cp æ (ii) Density of the fluid

So, specific heat ratio is g = = ç1 + ÷ (iii) Temperature of the medium
CV è fø
(iv) Viscosity of the medium
CBSE Term II Physics XI 97

Solved Examples
8RT
Example 1. If the mass of each molecule of a gas is Average speed, v =
pM
halved and speed is doubled, then find the ratio of
initial and final pressure. where, T = temperature of the gas (in kelvin)
Sol. Given, m1 = m, v1 = v , V1 = V, = 273 + 27 ° = 300 K
m 2 = m / 2, v 2 = 2 v , V2 = V1 = V and R = gas constant = 8 . 314 J mol -1 K -1
\ Pressure exerted by first gas molecule, 8 ´ 8 . 314 ´ 300
1 m1 2 \ v= = 673.72 ms -1
p1 = v1 3 .14 ´ 14 ´ 10 -3
3 V1
and pressure exerted by second gas molecule, Example 4. The molecules of a given mass of a gas
1 m2 2 have root-mean-square speed of 100 ms -1 at 27°C
p2 = v2
3 V2 and 1.00 atm pressure. What will be the
root-mean-square speed of the molecules of the gas
p1 m1 V2 v12
\ = ´ ´ at 127°C and 2.0 atm pressure?
p 2 m 2 V1 v 22
Sol. Given, at 27°C, root-mean-square speed,
Putting all the given values in above equation, we get ( v rms )1 = 100 ms -1
2
p1 m V æ vö 1 1 3 p1 3 p1 V1
= ´ ´ç ÷ =2´ = ( v rms )1 = = …(i)
p 2 m /2 V è 2v ø 4 2 r1 M
Example 2. Oxygen is filled in a closed metal jar of According to ideal gas equation,
p1 V1 p 2 V2
=
-3 3 5
volume 10 m at a pressure of 1.5 ´ 10 Pa and T1 T2
temperature 400 K. The jar has a small leak in it. V1 p 2 T1 2 ´ 300 3
The atmospheric pressure is 10 5 Pa and the Þ = = =
V2 p1 T2 400 2
atmospheric temperature is 300 K. Find the mass of
the gas that leaks out by the time, the pressure and \ At 127°C, root mean square speed,
the temperature inside the jar equalise with the 3p 2 3 p 2 V2
( v rms )2 = = …(ii)
surrounding. r2 M
Sol. Given, volume, V = 10 -3 m3 From Eqs. (i) and (ii), we get
Pressure, p 1 = 1. 5 ´ 10 5 Pa, p 2 = 10 5 Pa V p
( v rms )22 = ( v rms )12 ´ 2 ´ 2
V1 p1
Temperature, T1 = 400 K , T2 = 300 K
From ideal gas equation, p1 V = n1RT1 2
= ( 100 )2 ´ ´ 2
pV 3
Þ n1 = 1 200
RT1 -1
or ( v rms )2 = ms
Similarly, p 2 V = n 2RT2 3
p V Example 5. Calculate the root-mean-square, average
Þ n2 = 2
RT2 and most probable speeds of oxygen molecules at
\Number of moles leaked, 27° C.
æp p öV Sol. Given, molar mass of oxygen,
Dn = n1 - n 2 = ç 1 - 2 ÷
è T1 T2 ø R M = 32 g mol -1 = 0.032 kg mol -1
æ 1.5 1 ö 10 5 ´ 10 -3 Temperature, T = 27 ° C = (27 + 273 ) K = 300 K
=ç - ÷´ 3RT
è 400 300 ø 8.3 (i) Root mean square speed, v rms =
= 5. 02 ´ 10 -3 M
Mass of gas leaked = DnM 0 = 5.02 ´ 10 -3 ´ 32 = 0.16 g 3 (8.314)(300)
= = 483.6 ms -1
(0.032)
Example 3. Find the average speed of nitrogen
8RT 8(8.314)(300)
molecules at 27° C. (ii) Average speed, v = =
-3 pM (3.14)(0.032)
Sol. Mass of 1 mole of nitrogen gas, M = 14 g = 14 ´ 10 kg
= 445.6 ms -1
98 CBSE Term II Physics XI

(iii) Most probable speed, = 7 .72 ´ 10 -21 J/mol


2RT 2( 8. 314 )( 300 ) (ii) 1 mole of gas contains N ( = 6. 02 ´ 10 23 ) mol
v mp = = = 394.8 ms -1
M ( 0. 032 ) Therefore, at 0° C, the kinetic energy of translation
of 1 mole of the gas
Example 6. A balloon has 5.0 g moles of helium at 7°C. = (5.65 ´ 10 -21 ) (6.02 ´ 10 23 ) » 3401 J/mol
Calculate At 100°C, the kinetic energy of translation of 1 mol
(i) the number of atoms of helium in the balloon and of gas
(ii) the total internal energy of the system. = ( 7.72 ´ 10 -21 ) (6.02 ´ 10 23 ) » 4647 J/mol
(Avogadro’s number, N A = 6.02 ´ 10 23 Example 8. One mole each of hydrogen, oxygen and
and Boltzmann’s constant, nitrogen are mixed in a close container of volume
10 L and temperature 27°C. Calculate the pressure
k B = 1.38 ´ 10 -23 Jmol -1 K -1 ).
exerted by the mixture. (R = 8. 314 J mol -1 K -1 )
Sol. Given, n = 5. 0 , T = 7 ° C = 7 + 273 = 280 K
Sol. Given, R = 8.314 J mol -1 K -1
(i) Number of atoms = nN A = 5. 0 ´ 6. 02 ´ 1023
Temperature, T = 27 ° C = 27 + 273 = 300 K
» 30 ´ 10 23
Volume, V = 10 L = 10 ´ 10 -3 m3
3
(ii) Average kinetic energy per molecule = k BT According to Dalton’s law of partial pressure,
2
3 p = p1 + p 2 + p 3
\ Total internal energy = k BT ´ N mRT mRT mRT 3mRT
2 Þ p= + + =
3 V V V V
= ´ 1. 38 ´ 10 -23 ´ 280 ´ 30 ´ 10 23 Here, m = 1 mole
2
3 3RT 3 ´ 8.314 ´ 300
= ´ 30 ´ 280 ´ 1. 38 = 1.74 ´ 10 4 J \ p= =
2 V 10 ´ 10 -3
= 7 . 48 ´ 10 Nm -2
5
Example 7. Given, Avogadro's number, N = 6.02 ´ 10 23
and Boltzmann’s constant, k B = 1.38 ´ 10 -23 J/K. Example 9. Calculate the ratio of the mean free path of
Calculate the molecules of two gases, if the ratio of the
numbers density per cm 3 of the gases is 5 : 3 and
(i) the average kinetic energy of translation of the
molecules of an ideal gas at 0°C and at 100°C the ratio of the diameters of the molecules of the
gases is 4 : 5.
(ii) and the corresponding energies per mole of the n1 5
gas. Sol. Given, ratio of the numbers density, =
n2 3
Sol. (i) According to the kinetic theory, the average kinetic d1 4
energy of translation per molecule of an ideal Ratio of the diameters, =
monoatomic gas at temperature T (in kelvin) is d2 5
æ3ö Mean free path of the molecules of first gas,
ç ÷ k BT, where k B is Boltzmann’s constant.
è2 ø 1
l1 =
At 0° C ( T = 273 K ), the kinetic energy of translation 2 pn1 d12
3 Similarly, mean free path of the second gas,
= k BT
2 1
3 l2 =
-23
= ´ (1. 38 ´ 10 ) ´ 273 2 pn 2 d 22
2
\ Ratio of l1 and l 2 ,
= 5. 65 ´ 10 -21 J/mol
l1 n 2 d 22 3 5 5 15
At 100° C (T = 373 K ), the energy is = = ´ ´ =
l 2 n1 d12 5 4 4 16
3
= ´ (1. 38 ´ 10 -23 ) ´ 373 Þ l1 : l 2 = 15 : 16
2
CBSE Term II Physics XI 99

Chapter
Practice
PART 1 6. According to the kinetic theory of gases, the
temperature of a gas is a measure of average
Objective Questions (a)
(b)
velocities of its molecules
linear momenta of its molecules
(c) kinetic energies of its molecules
l
Multiple Choice Questions (d) angular momenta of its molecules
1. According to atomic hypothesis, 7. A cylinder containing an ideal gas is in vertical
(a) atoms attract each other when they are little distance position and has a piston of mass M that is able to
apart move up or down without friction (figure). If the
(b) atoms repel if they being squeezed into one another temperature is increased, then
(c) Both (a) and (b)
M
(d) Neither (a) nor (b)
2. The mass of 22.4 L of any gas is equal to its
molecular weight in grams at
(a) 270 K and 1 atm
(b) 273 K and 1 atm
(c) 273 K and 10 atm
(d) 270 K and 10 atm
3. Which one of the following graphs represents the
(a) both p and V of the gas will change
behaviour of an ideal gas?
(b) only p will increase according to Charles’ law
(c) V will change but not p
pV pV (d) p will change but not V
(a) (b)
8. At what temperature, the kinetic energy of gas
T T molecule is half of the value at 27°C?
(a) 13.5°C (b) 150°C
(c) 75 K (d) − 123 ° C
(c) pV (d) pV
9. The root-mean-square speed of a nitrogen molecule
at 300 K is
T T (a) 534 ms −1 (b) 534 ms −1
4. Boyle’s law is applicable for an [NCERT Exemplar] (c) 267 ms −1 (d) 216 ms −1
(a) adiabatic process (b) isothermal process 10. Oxygen and hydrogen are at the same temperature
(c) isobaric process (d) isochoric process T. The ratio of the mean kinetic energy of oxygen
5. The collisions of the molecules of an ideal gas are molecules to that of the hydrogen molecules will be
(a) elastic (b) inelastic (a) 16 : 1 (b) 1 : 1
(c) completely inelastic (d) partially elastic (c) 4 : 1 (d) 1 : 4
100 CBSE Term II Physics XI

11. A molecule moving along a straight line possess …. 19. Assertion In gases, the molecules move faster and
degree of freedom. longer distances without colliding.
(a) one (b) two (c) three (d) four Reason In gases, the interatomic forces are
12. The internal energy of 2 moles of a monoatomic gas negligible.
is 20. Assertion A gas that satisfy pV = µ RT at all
3
(a) RT (b) 3 RT (c) 2 RT (d) 5 RT pressures and temperatures is known as ideal gas.
2
Reason A real gas at low pressure and high
13. The value of γ for a diatomic molecule (vibrational temperature behaves like ideal gas.
mode) is 21. Assertion The total translational kinetic energy of
9 7 7 5
(a) (b) (c) (d) all the molecules of a given mass of an ideal gas is
7 9 5 7 1.5 times the product of its pressure and volume.
14. During an adiabatic process, the pressure of a gas Reason The molecules of a gas collide with each
is found to be proportional to the cube of its other and the velocities of the molecules change
Cp due to the collision.
temperature. The ratio of for the gas is
CV
22. Assertion Total kinetic energy of the chamber will
4 5 3
(a) (b) 2 (c) (d) increase by the translational motion.
3 3 2
Reason If a gas chamber containing a gas is moved
15. As temperature tends to zero, i.e. T → 0 , specific translationally, then temperature of gas will
heat of all substances increase.
(a) approaches zero (b) approaches infinity
23. Assertion Kinetic energy of molecules with any
(c) may be zero or infinity (d) None of these
reference, must be positive.
16. The total energy for one mole of solid is Reason In the expression of kinetic energy, the
(a) 2 RT (b) 3 RT (c) 4 RT (d) ( 3 / 2 ) RT velocity appears with power 2.
17. The mean free path for air molecule with average 24. Assertion In pressure - temperature (p-T) phase
speed 18 . 5 ms −1 at STP is
diagram of water, the slope of melting curve is
(Take, d = 2 × 10 −10 m and n = 2.7 × 10 25 m −3 )
found to be negative.
(a) 3 . 5 × 10 −7 m (b) 4 × 10 −7 m
(c) 2 . 9 × 10 −7 m (d) 5 × 10 −7 m Reason Ice contracts on melting, to form water.

18. We took two separate gases with the same number 25. Assertion Pressure of an ideal gas is equal to
densities for both. If the ratio of the diameters of (1 / 3) nm v 2 .
their molecules is 4 : 1, then ratio of their mean free Reason Pressure of an ideal gas increases with
paths is increase of volume.
(a) 1 : 4 (b) 4 : 1 (c) 2 : 1 (d) 1 : 16 26. Assertion Degree of freedom of a monoatomic gas
is always three, whether we consider vibrational
l
Assertion-Reasoning MCQs effect or not.
Direction (Q. Nos. 19-26) Each of these questions Reason At all temperatures (low or high),
contains two statements Assertion (A) and Reason (R). vibrational kinetic energy of an ideal gas is zero.
Each of these questions also has four alternative
choices, any one of which is the correct answer. You l
Case Based MCQs
have to select one of the codes (a), (b), (c) and (d) given
below. 27. Direction Read the following passage and answer
the questions that follows
(a) Both A and R are true and R is the correct
explanation of A. Law of Equipartition of Energy
For a dynamic system in thermal equilibrium, the
(b) Both A and R are true, but R is not the correct
total energy is distributed equally amongst all the
explanation of A.
degree of freedom and the energy associated with
(c) A is true, but R is false.
each molecule per degree of freedom is 1 k B T,
(d) A is false and R is also false. 2
CBSE Term II Physics XI 101

where k B = 1.38 × 10 − 23 JK −1 is Boltzmann constant 2. Estimate the total number of air molecules
and T is absolute temperature of system on the (inclusive of oxygen, nitrogen, water vapour and
kelvin scale. For a monoatomic gas in thermal other constituents) in a room of capacity 25.0 m 3 at
equilibrium at temperature T, the average value of a temperature of 27°C and 1 atm pressure. (Take,
translational energy of the molecule is k B = 1.38 × 10 −23 JK −1 ) [NCERT]
1 1 1 3. Estimate the fraction of molecular volume to the
Et = mv x2 + mv y2 + mv z2 actual volume occupied by oxygen gas at STP. Take,
2 2 2
the diameter of an oxygen molecule to be 3 Å.
Translational energy of the molecules, [NCERT]
3 4. Why temperature less than 0K is not possible?
E t = kB T
2 5. A gas is contained in a closed vessel. How pressure
(i) Law of equipartition of energy is used to predict due to the gas will be affected, if force of attraction
the specific heats of between the molecules disappear suddenly?
(a) gases (b) solids
6. A flask contains argon and chlorine in the ratio of
(c) Both (a) and (b) (d) Neither (a) nor (b)
2 : 1 by mass. The temperature of the mixture is
(ii) Diatomic molecule (rigid rotator) has 27°C. Obtain the ratio of
(a) three translational degrees of freedom
(i) average kinetic energy per molecule and
(b) two rotational degrees of freedom
(c) Both (a) and (b)
(ii) root-mean-square speed ( v rms ) of the molecules of
the two gases.
(d) All of the above
(Take, atomic mass of argon = 39. 9 u and
(iii) Choose the correct option. molecular mass of chlorine = 70. 9 u) [NCERT]
1
(a) Each translational mode contributes k BT average 7. We have 0.5 g of hydrogen gas in a cubic chamber
2
energy.
of size 3 cm kept at NTP. The gas in the chamber is
1 compressed keeping the temperature constant till a
(b) Each rotational mode contributes k BT average final pressure of 100 atm. Is one justified in
2
energy. assuming the ideal gas law, in the final state?
(c) Vibrational mode contributes k BT average energy. (Hydrogen molecules can be considered as spheres
(d) All of the above of radius 1 Å). [NCERT Exemplar]
(iv) Molecules of CO at moderate temperature have 8. Three vessels of equal capacity have gases at the
energy same temperature and pressure. The first vessel
7 5 3 1 contains neon (monoatomic), the second contains
(a) k BT (b) k BT (c) k BT (d) k BT chlorine (diatomic) and the third contains uranium
2 2 2 2
hexafluoride (polyatomic). Do the vessels contain
(v) The mean kinetic energy of one mole of gas per equal number of respective molecules?
degree of freedom (on the basis of kinetic theory of
Is the root-mean-square speed of molecules, the
gases) is same in the three cases? If not, in which case is v rms ,
1 3 3 1
(a) kT (b) kT (c) RT (d) RT the largest? [NCERT]
2 2 2 2
9. A tank used for filling helium balloons has a volume
of 0.6 m 3 and contains 2 mole of helium gas at
20.0°C. Assuming that, the helium behaves like an
PART 2 ideal gas.

Subjective Questions (i) What is the total translational kinetic energy of


the molecules of the gas?
(ii) What is the average kinetic energy per
l
Short Answer (SA) Type Questions molecule?
1. Molar volume is the volume occupied by one mole 10. At what temperature, is the root-mean-square
of any (ideal) gas at standard temperature and speed of an atom in an argon gas cylinder equal to
pressure (STP : 1 atm pressure, 0°C). Show that, it the rms speed of a helium gas atom at –20°C?
is 22.4 L. [NCERT] (Take, atomic mass of Ar = 39. 9 u and He = 4. 0 u)
[NCERT]
102 CBSE Term II Physics XI

11. What do you understand by degree of freedom? 26. Estimate the average thermal energy of a helium atom
12. Equal masses of monoatomic and diatomic gases at (i) room temperature (27 ° C), (ii) the temperature on
are supplied heat at the same temperature, the surface of the sun (6000 K) and (iii) the
pressure and volume. temperature of 10 million kelvin (the typical core
If same amount of heat is supplied to both the temperature in the case of a star). [NCERT]
gases, then which of them will undergo greater 27. A meter long narrow bore held horizontally (and
temperature rise? closed at one end) contains a 76 cm long mercury
13. At room temperature, diatomic gas molecule has thread, which traps a 15 cm column of air. What
five degrees of freedom. At high temperature, it happens, if the tube is held vertically with the open
has seven degrees of freedom, explain. end at the bottom? [NCERT]

14. Calculate the number of degrees of freedom in 28. An oxygen cylinder of volume 30 L, has an initial
3
15 cm of nitrogen at NTP. gauge pressure of 15 atm and a temperature of 27 °C.
After some oxygen is withdrawn from the cylinder, the
15. What is basic law followed by equipartition of gauge pressure drops to 11 atm and its temperature
energy? drops to 17 °C. Estimate the mass of oxygen taken out
16. A diatomic gas is heated in a vessel to a of the cylinder.
temperature of 10000 K, then each molecule ( Take, R = 8.31 mol −1K −1 and molecular mass of
possess an average energy E1 . After sometime, a O 2 = 32 u)
few molecule escape into the atmosphere at 300 29. The container shown in figure has two chambers,
K. Due to which, their energy changes to E 2 . separated by a partition of volumes V1 = 2. 0 L and
E V2 = 3. 0 L. The chambers contain µ1 = 4. 0 and
Calculate the ratio of 1 .
E2 µ 2 = 5. 0 moles of a gas at pressures p1 = 1. 00 atm and
p 2 = 2. 00 atm. Calculate the pressure after the
17. What will be the internal energy of 8 g of partition is removed and the mixture attains
oxygen at STP?
equilibrium. [NCERT Exemplar]
18. Find the value of specific heat capacity for V1 V2
solids. µ1 µ2
19. Calculate the value of specific heat capacity for p1 p2
one mole of water in J/kg.
20. Calculate the mean free path of a molecule of a 30. Explain, why
gas at a room temperature and one atmospheric (i) there is no atmosphere on moon.
pressure. The radius of the gas molecules (ii) there is fall in temperature with altitude.
(average) is 2 × 10 −10 m. [NCERT Exemplar]
21. Explain qualitatively, how the extent of 31. Calculate the temperature of atoms at which rms
Brownian motion is affected by the speed of argon gas is equal to the rms speed of Helium
(i) size of the Brownian particle, gas atoms at −10 ° C ? (Take, atomic mass of Ar = 39. 9 u
and that of He = 4u)
(ii) density of the medium,
(iii) temperature of the medium 32. If one mole of a monoatomic gas is mixed with three
moles of a diatomic gas, what is the molar specific heat
(iv) and viscosity of the medium. [NCERT]
of mixture at constant value?
22. Although velocity of air molecules is very fast (Take, R = 8.31 J mol −1 K −1 )
but fragnance of a perfume spreads at a much 33. A cylinder of fixed capacity contains 44.8 L of helium
slower rate, explain. gas at STP. Calculate the amount of heat required to
23. Write the differences between ideal gas and raise the temperature of container by 15°C? (Take,
real gas. R = 8. 31 J mol −1K −1 )
24. Write the assumptions of kinetic theory of gases. 34. A gaseous mixture contain 16 g of helium and
25. Find the expression for the average kinetic 16 g of oxygen, then calculate the ratio of C p / C V of the
energy of a molecule of an ideal gas. mixture.
CBSE Term II Physics XI 103

35. Three moles of a diatomic gas is mixed with two 40. A gas in equilibrium has uniform density and
moles of monoatomic gas. What will be the pressure throughout its volume. This is strictly true
molecular specific heat of the mixture at constant only, if there are no external influences. A gas
volume? (Take, R = 8.31 J mol −1K −1 ) column under gravity, e.g. does not have uniform
density (and pressure). As you might expect, its
36. Give a formula for mean free path of the molecules density decreases with height. The precise
of a gas. Briefly explain, how its value is affected by dependence is given by the so called law of
(i) change in temperature and atmosphere.
(ii) change in pressure. n 2 = n1 exp [ − mg ( h 2 − h1 )/ k B T]
where, n 2 and n1 refer to number density at heights
l
Long Answer (LA) Type Questions
h 2 and h1 , respectively. Use this relation to derive
37. Given below are densities of some solids and the equation for sedimentation equilibrium of a
liquids. Give rough estimate of the size of their suspension in a liquid column.
atoms. [NCERT] n 2 = n1 exp [ − mg NA (ρ − ρ′ )( h 2 − h1 )/(ρ RT)]
Substance Atomic mass (u)
Density where, ρ is the density of the suspended particle
(10 −3 kgm −3 ) and ρ′ is that of surrounding medium.
Carbon (diamond) 12.01 2.22
(QN A is Avogadro’s number and R is the universal
Gold 197.00 19.32
gas constant) [NCERT]
Nitrogen (liquid) 14.01 1.00
Lithium 6.94 0.53
41. Consider an ideal gas with following distribution of
speeds. [NCERT Exemplar]
Fluorine (liquid) 19.00 1.14
Speed (m/s) % of molecules
38. Figure shows plot of pV/ T versus p for 1.00 ×10 −3 kg
of oxygen gas at two different temperatures. 200 10
Y 400 20
600 40
pV
(JK–1) T1 800 20
T T2
1000 10
O X
p [NCERT] (i) Calculate v rms and hence T.
(i) What does the dotted plot signify? (Take, m = 3.0 × 10 −26 kg)
(ii) Which is true T1 > T2 or T1 < T2 ? (ii) If all the molecules with speed 1000 m/s escape
(iii) What is the value of pV / T, where the curves from the system, calculate new v rms and hence T.
meet on the Y-axis?
42. A box of 1.00 m 3 is filled with nitrogen at 1.50 atm
(iv) If we obtain similar plots for 1 × 10 −3 kg of at 300 K. The box has a hole of an area is
hydrogen, would we get the same value of pV/T at 0.010 mm 2 . How much time is required for the
the point where the curves meet on the Y-axis? If pressure to reduce by 0.10 atm, if the pressure
not, what mass of hydrogen yields the same value outside is 1 atm ? [NCERT Exemplar]
of pV/T (for low pressure, high temperature
region of the plot)? (Take, molecular mass of 43. (i) What do you understand by specific heat
capacity of water?
H 2 = 2. 02 u, O 2 = 32.0 u and R = 8.31 Jmol −1K −1 )
(ii) If one mole of ideal monoatomic gas ( γ = 5/ 3) is
39. You are given, the following data about a group of mixed with one mole of diatomic gas ( γ = 7 / 5),
particles, where n i represents the number of what is the value of γ for the mixtures?
molecules with speed v i (Here, γ represents the ratio of specific heat at
ni 2 4 8 6 3 constant pressure to that at constant volume)
Vi ( ms −1 ) 1.0 2.0 3.0 4.0 5.0 44. (i) Define mean free path.
(ii) Derive an expression for mean free path of a gas
Calculate (i) average speed, (ii) rms speed molecule.
(iii) and most probable speed.
104 CBSE Term II Physics XI

45. Estimate the mean free path and collision frequency l


The size of a gas particle is as neglighible as
of a nitrogen molecule in a cylinder containing compared to the volume of the container in which
nitrogen at 2.0 atm and temperature 17°C. Take, the the gas is placed.
radius of a nitrogen molecule to be roughly 1.0 Å. Gases are mostly empty space and this is evident
Compare the collision time with the time, the because gases can be easily compressed.
molecule moves freely between two successive l
Gases are in rapid motion, Brownian motion and
collisions. they undergeo elastic collisions with each other and
(Take, molecular mass of N 2 = 28. 0 u ) [NCERT] the walls of the container, i.e. momentum and
energy is transferred during collisions.
l
Case Based Questions l
Gas molecules do not interact with each other
46. Direction Read the following passage and answer except for colliding with each other.
the questions that follows (i) Define an isolated system.
Kinetic Molecular Theory (ii) Write the equation of state of an ideal gas and
The kinetic molecular theory is a model that identify each term of this equation.
explains some of the behavious of gases. The kinetic (iii) Why the collisions between the gas molecules are
molecular theory is given by some assumptions considered elastic?
Chapter Test
Multiple Choice Questions
1. Temperature remaining constant, the pressure of gas is decreased by 20%. The percentage change in volume
(a) increases by 20% (b) decreases by 20%
(c) increases by 25% (d) decreases by 25%
2. The internal energy of an ideal gas is in the form of
(a) kinetic energy of molecules
(b) potential energy of molecules
(c) Both kinetic and potential energies of molecules
(d) gravitational potential energy of molecules
3. An inflated rubber balloon contains one mole of an ideal gas, has a pressure p, volume V and temperature T. If the
temperature rises to 1.1 T and the volume is increased to 1.05 V, the final pressure will be [NCERT Exemplar]
(a) 1.1 p (b) p
(c) less than p (d) between p and 1.1 p
4. The two gases with the ratio 3 : 2 of their masses in a container are at a temperature T. The ratio of the kinetic energies
of the molecule of two gases is
(a) 3 : 2 (b) 9 : 4
(c) 1 : 1 (d) 4 : 9
5. The ratio of the molar heat capacities of a diatomic gas at constant pressure to that at constant volume is
7 3
(a) (b)
2 2
3 7
(c) (d)
5 5

Short Answer Type Questions


6. Draw p-V curves showing deviations from ideal behaviour for a given mass of a gas for two different temperatures.
7. Why does specific heat of gases increases with their atomicity?
8. Calculate the temperature at which the rms speed of CO 2 gas molecule will be 1 kms −1. (Take, molecular mass of
CO 4 = 44 u) (Ans. 1 .8 × 10 3 K)
9. What is the kinetic energy of translation of one molecule of a gas at 300 K ?
Gas is having three degree of freedom and Boltzmann constant, kB = 1 . 38 × 10 −23 JK −1 . (Ans. 6.21 × 10 −21 J)
10. State Charles’ law, obtain it from kinetic theory of gases.
Long Answer Type Questions
11. Using the law of equipartition of energy, show that for an ideal gas having f degrees of freedom is given by γ = 1 + 2/f.
12. What is Maxwell-Boltzmann law with regard to speed distribution amongst molecules, state and explain?

Answers
Multiple Choice Questions For Detailed Solutions
1. (c) 2. (a) 3. (d) 4. (c) 5. (d) Scan the code
106 CBSE Term II Physics XI

EXPLANATIONS
1
PART 1
= (27 + 273 ) × 2 = 150 K
1. (c) Atoms attract when they are little distance apart and 1
repel, if they being squeezed into one another. = − 123 ° C
2. (b) The mass of 22.4 L of any gas at STP (standard
9. (c) Root-mean-square speed,
temperature 273 K and pressure 1 atm) is equal to its
molecular weight in grams. 3RT 3 × 8. 314 × 300
v rms = = ≈ 267 ms −1
3. (c) We know that, ideal gas equation, M 28
pV = nRT 10. (b) The mean kinetic energy for gas molecules,
⇒ pV ∝ T 3
E = k BT
pV 2
⇒ = constant
T ⇒ E ∝T
So, the graph passes through origin. E O T1
∴ =
4. (b) Boyle’s law is applicable, when temperature is EH T2
constant. According to the question, both gases are at the same
i.e. pV = nRT = constant temperature T.
⇒ pV = constant (at constant temperature) EO T 1
So, = =
1 EH T 1
i.e. p∝ (where , p = pressure and V = volume)
V ⇒ E O : EH = 1 : 1
So, this process can be called as isothermal process. 11. (a) As, the molecule is moving along a straight line, so it
5. (a) According to the kinetic theory of gases, the collision has only one translational degree of freedom.
among molecules and the collision of molecule with the 3 
12. (b) Internal energy, U =  k BT 2N A
walls of container are elastic. 2 
6. (c) According to the kinetic theory of gases, the = 3 ( k B × N A ) T = 3RT
temperature of a gas is a measure of kinetic energies of
13. (a) The diatomic molecule has 3 translational and 2
molecules of a gas.
rotational degree of freedom.
7. (c) Consider a frictionless cylinder and piston system of
As, vibrational mode is also considered here, so total
mass M, filled with an ideal gas.
degree of freedom, f = 5 + 2 = 7
M C 2
As, we know, γ = p = 1 +
pa CV f
2 9
=1 + =
pa Mg/A 7 7
p 14. (d) As per question, p ∝ T 3 … (i)
Also, pV = nRT
A
p ⇒ pV ∝ T … (ii)
From Eqs. (i) and (ii), we get
The pressure inside the gas will be p = p a + Mg/ A p ∝ ( pV )3
where, p a = atmospheric pressure, ⇒ p V = constant
2 3

A = area of cross-section of the piston or pV 3/ 2 = constant


and Mg = weight of piston. In adiabatic process, pV γ = constant
Hence, p = constant. 3
∴ γ=
When temperature increases as pV = nRT, hence 2
volume V increases at constant pressure. 15. (a) The prediction of specific heats, based on law of
8. (d) The kinetic energy of gas molecules is given by equipartition of energy, are independent of temperature.
3 (KE1) T1 But as we go to low temperatures, there is a marked
KE = RT ⇒ = departure from this prediction. Specific heat of all
2 (KE 2 ) T2
substances approach zero as T → 0 K.
(KE 2 )
⇒ T2 = T1 × This is related to the fact that, degrees of freedom get
(KE1) frozen and ineffective at low temperatures.
CBSE Term II Physics XI 107

16. (b) By law of equipartition of energy, average energy Also, ice contracts on melting.
associated with an atom due to its oscillation in one Therefore, both A and R are true but R is not the correct
1  explanation of A.
dimension = 2 ×  k BT = k BT
2  25. (c) Pressure of an ideal gas
∴In three dimension (3D) average energy per atom = 3 k BT. 1
= nm v 2
∴ Total energy of one mole of solid, 3
U = ( 3 k BT ) × N A = 3 RT (Q k BN A = R) Also, pressure of an ideal gas,
17. (c) For air at STP, n = 2.7 × 10 m25 −3 µRT
p=
1 V
d = 2 × 10 −10 m ⇒ l = 1
2 nπd 2 If µ and T are constants, then p ∝
V
On putting values, l = 2.9 × 10 −7 m So, pressure of an ideal gas increases with the decrease of
1 volume.
18. (d) As, mean free path, l =
2 n πd 2 Therefore, A is true but R is false.
1 26. (c) In an ideal gas, there is no inter-particle interaction,
⇒ l∝ 2 (Q n1 = n 2 )
d so vibrational kinetic energy remains constant,
2
l1 d 22  1  1 irrespective of temperature.
⇒ = 2 =  =
l2 d1  4  16 If the gas is monoatomic, then no vibrational energy is
there, so the degree of freedom is always three.
19. (a) The interatomic forces are negligible in gases.
Therefore, A is true but R is false.
Therefore in gases, the molecules are free and move
longer distances without colliding, with faster speed. 27. (i) (c) Law of equipartition of energy is used to predict
the specific heat of gases and solids.
Therefore, both A and R are true and R is the correct
explanation of A. (ii) (c) The diatomic molecules (without vibrational
mode) like O2 and N2 has three translational degrees
20. (b) A real gas at low pressure and high temperature
of freedom and two rotational degrees of freedom.
behaves like an ideal gas
(iii) (d) According to law of equipartition of energy, each
and satisfies pV = µRT.
translational mode and rotational mode contributes
Therefore, both A and R are true but R is not the correct 1
explanation of A. k BT average energy to the total energy. If the
2
21. (b) Total translational kinetic energy of all molecules molecule also has vibrations, then it contributes k BT
(n moles) of a given mass of an ideal gas average energy.
3 3
= µRT = pV = 1. 5 pV (QpV = µRT ) (iv) (a) Molecules of CO at moderate temperature
2 2 possess vibrational modes, i.e. it oscillates like
Due to collision of molecules of gas with each other, their one-dimensional oscillator.
velocities continuously change. As, each vibrational mode contributes two square
Therefore, both A and R are true but R is not the true 1 
terms, so this mode will contribute 2  k BT
explanation of A. 2 
22. (c) The translational motion of the gas chamber increases energy.
the total kinetic energy of the chamber but it does not CO molecule has 3 translational and 2 rotational
contribute to the internal energy of the system. 1
So, the temperature of gas in the chamber remains same. modes and each of them contributes k BT to the
2
Therefore, A is true but R is false. energy.
23. (a) Kinetic energy of the molecules are given by ∴ Total energy,
1 3
mv 2 = k BT 1  1  1 
2 2 E = 3  k BT + 2  k BT + 2  k BT
2  2  2 
Since, temperature in kelvin scale cannot be negative and 14243 14243 14243
Translational mode Rotational mode Vibrational mode
square of the velocity also remains positive, therefore
kinetic energy of molecules with any reference must be 7
= k BT
positive. 2
Therefore, both A and R are true and R is the correct (v) (d) Mean kinetic energy of one mole of gas per degree
explanation of A. of freedom,
24. (b) The negative slope of the melting curve indicates that f RT 1
E= = RT
the melting point of ice decreases with increase in 2 f 2
pressure.
108 CBSE Term II Physics XI

PART 2 (ii) If m is mass of single molecule and M is the


molecular mass, then
1. As, for one mole of ideal gas, pV = µRT (here, µ = 1) 1 3
mv 2rms = k BT
RT 2 2
pV = RT ⇒ V =
p = constant at a given temperature
Putting, R = 8. 31 J mol −1 K −1, T = 273K (v2 ) ( m )Cl 2 M Cl 2 70. 9
∴ 2rms Ar = = = = 1.777
p = 1 atm = 1.013 × 10 5 N - m −2 ( v rms )Cl 2 ( m )Ar M Ar 39. 9
8.31 × 273 ( v rms )Ar
V= = 0.0224 m 3 o = 1.777 = 1. 333
1.013 × 10 5 ( v rms )Cl2
= 22.4 L (Q 1m 3 = 10 3L) 7. We have, 0.25 × 6 × 10 23 molecules and each of volume
2. Given, V =25. 0 m , T = 273 + 27 = 300 K
3
10 −30m3.
k B = 1. 38 × 10 −23 JK −1 Molecular volume = 2.5 × 10 −7 m 3
Now, pV = µRT ⇒ pV = µ (N A k B ) T Supposing, ideal gas law is valid.
⇒ pV = N ′ k BT V (3)3 × 10 −6
Final volume = in = ≈ 2.7 × 10 −7 m 3
(N ′ is total number of molecules) 100 100
pV (1.01 × 10 5 ) × 25 which is about the molecular volume. Hence,
∴ N′ = = = 6.10 × 10 26
k BT (1.38 × 10 −23 ) × 300 intermolecular forces cannot be neglected, therefore the
3. Given, d = 3 Å ideal gas situation does not hold.
d 8. As three vessels are identical, i.e. they have same volume.
r= = 1 . 5 × 10 −10 m = 1 . 5 × 10 −8 cm Now at constant pressure, temperature and volume, the
2 three vessels will contain equal number of molecules (by
4
Molecular volume, V = πr 3N Avogadro’s law) and is equal to Avogadro’s number,
3 N A = 6.023 × 10 23.
4
= × 3.14 × (1.5 × 10 −8 )3 × 6.023 × 10 23 = 8 . 52 cc Q v rms =
3 k BT
⇒ v rms ∝
1
3 m m
Actual volume occupied by 1 mole of oxygen at STP,
where, m is mass of single gas molecule as neon has the
V′ = 22400 cc smallest mass, so rms speed will be greatest in case of
V 8. 52
∴ = = 3. 8 × 10 −4 neon.
V ′ 22400 9. (i) We know that, (KE )trans = nRT
3
4. According to the kinetic interpretation of temperature, 2
Absolute temperature ∝ Average KE of molecules Given, n = 2 mole, T = 273 + 20 = 293 K
As, the heat is removed, the temperature falls and 3
⇒ (KE )trans = (2) (8.31) (293)
velocity of molecules decreases. At absolute zero, the 2
molecule motion ceases, i.e. kinetic energy becomes zero. = 7.3 × 10 3 J
As, kinetic energy cannot be negative, so no further (ii) Average KE per molecule
decrease in kinetic energy is possible. Hence, 1 3 3
temperature cannot be decreased below 0K. (KE )av = mv 2rms = k BT = (1.38 × 10 −23 ) (293)
2 2 2
5. As force of attraction between molecules disappears, then
= 6.07 × 10 −21 J
molecules will hit the wall with more speeds, hence rate
of change of momentum will increase. 10. Let C and C′ be the rms velocity of argon and helium gas
∆p atoms at temperatures T K and T ′K, respectively.
As, we know, F =
∆t Given, M = 39.9, M ′ = 4. 0, T =?,
where, F is average force on the wall due to molecules, T ′ = − 20 + 273 = 253 K
∆p is change in momentum and ∆t is the time duration. Now, v=
3RT
=
3RT
Due to increase in ∆p, force F will also increase, hence M 39. 9
F
pressure, P = will increase. Here, A is area of one wall. 3RT ′ 3R × 253
A and v′ = =
M′ 4
6. (i) The average kinetic energy per molecule of any gas
3RT 3R × 253
is ( 3 / 2 ) k BT. It depends only on temperature and Since, v = v ′, therefore =
not on the nature of the gas. As both argon and 39. 9 4
chlorine have the same temperature in the flask, the 39.9 × 253
or T= = 2523.7K
ratio of average KE per molecule of the two gases is 4
1 : 1.
CBSE Term II Physics XI 109

11. The total number of coordinates or independent ∴ Internal energy of 8 g of oxygen,


quantities required to describe completely the position 5
and configuration of a dynamical system is known as = 0.25 × RT
2
number of degrees of freedom of the system. It is 5
represented by f and expressed as = 0.25 × × 8.31 × 273
2
Degree of freedom, f = 3N − K
= 1417.9 J
where, N is the number of particles in a system and K is
18. In solids, each atom vibrates about its mean position, let
number of independent relations between the particles.
us consider one mole of solid containing N A atoms. By
12. For monoatomic gas, temperature rise will be greater law of equipartition of energy, average energy associated
because monoatomic gas possesses only translational with an atom due to its oscillation in one dimension
degree of freedom whereas diatomic gas translation,
1 
rotation and vibrational (at higher temperature), so = 2 ×  k BT = k BT
2 
temperature rise for diatomic gases is lower.
13. At low temperature, diatomic gas has three translational
∴In three dimension (3D), average energy per atom= 3 k BT.
and two rotational degrees of freedom, so total number of ∴ Total energy of one mole of solid,
degrees of freedom is five. U = ( 3 k BT ) × N A = 3 RT (Q k BN A = R)
But at high temperature, gas molecule starts to vibrate
According to first law of thermodynamics,
which give two additional degrees of freedom, i.e. seven.
∆Q = ∆U + ∆W or ∆Q = ∆U + p∆V
14. Number of nitrogen molecules in 22400 cm 3 of gas at
NTP = 6.023 × 10 23 In case of solid, ∆V is negligible.
∴ Number of nitrogen molecules in 15 cm 3of gas at NTP ∴ ∆Q ≈ ∆U
6.023 × 10 23 × 15 ∆Q ∆U
= = 4.03 × 10 20 Molar specific heat of solid, C = =
22400 ∆T ∆T
Nitrogen is a diatomic gas. Specific heat capacity of solids, C = 3R
Number of degrees of freedom of nitrogen (diatomic) ⇒ C = 24.93 J mol − 1K − 1 (QR = 8.31 Jmol −1K −1)
molecule at 273 K = 5 19. If we consider water molecule as a solid made up of
∴ Total degrees of freedom of 15 cm 3 of gas 3 atoms (2 hydrogen and 1 oxygen) and each atom is free
to vibrate in three dimensions about its mean position.
= 4.03 × 10 20 × 5 = 2.015 × 10 21
By using law of equipartition of energy, average energy
15. The law of equipartition of energy for any dynamical
associated with one atom of water molecule
system in thermal equilibrium, the total energy is
1 
distributed equally amongst all the degrees of freedom. = 2 ×  k BT = k BT
2 
The energy associated with each molecule per degree of
1 In three dimension, the average energy per atom of water
freedom is k BT, where k B is Boltzmann’s constant and T
2 molecule = 3k BT
is temperature of the system. ∴ Total energy of one molecule of water,
16. Number of degrees of freedom of diatomic gas at = 3 × ( 3 k BT ) = 9 k BT
10000 K = 7 . Now, total energy of 1 mole of water,
Number of degrees of freedom of diatomic gas at U = 3 × 3 k BT × N A = 9RT
300 K = 5
∴ Molar specific heat of water,
7  ∆ Q ∆U
  k BT1 C= = = 9R
E1  2  7 T 7 10000 140
∴ = = × 1 = × = ∆T ∆T
E 2  5 5 T2 5 300 3
  B 2
k T For one mole, C = 9 × 8. 31 ≈ 75 J/kg
 2
20. Given, T = 27 ° C = 273 + 27 = 300 K,
17. Oxygen is a diatomic gas.
p = 1 atm = 1.01 × 10 5N / m 2
Number of moles of O2 gas and d = 2 × 2 × 10 −10 m
Atomic weight 8 1 = 4 × 10 −10 m
= = = = 0.25
Molecular weight 32 4 Q Mean free path, λ = k BT / 2 πd 2p
∴ Energy associated with 1 mole of oxygen, 1.38 × 10 −23 × 300
5 =
U = RT 1.414 × 3.14(4 × 10 −10 )21.013 × 10 5
2
= 5.75 × 10 −8m
110 CBSE Term II Physics XI

21. The effect of the various factors on the Brownian motion (iii) The size of a molecule is much smaller than the
is as follows average separation between the molecules. At
ordinary pressure and temperature, the average
Factors Effects distance between molecules is about 20 Å, whereas
(i) Decrease in the size of Increase of Brownian size of a molecule is 2 Å.
the Brownian particle. motion. (iv) There is no intermolecular forces between
(ii) Decrease in the density Increase of Brownian molecules of gas except during collision.
of the medium. motion. (v) The collision between molecules among themselves
or between molecules and walls are perfectly elastic
(iii) Increase in temperature Increase of Brownian
(i.e. total momentum and total kinetic energy of
of the medium. motion.
molecules are conserved, however only their
(iv) Increase in viscosity of Decrease of Brownian velocities will change).
the medium. motion. (vi) The duration of collision between two molecules is
negligible as compared to time interval of two
22. This is because scent vapour molecules do not travel successive collisions, i.e. collisions are instantaneous.
uninterrupted, they undergo a number of collisions and
(vii) The density and the distribution of molecules is
trace a zig-zag path. Due to which their effective uniform throughout the gas.
displacement per unit time is small, so spreading occurs
25. The average kinetic energy of a molecule depends on the
at a much slower rate.
absolute temperature of the gas. It is the kinetic
23. Differences between ideal gas and real gas are given below interpretation of temperature.
Let us consider a sample of an ideal gas having N number
Ideal gas Real gas of molecules. Let the volume of the gas be V, pressure is
(i) It obeys ideal gas It does not obey, p and temperature be T.
equation, pV = µRT at all pV = µRT at all values of From the expression of an ideal gas, pressure,
temperatures and temperature and pressure. 1
pressures. p = nmv 2 …(i)
3
(ii) The volume of the The volume of the Multiplying both sides with V, we get
molecules of an ideal gas molecules of a real gas 1
is zero. is non-zero. ⇒ pV = ( nV ) mv 2
3
(iii) There is no intermolecular There is intermolecular 2 1
force between the force of attraction or or pV = ( nV ) mv 2
molecules. repulsion depending on 3 2
whether intermolecular 2 1 
⇒ pV = N  mv 2 …(ii)
separation is larger or 3 2 
small.
Here, number of molecules in the sample, N = nV
(iv) There is no intermolecular Potential energy ( U) does 1
potential energy ( U) not equal to zero as and mv 2 is the average kinetic energy of the molecules
2
because intermolecular intermolecular force (F ) is
of the gas.
force (F ) is zero. not zero.
So, total internal energy of the gas,
(v) It has only kinetic energy. It has both kinetic and 1 
potential energies. E = N  mv 2 …(iii)
2 
(vi) At absolute zero, the All real gases get liquified
From Eqs. (ii) and (iii), we get
volume, pressure and before reaching absolute
internal energy become zero. The internal energy 2
pV = E …(iv)
zero. of the liquified gas is not 3
zero. For ideal gas, we can write
pV =µRT
24. Following are the assumptions of kinetic theory of gases
or, pV = µk BN A T = k B(µN A ) T
(i) A given amount of gas consists of a very large
number of molecules (of the order of Avogadro’s or, pV = k BNT …(v)
number10 23) and all molecules are identical in all (N = µN A = total number of molecules)
respect. Combining Eqs. (iv) and (v), we get
(ii) The molecules of a gas are in a state of incessant 2 E 3
E = k BNT ⇒ = k BT …(vi)
random motion in all directions with different speeds, 3 N 2
move freely in straight lines following Newton’s first 1 3
law. ∴ Average kinetic energy = mv 2 = k BT
2 2
CBSE Term II Physics XI 111

26. (i) Given, T = 27 ° C = (273.15 + 27 ) = 300.15 K 28. Given, absolute pressure, p1 = (15 + 1 ) atm
Average thermal energy, E = k BT
3 (Q absolute pressure = gauge pressure + 1 atm)
2 = 16 × 1. 013 × 10 5 Pa
(where, k B = Boltzmann constant V1 = 30 L = 30 × 10 −3 m 3
= 1.38 × 10 −23 J K −1) T1 = 273.15 + 27 = 300.15 K
3 Using ideal gas equation, pV = nRT
E = × 1.38 × 10 −23 × 300.15 = 6.21 × 10 −21J
2 pV p1V1 16 × 1.013 × 10 5 × 30 × 10 −3
or n= = =
(ii) At the temperature, T = 6000 K (surface of the sun) RT RT1 8.314 × 300.15
3 = 19. 48
Average thermal energy, E = k BT
2 Final p 2 = (11 +1 ) = 12 atm = 12 × 1.013 × 10 5 Pa
3 −23 V2 = 30 L = 30 × 10 −3 m 3
= × 1. 38 × 10 × 6000
2 T2 = 273.15 + 17 = 290.15 K
= 1.241 × 10 −19 J Number of moles
(iii) At temperature, T = 10 7K p V 12 × 1.013 × 10 5 × 30 × 10 −3
= 2 2= = 15.12
Average thermal energy, RT2 8.314 × 290.15
3 3
E = k BT = ×1.38 × 10 −23 × 10 7 Hence, moles removed = 19.48 − 15.12 = 4.36
2 2 Mass removed = 4.36 × 32 g
= 2.07 × 10 −16 J = 139.52g = 0.1395 kg
27. If the tube is held horizontally, the mercury thread of 29. Given, V1 = 2. 0 L, V2 = 3. 0 L, µ 1 = 4. 0 moles,
length 76 cm traps a length of air = 15 cm. A length of
µ 2 = 5.0 moles
9 cm of the tube will be at the open end, [Fig. (a)]. The
pressure of air enclosed in tube will be atmospheric p1 = 1.00 atm, p 2 = 2.00 atm
pressure. Let area of cross-section of the tube be p1V1 = µ 1RT1, p 2V2 = µ 2RT2
1 sq. cm. µ = µ 1 + µ 2, V = V1 + V2
2
Mercury
(24 + h) cm For 1 mole, pV = E
3
2
For µ 1 mole, p1V1 = µ 1E1
(76 – h) cm 3
15 cm 76 cm 9 cm 2
For µ 2 mole, p 2V2 = µ 2E 2
3
(a) (b)
3
Total energy is (µ 1E1 + µ 2E 2 ) = ( p1V1 + p 2V2 )
∴ p1 = 76 cm and V1 = 15 cm3 2
If the tube is held vertically, 15 cm air gets another 9 cm 2 2
of air (filled in the right handside in the horizontal pV = E total = µ Eper mole
3 3
position) and let h cm of mercury flow out to balance the 2 3
atmospheric pressure, [Fig. (b)]. Then, the heights of air p ( V1 + V2 ) = × ( p1V1 + p 2V2 )
column and mercury column are (24 + h ) cm and 3 2
(76 − h ) cm, respectively. p1V1 + p 2V2
p=
The pressure of air = 76 − (76 − h ) = h cm of mercury V1 + V2
∴ V2 = (24 + h ) cm3 1. 00 × 2. 0 + 2. 00 × 3. 0  8. 0
= = = 1. 60 atm
and p 2 = h cm  2. 0 + 3. 0  5. 0
If we assume that temperature remains constant, then 30. (i) The moon has small gravitational force and hence the
p1V1 = p 2V2 or 76 × 15 = h × (24 + h ) escape velocity is small. As the moon is in the
or h 2 + 24 h − 1140 = 0 proximity of the earth as seen from the sun, the
moon has the same amount of heat per unit area as
− 24 ± (24 )2 + 4 × 1140
or h= that of the earth. The air molecules have large range
2 of speeds.
= 23.8 cm or − 47.8 cm Even though the rms speed of the air molecules is
Since, h cannot be negative (because more mercury smaller than escape velocity on the moon, a
cannot flow into the tube), therefore h = 23.8 cm. Thus, significant number of molecules have speed greater
in the vertical position of the tube, 23.8 cm of mercury than escape velocity and they escape.
flows out.
112 CBSE Term II Physics XI

Now, rest of the molecules are arranged in the speed 16


34. Moles of helium, µ He = =4
distribution for the equilibrium temperature. Again, 4
a significant number of molecules escape as their 16 1
speeds exceed escape speed. Hence, over a long Moles of oxygen, µ O2 = =
32 2
time the moon has lost most of its atmosphere.
As helium is monoatomic, so degrees of freedom of
(ii) As the molecules move higher, their potential f 3
energy increases and hence kinetic energy decreases helium, f = 3, so CVHe = R = R
2 2
and hence temperature reduces.
As oxygen is diatomic, so degrees of freedom of oxygen,
At greater height, more volume is available and gas
f = 5, so
expands and hence some cooling takes place.
f 5
3RT CVO 2 = R = R
31. As, we know that, v rms = 2 2
M µ He CVHe + µ O 2 CVO2
Thus, v rms / Ar = v rms / He ∴ CV mixture =
µ He + µ O 2
TAr THe
⇒ = 3 1 5
M Ar M He 4× R+ × R
= 2 2 2 = 29 R
TAr = ? THe = 273 − 10 = 263 K 1 18
4+
M Ar = 39. 9 u , M He = 4 u 2
TAr 263 Cp
Thus, = γ= (of mixture)
39. 9 4 CV
263 × 39. 9 R R
TAr = = 2623. 43 K γ mixture = 1 + =1 + = 1. 62
4 CV mixture 29
3 R
32. Given, for monoatomic gas, µ 1 = 1, CV1 = R and for a 18
2 (as Cp − CV = R)
5
diatomic gas, µ 2 = 3 and CV2 = R. 5
2 35. For a monoatomic gas, γ =
3
∴ Total heat energy required to raise the temperature of R R 3
mixture by ∆T. CVγ = = = R
γ −1 5
−1 2
∆U = µ 1 CV1 ∆T + µ 2 CV2 ∆T 3
3 5 7
∆U = 1 × R ∆T + 3 × R ∆ T = 9R ∆T …(i) For a diatomic gas, γ =
2 2 5
Let CV m be the molar specific heat of the mixture at R 5
constant volume and as total number of moles of mixture. CV = = R
7
µm =1 + 3= 4 −1 2
5
∴ Heat energy required, ∆U = µ m CV m ∆T By conservation of energy,
⇒ ∆U = 4 CV m ∆T …(ii)
µ 1CV1 + µ 2CV2
From Eqs. (i) and (ii), we have CVmixture =
9 µ1 + µ 2
9R∆T = 4 CV m ∆T ⇒ ∆CV m = R = 2 . 25 R 3 5
4 2× R+3× R
2 2 3R + 7 . 5 R
33. At STP, 1 mole of gas occupy 22.4 L of volume. = = = 2 .1 R
44. 8 2+3 5
∴ Moles of helium in container, µ = = 2 moles
22. 4 36. As, we know that, the value of mean free path of the
3 molecules of a given gas is given by
Now, helium is monoatomic, so CV = R. 1
2 Mean free path, λ =
Change in temperature, 2 π nd 2
∆T = T2 − T1 = 15 ° C = 15 K where, n = number of gas molecules present in unit
∴ Volume of gas remains constant. volume of given gas and d = molecular diameter.
∴ ∆ W = p∆ V = 0 ⇒ ∆ Q = ∆ U + ∆ W (i) Effect of temperature As temperature of a gas is
Amount of heat required, ∆Q = ∆U = µCV ∆T increased at constant pressure, volume of gas
increases and hence the number of molecules per
3
= 2 × R × 15 = 45 R unit volume decrease. In fact,
2 1
= 45 × 8. 31 = 374 J n∝ and V ∝ T
V
CBSE Term II Physics XI 113

Thus, n∝
1 ∴ Radius of lithium atom,
1
T
 3 × 6. 94 × 10 −3 3
Due to decrease in molecular number density, the r =  3
 4 × 3.14 × 6 × 10 × 0. 53 × 10 
23
value of mean free path of the gas increase. i.e.
1 = 1.73 × 10 −10 m = 1.73 Å
λ ∝ ∝ T. Thus, pressure remaining constant, the
n (v) For flourine (liquid),
mean free path of a gas is directly proportional to its
M = 19. 00 and ρ = 1.14 × 10 3 kg / m 3
absolute temperature.
∴ Radius of fluorine atom,
(ii) Effect of pressure At constant temperature, 1
volume V decreases, upon increasing pressure, as  3 × 19. 0 × 10 −3 3
molecular number density n also increases. r =  3
 4 × 3.14 × 6 × 10 × 1.14 × 10 
23
Therefore, the mean free path decreases.
1 = 1.88 × 10 −10 m = 1.88 Å
i.e. p ∝ ∝n
V pV
38. (i) Dotted plot shows that is a constant quantity
1 T
∴ λ∝
n  pV 
 = mR which is independent of pressure. It
1  T 
or λ∝
p signifies the ideal gas behaviour.
Thus, at a constant temperature, the mean free path (ii) As, curve at temperature T1 is closer to the dotted
of a gas is inversely proportional to its pressure. plot than the curve at temperature T2. Since, the
37. We know that, density of an element, behaviour of a real gas approaches the behaviour of
Mass Mass of 1 mole
ρ= = perfect gas when temperature is increased, so
Volume Total volume of molecules in T1 > T2.
1 mole when closely packed pV
M ( in grams ) 3 M × 10 −3 kg (iii) The value of , where the curves meet on Y-axis is
ρ= = T
 4 3 4π r 3 ⋅ N A equal to µR.
 πr  N A
3 
Now, given mass of oxygen gas = 1 × 10 −3 kg = 1 g
1/ 3
 3 M ×10  −3
pV 1
⇒ r=  ∴ = µR =   × 8.31 JK −1 = 0.26 JK −1
 4 πN A ⋅ ρ  T  32 
where, N A = Avogadro’s number • 6 × 10 23 (iv) If we use1 × 10 −3 kg of hydrogen, then we will not
(i) For carbon (diamond), pV
get same value of at the point where the curves
M = 12 . 01 , ρ = 2 . 22 × 10 3 kg / m 3 T
∴ Radius of carbon atom, meet on Y-axis because molecular mass of hydrogen
1/ 3 is different from oxygen.
 3 × 12. 01 × 10 −3 
r=  3
pV
 4 × 3.14 × 6 × 10 × 2.22 × 10  Now, to get same value of , mass of hydrogen
23
T
= 1.29 × 10 −10 m = 1.29 Å required is obtained from
(ii) For gold, M = 197 . 00 , ρ = 19. 32 × 10 3 kg /m 3 pV m
= µR = × 8. 31 = 0.26
∴ Radius of gold atom, T 2. 02
1
 3 × 197 × 10 −3 3 2. 02 × 0.26
r=  ⇒ m= = 6. 32 × 10 −2 g
3 8. 31
 4 × 3.14 × 6 × 10 × 19. 32 × 10 
23

39. (i) Average speed,


= 1. 59 × 10 −10 m = 1. 59 Å
n v + n 2v 2 + n 3v 3 + n 4v 4 + n 5v 5
(iii) For nitrogen (liquid), v av = 1 1
n1 + n 2 + n 3 + n 4 + n 5
M = 14. 01 , ρ = 1. 00 × 10 3 kg / m 3
2 ×1 + 4 × 2 + 8 × 3 + 6 × 4 + 3 × 5
∴ Radius of nitrogen atom, =
1 2+4+8+6+3
 3 × 14. 01 × 10 −3 3 = 3.17 m / s
r=  3
 4 × 3.14 × 6 × 10 × 1. 0 × 10 
23
(ii) Root-mean-square speed,
= 1.77 × 10 −7 m = 1.77 Å n1v12 + n 2v 22 + n 3v 32 + n 4v 42 + n 5v 52
v rms =
(iv) For lithium, M = 6. 94 , ρ = 0. 53 × 10 3 kg/m 3 n1 + n 2 + n 3 + n 4 + n 5
114 CBSE Term II Physics XI

2 × 12 + 4 × 22 + 8 × 32 + 6 × 42 + 3 × 52 ∴ v rms = 639 m /s
=
2+4+8+6+3 1 3
mv 2rms = k BT
= 3. 36 m / s 2 2
(iii) The most probable speed is that speed which is 1 mv 2rms
∴ T=
possessed by maximum number of molecules. 3 kB
2 k BT 3 k BT
Most probable speed, v mp = = × 2/ 3 1 3. 0 × 10 −26 × 4. 08 × 10 5
m m = ×
3 1. 38 × 10 −23
2 3 k BT 2 2
v mp =× = v rms = × 3.36 m/s = 2.96 × 10 2 = 296 K
3 m 3 3
= 0.816 × 3.36 m / s = 2.74 m/s 10 × (200 )2 + 20 
 2
40. According to the law of atmosphere, × ( 400 ) + 40 × ( 600 ) + 20 × ( 800 ) 
2 2
2
(ii) v rms =
 mg  90
n 2 = n1 exp − ( h 2 − h1) …(i)
 k BT  10 ×100 2 × (1 × 4 + 2 × 16 + 4 × 36 + 2 × 64 )
=
where, n 2 and n1 refer to number density of particles at 90
heights h 2 and h1, respectively. 308
= 10000 ×
If we consider the sedimentation equilibrium of 9
suspended particles in a liquid, then in place of mg, we will
have to take effective weight of the suspended particles. = 342 ×1000 m 2/s 2
Let, V = average volume of a suspended particle, = 584 m/s
ρ = density of suspended particle, ρ′ = density of liquid, 1 mv 2rms
m = mass of one suspended particle and m ′ = mass of ∴ T= = 248 K
3 kB
equal volume of liquid displaced.
According to Archimedes’ principle, effective weight of 42. Given, volume of the box, V = 1. 00 m 3
one suspended particle Area, a = 0.010 mm 2 = 10 −8 m 2
= actual weight − weight of liquid displaced
= mg − m ′g
 m  ρ′  N2 gas
= mg − V ρ′ g = mg −   ρ′ g = mg 1 −  p=1.5 atm Hole
 ρ  ρ T=300K
R
Also, Boltzmann constant, k B = poutside = 1 atm
NA
where, R is gas constant and N A is Avogadro’s number. Temperature outside = Temperature inside
 ρ′  Initial pressure inside the box = 1 . 50 atm.
Putting, mg 1 −  in place of mg and value of k B in
 ρ Final pressure inside the box = 0.10 atm.
Eq. (i), we get Assuming,
 mg N A  ρ′   v ix = speed of nitrogen molecule inside the box along
n 2 = n1 exp − 1 −  ( h 2 − h1), which is
 RT  ρ 
x-direction.
required relation. n1 = number of molecules per unit volume in a time
interval of ∆t, all the particles at a distance ( v ix ∆t ) will
41. This problem is designed to give an idea about cooling by
collide the hole and the wall, the particle colliding along
evaporation
the hole will escape out reducing the pressure in the box.
Σ nivi
2
Let area of the wall is A, number of particles colliding in
(i) v 2rms = i
1
Σn i time ∆t = n1( v ix ∆t )A
2
10 × (200 )2 + 20 × ( 400 )2 
 1
2 is the factor because all the particles along x- direction
+ 40 × ( 600 ) + 20 × ( 800 ) + 10 × (1000 ) 
2 2
=  2
100 are behaving randomly. Hence, half of these are colliding
against the walls on either side.
10 × 1002 × (1 × 4 + 2 × 16 + 4 × 36 + 2 × 64 + 1 × 100)
= Inside the box, v 2ix + v 2iy + v 2iz = v 2rms
100
= 1000 × ( 4 + 32 + 144 + 128 + 100 ) v2
⇒ v 2ix = rms (Q v ix = v iy = v iz )
3
= 408 ×1000 m 2/s 2
CBSE Term II Physics XI 115

If particles collide along hole, they move out. Similarly, Let µ and µ ′ be moles of mono and diatomic gases,
outer particles colliding along hole will move in. µCV + µ ′ CV′
then CV (mixture) =
If a = area of hole, then net particle flow in time, µ + µ′
1 k T  3 k BT  3
1× R +1 × R
5
∆t = ( n1 − n 2 ) B ∆ta Q v rms = 
2 m  m  CV = 2 2 =2R
1 +1
(temperature inside and outside the box are equal)
R R
Let n = number of density of nitrogen γ (mixture) = 1 + =1 + = 1. 5
CV ( mixture) 2R
µN A pN A  µ p 
n= = Q = 
V RT  V RT  44. (i) The mean free path of a gas molecule is defined as
the average distance travelled by a molecule
where, N A = Avogadro’s number. between two successive collisions.
If after time ∆t, pressure inside changes from p to p1′
p ′N λ2 λ4 λ5 λ5
λ1
∴ n1′ = 1 A
RT
Now, number of molecules coming out = n1V − n1′V λ3
1 k T
= ( n1 − n 2 ) B ∆ta According to figure, if a molecule covers free path
2 m λ 1, λ 2 , λ 3 ⋅⋅⋅ after successive collisions, then its mean
p1N A p ′N 1 N k BT free path is given by
∴ V − 1 A V = ( p1 − p 2 ) A ∆ta
RT RT 2 RT m λ 1 + λ 2 + λ 3 ⋅⋅⋅
λ=
 p − p1′  V m Total number of collisions
⇒ ∆t = 2  1 
 p1 − p 2  a k BT (ii) Let d be the diameter of each molecule of the gas,
then a particular molecule will suffer collision with
Putting the values from the data given, any molecule that comes within a distance d
1. 5 − 1. 4  1 × 1. 00 46.7 × 10 −27 between centres of two molecules.
∆t = 2  
1. 5 − 1. 0  0. 01 × 10 −6
1.38 × 10 −23 × 300
2 6.717
= × 3. 358 × 10 5 = × 10 5 = 1. 343 × 10 5s
5 5
43. (i) If we consider water molecule as a solid made up of
3 atoms (2 hydrogen and 1 oxygen) and each atom is
d

free to vibrate in three dimensions about its mean


t
v∆

position.
By using law of equipartition of energy, average
d

energy associated with one atom of water molecule


1 
= 2 ×  k BT = k BT If v is average speed of molecule, then from figure,
2  the volume swept by the molecule in small time ∆ t
In three dimension, the average energy per atom of in which any molecule will collide with it
water molecule = 3k BT = πd 2 v ∆t
∴ Total energy of one molecule of water is If n is number of molecules per unit volume of the
= 3 × ( 3 k BT ) = 9 k BT gas, then number of collision suffered by the
Now, total energy of 1 mole of water is molecule in time ∆ t
U = 3 × 3 k BT × N A = 9RT = πd 2 v ∆t × n
∴ Molar specific heat of water, So, number of collisions per second
∆ Q ∆U πd 2 v ∆ t × n
C= = = 9R = = n πd 2 v
∆T ∆T ∆t
= 9 × 8. 31≈ 75 mol −1K −1 ∴ Average time between two successive collisions,
1
3 τ=
(ii) For monoatomic gas, CV = R n πd 2 v
2
5 ∴ Mean free path = Average distance between two
For diatomic gas, C′V = R
2 successive collisions
116 CBSE Term II Physics XI

⇒ λ = τ × mean velocity =
1
×v=
1 3RT 3 × 8.31 × 290
Now, v rms = = = 508.14 m/s
n πd v
2
n πd 2 M 28 × 10 −3
1
Mean free path, λ = v
∴ Collision frequency, f = rms =
508.14
n πd 2 λ 1.11 × 10 −7
where, d = diameter of each molecule = 4. 58 × 10 9 collisions/s
and n = number of molecules per unit volume. 46. (i) An isolated system is the set of particle and walls
45. Let n be the number of molecules per unit volume of the such that it cannot transfer either energy or mass to
gas. Now, pV = NkT, where N = number of molecules in its surroundings.
volume V of the gas. (ii) The ideal gas equation is given by pV = nRT
N p where, p is the pressure of ideal gas,
∴ n= =
V kT V is the volume of ideal gas,
Here, p = 2 × 1.01 × 10 5 N / m 2, k = 1.38 × 10 −23 J / K , n is the amount of ideal gas measured in terms of
T = 273 + 17 = 290 K moles,
2 × 1. 01 × 10 5 R is the universal gas constant
∴ n =
(1. 38 × 10 −23 ) × (290 ) and T is the temperature.
(iii) Gas molecules influence each other only by
= 5. 05 × 10 25 molecules/m 3
collision. They do not exert force on each other, they
1
Mean free path, λ = do not stick to each other.
4 π 2 r 2n If collisions are not elastic, the molecules will stick
Here, r = 1. 0 Å = 1. 0 × 10 −10 m, to each other upon collision, the distribution of gas
n = 5. 05 × 10 25 molecules/m 3 in a container will become uneven and temperature
1 will not remain constant.
∴ λ=
4 π 2 × (1. 0 × 10 −10 )2 × 5. 05 × 10 25 The molecules are tiny, the energy and momenta of
the molecules are conserved during the collisions.
λ = 1.11 × 10 −7 m
CBSE Term II Physics XI 117

CHAPTER 06

Oscillations
In this Chapter...
l Periodic Motion l Some Systems Executing
l Oscillatory Motion Simple Harmonic Motion
l Simple Harmonic Motion l Undamped and Damped
Oscillations
l Simple Harmonic Motion and
Uniform Circular Motion l Free and Forced Oscillations
l Characteristics of SHM l Resonance
l Energy in Simple Harmonic
Motion

Frequency can also be defined as reciprocal of time period, it


Periodic Motion is denoted by ν.
A motion that repeats itself after regular intervals of time is The SI unit of frequency of oscillation is hertz (Hz).
called periodic motion. e.g. Revolution of planets, rotation of 1
the earth, etc. Frequency, ν =
T
and 1 hertz = 1 Hz = 1 oscillation per sec = 1 s −1
Oscillatory Motion When frequency of an oscillatory motion is very high, then
A motion in which a body moves to and fro or back and forth
motion is called vibrational motion.
repeatedly about a fixed point in a regular interval of time is
called oscillatory motion. This fixed point is known as Displacement
mean/equilibrium position or centre of oscillation.
In oscillation, displacement is the change in position with
e.g. Motion of a simple pendulum
respect to mean position (or equilibrium position) or
Every oscillatory motion is periodic, but every periodic reference position.
motion need not be oscillatory. e.g. Motion of the planet
around the sun is periodic but not oscillatory.
Periodic Function
Some important characteristics of these motions are given
below The displacement can be represented by a mathematical
function of time. The function which repeats its value in
Period regular interval of time or period is called periodic function.
One of the simplest periodic function is given by
The smallest interval of time after which a periodic motion is
repeated, is called its period. f ( t ) = A cos ωt
Its SI unit is second and it is denoted by the symbol T. The time period for this function will be T = 2π / ω .
Similarly, if we consider a sine function, then
Frequency f ( t ) = A sin ωt
The number of oscillations/vibrations preformed by a
oscillating body about its mean position in a unit time is Also, f ( t ) = A sin ωt + B cos ωt
known as its frequency.
118 CBSE Term II Physics XI

is also a periodic function with time period T. vmax


Then, f ( t ) = D sin (ωt + φ) –A O A –A O A
t=0 t = T/4
where, D = A2 + B2 vmax
–A O A –A A
O
φ = tan  
−1 B t = T/2 t = 3T/4
and
 A vmax
Hence, any periodic function can be expressed as a –A O A –A O A
superposition of sine and cosine functions of different time t=T t =5T/4
periods with suitable coefficients. Location of a particle executing SHM with different time

Observations from the figures are given below


Simple Harmonic Motion (SHM) (i) The time after which motion repeats itself is T.
A special type of periodic motion in which a particle moves (ii) T will remain fixed, i.e. it does not vary with variation
to and fro repeatedly about a mean position under the in initial location ( t = 0 ).
influence of a restoring force is known as Simple Harmonic (iii) The speed is maximum for zero displacement, i.e. at
Motion (SHM). x = 0 and zero at the extremes of motion.
Equation of SHM
Consider a particle oscillating back and forth about the origin SHM and Uniform Circular Motion
on an X-axis between the limits +A and −A in time T. (Geometrical Interpretation of SHM)
Origin or mean position
Consider a particle P starting from X is moving with a uniform
speed along the circumference of a circle of radius A, with
−A +A centre at O. This circle is known as circle of reference, while
A particle executing SHM the particle P is known as reference particle.
Let P′ be the foot of perpendicular drawn from the point P to
Then, this oscillatory motion of the particle is said to be the diameter XOX′. P ′ is known as projection of the particle
SHM, if the displacement x of the particle from the origin P at diameter XOX′.
varies with time as Y
P
Displacement, x( t ) = A cos (ωt + φ) M
where, A, ω and φ are constants. A φ
+
ωt
Given, the time t is taken as zero when the particle is at X′
O P′
X
position +A and it returns to same point with position +A at
time t = T.
Let at an instant t, the particle be at P, if O is taken as mean
Y′
position of the particle, then OP = x (say), i.e. the
Circular representation of SHM
displacement of the particle from the mean position.
x As P moves along the circle from X to Y , Y to X′, X′ to Y′ and
–A O +A Y′ to X, the projection of the particle P, i.e. P′ moves from X
P to O, O to X′, X′ to O and O to X, respectively.
The restoring force F acting on the particle at that instant, Thus, P revolves along the circumference of the circle while
F = − kx its projection P′ moves to and fro about the point O along the
diameter XOX′. Hence, the motion of P′ about point O is said
where, k is a force constant having SI unit N/m. The negative to be simple harmonic.
sign shows that, the restoring force F is always directed
towards the mean position. Therefore, the projection of uniform circular motion upon a
The above relation is known as force law for SHM. diameter of the circle executes simple harmonic motion.
The motion of projection of reference particle along any
Location of the Particle Executing SHM other diameter of the circle of reference will also be simple
at the Discrete Value of Time t harmonic motion.
The figure below shows the positions of a particle executing Hence, SHM can be geometrically defined as the projection
SHM are at discrete values of time. It must be noted that of a uniform circular motion on any diameter of the circle of
T
each interval of time being . reference.
4
CBSE Term II Physics XI 119

t is determined by the argument (ωt + φ0 ) in the sinusoidal


Characteristics of SHM function. This quantity (ωt + φ0 ) is called phase of the
Some of the important characteristics of a simple harmonic motion, where φ0 is called phase constant or phase angle.
motion are given below
(iv) Angular Frequency
(i) Displacement
Angular frequency of a body executing periodic motion
The displacement of a particle executing SHM at an instant is
given by the distance of the particle from the mean position is equal to the product of frequency of the particle with
at that instant. factor 2π.
The values of displacement as a continuous function of time It is denoted by ω and its SI unit is radian per second.
can be represented as a graph given below 2π
ω= = 2 πν
x (t) T
A
where, ν = frequency of the particle.
Displacement

O
t (v) Velocity
The velocity of a particle executing SHM at any instant, is
–A defined as the rate of change of its displacement at that
Displacement as a continuous function of time instant.
Velocity, v = ω A 2 − x 2
Displacement, x ( t ) = A cos (ωt + φ)
At mean position, x = 0
(ii) Amplitude
The magnitude of maximum displacement of a particle ∴ v = ωA (maximum velocity)
executing SHM is called amplitude of the oscillation of that At extreme position, x=A
particle. Amplitude is measured on either side of mean
position. ∴ v=0 (minimum velocity)
The displacement varies between the extremes + A and −A Also, velocity, v( t ) = − ωA sin (ωt + φ)
because the sinusoidal function of time varies from +1 to −1.
Two SHMs may have same ω (angular frequency) and φ (vi) Acceleration
(phase constant) but different amplitudes A and B. The acceleration of the particle executing SHM at any instant
x is defined as the time rate of change of its velocity at that
2
B instant.
A
1 Acceleration, a = − ω 2 x
O
t At mean position, x = 0
–A ∴ a=0 (minimum acceleration)
–B
Different amplitudes in SHM
At extreme position, x = A,

∴ a = −ω 2A (maximum acceleration)
(iii) Phase
Also, acceleration, a( t ) = − ω 2 A cos (ωt + φ)
If amplitude A is fixed for a given SHM, then the state of
motion, i.e. position and velocity of the particle at any time
120 CBSE Term II Physics XI

Displacement, Velocity and Acceleration of a Body Executing SHM


Displacement x (t ) = A cos ωt Velocity v (t ) = − ω A sin ωt Acceleration a (t ) = − ω 2 A cos ωt
x v x
Displacement

Acceleration
+A +ω A + ω2A

Velocity
0 t 0 t 0 t

–A –ωA –ω A 2
T
(a) (b) (c)
Sinusoidal in nature. x varies from − A to A. Sinusoidal in nature. v ( t ) varies from − ωA Sinusoidal in nature. a ( t ) varies from
It has zero phase difference. to ωA. − ω 2A to ω 2A.
π
It has a phase difference of w.r.t. x ( t ). It has a phase difference of π w.r.t. x ( t ).
2

Force Law for SHM Total Energy


It states that simple harmonic motion is the motion At any displacement x, the total energy of a harmonic oscillation
executed by a particle subjected to a force, which is is given by
proportional to the displacement of the particle and is 1 1 1
E = K + U = k( A 2 − x 2 ) + kx 2 = k A 2
always directed towards the mean position. 2 2 2
1
For a body executing SHM, Also, E = mω A = 2 π mν A
2 2 2 2 2
(Qω = 2 πν)
2
⇒ Restoring force is given by F( t ) = − kx( t ) Also, the total mechanical energy of a harmonic oscillation is
= −m ω 2 x ( t ) independent of time or displacement, while it depends on the
maximum displacement i.e. amplitude.
k
where, k = mω 2 or ω =
m Graphical Representation of Energy in SHM
At mean position, the entire mechanical energy is in the form of
Energy in SHM kinetic energy and at the two extreme positions, the energy is
totally potential, i.e. kinetic energy is zero.
A particle executing SHM posseses both kinetic energy
So, the graph for kinetic energy K, potential energy U and total
and potential energy. When a body is displaced from its
energy E with displacement x is given below
equilibrium position by doing work upon it, it acquires
potential energy. When the body is released, it begins to Energy
move back with a velocity, thus acquiring kinetic energy. E=K+U

U
Kinetic Energy
Kinetic energy of the particle at any displacement x is K
given by
x = –A 0 x = +A
1 1
KE = k ( A 2 − x 2 ) = mω 2 A 2 sin 2 (ωt + φ) Displacement
2 2 K, U and E as functions of displacement x for a
Hence, kinetic energy is also a periodic function of time, harmonic oscillation
being zero when the displacement is maximum. The variation of energies K , U and E of a harmonic oscillator with
Kinetic energy is maximum when the particle is at the time t is given below
mean position. Period of kinetic energy is T/ 2 .

Potential Energy E=K+U


Energy

Potential energy of a particle at displacement x is given U


by
K
1 2 1
U=
kx = kA 2 cos 2 (ωt + φ) 0 T/4 T/2 3T/4 T
2 2 Time (t)
Thus, potential energy of a particle executing simple K, U and E as functions of time t for a harmonic oscillation
harmonic motion is also periodic with period T/ 2 .
From the graph, the frequency of PE or KE is double than that
Potential energy is zero at the mean position and of SHM. But the total energy remains constant at all t
maximum at the extreme position. or x.
CBSE Term II Physics XI 121

Some Systems Executing SHM Expression for Time Period of


a Simple Pendulum
Some systems executing SHM are given below
Consider a simple pendulum having a small bob of mass m
1. Oscillation Due to a Spring tied to an inextensible massless string of length L.
If the block is pulled towards right through a small distance x In the equilibrium position, the bob of a simple pendulum
and released, it starts oscillating back and forth about lies vertically below the point of suspension. If the bob is
its equilibrium position under the action of the restoring slightly displaced on either side and then released, it begins
force. F = − kx oscillation about the mean position.
Let us consider that at any instant during oscillation, the bob
where, k is the force constant.
lies at position A when its displacement is OA = X and the
Equilibrium F= 0 thread makes an angle θ with the vertical.
m Pivot point

F =– kx θ
Stretched T
m L

x A
F = – kx x
O θ mg cos θ
Compressed
m mg sin θ
mg
x
Oscillating pendulum
Three different positions of the oscillation due to a spring
The forces acting on the bob are given below
d x −k
2
(i) Weight mg of the bob acting vertically downwards.
⇒ a= = x = −ω 2x
dt 2 m (ii) Tension T along the string.
Thus, acceleration is proportional to displacement x and acts The force mg has two rectangular components are given
opposite to it. below
The block executes simple harmonic motion and its time (i) The component mg cos θ acting along the thread is
m balanced by the tension T in the thread.
period is given by T = 2π
k (ii) The tangential force mg sin θ will provide the
restoring torque, which tends to bring the bob back to
and frequency of oscillation will be
its mean position. Thus, the restoring torque of the
1 1 k force mg sin θ about the pivot point is given by
Frequency, ν = =
T 2π m τ = − ( mg sin θ)L = − mgL sin θ …(i)
where, the negative sign shows that the torque acts to
2. Simple Pendulum reduce θ and L is the length of the simple pendulum.
An ideal simple pendulum consists of a point mass suspended For rotation, the torque can be given as
by an inextensible and weightless string which is fixed at the τ = Iα …(ii)
other end. where, I = pendulum’s moment of inertia about the
A simple pendulum is obtained by suspending a small pivot point
metal bob by a long and fine cotton thread from a support. and α = angular acceleration about the pivot
point.
Support From Eqs. (i) and (ii), we get
Pivot point
−mgL sin θ
L − mgL sin θ = Iα ⇒ α =
I
If θ is in radian, sin θ can be expressed as
m
θ3 θ5
sin θ = θ − + − ...
Simple pendulum 3! 5!
122 CBSE Term II Physics XI

Now, if θ is small, sin θ can be approximated by θ Undamped and Damped Oscillations


−mgLθ
∴ α≈ When a simple harmonic system oscillates with a constant
I amplitude which does not change with time, its oscillations are
Therefore, the acceleration of the pendulum is called undamped oscillations.
proportional to the angular displacement but in opposite
sign. Thus, as the pendulum moves to the right, the pull
to the left increases until it stops and begins to return to
the left.
Similarly, when it moves towards left, its acceleration to
the right tends to return it to the right and so on as it Undamped oscillation
swings to and fro as SHM. Hence, the motion of a simple
When a simple harmonic system oscillates with a decreasing
pendulum swinging through small angles is amplitude with time, its oscillations are called damped
approximately SHM. oscillations.
−mgL
Equation α = θ is the angular analogue of
I
equation a = − ω 2 x.
mgL
On comparing these equations, we have ω =
I Damped Oscillation
2π mgL
⇒ =
T I Free and Forced Oscillations
I A body capable of oscillating is said to be executing free
⇒ T = 2π ...(iii)
mgL oscillations, if it vibrates with its own natural frequency without
the help of any external periodic force.
where, T = time period of pendulum. When a body oscillates with the help of an external periodic force
All the masses of a simple pendulum is centered in the with a frequency different from the natural frequency of the
mass m of the bob (taken as a point) which is at a distance body. Then, its oscillations are called forced oscillations.
of L from the pivot point. Therefore, I = mL 2 .
On putting this value in Eq. (iii), we get Resonance
mL 2
L When a body oscillates with its own natural frequency ν 0 , with
T = 2π = 2π the help of an external periodic force whose frequency ν d is
mgL g
equal to the natural frequency of the body, the oscillations of the
L body are called resonance.
⇒ Time period of pendulum, T = 2π
g Condition for resonance, ν 0 = ν d
CBSE Term II Physics XI 123

Solved Examples
Example 1. An object performs SHM of amplitude Example 4. A linear harmonic oscillator has a total
5 cm and time period 4 s. If timing is started when mechanical energy of 200 J. Potential energy of it
the object is at the centre of the oscillation, i.e. at mean position is 50 J. Find
x = 0, then calculate (i) the minimum potential energy,
(i) frequency of oscillation and (ii) the maximum kinetic energy and
(ii) displacement at 0.5 s. (iii) the potential energy at extreme positions.
1 1 Sol. (i) At mean position, potential energy is minimum.
Sol. (i) Frequency, ν = = = 0.25Hz
T 4 Hence, U min = 50 J
(ii) The displacement equation of object, (ii) At mean position, kinetic energy is maximum.
x = A sin ωt ∴ K max = E − U min
= A sin(2πνt ) = 200 − 50 = 150 J
So, at t = 0.5 s, (iii) At extreme positions, kinetic energy is zero and
x = 5 sin(2 π × 0.25 ×0.5) potential energy is maximum.
π 5 ∴ U max = E = 200 J
= 5 sin = cm (from law of conservation of energy)
4 2
Example 2. Amplitude of a harmonic oscillator is A. Example 5. A particle executes SHM, at what value of
When particle’s velocity of this oscillator is half of displacement are the kinetic and potential energies
maximum velocity, then determine position of equal?
particle. 1
Sol. We know that, kinetic energy, K = mω 2 ( A 2 − x2 )
v max Aω 2
Sol. As, v = ω A 2 − x 2 but it is given that v = = 1
2 2 and potential energy, U = mω 2 x2
Aω 2
⇒ = ω A 2 − x2 Since, K =U
2
1 1
⇒ A 2 = 4 ( A 2 − x2 ) ⇒ mω ( A 2 − x2 ) = mω 2 x2
2
2 2
4A2 − A2 3A or 2 x2 = A 2
⇒ x2 = ⇒ x=±
4 2 A
or x= = 0.707A
Example 3. Frequency of oscillation of a body is 6 Hz 2
when force F1 is applied and 8 Hz when F2 is Example 6. The length of a simple pendulum is 16 cm.
applied. If both forces F1 and F2 are applied It is suspended by the roof of a lift which is moving
together, then find out the frequency of oscillation. upwards with an acceleration of 6.2 ms −2 . Find the
Sol. According to question, F1 = − k 1 x and F2 = − k 2 x time period of pendulum.
1 k1 Sol. Given, length of the pendulum, l = 16 cm = 0.16 m
So, ν1 = = 6 Hz,
2π m Acceleration of the lift, a = 6.2 ms −2
1 k2 l
ν2 = = 8 Hz Q Time period, T = 2π
2π m (g + a)
Now, F = F1 + F2 = − ( k 1 + k 2 )x 0.16
1 k1 + k 2 = 2 × 3.14
Therefore, ν= (9.8 + 6.2)
2π m
0.16
1 4 π 2 ν12 m + 4 π 2 ν 22 m = 6.28 ×
⇒ ν= 16
2π m 1 6.28
(as, k = 4 π 2 ν 2 m) = 6.28 × =
100 10
= ν12 + ν 22 = 8 2 + 6 2 = 10 Hz = 0.628 s
124 CBSE Term II Physics XI

Example 7. A spring-mass system is hanging from the Extension in spring in equilibrium, when lift starts
m (g + a)
ceiling of an elevator in equilibrium. The elevator accelerating upwards =
suddenly starts accelerating upwards with k
m ( g + a ) mg ma
acceleration a, find ∴ Amplitude = − =
k k k

k
Example 8. The amplitude of a damped oscillator
becomes half in one minute. The amplitude after
3 min will be 1/x times of the original. Determine
m
the value of x.
Sol. Amplitude of damped oscillations, A = A 0 e − γt
(i) the frequency and
As, A = A 0 / 2 at t = 1 min
(ii) the amplitude of the resulting SHM. A0
Sol. (i) Frequency = 2π m/ k So, = A 0 e − γ or e γ = 2
2
(frequency is independent of g in spring-mass system) After 3 min, the amplitude will be A 0 / x.
(ii) Initial extension in spring in equilibrium A0
mg So, = A 0 e − γ 3 or x = e 3 γ = ( e γ )3 = 2 3 = 8
= x
k
CBSE Term II Physics XI 125

Chapter
Practice
PART 1 8. A particle executing SHM has a maximum speed of
30 cm/s and angular frequency 10 rad/s. The
Objective Questions amplitude of oscillation is
(a) 3 cm (b) 6 cm
(c) 1 cm (d) 60 cm
l
Multiple Choice Questions
9. The relation between acceleration and
1. The motion of satellites and planets is displacement of four particles are given below.
(a) periodic (b) oscillatory Which one of the particle is suggesting simple
(c) simple harmonic (d) non-periodic harmonic motion? [NCERT Exemplar]
2. The motion of a swing is (a) a x = + 2 x (b) a x = + 2 x2
(a) periodic but not oscillatory (c) a x = − 2 x2 (d) a x = − 2 x
(b) oscillatory 10. For simple harmonic motion of an object of mass m,
(c) linear simple harmonic (a) F = − mω 2 x
(d) circular motion (b) F = − mω x
3. When frequency of oscillations is high, then motion (c) force always acts in the opposite direction of
displacement
is called
(d) Both (a) and (c)
(a) periodic (b) non-periodic
(c) vibratory (d) rotatory 11. If we do an experiment by swinging a small ball by
a thread of length 100 cm, what will be the
4. The periodic function f (t ) = A sin ωt repeats itself approximate time for complete to and fro periodic
after periodic of motion? (Take, g = π 2 )
(a) 2π (b) 3π (c) π (d) π /2 (a) 4 s (b) 2 s (c) 6 s (d) 1 s
5. A particle executing a simple harmonic motion has 12. A simple pendulum suspended from the roof of a
a period of 6 s. The time taken by the particle to move lift oscillates with frequency ν, when the lift is at
from the mean position to half of the amplitude, rest. If the lift falls freely under gravity, its
starting from the mean position is frequency of oscillation becomes.
(a) (1 / 4 ) s (b) ( 3 / 4 ) s (a) zero (b) ν
(c) (1 / 2 ) s (d) ( 3 / 2 ) s (c) 2ν (d) infinite
6. The displacement of a particle in SHM varies 13. A body of mass 400g connected to a spring with
according to the relation x = 4 (cos π t + sin πt ). The spring constant 10 Nm −1 , executes simple harmonic
amplitude of the particle is motion, the time period of oscillation is
(a) − 4 (b) 4 (a) 4 π × 10 −1 s (b) 0.3 π s
(c) 4 2 (d) 8 (c) 2 s (d) 5 × 10 −1 s
7. At extreme position, velocity of the particle executing 14. Two spring of force constants k1 and k 2 are
SHM that has amplitude A is connected to a mass m as shown in figure. The
(a) ω 2A (b) 0 frequency of oscillation of the mass is f. If both k 1
ωA
(c) ωA (d) and k 2 are made four times their original values, the
2 frequency of oscillation becomes
126 CBSE Term II Physics XI

k1 k2 18. Assertion Vibrations and oscillations are two


m different types of motion.
Reason For vibration, frequency is small and for
(a) f/2 (b) f/4 oscillation, the frequency is high.
(c) 4f (d) 2f 19. Assertion x = A cos ωt and x = A sin ωt can
15. Natural length of the spring is 40 cm and its spring represent same motion depending on initial
−1
constant is 4000 Nm . A mass of 20 kg is hung position of particle.
from it. The extension produced in the spring is Reason If the argument of x = A cos ωt, i.e. ωt is
(g = 9.8 ms −2 ) increased by 2π rad, the value of x remains same.
(a) 4.9 cm (b) 0.49 cm
20. Assertion x = A cos ωt represents a periodic
(c) 9.4 cm (d) 0.94 cm
function. The value of x varies between +A and −A.
16. A block is left in the equilibrium position as shown Reason Amplitude is a vector quantity.
mg
in the figure. If now it is stretched by , the net
k 21. Assertion If the amplitude of SHM of a spring-
stretch of the spring is mass system is increased, then time period of SHM
will remain constant.
Reason If amplitude is increased, body will have
k to travel more distance to complete one oscillation.
22. Assertion A block of mass m attached to a stiff
m spring have large oscillation frequency.
Reason Stiff spring have high value of spring
mg mg 2mg mg
(a) (b) (c) (d) constant k.
k 2k k 4k
23. Assertion In damped oscillations, the motion is
17. I. Time period of a spring-mass system depends on periodic.
its amplitude.
II. Time period of a spring-mass system depends on Reason In damped oscillations, the amplitude
its mass. increases due to dissipative forces.
III. Time period of a spring-mass system depends on 24. Assertion In damped oscillations, the total
spring constant. mechanical energy decreases with time.
Which of the following statement(s) is/are correct? Reason Total mechanical energy of oscillator
(a) Both I and II (b) Both I and III 1
executing SHM is given by kA 2 ,where A is
(c) Both II and III (d) I, II and III 2
amplitude at time t.
l
Assertion-Reasoning MCQs
Direction (Q. Nos. 18-24) Each of these questions
l
Case Based MCQs
contains two statements Assertion (A) and Reason (R). Direction Read the following passage and answer the
Each of these questions also has four alternative questions that follows
choices, any one of which is the correct answer. You
25. Energy in SHM
have to select one of the codes (a), (b), (c) and (d) given
below. A particle executing SHM posseses both kinetic
energy and potential energy. When a body is
(a) Both A and R are true and R is the correct
displaced from its equilibrium position by doing
explanation of A
work upon it, it acquires potential energy. When
(b) Both A and R are true, but R is not the correct the body is released, it begins to move back with a
explanation of A velocity, thus acquiring kinetic energy.
(c) A is true, but R is false Both kinetic and potential energies of a particle in
(d) A is false and R is also false SHM vary between zero and their maximum
values.
CBSE Term II Physics XI 127

(i) In SHM, (iii) A hydrogen molecule rotating about its centre of


(a) PE is stored due to elasticity of system mass.
(b) KE is stored due to inertia of system (iv) An arrow released from a bow. [NCERT]
(c) Both KE and PE are stored by virtue of elasticity of
system 2. On an average, a human heart is found to beat
(d) Both (a) and (b) 75 times in a minute. Calculate its beat frequency and
(ii) The expression for displacement of an object in period. [NCERT]
SHM is x = A cos(ωt ). The potential energy at 3. The equation of a wave is given by
t = T / 4 is y = 6 sin 10πt + 8 cos 10πt, where y is in centimetre
(a) (1 / 2 ) kA 2 (b) (1 / 8 ) kA 2 and t in second. Determine the constants involved in
(c) (1 / 4 ) kA 2 (d) zero the standard equation of the wave.
(iii) For a SHM, if the maximum potential energy 4. Which of the following functions of time represent
become double, choose the correct option. (a) simple harmonic motion (b) and periodic but not
(a) Maximum kinetic energy will become double simple harmonic?
(b) Total mechanical energy will become double Give the period for each function, when functions are
(c) Both (a) and (b) (i) sin ωt − cos ωt and (ii) sin 2 ωt. [NCERT]
(d) Neither (a) nor (b)
5. Two particles execute SHM of the same amplitude
(iv) A block is in simple harmonic motion as shown in and frequency along close parallel lines. They pass
the figure on a frictionless surface, i.e. µ = 0. each other moving in opposite directions, each time
Choose the correct option. their displacement is half of their amplitude. What is
their phase difference?
6. Figure below shows circular motion of a particle. All
k the parameters are labelled in the figure. Obtain the
m corresponding equation for simple harmonic motion
µ=0 x=0 of the revolving particle P.
Y
(a) The kinetic energy varies between a maximum value
and zero
(b) The potential energy varies between a maximum
value and zero T=2s X
2 cm
(c) Total energy remains constant
(d) All are correct
(v) In simple harmonic motion, let the time period of P (t = 0)
variation of potential energy is T1 and time period
of variation of position is T2 , then relation 7. A simple harmonic motion is represented by
between T1 and T2 is x = 12 sin (10 t + 0. 6)
(a) T1 = T2 (b) T1 = 2 T2 Find out the amplitude, angular frequency, frequency,
(c) 2 T1 = T2 (d) None of these time period and initial phase, if displacement is
measured in metre and time in seconds.
8. Figures depict four x-t plots for linear motion of a
PART 2 particle. Which of the plots represent periodic
motion? What is the period of motion (in case of
Subjective Questions periodic motion)?
x

(i)
l
Short Answer (SA) Type Questions
1. Which of the following examples represent O t(s)
periodic motion? x
(i) A swimmer completing one (return) trip from
one bank of a river to the other and back. (ii) –1
(ii) A freely suspended bar magnet displaced from –3 0 1 3 t(s)
its N-S direction and released.
128 CBSE Term II Physics XI

x
16. Consider an SHM as x(t ) = 2 cos(4π t + π / 6)
(iii) where, x is in metres and t in seconds. Determine
the time period and initial velocity of the oscillating
O
1 4 7 10 13 t(s) body.
x
17. A particle executes SHM with a time period of 2 s
(iv) and amplitude 5 cm. Find
–3 –2 –1 0 1 2 3 t(s)
(i) displacement, (ii) velocity and

9. Which of the following examples represent (nearly) (iii) acceleration after (1/3) s starting from the mean
simple harmonic motion and which represent position.
periodic but not simple harmonic motion? 18. A body oscillates with SHM according to the
(i) The rotation of earth about its axis. equation
(ii) Motion of an oscillating mercury column in a x = (5. 0 m) cos [(2 π rad s −1 ) t + π/4].
U-tube. At t = 1 . 5 s, calculate displacement, speed and
(iii) Motion of a ball bearing inside a smooth curved acceleration of the body. [NCERT]
bowl, when released from a point slightly above 19. A particle is in linear simple harmonic motion
the lower most point. between two points A and B, 10 cm apart. Take the
(iv) General vibrations of a polyatomic molecule direction from A to B as the positive direction and
about its equilibrium position. [NCERT] give the signs of velocity, acceleration and force on
the particle when it is [NCERT]
10. Every SHM is periodic motion, but every periodic
motion need not to be a simple harmonic motion. (i) at the end A,
Do you agree? Give an example to justify your (ii) at the end B,
statement. (iii) at the mid-point of AB going towards A,
11. Which of the following relationships between the (iv) at 2 cm away from B going towards A,
acceleration a and the displacement x of a particle (v) at 3 cm away from A going towards B
involve simple harmonic motion? [NCERT] (vi) and at 4 cm away from B going towards A.
(i) a = 0.7 x (ii) a = − 200x 2
20. A body describes simple harmonic motion with an
(iii) a = − 10x (iv) a = 100x 3
amplitude of 5 cm and a period of 0.2 s. Find the
12. The maximum acceleration of a simple harmonic acceleration and velocity of the body when the
oscillator is a 0 and the maximum velocity is v 0 . displacement is (i) 5 cm, (ii) 3 cm and
What is the displacement amplitude? (iii) 0 cm. [NCERT]
13. A particle is executing SHM of amplitude A. At 21. A particle performs SHM on a rectilinear path.
what displacement from the mean position is the Starting from rest, it travels x1 distance in first
energy half kinetic and half potential? second and in the next second, it travels x 2
14. Justify the following statements distance. Find out the amplitude of this SHM.
(i) The motion of an artificial satellite around the 22. Figures correspond to two circular motions. The
earth cannot be taken as SHM. radius of the circle, the period of revolution, the
initial position and the sense of revolution (i.e.
(ii) The time period of a simple pendulum will get
clockwise or anti-clockwise) are indicated on each
doubled, if its length is increased four times.
figure.
15. Which of the following functions of time represent
Y
(a) periodic and (b) non-periodic motion? Give the Y

period for each case of periodic motion. (ω is any


positive constant) [NCERT] T=2s X P 2m T=4s
(i) sin ωt + cos ωt 3m (t = 0) X

(ii) sin ωt + cos 2ωt + sin 4ωt


P(t = 0)
(iii) e − ωt (a) (b)
(iv) log (ωt )
CBSE Term II Physics XI 129

Obtain the corresponding simple harmonic motions Calculate


of the X-projection of the radius vector of the (i) the period of oscillation,
revolving particle P, in each case.
(ii) the maximum speed and
23. A man stands on a weighing machine placed on a (iii) maximum acceleration of the collar. [NCERT]
horizontal platform. The machine reads 50 kg. By
means of a suitable mechanism, the platform is 30. The acceleration due to gravity on the surface of
made to execute harmonic vibrations up and down moon is 1.7 m/s 2 . What is the time period of a
with a frequency of two vibrations per second. simple pendulum on the surface of moon, if its time
What will be the effect on the reading of the period on the surface of earth is 3.5 s? (Take, g on
weighing machine? The amplitude of vibrations of the surface of earth is 9.8 m/s 2 ) [NCERT]
platform is 5 cm. (Take, g = 10 ms −2 ) l
Long Answer (LA) Type Questions
24. A particle is executing SHM. If v1 and v 2 are the
speeds of the particle at distance x1 and x 2 from the 31. Which of the following functions of time represent
equilibrium position, show that the frequency of (i) simple harmonic, (ii) periodic but not simple
oscillations is harmonic and (iii) non-periodic motion? Give
1/ 2 period for each case of periodic motion (ω is any
1  v12 − v 22 
f=   positive constant). [NCERT]
2 π  x 22 − x12 
(a) sin ωt − cos ωt
25. A body of mass 0.2 kg and velocity after 1s, it (b) sin 3 ωt
passes through its mean position be 6 m/s executes π 
SHM. Find out the total energy and potential (c) 3 cos  − 2ωt 
4 
energy of the body, if time period of the body is 8s
during the SHM. (d) cos ωt + cos 3ωt + cos 5ωt
26. A block whose mass is 1 kg is fastened to a spring. (e) exp ( −ω 2 t 2 )
The spring has a spring constant of 50 Nm−1 . The (f) 1 + ωt + ω 2 t 2
block is pulled to a distance x = 10 cm from its 32. The motion of a particle executing simple harmonic
equilibrium position at x = 0 on a frictionless motion is described by the displacement function,
surface from rest at t = 0. Calculate the kinetic,
x( t ) = A cos (ωt + φ )
potential and total energies of the block when it is
5 cm away from the mean position. [NCERT] If the initial ( t = 0) position of the particle is 1 cm
and its initial velocity is ω cm /s, then what are its
27. Two identical springs of spring constant k are amplitude and initial phase angle? The angular
attached to a block of mass m and to fixed supports frequency of the particle is π s −1 . If instead of the
as shown in figure. Show that when the mass is cosine function, we choose the sine function to
displaced from its equilibrium position on either describe the SHM, x = B sin(ωt + α ), then what are
side. It executes a simple harmonic motion. Find the amplitude and initial phase of the particle with
the period of oscillations. [NCERT] the above initial conditions? [NCERT]
33. A cylindrical piece of cork of base area A, density ρ
k
m
k and height L floats in a liquid of density ρL . The
cork is depressed slightly and then released. Show
that the cork oscillates up and down simple
28. A spring balance has a scale that reads from 0 to harmonically and find its time period of
50 kg. The length of the scale is 20 cm. A body oscillations. [NCERT]
suspended from this spring, when displaced and 34. A person normally weighing 50 kg stands on a
released, oscillates with a period of 0.60 s. What is massless platform which oscillates up and down
the weight of the body? [NCERT] harmonically at a frequency of 2.0 s −1 and an
29. A 5 kg collar is attached to a spring of spring amplitude 5.0 cm. A weighing machine on the
constant 500 Nm −1 . It slides without friction over a platform gives the person’s weight against time.
horizontal rod. The collar is displaced from its (i) Will there be any change in weight of the body,
equilibrium position by 10.0 cm and released. during the oscillation?
130 CBSE Term II Physics XI

Head
(ii) If answer to part (i) is yes, then what will be
the maximum and minimum reading in the
machine and at which position?
[NCERT Exemplar]
35. Consider a block of mass 700 g is fastened to a
spring having spring constant of 70 N/m. Find Connection
to the shaft
out the following parameters, if block is pulled a
distance of 14 cm from its mean position on a Piston
frictionless surface and released from rest at In ideal condition, piston can be assumed to be
t = 0. frictionless and hence whenever a force is applied by
the means of combustion, the piston undergoes
oscillation.
k This oscillation may die out quickly but for a brief
m moment the motion of the piston can be expressed as
x pure harmonic.
– xm x=0 xm
(i) Give the name of three important characteristics of a
( i) The angular frequency, the frequency and the SHM.
period of the resulting motion.
(ii) If the piston is given a small displacement from the
(ii) The amplitude of the oscillation. mean position, a force comes into play which tends to
( iii) The maximum speed of the oscillating block. bring the piston back to the mean point, this give rise
( iv ) The maximum acceleration of the block. to vibrations. Define phase of a vibrating piston.
( v ) The phase constant and hence the (iii) The piston in the cylinder head of a locomotive has a
displacement function x ( t ). stroke (twice the amplitude) of 1.0 m. If the piston
executes simple harmonic motion with an angular
l
Case Based Questions frequency of 200 rad/min, then what is its maximum
Direction Read the following passage and answer speed? [NCERT]
the questions that follows (iv) The vertical motion of a huge piston in a machine is
simple harmonic with a frequency of 0 . 50 s −1 . A block
36. Oscillating Piston of 10 kg is placed on the piston, what is the maximum
In locomotives, a moving component that is amplitude of the piston’s SHM for the block and the
contained by a cylinder is called piston. piston to remain together?
Chapter Test 7. Two identical springs of spring constant k are attached
to a block of mass m and to fixed supports as shown in
figure. When the mass is displaced from equilibrium
Multiple Choice Questions position by a distance x towards right, find the restoring
1. The function log a(ωt ) force.
(a) is a periodic funciton m
k k
(b) is a non-periodic function
(c) could represents osicllatory motion
(d) can represent circular motion
2. A particle executing simple harmonic motion with an 8. A body of mass m is situated in a potential field
amplitude A and angular frequency ω. The ratio of U( x) = U0 ( 1 − cos α x), where U0 and α are constants. Find
maximum acceleration to the maximum velocity of the the time period of small oscillations.
particle is (Ans. T = 2 π m / U0α 2 )
ω2
(a) ωA (b) ω 2 A (c) ω (d)
A 9. Draw a graph to show the variation of potential energy,
3. The ratio of frequencies of two oscillating pendulums kinetic energy and total energy of a simple harmonic
are 2 : 3, then their lengths are in ratio oscillator with displacement.
(a) 2 / 3 (b) 3 / 2 10. Show that for a particle in linear SHM, the average
kinetic energy over a period of oscillation equals the
(c) 4/9 (d) 9/4 average potential energy over the same period.
[NCERT]
4. Four pendulums A, B, C and D are suspended from the
same elastic support as shown in figure. A and C are of Long Answer Type Questions
the same length, while B is smaller than A and D is
11. Fig. (a) shows a spring of force constant k clamped
larger than A. If A is given a transverse displacement,
rigidly at one end and a mass m attached to its free
G G end. A force F applied at the free end stretches the
spring. Fig. (b) shows the same spring with both ends
free and attached to a mass m at either end. Each end
of the spring in Fig. (b) is stretched by the same force F.
B A m
C k
D F

(a) D will vibrate with maximum amplitude (a)


(b) C will vibrate with maximum amplitude m m
k
(c) B will vibrate with maximum amplitude F F
(d) All the four will oscillate with equal amplitude
(b)
5. The equation of motion of a particle is x = a cos (α t ) 2.
(i) What is the maximum extension of the spring in both
The motion is the cases ?
(a) periodic but not oscillatory
(ii) If the mass in Fig. (a) and the two masses in Fig. (b)
(b) periodic and oscillatory
are released, then what is the period of oscillation in
(c) oscillatory but not periodic
each case ? [NCERT]
(d) Neither periodic nor oscillatory
(Ans. (ii) T = 2π m /2k )
Short Answer Type Questions
12. Plot the corresponding reference circle for each of the
6. A spring compressed by 0.1 m develops a restoring following simple harmonic motions. Indicate the initial
force of 10 N. A body of mass 4 kg placed on it. Deduce (t = 0 ) position of the particle, the radius of the circle
(i) the force constant of the spring, and the angular speed of the rotating particle. For
(ii) the depression of the spring under the weight of the simplicity, the sense of rotation may be fixed to be
body (Take, g = 10 N/kg), anti-clockwise in every case (x is in cm and t is in
second). [NCERT]
(iii) the period of oscillation, when the body is  π π 
(i) x = − 2 sin  3t +  (ii) x = cos  − t 
distributed and  3 6 
(iv) frequency of oscillation.  π
(iii) x = 3 sin  2 πt +  (iv) x = 2cos πt
(Ans. (i) 100 Nm −1 , (ii) 0.4 m, (iii) 1.26 s and (iv) 0.8 Hz)  4

Answers For Detailed Solutions


Multiple Choice Questions Scan the code
1. (b) 2. (c) 3. (d) 4. (b) 5. (c)
132 CBSE Term II Physics XI

EXPLANATIONS
PART 1 10. (d) In SHM, the acceleration is given by a = − ω 2 x
Hence, force (−mω 2x) always acts in the opposite
1. (a) The motion of planets and satellites are repetitive and direction of the displacement.
repeats itself after a fixed interval of time. These type of 11. (b) Given, length, L = 100 cm = 1 m
motions are known as periodic motion.
L
2. (b) A swing executes to and fro motion about a fixed point Time period of pendulum, T = 2π
in a set interval of time. Hence, the motion of a swing is g
oscillatory in nature. 1 2π
3. (c) Vibratory motion is an oscillatory motion in which = 2π = (Q g ≈ π 2 )
objects moves around a point back and forth. In the
g π2
vibratory motion, the frequency of oscillations are very 2π
= ≈ 2s
high. π
4. (a) A periodic function repeats itself after a time period T. 12. (a) When lifts falls freely, acceleration on pendulum
f (t ) = f (t + T) becomes zero. Therefore, the frequency of oscillation also
becomes zero.
As, sin (ωt ) = sin (ωt + 2 π )
13. (a) Given, m = 400g = 400 × 10 −3 kg
∴ Period of function is 2π.
and k = 10 Nm −1
5. (c) Given, time period of SHM = 6 s
m
When the motion starts from origin, its displacement is ∴ T = 2π
given by x = A sin ωt k
A 400 × 10 −3
According to question, = A sin ωt = 2π = 4 π × 10 −1s
2 10
1 14. (d) The frequency of oscillation of a spring system is
⇒ sin ωt =
2 given by
π T 1 k
⇒ ωt = ⇒ t = f =
6 12 2π m
or t =(1 / 2 ) s ⇒ f ∝ k
6. (c) Given, equation x ( t ) = 4 (cos πt + sin πt ) For given combination of spring, the equivalent force
Now, comparing above equation with general form constant,
x( t ) = A cos ωt + B sin ωt k = k1 + k 2
We get, A = 4 and B = 4 When both k 1 and k 2 are made four times, then
As, the resultant amplitude for such a equation k ′ = 4 k 1 + 4 k 2 = 4( k 1 + k 2 ) = 4k
= A2 + B2 f′ k′ 4k
∴ = = =2
∴ Amplitude = 4 2 + 4 2 = 4 2 f k k
or f ′ = 2f
7. (b) Velocity of the particle executing SHM is given as
15. (a) In equilibrium, kx = mg
v = ω A 2 − x2
mg
At extreme position, x = A ⇒ v = 0 ∴ Extension, x =
k
8. (a) Given, maximum speed, v max = 30 cm/s 20 × 9. 8
x=
and angular frequency, ω = 10 rad/s 4000
We know that, v max = ωA x = 0. 049 m
v 30 x = 4. 9 cm
⇒ A = max = = 3 cm
ω 10 16. (c) According to law of conservation of energy for the
9. (d) In SHM, the acceleration is directly proportional to spring system,
the displacement. The acceleration is always directed Initial total energy = Final total energy
towards the mean position and so is always opposite to
⇒ Potential energy = Kinetic energy + Spring elastic
displacement.
energy
i.e. a ∝ − x or a = − ω 2x 1 1
Therefore, (d) is the correct option. ⇒ mgx = mv 2 + kx2
2 2
CBSE Term II Physics XI 133

Putting v = 0, we get which is independent of the amplitude.


2 mg Thus, if the amplitude of the system is increased, then T
x=
k will remain same.
Hence, option (c) is correct. Therefore, A is true but R is false.
17. (c) We know that, for a spring-mass system, restoring 22. (a) A stiff spring has large spring constant k and a soft
force, Fs = − k x (spring force) …(i) spring has small k.
where, k is spring constant. As, frequency of oscillation, for a spring-mass system,
F = − mω 2 x (for SHM condition) …(ii) k
ν = 2 πω = 2 π … (i)
On comparing Eqs. (i) and (ii), we get m
k m  2π  This means, a block of mass m attached to stiff spring
ω= ⇒T =2 π Q T = 
m k  ω have large frequency of oscillation according to Eq. (i).
1 Therefore, both A and R are true and R is the correct
⇒ T ∝ m and T ∝
k explanation of A.
So, time period T does not depends on the amplitude of 23. (d) In damped oscillations, the energy of the system is
the oscillation, but depend on m and k. dissipated continuously although the motion is
approximately periodic for small damping but not strictly
So, statements II and III are correct but statement I is
periodic.
incorrect.
This is because, due to the presence of dissipative forces,
18. (d) If the body is given a small displacement from the
such as drag friction, etc. the amplitude of oscillation
equilibrium position, a force comes into play which tries
decreases.
to bring the body back to the equilibrium point, giving
rise to oscillations or vibrations. Therefore, A is false and R is also false.
There is no significant difference between oscillations and 24. (a) Total mechanical energy of an oscillator
1
vibrations. It seems that, when frequency is small, then it executing SHM, E = kA 2
is called oscillation (like, the oscillation of branch of a tree), 2
while when frequency is high, then it is called vibration But for damped oscillation, A ( t ) = Ae − bt / 2m
(like, the vibration of a string of musical instrument). So, for damped oscillation,
Thus, vibrations and oscillations are not two different 1 1
E = k ( Ae − bt / 2m )2 = kA 2 e − bt / m
types of motion. 2 2
Therefore, A is false and R is also false. Thus, E decreases with time t.
19. (b) x = A cos ωt and x = A sin ωt both represent the Therefore, both A and R are true and R is the correct
displacement of particle undergoing periodic motion. explanation of A.
At t = 0, 25. (i) (d) In SHM, potential energy depends on elastic
If x = A, we can represents its displacement by behaviour and kinetic energy on inertial behaviour
of system. In case of mass m oscillating on spring,
x = A cos ωt
kinetic energy is due to motion of m and potential
and if x = 0, we can represent its displacement by energy is due to stretching of spring.
x = A sin ωt
 T  2π T π
This implies that, both x = A cos ωt and x = A sin ωt , (ii) (d) x  = A cos  ×  = 0, as cos = 0
 4 T 4 2
represents the same motion depending on initial position
of particle. 1
At x = 0 , PE = kx2 = 0
Since, x = A cos ωt represents a periodic function with 2
time period of 2π rad. So, if the argument of this function (iii) (c) Both (a) and (b) are correct. Both maximum
ωt is increased by an integral multiple of 2π rad, the kinetic energy and total mechanical energy (TME)
value of the function remains the same. will become double.
Therefore, both A and R are true but R is not the correct Since, TME = (KE )max = (PE )max
explanation of A. (iv) (d) Both the kinetic and potential energies varies
20. (c) In x = A cos ωt , since cos ωt varies between +1 to − 1, between zero and maximum value. Since, total
thus the value of x varies between + A and − A. mechanical energy (KE + PE ) is constant for this
Amplitude A is a scalar quantity. system, there is interconversion of KE and PE
during motion, KE is maximum at mean position (i.e.
Therefore, A is true but R is false.
x = 0) and PE is maximum at extreme position. (i.e.
21. (c) Time period of oscillation of spring-mass system, x = ± A)
T =2π
m (v) (c) For SHM, x( t ) = A cos (ωt + φ ) … (i)
k ⇒ T2 = 2 π / ω
134 CBSE Term II Physics XI

1 2 1 2 4. (i) Given function is


PE = kx = kA cos 2(ωt + φ ) … (ii)
2 2 π 
sin ωt − cos ωt = sin ωt − sin  − ωt 
π 2 
⇒ T1 =
ω  π   π 
The displacement x( t ) is a cosine function whose  ωt + − ωt   ωt − + ωt 
= 2 sin  2  ⋅ sin  2 
time period ( T2 ) is 2π, while PE (potential energy)
 2   2 
is square of cosine function whose time period ( T`1)    
is π.
 π  π

T
T1 = 2 or 2 T1 = T2 = 2 sin   .sin ωt − 
 4  4
2
 π
= 2 sin ωt − 
 4
PART 2
This function represents a simple harmonic motion
1. (i) There is no repetition of the motion as the swimmer 2π  π
just completes one trip, hence not periodic. having a period of T = and a phase angle  − 
ω  4
(ii) The motion is repeated after a certain interval of  7 π
time, hence periodic. In fact, the bar magnet or  − .
oscillates about its mean position with a definite  4 
period of time. (ii) Given function is
(iii) Rotatary motion is periodic as repeating after fixed 1 − cos 2ωt 1 1
sin 2 ωt = = − cos 2ωt
time interval. 2 2 2
(iv) There is no repetition, hence not periodic. π
The function is periodic having a period of T =. It
2. The beat frequency of heart ω
Number of beats also represents a harmonic motion with the point of
= 1
Time taken equilibrium occurring at instead of zero.
2
75
= = 1.25 s−1 5. In SHM, x = A sin (ωt + φ ) …(i)
60 dx
= 1.25 Hz Velocity, v = = A ω cos (ωt + φ ) …(ii)
dt
1 1 A A
Beat period, T = = = 0.8 s At t = 0 , x = , then from Eq. (i), = A sin φ
Frequency 1.25 s−1 2 2
y = 6 sin10 π t + 8 cos10 π t 1 π 5π
3. Given, …(i) or sin φ = = sin or sin
The general equation of wave is 2 6 6
y = A sin (ωt + φ ) π 5π
∴ φ = or
= A sin ωt cos φ + A cos ωt sin φ 6 6
= ( A cos φ ) sin ωt + ( A sin φ ) cos ωt ...(ii)
Comparing Eqs. (i) and (ii), we get
A cos φ = 6 ...(iii) x =− A x=0 x= A x=A
A sin φ = 8 ...(iv) 2
ωt = 10 πt or ω = 10 π rad/s π
and If φ = , displacement and velocity both are positive.
2π 2π 6
∴ Time period, T = = = 0.2 s 5π
ω 10 π When φ = , displacement is positive and velocity is
6
Squaring and adding Eqs. (iii) and (iv), we get negative. Therefore, displacement-time equations of two
A 2 (cos 2 φ + sin 2 φ ) = 6 2 + 8 2 particles will be
= 36 + 64 = 100  π
x1 = A sin ωt + 
or A 2 = 100  6
∴ A = 10 cm  5 π
and x2 = A sin ωt + 
Dividing Eq. (iv) by Eq. (iii), we get  6 
8
tan φ = = 1.3333 ∴ Phase difference,
6 5π π 4π 2π
∆φ = − = = rad
∴ φ = tan −1 (1.3333) = 53 ° 8 ′ 6 6 6 3
CBSE Term II Physics XI 135

6. Suppose the particle moves from P to P ′ in time t. 10. Yes, every periodic motion need not to be SHM. e.g. The
Y motion of the earth round the sun is a periodic motion,
but not simple harmonic motion as the back and forth
x(t) motion is not taking place.
T=2s N O 11. (i) No negative sign on RHS, hence not SHM.
X′ X
90°– θ (ii) Displacement on RHS is squared, hence not SHM.
θ
(iii) a = −10 x follows the condition of SHM.
P′
P (t = 0) Acceleration ∝ – Displacement, hence SHM.
(iv) No negative sign on RHS and displacement appears
The angle swept by the radius vector, as cubed, hence not SHM.
2π 2π 12. Let A be the displacement amplitude and ω be the
θ = ωt = t= ⋅ t = πt rad
T 2 angular frequency of the simple harmonic oscillator,
π  π  then a 0 = ω 2A and v 0 = ω A
Displacement, ON = OP ′ cos  − θ = 2 cos  − θ
2  2  v 20 ω 2A 2
− x( t ) = 2 sin θ On dividing, = =A
a 0 ω 2A
(negative sign shows displacement being to the left
v 20
from O) or A=
a0
⇒ x( t ) = 2 sin πt
7. Given equation, x = 12 sin (10 t + 0.6 ) 13. As, E k = E p
On comparing with x( t ) = A sin (ωt + φ ) 1 1
∴ mω 2 ( A 2 − x 2 ) = mω 2 x 2
We have, 2 2
(i) Amplitude, A = 12 m ⇒ A − x = x or 2 x = A 2
2 2 2 2

A2 A
(ii) Angular frequency, ω = 10 rad/s ⇒ x2 = or x = ±
ω 10 2 2
(iii) Frequency, ν = = = 1. 59 Hz Thus, the energy will be half kinetic and half potential at
2π 2π
A
1 1 displacement on either side of the mean position.
(iv) Time period, T = = = 0. 628 s 2
ν 1. 59
14. (i) The motion of an artificial satellite around the earth is
(v) Initial phase, ωt + φ|t = 0 = 10 t + 0. 6|t = 0 = 0. 6 rad
periodic, as it repeats after a regular interval of time. But
8. (i) No repetition of motion. Its a unidirectional and it cannot be taken as SHM because it is not to and fro
linear but non-uniform motion of the particle. motion about any fixed point that is, mean position.
(ii) Motion repeats after every 2 s, hence periodic with
(ii) Time period of simple pendulum,
time period 2 s. l
(iii) Repetition of one position or a few positions (but not T =2 π ,
g
all) is not enough for motion to be periodic, the
entire motion during one period must be repeated i.e. T∝ l
successively. Hence, the given x-t plot is not Clearly, if the length is increased four times, the time
periodic, though there is repetition of a single period gets doubled.
position (x = 0 at every 3 s) but other positions are
15. (i) Given, x ( t ) = sin ωt + cos ωt
not repeated.
 π π
(iv) Clearly, the motion repeats itself after every 2 s. = 2 sin ωt cos + cos ωt sin 
Hence, periodic motion having a time period of 2s.  4 4
9. (i) There is no to and fro motion which is a must for a  π
x( t ) = 2 sin ωt + 
periodic motion to be SHM. Hence, rotation of earth  4
about its axis is not SHM.
 2 π   2π  π 
(ii) This is a periodic motion and as it follows F = − kx Moreover, x  t +  = 2 sin ω  t +  + 
 ω    ω  4
(about mean position to and fro motion), hence SHM.
(iii) It is a periodic motion, oscillatory in nature about  π
= 2 sin ωt + 2 π + 
lower most point as mean position following SHM  4
force law. Hence, it is SHM.  π
(iv) A polyatomic molecule has a number of natural = 2 sin ωt +  = x ( t )
 4
frequencies. So, in general, its vibration is a
superposition of SHMs of a number of different [Q sin (2π + θ ) = sin θ ]
frequencies. Thus, superposition is periodic but not
necessarily SHM.
136 CBSE Term II Physics XI

Hence, sin ωt + cos ωt is a periodic function with 17. Given, T = 2 s, A = 5 cm, t = (1 / 3 ) s


2π (i) For the particle starting from mean position
time period equal to .
ω ( i. e. φ = 0 ), displacement,
(ii) Given, x ( t ) = sin ωt + cos 2 ωt + sin 4 ωt 2π
x = A sin ωt = A sin t
sin ωt is a periodic function with period T
2π 2π 1 π
= =T = 5 sin × = 5 sin
ω 2 3 3
cos 2 ωt is a periodic function with period 3
2π π T =5× = 4. 33 cm
= = = 2
2ω ω 2 dx d( A sin ωt )
(ii) Velocity, v = = = Aω cos ωt
sin 4ωt is a periodic function with period dt dt
2π π T 2π A 2π 2π × 5 π
= = = = cos t= cos
4ω 2ω 4 T T 2 3
Clearly, the function x ( t ) repeats after a minimum  π 
= 5 × 3.14 × 0. 5 Q cos = 0.5
2π  3 
time, T = . Hence, the given function is periodic.
ω = 7 . 85 cm s −1
(iii) The function e − ωt decreases monotonically to zero as dv d( Aω cos ωt )
t → ∞. It is an exponential function with a negative (iii) Acceleration, a = = = − Aω 2 sin ωt
dt dt
exponent of e, where e ≈ 2.71828. It never repeat its
4 π 2A 2π
value, so it is non-periodic. =− sin t
T2 T
(iv) The function log (ωt ) increases monotonically with
time. As t → ∞ , log (ωt ) → ∞. It never repeat its 4 × 9. 87 × 5 π
=− sin
value, so it is non-periodic. 4 3
16. Given equation of SHM 3
= − 9. 87 × 5 × = − 42 .73 cm s −2
x(t) = 2 cos ( 4 πt + π/ 6 ) …(i) 2
We know that, x( t ) = A cos(ω t + φ ) …(ii) ∴ |a|= 42 . 73 cm s −2
Comparing Eq. (i) with Eq. (ii), we get 18. Equation of SHM,
x ( t ) = (5.0 m) co s [(2π rad s −1) t + π/4]
∴ Amplitude, A = 2 m,
Time, t = 1.5 s
ω = 4 π rad/s
Displacement,

or = 4π x ( t ) = 5. 0 × cos (2 π × 1. 5 + π/4)
T
 π π
1 = 5. 0 × cos  3 π +  = − 5. 0 cos
⇒ T = = 0. 5 s  4 4
2
The velocity of the particle can be found by = − 5. 0 × 0.707
differentiating displacement w.r.t. time. ∴ x ( t ) = − 3 . 535 m
dx d dx d   π
∴ Velocity, v = = [2 cos( 4 πt + π / 6 )] Speed, v = = 5. 0 cos 2 πt + 
dt dt dt dt  4
 π  π
= −2 sin  4 π t +  × 4 π = − 5. 0 × sin 2 πt +  × 2 π
 6  4
 π  π
or Velocity, v = − 8 π sin  4 πt +  = − 10 π sin 2 πt + 
 6  4
The initial velocity is the velocity of the oscillating body dx  π
∴ = −10 π sin  3 π +  = 10 π × 0.707
at time t = 0. dt t =1.5
 4
dx  π
∴ Initial velocity = = −8 π sin  4 π × 0 +  = 22.22 m/s [Q sin ( 3π + θ ) = − sin θ ]
dt t = 0  6
dv d   π 
 π Acceleration, a = = − 10 π sin 2 πt + 
= −8 π sin   dt dt   4 
 6
 π
 π 1 = − 10 π × 2 π cos 2 πt + 
Initial velocity = −4 π m/ s Q sin =   4
 6 2
dv  π
= −20 π 2 cos  3 π +  = 139 . 56 m/s 2
dt t=1.5
 4
CBSE Term II Physics XI 137

19. Visualise the situation using the diagram below From Eqs. (i) and (ii), we have
–ve P2 P3 P1 +ve  A− x 2 
A − ( x1 + x2 ) = A 2 ⋅  1
 − 1
A O B   A  
– 5 cm (Mean position) +5 cm
1
Now, = (2 A 2 + 2 x12 − 4 Ax1 − A 2 )
A
Velocity Acceleration Force ⇒ A 2 − A( x1 + x2 ) = A 2 + 2 x12 − 4 Ax1
(i) A 0 + + ⇒ A ( 3 x1 − x2 ) = 2 x12
(ii) B 0 − − 2 x12
∴ A=
(iii) O − 0 0 3 x1 − x2
(iv) P1 − − − 22. If the particle moves from P to P ′ in time interval t, then
(v) P2 + + + angle moved by position vector (or radius vector),
(vi) P3 − − − 2π 2π
θ = ωt = t= t = ( πt ) rad
T 2
20. Given, amplitude, A = 5 cm = 0. 05 m
Time period, T = 0.2 s Y

(i) When displacement is x = 5 cm = 0. 05 m


2
 2π 
Acceleration, a = − ω 2( x) = −   ( x)
2
 T T=2s N O X
 2π  π θ
= −   ( 0. 05 ) = − 5 π 2 m/s 2 −θ
 0.2  2
P′
Velocity, v = ω A 2 − x2
P(t = 0)
 2π 
=   ( 0. 05 )2 − ( 0. 05 )2 Displacement suffered by the particle,
 T
π 
 2π  ON = OP ′ cos  − θ = OP ′ sinθ
=  ×0=0 2 
 T
But ON = − x( t )
(ii) When displacement is x = 3 cm = 0. 03 m As it is left to mean position O, hence negative sign.
2
 2π  ⇒ − x( t ) = 3 sin θ (Q OP = 3 cm)
Acceleration, a = −   ( 0. 03 ) = − 3 π 2 m/s 2
 0.2  or x( t ) = − 3 sin θ
or x( t ) = − 3 sin πt as θ = πt
and velocity, v = ω A 2 − x2
Y
 2π 
=  ( 0. 05 )2 − ( 0. 03 )2 = 0. 4 π m/s
 0.2 
(iii) When displacement is x = 0 T=4s
P N O 2m
X
Acceleration, a = − ω 2x = 0 T=0
θ
Velocity, v = ω A 2 − x2 P′
 2π 
=   ( 0. 05 )2 − ( 0 )2 = 0 . 5 π m/s
 0.2 
21. Because the particle starts from rest, so its starting point If the particle moves from P to P ′ in the time interval t,
will be extreme position, then angle swept by radius vector,

Thus, the displacement of the particle from the mean θ = ωt = t
position after one second, T
2π πt
A − x1 = A cosωt ...(i) = t= rad
4 2
= A cosω (putting t = 1 s)
Displacement, ON = OP ′ cosθ
where, A is amplitude of the SHM. πt
After next second, A − ( x1 + x2 ) = A cos ωt or − x( t ) = 2 cos (Q OP = 1cm)
2
= A cos2ω (putting t = 2 s) Negative sign to indicate left to mean position.
= A(2 cos 2 ω − 1 ) ...(ii)  πt 
⇒ x( t ) = − 2 cos  
(Q cos 2ω = 2 cos 2 ω − 1) 2
138 CBSE Term II Physics XI

23. Given, m = 50 kg, ν = 2 s−1, A = 5 cm = 0. 05 m 1


Potential energy, PE = E − KE = 7 .2 – mv 2
Maximum acceleration, 2
a max = ω 2A = (2 πν )2 A = 4 π 2ν 2A 1
= 7 .2 – × 0.2 × ( 6 )2 = 7 .2 − 3. 6
 22 
2 2
= 4 ×   × (2 )2 × 0. 05 = 7 . 9 ms−2 = 3. 6 J
7
26. Given, m = 1 kg, k = 50 Nm−1, A = 10 cm = 0.1m,
∴ Maximum force felt by the man = m ( g + a max ) x = 5 cm = 0.05 m
= 50 (10 + 7 . 9 ) = 895. 0 N = 89 . 5 kgf 1
Minimum force felt by the man Kinetic energy, K = k ( A 2 − x2 )
2
= m ( g − a max ) 1
K= × 50 × [( 0.1 )2 − ( 0. 05 )2 ] = 0.1875 J
= 50 (10 − 7 . 9 ) 2
= 105. 0 N = 10. 5 kgf Potential energy of a block,
Hence, the reading of the weighing machine varies 1 1
U = k x2 = × 50 × ( 0. 05 )2 = 0. 0625 J
between 10.5 kgf and 89.5 kgf. 2 2
24. The displacement of a particle executing SHM is given Total energy, E = K + U = 0.1875 + 0. 0625
by E = 0.25 J
x = A cos ωt We also know that at maximum displacement, KE is zero
dx and hence the total energy of the system is equal to PE.
= −ωA sin ωt
dt Therefore, the total energy of the system
1 1
∴ Velocity, v =
dx = ( k × A 2 ) = × 50 × 0.1 × 0.1
dt 2 2
or v 2 = A 2 ω 2 sin 2 ωt = 0.25 J (Q x = A = 0.1m )
= A 2ω 2 (1 − cos 2 ωt ) which is same as the sum of two energies at a
= ω 2 ( A 2 − x2 ) displacement of 5 cm. This is in confirmity with the
Hence, v1 = ω 2 ( A 2 − x12 )
2
...(i) principle of conservation of energy.
and v 22 = ω 2 ( A 2 − x22 ) ...(ii) 27. Force exerted by left spring, trying to pull the mass
Subtracting Eqs. (i) and (ii), we get towards the mean position is given by
v12 − v 22 = ω 2 ( x22 − x12 ) F1 = − kx
v 2 − v 22
ω 2 = 12
x2 − x12 F1 F2
1/ 2
 v 2 − v 22  O
⇒ ω =  12 
 x2 − x12  x

But ω = 2 πf Similarly, force exerted by the right spring, trying to push


1/ 2
1  v12 − v 22  the mass towards the mean position is given by
f =  
2 π  x22 − x12  F2 = − k x
2π 2π π The net force acting on the mass due to both the springs,
25. Given, m = 0.2 kg, T = 8 s, ω = = = rad/s F = F1 + F2 = −2 k x
T 8 4
Thus, the force acting on the mass is proportional to its
When t = 1 s, v = 6 m / s
displacement x and is directed towards its mean position.
As, v ( t ) = ω A cos ωt Hence, the motion of the mass m is simple harmonic.
π π  π 1 Comparing the equation, F = −2 kx with F = − k ′ x.
6 = × A cos  × 1 ⇒ 6= ×A×
4 4  4 2 We have, force constant, k ′ = 2 k .
4 2 × 6 24 2 ∴ The time period of oscillation,
⇒ A= = m
π π m m
The total energy of the body, T = 2π = 2π
k′ 2k
2
1 1  π
2
 24 2  28. The length of the scale 20 cm reads upto 50 kg,
E= mω 2A 2 = × 0.2 ×   ×  
2 2  4  π  so F = mg = 50 × 9. 8 N and y = 20 cm = 0.20 m.
F 50 × 9. 8
=
230. 4
= 7 .2 J Now, force constant, k = = = 2450 Nm−1
32 y 0.20
CBSE Term II Physics XI 139

Suppose the spring oscillates with time period of 0.60 s  π


= 2 sin ωt − 
when loaded with a mass of M kg, then  4
M M Clearly, the above equation is of the form
T = 2π or T2 = 4π 2
k k x = A sin(ωt ± φ )
T 2k ( 0. 60 )2 × 2450 So, it represents SHM.
⇒ M= = = 22. 36 kg
4π 2 4 × ( 3.14 )2 Again, if t =

is the period of SHM, then
∴ Weight = Mg = 22. 36 × 9. 8 = 219.13 N ω
29. Draw the figure containing collar of 5 kg attached to a  2π    2π  π 
x t +  = 2 sin ω  t +  − 
spring of spring constant 500 N /m.  ω   ω  4
5 kg  π
k = 500 N/m = 2 sin ωt + 2 π − 
 4
Collar
 π
(i) Given, m = 5 kg, k = 500 N/m, A = 10 cm = 0.1 m = 2 sin ωt −  = x( t )
 4
The period of oscillation is given by

m 5 Hence, period, T =
T = 2π = 2π × = 0.628 s ω
k 500 π  π
(ii) Maximum speed of the collar, v max = ωA and phase angle = − or 2 π −  .
4  4
k 500 1
v max = ⋅A = × 0.1 = 1 m /s (b) Let x( t ) = sin 3 t ⇒ x( t ) = ( 3 sin ωt − sin 3ωt )
m 5 4
(iii) Maximum acceleration of the collar, Using sin 3θ = 3 sin θ − 4 sin 3 θ
k 500 The above equations represent two SHMs in
a max = ω 2A = ⋅ A = × 0.1 = 10 m/s 2
m 5 combination. The combination is periodic but not
30. Given, acceleration due to gravity on the moon , SHM.
g m = 1.7 m/s 2 3 2π
Period of sinωt = =T
Acceleration due to gravity on the earth, ge = 9. 8 m/s 2 4 ω
Time period on the earth, Te = 3. 5 s 1 2π T
Period of sin 3ωt = = T′ =
Time period on the moon, Tm = ? 4 3ω 3
On the surface of the earth, time period = Te Thus, period of the combination = minimum time
l after which the combined function repeats
∴ Te = 2π …(i) T
ge = LCM of T and = T
3
On the surface of the moon, time period = Tm
π   π
l (c) x( t ) = 3 cos  − 2ωt  = 3 cos 2ωt − 
∴ Tm = 2π …(ii) 4   4
gm
As, cos( − θ ) = cos θ
Suppose ge , gm are accelerations due to gravity on the earth
The above equation is of the form
and the moon surface, respectively.
On dividing Eq. (i) by Eq. (ii), we get x( t ) = A cos(ωt ± φ )
2π π
Te 2 π l gm Hence, it is SHM with period, T = =
= × 2ω ω
Tm 2 π l ge
(d) x( t ) = cos ωt + cos 3ωt + cos 5ωt
Te gm  ge  2π
⇒ = ⇒ Tm =   Te cosωt represents SHM with period = = T (say)
Tm ge  gm  ω
Putting the values, we get 2π T
cos 3ωt represents SHM with period = =
9. 8 3ω 3
Tm = × 3. 5 = 8. 4 s
1.7 2π T
cos 5ωt represents SHM with period = =
31. (a) Assuming, x ( t ) = sin ωt − cos ωt 5ω 5
 1 1  The minimum time after which the combined
= 2  sin ωt − cos ωt  function repeats its value is T. Hence, the given
 2 2 
function represents periodic function but not SHM,
 π π 
= 2  cos sin ωt − sin cos ωt  with period T.
 4 4  (e) x( t ) = exp ( −ω 2t 2 )
140 CBSE Term II Physics XI

The given function is an exponential function. It 33. Consider a cylinder of mass m, length L, density of
decreases monotonically x( t ) → 0 as t → ∞. material ρ and uniform area of cross-section A. Therefore,
There is no repetition of the values, hence it m = AL ρ ...(i)
represents a non-periodic function.
(f) x( t ) = 1 + ωt + ω 2t 2
Here, as t → ∞, x( t ) → ∞
l

l+y
No repetition of values, hence it represents
non-periodic function.
32. Given, x( t ) = A cos(ωt + φ )
At t = 0; x( 0 ) = 1 cm, velocity v = ω cm/s
Let the cylinder be floated in the liquid of density ρL .
Angular frequency, ω = π s−1
In equilibrium, let l be the length of cylinder dipping
⇒ 1 = A cos(ωt + φ )
in liquid.
For t = 0, 1 = A cos φ …(i)
In equilibrium, weight of cylinder = weight of liquid
dx( t ) d
Now, v( t ) = = A cos(ωt + φ ) displaced
dt dt mg = Al ρL g
= − Aω sin(ωt + φ ) m = Al ρL ...(ii)
Again at t = 0, v = ω cm/s Let the cylinder be pushed down by y, then
⇒ ω = − Aω sin φ Total upward thrust, F2 = A ( l + y ) ρL g
⇒ −1 = Asin φ …(ii) Restoring force, F = − (F2 − mg )
Squaring and adding Eqs. (i) and (ii), we get F = − [ A ( l + y ) ρL g − Al ρL g ] = − AρL gy ...(iii)
A 2 cos 2 φ + A 2 sin 2 φ = (1 )2 + ( −1 )2 In SHM, F ∝ − y
A 2 = 2 ⇒ A = ± 2 cm F = − ky ...(iv)
Hence, amplitude = 2 cm From Eqs. (iii) and (iv), we get
Dividing Eq. (ii) by Eq. (i), we have Spring factor, k = AρL g
A sin φ −1 and inertia factor, m = AL ρ
= or tan φ = −1
A cos φ 1 Time period, T = 2 π
Inertia factor
π 7π Spring factor
⇒ φ=− or
4 4 ALρ Lρ
T=2π =2π ...(v)
Now, if instead of cosine, we choose the sine function, AρL g ρL g
then x( t ) = B sin(ωt + α ).
Using m = Al ρL = ALρ
At t = 0, x = 1 cm ⇒ 1 = Bsin( 0 + α )
So, lρL = Lρ
or Bsinα = 1 …(iii)
dx( t ) d So, another form of time period,
Velocity, v( t ) = = [B sin(ωt + α )] l ρL l
dt dt T=2π =2π
= + Bω cos(ωt + α ) g ρL g
Again at t = 0, v( t ) = ω cm/s 34. (i) This is a case of variable acceleration. In accelerated
ω = + B ω cos( 0 + α ) motion, weight of body depends on the magnitude
B cosα = +1 …(iv) and direction of acceleration for upward or
downward motion.
Squaring and adding Eqs. (iii) and (iv), we get
Hence, the weight of body changes.
B 2 sin 2 α + B 2 cos 2 α = (1 )2 + (1 )2
⇒ B 2 sin 2 α + B 2 cos 2 α = 2 (ii) Considering the situation in two extreme positions,
B 2(sin 2 α + cos 2 α ) = 2 as their acceleration is maximum in magnitude.
B 2 × 1 = 2 ⇒ B = ± 2 cm N
Hence, amplitude of motion = 2 cm
Dividing Eq. (iii) by Eq. (iv), we get
Top position (highest point)
B sin α 1
= or tanα = 1
B cos α 1 a
π
∴ α=
4 mg
Mean position
CBSE Term II Physics XI 141

We have, mg − N = ma Since, there is no friction involved, the mechanical


At the highest point, the platform is accelerating energy of the spring-block system is conserved. The
downward. block is released from rest 14 cm from its
⇒ N = mg − ma equilibrium or mean position. The block will have
zero kinetic energy whenever it is again 14 cm from
But a = ω 2A (in magnitude) its equilibrium position, which means it will never
∴ N = mg − mω A 2
farther than 14 cm.
where, A = amplitude of motion. (ii) The maximum amplitude of the oscillation
Given, m = 50 kg, frequency ν = 2 s −1 = maximum displacement
∴ ω = 2 πν = 4 π rad/s ∴ xm = 14 cm = 0.14 m
A = 5 cm = 5 × 10 −2 m (iii) The maximum speed v m is given by
∴ N = 50 × 9. 8 − 50 × ( 4 π )2 × 5 × 10 −2 v m = ω xm = 10 × 0.14 = 1.4 m/s
= 50 ( 9. 8 − 16 π 2 × 5 × 10 −2 ) ( iv ) The maximum acceleration of the block is given by
= 50 ( 9. 8 − 7 . 89 ) = 50 × 1. 91 = 95 . 5 N a m = ω 2xm = 100 × 0.14 = 14 m/s 2
When the platform is at the lowest position of its At time t = 0, the block is located at position, x = xm .
oscillation, (v) From x( t ) = xm cos(ωt + φ )
Mean position xm = xm cos( 0 × ω + φ )
N cos φ = 1 ⇒ φ = 0
Displacement,
x( t ) = xm cos (ωt + φ ) = 0.14 × cos (10 t + 0 )
a
x( t ) = 0.14 cos 10 t
36. (i) Three important characteristics of an SHM are
amplitude, time period (or frequency) and phase.
mg (ii) The phase of a vibrating piston at any instant of
time is the state of piston as regards to its position
It is accelerating towards mean position, i.e. vertically
and state of motion.
upwards.
Writing equation of motion, (iii) Given, angular frequency of the piston,
N − mg = ma = mω 2A ω = 200 rad/min
or N = mg + mω 2A Stroke length = 1 m
= m ( g + ω 2A ) ∴ Amplitude of SHM,
Putting the data, Stroke length 1
A= = = 0. 5 m
N = 50 [9.8 + ( 4 π )2 × 5 × 10 −2 ] 2 2
= 50 [ 9. 8 + (12 . 56 )2 × 5 × 10 −2 ] Now, v max = ωA = 200 × 0 . 5 = 100 m/min
= 50 ( 9. 8 + 7 . 88 ) = 50 × 17 . 68 (iv) As, ν=
1 k
= 884.4 N 2π m
Now, the machine reads the normal reaction. It is clear that, ∴ k = 4 π 2m ν2
maximum weight = 884. 4 N (at lowest point) For maximum displacement, y max = A
and minimum weight = 95 . 5 N (at top point). Maximum restoring force,
35. ( i ) The angular frequency is given by F = − k A = − mg
mg mg g
ω=
k
=
70 N / m
= 10 rad/s or A= = =
m 0.700 kg k 4π m ν
2 2
4 π 2ν2
9. 8
ω 10 =
Frequency, f = = •1 . 59 Hz 4 × ( 3.14 )2 × ( 0. 50 )2
2π 2π
1 1 = 0. 99 m
Period, T = = = 0. 63 = 630 ms
f 1. 59
142 CBSE Term II Physics XI

CHAPTER 07

Waves

In this Chapter...
l Waves and Their Superposition l Reflection of Waves
l Characteristics of Wave l Standing Waves and Normal Modes
Motion
l Beats
l Transverse & Longitudinal
Waves
l Principle of Superposition of
Waves

Waves and Their Electromagnetic waves do not require any material medium for
Superposition their propagation and are also called non-mechanical waves.
Electromagnetic waves travel through the vacuum at the same
Waves occur when a system is disturbed from its speed c given as c• 3 × 10 8 m/s.
equilibrium position and this disturbance travels or
propagates from one region of the system to other. Matter Wave
In a wave, both information and energy propagate (in the These types of waves, associated with microscopic particles, i.e.
form of signals) from one point to another but there is no electrons, protons and even atoms and molecules when they are in
motion of matter as a whole through a medium. motion are called matter waves or de-Broglie waves.
Types of Waves Characteristics of Wave Motion
Usually, there are three types of waves as given below l
In a wave motion, the disturbance travels through the medium
Mechanical Wave due to repeated periodic oscillations of the particles of the
medium about their mean positions.
The waves requiring a material medium for their
propagation are called mechanical waves. These waves l
The energy is transferred from one place to another without
are also called elastic waves because their propagation any actual transfer of the particles of the medium.
depend on the material media, which possess elasticity l
There is a continuous phase difference between two successive
and inertia. e.g. Water waves, sound waves, etc. particles because each particle receives disturbance a little
Electromagnetic Wave later than its preceding particle.
These types of waves, travelling in the form of oscillating
l
The velocity with which a wave travels is different from the
electric and magnetic fields are called electromagnetic velocity of the particles with which they vibrate simple
waves. harmonically about their mean positions.
CBSE Term II Physics XI 143

l
In a given medium, the wave velocity remains constant, The angular wave number also known as propagation
while the particle velocity changes continuously during its constant is 2π times the number of waves that can be
vibration about the mean position. accommodated per unit length.
The particle’s velocity is maximum at the mean position and Period, Angular Frequency and Frequency
zero at the extreme position. The time taken by a wave to travel a distance equal to one
l
For the propagation of a mechanical wave, the medium must wavelength is known as time period of wave.
possess the properties of inertia, elasticity as well as friction.
y

Transverse Wave a
t1
In transverse wave, individual particles of the medium execute t
simple harmonic motion about their mean positions in a
direction perpendicular to the direction of propagation of the
wave. e.g. Waves in string. T
Graph for displacement, time period,
These waves can propagates only through solids.
amplitude and frequency

Longitudinal Wave Angular frequency, ω =



and its SI unit is rads −1 .
The waves in which the individual particles of the medium T
execute simple harmonic motion about their mean positions The number of complete vibrations or oscillations produced
along the direction of propagation of the wave are called by a wave in one second is known as the frequency of the
longitudinal waves. e.g. Waves in spring. wave.
These waves can propagate in all elastic media. 1 ω
Frequency, ν = =
T 2π
Displacement Relation in a Progressive Wave
ν is usually measured in hertz (Hz).
The displacement y( x, t ) denotes the transverse displacement
of the element at position x at time t and is given by Relation between wavelength λ and velocity v of a wave
is given by
y( x, t ) = A sin ( kx − ωt ) + B cos ( kx − ωt )
v
= a sin ( kx + ωt + φ) λ=
ν
where, a = amplitude of wave = A 2 + B 2 ,
φ phase constant at x = 0, t = 0 = tan −1 (B/A) Speed of a Travelling Wave
ω = angular frequency of the wave, λ
Speed of travelling wave is given by v = νλ =
x = position of vibrating element T
and k = angular wave number. The above equation is general relation for all progressive
waves, shows that the wave pattern travels a distance equal
Amplitude and Phase to the wavelength of the wave.
The magnitude of the maximum displacement of the particles
from their equilibrium position, is known as amplitude of the Speed of a Transverse Wave
particle. on Stretched String
The quantity ( kx − ωt + φ) is called the phase of the wave, The speed of the wave along a stretched ideal string
which appears as the argument of the sine or cosine function. depends only on the tension and the linear mass density of
the string and does not depend on the frequency of the
Wavelength and Angular wave. It is given by
Wave Number v=
T
The minimum distance between two particles having the same µ
phase is called the wavelength of the wave, usually denoted
by λ. Speed of a Longitudinal Wave
2π (Speed of Sound)
Wavelength, λ =
k In a longitudinal wave, the constituents of the medium

⇒ Angular wave number, k = rad m−1 . oscillate forward and backward in the direction of wave
λ propagation.
144 CBSE Term II Physics XI

In case of sound wave, it will be compressions and γp


Speed of sound, v =
rarefactions formed in air, then speed of wave is given by ρ
B This modification of Newton’s formula is referred to as the
v= Laplace correction. For air, γ = 7 5.
ρ
Speed of longitudinal wave in term of temperature of the gas,
where, B is the Bulk modulus and ρ is the density of the
medium. γRT
v=
M
The speed of longitudinal waves in a solid bar is given by
where, T is temperature of the gas and M is molar mass of
Y
v= the gas.
ρ
where, Y is the Young’s modulus of the material of the bar. Principle of Superposition
Speed of Mechanical Waves of Waves
The speeds of different types of sound waves, infrasonic, This principle states that when two or more pulses overlap,
audible or ultrasonic, in a given medium, are all given by the resultant displacement is the algebraic sum of the
displacements due to each pulse.
E
Speed of mechanical waves, v =
ρ Interference of Wave
where, E is the (appropriate) elastic constant and ρ is the Wave interference is the phenomenon that occurs, when two
density of the medium. or more waves pass through the same medium.
Let the two waves disturbances in the medium having
Speed of Electromagnetic Waves displacements y 1 ( x, t ) and y 2 ( x, t ).
In vacuum, the speed of propagation c of all types of Then, the net displacement y( x, t ) is
1
electromagnetic waves, is given by c = y( x, t ) = y 1 ( x1 , t ) + y 2 ( x 2 , t )
µ 0 ε0
For n waves superimposing with each other
where, µ 0 = permeability of vacuum and ε 0 = permittivity of
vacuum. y = f 1 ( x − vt ) + f 2 ( x − vt ) +…+ f n ( x − vt )
n
In any other material medium, the speed of propagation v of
electromagnetic waves, is given by = ∑ f i ( x − vt )
i =1
1
Speed of electromagnetic waves, v = This principle of superposition is basic to the phenomenon of
µε interference.
where, µ = permeability of the medium and ε = permittivity By the principle of superposition, the net displacement of the
of the medium. resultant wave,
φ  φ
Speed of the Longitudinal Wave in an y( x, t ) = 2 a cos sin  kx − ωt + 
2  2
Ideal Gas
φ
p The phase angle of resultant wave is
Speed of longitudinal waves in an ideal gas, v = 2
ρ φ
and the amplitude is A( φ) = 2 a cos .
where, p is the pressure of gas. 2
This is known as Newton’s formula. Case I For φ = 0, i.e. the two waves are in phase.
y( x, t ) = 2 a sin ( kx − ωt )
Laplace Correction
i. e. The resultant wave will have amplitude 2a
Laplace told that, the changes in pressure and volume of a (amplitude will be maximum).
gas, when sound waves are propagated through it, are not
isothermal but adiabatic. Hence, we can apply, Case II For φ = π, i.e. the two waves are out of
pV γ = constant ⇒ pγV γ −1 ∆V + V γ ∆p = 0 phase by 180°.
y( x, t ) = 0
If γ is the ratio of two specific heats C p C V . The speed of (i.e. zero displacement everywhere at all times)
sound is therefore, given by
CBSE Term II Physics XI 145

When waves interfere in phase, it is called constructive Vibration of String Fixed at Both Ends
interference and when they interfere in opposite phase that
is called destructive interference. A stretched string having length L fixed at both ends. The
one end be at x = 0 fixed while the other one at x = L.
For interference between sources of amplitudes A1 , A 2 and ⇒ Possible wavelengths of stationary waves are
associated intensities I 1 , I 2 as I ∝ A 2 . 2L
2
λ= , n = 1, 2, 3 K .
I max  I 1 + I 2  n
=  nv
Frequency of the vibration, ν = , where n = 1 , 2 , 3 K .
I min  I 1 − I 2  2L
When I 1 = I 2 = I 0 , then I resultant = 4I 0 cos 2 ( φ 2 ) Vibration of a String Fixed at One End
where, φ is the phase difference. Standing waves can also be produced on a string which is
fixed at one end and whose other end is free to move in a
Reflection of Waves transverse direction.
(2 n + 1 )
When a pulse or wave meets a rigid boundary, they get Frequency of the vibration, ν = v
reflected, e.g. echo. If the boundary is not completely rigid, 4L
a part of the incident wave is reflected and a part is where, n = 0 , 1 , 2,... .
transmitted into second medium. If a wave is incident
obliquely on the boundary between two different media, the Vibrations of Air Column
transmitted wave is called the refracted wave. The incident An organ pipe is a cylindrical tube which may be closed at
and refracted waves Snell’s law of refraction and incident and one end (closed organ pipe) or open at both ends (open organ
reflected waves obey usual law of reflection. pipe).
By the principle of superposition, the reflected wave must If the air in pipe at its open end is made to vibrate,
have equal amplitude as well as phase difference of π. longitudinal wave is produced. Due to superposition of
If boundary point is completely free to move, the reflected incident and reflected waves, stationary waves are formed in
pulse has the same phase and amplitude as the incident pulse. pipe.

Closed Organ Pipe


Standing Waves and Normal Modes
In closed organ pipe,
Standing Waves v
A new set of waves formed when two sets of progressive Frequency, ν = (2 n + 1 ) , where n = 0 , 1 , 2 , 3 , K
4L
wave trains of the same type, i.e. both transverse or both
longitudinal having the same amplitude and same time Open Organ Pipe
period/ frequency/ wavelength and travelling with same
It is easily seen that, an open air column at both ends
speed along the same straight line in opposite directions,
generates all harmonics. The equation for the frequency will
superimpose are called standing waves or stationary waves. be same as that of string fixed at both ends.
Net resultant wave, y ( t ) = 2 a sin kx cos ωt
nv
Frequency, ν = , where n = 1 , 2 , 3 , K .
Normal Modes 2L
Points having zero amplitude (i.e. where, there is no motion,
are called nodes and the points having largest amplitude are Beats
known as anti-nodes. The periodic variation in the intensity of sound wave caused

⇒ For nodes, x = , where n = 0 , 1 , 2 , 3 ,K . . by the superposition of two sound waves of nearly same
2 frequencies and amplitude travelling in the same direction
λ are called beats.
Distance between two successive nodes is .
2 One rise and one fall in the intensity of sound constitutes one
λ
⇒ For anti-nodes, x = (2 n + 1 ) beat and the number of beats per second is called beat
4 frequency.
 2 π
n = 0, 1, 2, K Q k =  The frequency of two sources or two waves should not differ
 λ  by more than 10 Hz.
Beat frequency,
λ
Distance between two consecutive anti-nodes is . νbeat = ν1 − ν 2
2
146 CBSE Term II Physics XI

Solved Examples
Example 1. The speed of a wave in a medium is (ii) If the equilibrium pressure of air is
960 ms −1 . If 3600 waves pass through a point in the 1.01 × 10 5 Nm −2 , what are the maximum and
medium in 1 min, then determine its wavelength. minimum pressure at a point as the wave passes
Sol. Given, speed of the wave, v = 960 ms −1 through that point?
Frequency of the wave, ν = 3600 min −1 Sol. The given equation,
3600 y = 0.02 sin ( 500 t − 3 x) …(i)
= = 60 s −1 The standard equation is y = y 0 sin (ωt − kx) …(ii)
60
v 960 Comparing Eqs. (i) and (ii), we get
∴ Wavelength, λ = = = 16 m ⇒ λ = 16 m y 0 = 0.02 Nm −2 , ω = 500 rad s −1 , k = 3 m−1
ν 60
ω 500 250
Example 2. The equation of a wave is (i) Frequency, f = = = Hz
2π 2π π
π π 2π 2π
y ( x , t ) = 0.05 sin  (10x − 40t ) −  m Wave number k =

⇒ λ= = m
2 4 λ k 3
Find 250 2 π 500
and velocity, v = f λ = × = ms −1
(i) the wavelength, frequency and wave velocity, π 3 3
(ii) the particle velocity and acceleration at x = 0.5 m (ii) Here, pressure, p max = p ′0 + p 0
and t = 0.05 s. = (1.01 × 10 5 + 0.02) Nm −2
Sol. (i) The given equation may be rewritten as = 101000. 02 Nm−2
 π and p min = p ′0 − p 0 = (1.01 × 10 5 − 0.02) Nm −2
y ( x, t ) = 0.05 sin  5 πx − 20 πt −  m
 4 = 100999.98 Nm −2
Comparing this with standard equation of plane Here, p ′0 = atmospheric pressure.
progressive harmonic wave,
y ( x, t ) = A sin ( kx − ωt + φ ), we get
Example 4. If the phase difference between two waves
is 60°, then find the value of path difference

Wave number, k = = 5 π rad m −1 between them.
λ π
∴ λ = 0.4 m Sol. Given, ∆φ = 60 ° =
3
Angular frequency, 2π
−1 As we know, ∆φ = × ∆x
ω = 2 π f = 20 π rads ⇒ f = 10 Hz λ
ω λ λ×π λ
Wave velocity, v = f λ = = 4 ms −1 ⇒ ∆x = × ∆φ = ⇒ ∆x =
k 2π 3 × 2π 6
in + x-direction
Example 5. Two coherent sound sources are at
(ii) The particle velocity and acceleration at x = 0. 5 m
distances x 1 = 0.2 m and x 2 = 0.48 m from a point.
and t = 0 . 05 s are
Calculate the intensity of the resultant wave at that
dy  5π π
point, if the frequency of each wave is f = 400 Hz
= − (20 π ) (0.05) cos  − π −  = 2.22 ms −1
dt  2 4 and velocity of wave in the medium is v = 448
d2y  5π π ms −1 . The intensity of each wave is I 0 = 60 Wm −2 .
= − (20 π )2 (0.05) sin  − π −  = 140 ms −2
dt 2  2 4 Sol. Path difference, ∆x = x 2 − x1 = 0.48 − 0.2 = 0.28 m
Example 3. The equation of a sound wave in air is 2π  2 πf 
Now, phase difference, φ = ∆x =   ∆x
given by λ  v 
y = ( 0.02 Nm −2 )[sin ( 500 s −1 ) t − (3 m −1 )x ] 2 π (400) (0.28) π
= =
(i) Find the frequency, wavelength and the speed of 448 2
sound wave in air.
CBSE Term II Physics XI 147

Now, intensity of resultant wave, Since, it is given that, the tension in one string is 2%
larger than the other.
I = I1 + I 2 + 2 I1I 2 cos φ 2T
∴ T2 = T1 + 1 = 1. 02 T1
 π 100
or I = I 0 + I 0 + 2I 0 cos  
2 ν2 1.02 T1
∴ = = 1.01
= 2I 0 = 2 ( 60 ) = 120 Wm−2 ν1 T1
Now, the frequency of the tightened string,
Example 6. Figure shows a tube structure in which a
sound signal is sent from one end and received at ν 2 = ν1 (1.01) = 1.01 × 420
the other end. The semi-circular part has a radius = 424.2 Hz
of 20.0 cm. The frequency of the sound source ∴ Beat frequency,
can be varied electronically between 1000 and ν beat = ν 2 − ν1 = 424.2 − 420 = 4.2 Hz
4000 Hz. Example 8. Two tuning forks P and Q, when set
Find the frequencies at which maxima of intensity vibrating, give 4 beats/s. If a prong of the fork P is
are detected. The speed of sound in air = 340 m/ s . filled, the beats are reduced to 2 s −1 . Determine the
frequency of P, if that of Q is 250 Hz.
Sol. There are four beats between P and Q, therefore the
possible frequencies of P are 246 Hz or 254 Hz (i.e.
Sol. According to the question, 250 ± 4) Hz.
When the prong of P is filled, its frequency becomes
greater than the original frequency.
S D
2r If we assume that the original frequency of P is 254 Hz,
then on filling, its frequency will be greater than 254 Hz.
Path difference, ∆x = πr − 2 r = ( π − 2 ) r The beats between P and Q will be more than 4. But it is
For maximum intensity, given that, the beats are reduced to 2, therefore 254 Hz is
∆x = nλ not possible.
nv Therefore, the required frequency must be 246 Hz.
⇒ ( π − 2 )r = nλ =
f (This is true because on filling, the frequency may
nv n × 340 increase to 248 Hz, giving 2 beats with Q of frequency
⇒ f = =
( π − 2 )r (3.14 − 2) × 0.2 250 Hz).
(Q r = 20 cm = 0.2 m) Example 9.
= 1491n Hz (i) An engine approaches a hill with a constant
If n = 1, then f = 1491 Hz speed. When it is at a distance of 0.8 km, it blows
If n = 2, then f = 2982 Hz a whistle whose echo is heard by the driver after
If n = 3, then f = 4473 Hz 4 s.
The frequencies in the given range are 1491 Hz If speed of engine in air is 330 m/s, calculate the
and 2982 Hz. speed of engine.
Example 7. Consider the two identical piano strings, (ii) A person standing between two parallel hills
each tuned exactly to the 420 Hz. The tension in fires a gun. He hears the first echo after 1.5 s and
any one of the strings is increased by 2.0%. If they second after 2.5 s. If speed of sound is 332 m/s,
are now struck, what is the beat frequency between calculate the distance between the hills. When
the fundamentals of the two strings? (Take, length will he hear the third echo.
of the strings = 65 cm) Sol. (i) The given situation is shown in the following figure
Sol. If ν1 , v1 , T1 and ν 2 , v 2 and T2 are the frequencies, u
velocities and tensions in the first and second strings
respectively, then
v 2 ν 2 / 2L ν 2 x 800 − x
= =
v1 ν1 / 2L ν1 Distance travelled by sound when it again meets the
1 person
T2 / µ = 800 + ( 800 − x)
ν 2 2L T2
⇒ = = = 1600 − ut = 1600 − u × 4
ν1 1
T1 / µ T1
1600 − 4 u
2L Now, =4
v
148 CBSE Term II Physics XI

⇒ 1600 − 4 u = 4 v If the length of the string is 10.0cm, locate the nodes


⇒ 1600 − 4 u = 4 × 330 and the anti-nodes. How many loops are formed in
4 u = 1600 − 1320 the vibration?
4 u = 280 ⇒ u = 70 ms −1 Sol. The given equation can be written as
(ii) The given situation is shown in the following figure y = 5 sin(1.57 x)sin( 314 t )
 πx 
2nd hill 1st hill ` = 5 sin   sin(100 πt ) ...(i)
2
General equation of standing wave,
S y = 2 A sin ( kx) sin (ωt ) ...(ii)
d−x x Comparing Eqs. (i) and (ii), we get
d π
k = cm −1 , ω = 100 π rad s −1
where, d is the distance between two parallel hills 2
and x is the distance between the man and the hill in 2π π
∴ k= =
front of it (i.e. hill 1). λ 2
Q The person hears the first echo after 1.5 s, ⇒ λ = 4 cm
2x ω 100 π
1.5 = …(i) v= = = 200 cms −1 = 2 ms −1
v k π /2
2( d − x) λ 4
For second echo, 2.5 = …(ii) ∴ Length, L = n ⇒ 10 = n × ⇒ n = 5
v 2 2
From Eqs. (i) and (ii), we get Hence, string is vibrating in 5 loops.
2d
4= ⇒ d = 2 v = 2 × 332 = 664 m
v
S N N N N
A A A A A
N N

1 cm
For third echo, sound will be reflected by one hill 2 cm 2 cm 2 cm 2 cm 2 cm
and then by another hill and then to person nλ
2 d 2 × 664 Nodes are formed at L = , whereas anti-nodes are
= = = 4s 2
v 332 λ
formed at (2 n + 1 ) .
Example 10. The vibrations of a string fixed at both ends are 2
described by the equation ∴ Nodes, x = 0, 2, 4, 6, 8, 10 cm and anti-nodes,
x = 1, 3, 5, 7, 9 cm.
y = (5.00 mm) sin [(1.57cm −1 )x] sin [( 314 s −1 ) t ]
CBSE Term II Physics XI 149

Chapter
Practice
PART 1 7. The displacement of the wave given by equation
y( x , t ) = a sin ( kx − ωt + φ ), where φ = 0 at point x
Objective Questions and t = 0 is same as that at point
2 nπ
(a) x + 2 nπ (b) x +
k
l
Multiple Choice Questions (c) kx + 2 nπ (d) Both (a) and (b)
1. Water waves produced by a motorboat sailing in 8. A steel wire has linear mass density
water are [NCERT Exemplar] 6. 9 × 10 −3 kg m −1 . If the wire is under a tension of
(a) Only longitudinal 60 N, then the speed of the transverse waves on the
(b) Only transverse wire is
(c) Both longitudinal and transverse (a) 63 ms −1 (b) 75 ms −1
(d) Neither longitudinal nor transverse (c) 73 ms −1 (d) 93 ms −1
2. The frequency of a sound wave is n and its velocity 9. What does the speed of seismic wave depend
is v. If the frequency is increased to 4n, the velocity upon?
of the wave will be (a) On the size of of the medium
(a) v (b) 2v (c) 4v (d) v/ 4 (b) Shape of the medium
(c) Rigidity of the medium
3. Sound waves of wavelength λ travelling in a
medium with a speed of v m/s enter into another (d) All of the above
medium, where its speed in 2v m/s. Wavelength of 10. Two sine waves travel in the same direction in a
sound waves in the second medium is medium. The amplitude of each wave is A and the
(a) λ (b) λ /2 (c) 2λ (d) 4λ phase difference between the two waves is 120°.
The resultant amplitude will be
4. The wave generated from up and down jerk given
(a) A (b) 2A
to the string or by up and down motion of the
(c) 4 A (d) 2A
piston at end of the pipe is
(a) transverse (b) longitudinal 11. Speed of sound wave in air
(c) Both (a) and (b) (d) electromagnetic wave (a) is independent of temperature
5. With propagation of longitudinal waves through a (b) increases with pressure
medium, the quantity transmitted is (c) increases with increase in humidity
(a) matter (d) decreases with increase in humidity
(b) energy 12. A standing wave is generated on a string. Which of
(c) Both energy and matter the following statement(s) is/are correct for the
(d) energy, matter and momentum standing waves?
(a) The amplitude of standing wave varies from
6. Equation of progressive wave is point-to-point but each element of the string oscillates
 π with the same angular frequency ω or time period.
y = a sin  10πx + 11 πt + 
 3 (b) The string as a whole vibrates in phase with differing
amplitudes at different points.
The wavelength of the wave is (c) The wave pattern in neither moving to the right nor to
(a) 0.2 unit (b) 0.1 unit the left.
(c) 0.5 unit (d) 1 unit (d) All of the above
150 CBSE Term II Physics XI

13. Change in temperature of the medium changes 200 N = T


(a) frequency of sound waves
(b) amplitude of sound waves
(c) wavelength of sound waves
20 m
(d) loudness of sound waves
14. Which of the following statement(s) is/are true for (a) 1s (b) 0.5s
wave motion? (c) 2s (d) Data given is insufficient
(a) Mechanical transverse waves can propagate through 19. At nodes in stationary waves
all mediums. (a) change in pressure and density are maximum
(b) Longitudinal waves can propagate through solids only. (b) change in pressure and density are minimum
(c) Mechanical transverse waves can propagate through (c) strain is zero
solids only. (d) energy is maximum
(d) Longitudinal waves can propagate through vacuum.
20. Following two wave trains are approaching each
15. Let a wave y(x , t ) = a sin (kx − ωt ) is reflected from other.
an open boundary and then the incident and
y1 = a sin 2000 πt
reflected waves overlaps. Then, the amplitude of
resultant wave y 2 = a sin 2008 πt
(a) 2a cos ( kx) (b) 2a sin ( kx) The number of beats heard per second is
 kx (a) 8 (b) 4
(c) 2 a sin   (d) a sin ( kx)
2 (c) 1 (d) zero

16. Equation of a plane progressive wave is given by l


Assertion-Reasoning MCQs
 x
y = 0. 6 sin 2 π  t −  .On reflection from a denser Direction (Q. Nos. 21-28) Each of these questions
 2 contains two statements Assertion (A) and Reason (R).
medium, its amplitude becomes (2/3)rd of the Each of these questions also has four alternative
amplitude of the incident wave. The equation of choices, any one of which is the correct answer. You
the reflected wave is have to select one of the codes (a), (b), (c) and (d) given
 x below.
(a) y = 0.6 sin2π  t + 
 2 (a) Both A and R are true and R is the correct
 x explanation of A
(b) y = − 0.4 sin2π  t +  (b) Both A and R are true, but R is not the correct
 2
explanation of A
 x
(c) y = 0.4 sin2π  t +  (c) A is true, but R is false
 2
 x (d) A is false and R is also false
(d) y = − 0. 4 sin 2 π  t − 
 2 21. Assertion The light emitted by stars, which are
hundreds of light years away, reaches us through
17. A sound wave is passing through air column in the interstellar space even though the interstellar space
form of compression and rarefaction. In
is practically a vacuum.
consecutive compressions and rarefactions,
(a) density remains constant Reason Light is an electromagnetic wave and do
(b) Boyle’s law is obeyed not necessarily require a medium for propagation,
(c) bulk modulus of air oscillates so it can even travel in vacuum.
(d) there is no transfer of heat 22. Assertion Transverse waves are possible in solids
18. A string of mass 2.5 kg is under tension of 200 N. and strings (under tension) but not in fluids.
The length of the stretched string is 20 m. If the Reason Solids and strings have non-zero shear
transverse jerk is struck at one end of the string, modulus, but fluids have approximately zero shear
the disturbance will reach the other end in modulus.
CBSE Term II Physics XI 151

23. Assertion Ocean waves hitting a beach are always The Fig. (b) shows how the displacement of A
found to be nearly normal to the shore. varies with time.
Reason Ocean waves are longitudinal waves.
Cork A
24. Assertion Compression and rarefaction involve
changes in density and pressure.
Reason When particles are compressed, density of Cork B
medium increase and when they are rarefied, density
of medium decreases.
8.0cm
25. Assertion Longitudinal waves can be propagated
through solids and fluids both. (a)
Reason Solids as well as fluids have non-zero bulk
+2
modulus, i.e. they can sustain compressive stress.
Displacement 0
26. Assertion Speed of sound is more in liquids and (mm) 0.25 0.50 0.75 1.00 1.25
solids than gases. –2 Time(s)
(b)
Reason Liquids and solids have higher densities
than gases.
(i) Name the type of waves produced on water
27. Assertion The amplitude A(φ ) of the resultant of the surface.
two light travelling waves given by equations (a) Longitudinal wave
y1 ( x , t ) = A sin ( kx − ωt ) (b) Transverse wave
(c) Sound wave
and y 2 ( x , t ) = A sin ( kx − ωt + φ )
(d) EM wave
decreases, as φ increases from 0 to π .
(ii) What is the amplitude of the vibrations of A as
Reason The amplitude of the resultant of the two wave passes?
above mentioned waves is given by (a) 2 mm (b) 0.25 mm
A(φ ) = 2 A cos (φ / 2 ) (c) 0.50 mm (d) 8 mm
28. Assertion When two vibrating tuning forks having (iii) The horizontal distance between A and B is half
frequencies 250 Hz and 512 Hz are held near each the wavelength of the wave, then the
other, beats cannot be heard. displacement of B with time is
Reason The principle of superposition of valid only, (a) same as that of A with equal magnitude
if the frequencies of the oscillators are nearly equal. (b) opposite to that of A with equal magnitude
(c) double in magnitude as that of A
l
Case Based MCQs (d) half in magnitude as that of A
29. Direction Read the following passage and answer the (iv) What is the frequency of the wave?
questions that follows (a) 4 Hz (b) 0.4 Hz
(c) 2 Hz (d) 0.2 Hz
Displacement of wave
(v) If the distance between the centre of the ripple
A stone os dropped in a liquid at rest in a tank. The
tank and its edge is 40 cm, then the time taken
Fig. (a) below shows circular wavefronts. The waves
by the wave to travel from the centre of the tank
produced at the centre of a circular ripple tank. Two
to the edge is
corks A and B, floats on the water and moves up and
(a) 5 s (b) 2.5 s
down on the surface as the wave passes. The
wavelength of the wave is 8.0 cm. (c) 3 s (d) 4.5 s
152 CBSE Term II Physics XI

10. A guitar string is 100 cm long and has a frequency


PART 2 of 125 Hz. Where should it be pressed to produce a
frequency of 200 Hz ?
Subjective Questions 11. A metre-long tube open at one end, with a movable
piston at the other end, shows resonance with a
l
Short Answer (SA) Type Questions fixed frequency source (a tuning fork of frequency
340 Hz), when the tube length is 25.5 cm or
1. Given below are some examples of wave motion. 79.3 cm. Estimate the speed of sound in air at the
State in each case if the wave motion is transverse,
temperature of the experiment. The edge effects
longitudinal or a combination of both.
may be neglected. [NCERT]
(i) Motion of a kink in a longitudinal spring
produced by displacing one end of the spring 12. A steel rod 100 cm long is clamped at its middle.
side ways. The frequency of longitudinal vibrations of the rod
(ii) Waves produced in a cylinder containing a liquid are given to be 2.53 kHz. What is the speed of
by moving its piston back and forth. sound in steel? [NCERT]

(iii) Waves produced by a motorboat sailing in water. 13. (i) For the wave on a string described by
y = 0. 06 sin (2 π/3) x cos (120 π t ), do all the points
(iv) Ultrasonic waves in air produced by a vibrating
quartz crystal. [NCERT]
on the string oscillate with the same (a)
frequency, (b) phase (c) and amplitude? Explain
2. If a progressive wave travelling in positive your answers.
x-direction having the amplitude of 6 cm,
(ii) What is the amplitude of a point 0.375 m away
frequency is 200 Hz and velocity is 400 m/s, then
from one end? [NCERT]
write the equation of that progressive wave.
14. A narrow sound pulse (e.g. a short pip by a whistle)
3. A wave moves with speed 300 m/s on a wire which is sent across a medium.
is under tension of 400 N. Find how much tension
must be changed to increase the speed to 315 m/s. (i) Does the pulse have a definite (a) frequency,
(b) wavelength (c) and speed of propagation?
4. A uniform cord have a mass 0.2 kg and length 6 m.
If tension is maintained in the cord by suspending (ii) If the pulse rate is 1 after every 20 s, (i.e. the
a mass of 3 kg from one end, then find out the whistle is blown for a split of second after every
speed of a pulse on this cord. Also find the time, it 20 s), is the frequency of the note produced by
takes the pulse to travel from the wall to the pulley. the whistle equal to 1/20 or 0.05 Hz? [NCERT]

5. Estimate the speed of sound in air at STP. The mass 15. Two sitar strings A and B playing the note Ga are
of 1 mole of air is 29.0 × 10 −3
kg. [NCERT]
slightly out of tune and produce beats of frequency
6 Hz. The tension in the string A is slightly reduced
6. A steel wire has a length of 12 m and a mass of and the beat frequency is found to reduce to 3 Hz.
2.10 kg. What will be the speed of a transverse If the original frequency of A is 324 Hz, then what
wave on this wire, when a tension of 2.06 × 10 4 N is is the frequency of B? [NCERT]
applied? [NCERT Exemplar]
16. A bat emits ultrasonic sound of frequency
7. A string of mass 2.5 kg is under a tension of 200 N. 1000 kHz in air. If the sound meets a water surface,
The length of the stretched string is 20.0 m. If the then what is the wavelength of (i) the reflected
transverse jerk is struck at one end of the string, sound (ii) and the transmitted sound? (Take, speed
what is the time taken for disturbance to reach the of sound in air is 340 ms −1 and in water 1486 ms −1 )
other end? [NCERT Exemplar] [NCERT]
8. Two waves of equal frequencies have their 17. A wave travelling along a string is described by
amplitudes in the ratio of 3 : 5. They are y( x , t ) = 0. 005 sin(80. 0x − 3. 0t ) in which the
superimposed on each other. Calculate the ratio of numerical constants are in SI units (0.005 m,
I max I min . 80.0 rad m −1 and 3.0 rad s −1 ). Calculate
9. At what temperature (in °C), will the speed of sound (i) the amplitude of particle,
in air be 3 times its value at 0°C? [NCERT Exemplar] (ii) the wavelength
CBSE Term II Physics XI 153

(iii) and the time period and frequency of the wave. where, x and y are in cm and t in seconds. The
Also, calculate displacement y of the particle at a positive direction of x is from left to right.
distance x = 30. 0 cm and time t = 20s. [NCERT] (i) Is this a travelling wave or a stationary wave? If
18. Earthquakes generate sound waves inside the it is travelling, what are the speed and direction
earth. Unlike a gas, the earth can experience both of its propagation?
transverse S and longitudinal P sound waves. (ii) What are its amplitude and frequency?
Typically, the speed of S wave is about 4.0 kms −1 (iii) What is the initial phase at the origin?
and that of P wave is 8.0 kms −1 . A seismograph (iv) What is the least distance between two
records P and S waves from an earthquake. The successive crests in the wave? [NCERT]
first P wave arrives 4 min before the first S wave.
Assuming, the waves travel in straight line, at what 25. The pattern of standing waves formed on a
distance does the earthquake occur? [NCERT] stretched string at two instants of time are shown in
figure. The velocity of two waves superimposing to
19. A stone dropped from the top of a tower of height form stationary waves is 360 ms −1 and their
300 m in high splashes into the water of a pond frequencies are 256 Hz.
near the base of the tower. When is the splash
heard at the top, given that the speed of sound in B D x in m

Displacement
air is 340 ms −1 ? (Take, g = 9.8 m s −2 ) [NCERT] t=0 A A' C C' E
20. One end of a long string of linear mass density
8.0 × 10 −3 kg m −1 is connected to an electrically t=? x in m
driven tuning fork of frequency 256 Hz. The other
end passes over a pulley and is tied to a pan
containing a mass of 90 kg. (a) Calculate the time at which the second curve is
plotted.
The pulley end absorbs all the incoming energy, so
that reflected waves at the end have negligible (b) Mark nodes and anti-nodes on the curve.
amplitude. (c) Calculate the distance between A′ and C′.
At t = 0, the left and (fork end) of the string x = 0
has zero transverse displacement ( y = 0) and is
l
Long Answer (LA) Type Questions
moving along positive y-direction. 26. A travelling harmonic wave on a string is described
The amplitude of the wave is 5.0 cm. Write down by y( x , t ) = 7. 5 sin( 0. 0050x + 12 t + π / 4).
the transverse displacement y as function of x and t (i) What are the displacement and velocity of oscillation
that describes the wave on the string. [NCERT] of a point at x = 1 cm and t = 1 s? Is this velocity
21. For the wave y(x , t ) = 3.0 sin(36t + 0.018 x + π/4) , equal to the velocity of wave propagation?
plot the displacement y versus time t graph for (ii) Locate the points of the string which have the
x = 0, 2 and 4 cm. What are the shapes of these same transverse displacements and velocity as the
graphs? In which aspects, does the oscillatory x = 1 cm point at t = 2 s, 5 s and 11 s. [NCERT]
motion in travelling wave differ from one point to γp
27. Use the formula, v = to explain, why the speed
another amplitude, frequency or phase? [NCERT] ρ
22. A source of frequency 250Hz produces sound of sound in air
waves of wavelength 1.32 m in a gas at STP. (i) is independent on pressure,
Calculate the change in the wavelength, when (ii) increases with temperature
temperature of the gas is 40° C. (iii) and increases with humidity? [NCERT]
23. The two individual wave functions are 28. Explain why (or how)
y1 = ( 5 cm )sin ( 4x − t ) and y 2 = ( 5 cm )sin ( 4x + t ) (i) in a sound wave, a displacement node is a
where, x and y are in centimeters. Find out the pressure anti-node and vice-versa.
maximum displacement of the motion at x = 2. 0 cm.
Also, find the positions of nodes and anti-nodes. (ii) bats can as certain distances, directions, nature
and sizes of the obstacles without any eyes.
24. A transverse harmonic wave on a string is (iii) a violin note and sitar note may have the same
described by y ( x , t ) = 3. 0 sin ( 36 t + 0. 018 x + π / 4)
frequency, yet we can distinguish between the
two notes.
154 CBSE Term II Physics XI

(iv) solids can support both longitudinal and In the figure shown above, horizontal arrows represent
transverse waves but only longitudinal the oscillation of particles along the direction of wave
waves can propagate in gases. motion.
(v) the shape of a pulse gets distorted during Compression
propagation in a dispersive medium.
[NCERT]

l
Case Based Questions
29. Direction Read the following passage and
answer the questions that follows
Tuning Fork Rarefaction
Longitudinal wave set up in a tuning fork
A tuning fork is an acoustic resonator in the
form of a two-pronged fork with the prongs (i) A tuning fork A, marked 512 Hz, produces 5 beats per
formed form a U-shaped bar of elastic metal. second, where sounded with another unmarked tuning
When a tuning fork is set into vibrations, its fork B. If B is loaded with wax the number of beats is
prong compresses the air medium just again 5 per second, what is the frequency of the tuning
infront of it. As a wave of compression fork B when not loaded?
progresses in the air along horizontal, the (ii) Frequency of tuning fork depends on
particles of the air medium also execute (a) mass of its prongs
periodic motion along horizontal as shown in (b) area of its prongs
figure.
(c) stifness of its prongs
The longitudinal wave can also be set in a
(d) density of its prongs
clamped rod (by pulling it along its length) or
in an organ pipe. (iii) A sonometer wire is vibrating in resonance with a tuning
fork. Keeping the tension applied same, the length of the
Equilibrium position wire is doubled. Under what conditions, would the tuning
Wave motion
Oscillation fork still be is resonance with the wire?
Chapter Test (i) y = 5 cos( 4 x) sin(20 t )
(ii) y = 4 sin(5 x − t / 2 ) + 3 cos(5 x − t / 2 )
Multiple Choice Questions (iii) y = 10 cos[(252 − 250 ) πt ] cos[(252 + 250 ) πt ]
1. In a longitudinal wave, the elastic property of the (iv) y = 100 cos( 100 πt + 0.5 x)
constituents of the medium that determines the stress
State which of these represent
under compressional strain is
(a) a travelling wave along − x-direction
(a) Young’s modulus (Y ) (b) Bulk modulus (B )
(c) shear modulus ( S ) (d) Either (b) or (c) (b) a stationary wave
(c) beats
2. The picture of a progressive transverse wave at a
(d) a travelling wave along − x-direction
particular instant of time gives
(a) shape of the wave Given reasons for your answers.
(b) motion of the particle of the medium 9. In the given progressive wave, y = 8 cos ( 400 πt − 0.2 πx )
(c) velocity of the wave where, y and x are in metre, t is in second. What is the
(d) None of the above
(i) amplitude,
3. A student plotted the following four graphs (ii) wavelength and
representing the variation of velocity of sound in a gas (iii) frequency? [Ans. (i) 8 m, (ii) 10 m and (iii) 200 Hz]
with the pressure p at constant temperature. Which one
is correct? Long Answer Type Questions
10. The earth has a radius of 6400 km. The inner core of
v v 1000 km radius is solid. Outside it, there is a region from
(a) (b) 1000 km to a radius of 3500 km which is in molten state.
Then, again from 3500 km to 6400 km, the earth is solid.
Only longitudinal P waves can travel inside a liquid.
p p
Assume that, the P wave has a speed of 8 kms −1 in solid
n parts and of 5 kms −1 in liquid parts of the earth.
v v
(c) (d)
An earthquake occurs at some place close to the
surface of the earth. Calculate the time after which it
will be recorded in a seismometer at a diametrically
p p opposite point on the earth, if wave travels along
diameter. (Ans. 32 min 55 s)
4. A wave equation is given by
[NCERT Exemplar]
  t x 1 
y = 4 sin π  − +  11. In the given progressive wave, y = 5 sin ( 100 π t − 0.4π x )
  5 9 6 
where, y and x are in m, t is in seconds. What is the
where, x is in cm and t is in second. The wavelength of (i) amplitude, (ii) wavelength,
the wave is
(iii) frequency, (iv) wave velocity
(a) 18 cm (b) 9 c (c) 36 cm (d) 6 cm
(v) and particle velocity amplitude? [NCERT Exemplar]
5. The equation of a progressive wave can be given by
y = 15 sin (660 πt − 0 .02 πx ) cm. The frequency of the [Ans. (i) 5 m, (ii) 5 m, (iii) 50 Hz, (iv) 250 m/s and (v) 500π m/s]
wave is 12. For the harmonic travelling wave,
(a) 330 Hz (b) 342 Hz (c) 365 Hz (d) 660 Hz y = 2 cos 2 π ( 10t − 0.0080 x + 3.5 )
where, x and y are in cm and t is in second.
Short Answer Type Questions
What is the phase difference between the oscillatory
6. The displacement of an elastic wave is given by the motion at two points separated by a distance of
function y = 3 sin ωt + 4 cos ωt, where y is in cm and t is λ
in second. Calculate the resultant amplitude. (i) 4 m, (ii) 0.5 m, (iii)
2
(Ans. 5 cm) 3λ
(iv) and (at a given instant in time)?
7. A sitar wire is replaced by another wire of same length 4
and material but of three times the earlier radius. If the (v) What is the phase difference between the oscillation
tension in the wire remains the same, by what factor will of a particle located at x = 100 cm, at t = T s and
the frequency change? t = 5 s? [NCERT Exemplar]
8. Given below are some functions of x and t to represent [Ans. (i) 6.4π rad, (ii) 0.8π rad, (iii)π rad,
the displacement of an elastic wave. (iv) 3π/2 rad and (v) 98π rad]

Answers For Detailed Solutions


Multiple Choice Questions Scan the code
1. (b) 2. (a) 3. (d) 4. (a) 5. (a)
156 CBSE Term II Physics XI

EXPLANATIONS
PART 1 9. (c) Speed of the seismic wave which are longitudinal in
nature depend on the modulus of rigidity of the medium.
1. (c) Water waves produced by a motorboat sailing in water
10. (a) Resultant amplitude
are both longitudinal and transverse, because the waves
produced are due to the transverse as well as lateral = A 2 + A 2 + 2 A 2 cosθ
vibrations in the particles of the medium. = A 2 (2 + 2 cos θ )
2. (a) Velocity of sound is independent of frequency.
Therefore, it is same ( v ) for frequency n and 4n. θ
= A 2 cos 2
3. (c) Let the frequency in the first medium is ν and in the 2
second medium is ν. ′ = A 2 × cos 60 ° (Qθ = 120 °)
Frequency remains same in both the medium. 1
v v′  v′ =A 2× =A
So, ν = ν′ ⇒ = ⇒ λ′ =   λ 2
λ λ′  v
11. (c) Due to presence of moisture, density of air decreases.
λ and λ ′, v and v ′ are wavelengths and speeds in first and γp
second medium, respectively. We know that, speed of sound in air is given by v = .
ρ
 2v
So, λ ′ =   λ = 2λ For air, γ and p are constants.
 v
1
4. (c) The wave generated from up and down jerk given to v ∝ , where ρ is density of air.
the string produces both longitudinal and transverse ρ
waves in which the movement of particle is along the v2 ρ2
propagation of wave and also perpendicular to the motion =
v1 ρ1
along the propagation of wave.
5. (b) Propagation of longitudinal waves through a medium where, ρ1 is density of dry air and ρ2 is density of moist air.
leads to transmission of energy through the medium v
As, ρ2 < ρ1 = 2 > 1 ⇒ v 2 > v1
without matter being transmitted. v1
There is no movement of matter (mass) and hence Hence, speed of sound wave in air increases with
momentum is also not transferred. increase in humidity.
6. (a) The general equation of progressive wave, 12. (d) There is no phase difference between oscillations of
y = A sin (ωt − kx + φ 0 ) different elements of the wave. However, the string as a
From comparison, we get whole vibrates in phase with different amplitudes at
kx = 10πx different points. Also, there is zero movement of the wave
pattern. Hence, they are called standing or stationary
k = 10π waves.
2π 2π
We know that, k = ⇒ λ= 13. (c) Speed of sound wave in a medium v ∝ T , where T is
π k
temperature of the medium.
2π 1
or λ= = = 0.2 unit Clearly, when temperature changes, speed also changes.
10 π 5
As, v = νλ
7. (b) The displacement of wave equation,
where, ν is frequency and λ is wavelength.
y ( x, 0 ) = a sin kx = a sin ( kx + 2 nπ )
Frequency ν remains fixed
 2 nπ 
= a sin k  x +  ⇒ v ∝ λ or λ ∝v
 k 
 2 nπ  As does not change, so wavelength λ changes.
⇒ The displacement at points x and  x +  are the
 k  14. (c) When mechanical transverse wave propagates through
same, where n = 1 , 2 , 3 , ... . a medium, the constituent of the medium oscillate
8. (d) Linear mass density = 6.9 × 10 −3 kg m −1 perpendicular to wave motion causing change in shape.
i.e. Solids and strings have shear modulus, i.e. why they
Tension, T = 60 N can sustain shearing stress.
Thus, speed of wave on the wire is given by Fluids have no shape of their own, they yield to shearing
T 60 N stress. This is why, transverse waves are possible in solids
v= = = 93 ms −1
µ 6.9 × 10 −3 kg m −1 and strings but not in fluids.
CBSE Term II Physics XI 157

15. (b) We have, incident wave, y1 = a sin ( kx − ωt ) 19. (b) In stationary waves, all particles except nodes oscillate
So, the reflected wave, y 2 = a sin ( kx + ωt ) with same frequency but amplitude is zero at nodes and
maximum at anti-nodes. Thus, change in pressure and
From principle of superposition, the standing wave density is minimum at nodes.
equation obtained after superimposing y1 and y 2,
ω − ω1
we get 20. (b) Beat frequency = f 2 − f1 = 2

y ( x, t ) = 2 a sin kx cos ωt
2008 π − 2000 π
Thus, the resultant amplitude is = = 4 Hz

A ( x) = 2 a sin kx
21. (a) Light is an electromagnetic wave and can travel in
16. (b) Amplitude of reflected wave,
vacuum as well, i.e. it does not necessarily require a
2 2
A r = × A i = × 0.6 = 0.4 units medium to propagate.
3 3 So, the light emitted by star, which are hundreds of light
Given, equation of incident wave, years away reaches us through interstellar space even
 x though the intersteller space is practically a vacuum.
y i = 0. 6 sin 2 π  t − 
 2 Therefore, both A and R are true and R is the correct
explanation of A.
Equation of reflected wave,
 x  22. (a) In transverse waves, the particle motion is normal to
y r = A r sin2 π  t + + π the direction of propagation of the wave. Therefore, as
 2 
the wave propagates, each element of the medium
(Q at denser medium, phase changes by π) undergoes a shearing strain. Transverse waves, can
The positive sign is due to reversal of direction of therefore, be propagated in those media, which can
propagation. sustain shearing stress.
 x Since, solids and strings have shear modulus and hence
So, y r = − 0. 4 sin 2 π  t +  [Qsin( π + θ ) = − sin θ] they can sustain shearing stress. So, transverse waves can
 2
be generated in them.
17. (d) (a) Due to compression and rarefactions, density of
However, fluids have approximately zero shear modulus.
the medium (air) changes. At compressed regions,
Thus, they cannot sustain shearing stress and get
density is maximum and at rarefactions, density is
deformed, i.e. they do not have shape of their own and
minimum.
hence no transverse wave is possible in fluids.
(b) As density is changing, so Boyle’s law is not
Therefore, both A and R are true and R is the correct
obeyed.
explanation of A.
(c) Bulk modulus remains same.
23. (c) Ocean waves are tranverse waves travelling in
(d) The time of compression and rarefaction is too concentric circles of ever increasing radius. When they
small, i.e. we can assume adiabatic process and hit the shore, their radius of curvature is so large that
hence no transfer of heat takes place. they can be treated as plane waves. Hence, they hit the
18. (b) Given, mass, m = 2.5 kg shore nearly normal to the shore.
µ = mass per unit length Therefore, A is true but R is false.
m 2. 5 kg 1.25 24. (a) A compression is a region of medium in which
= = = = 0.125 kg/m particles come closer, i.e. distance between the particles
l 20 10
become less than the normal distance between them.
T 200
Speed, v = = Thus, there is temporary decrease in volume and a
µ 0.125 consequent increase in density of medium. Similarly, in
(speed of transverse waves in any string) rarefaction, particles get farther apart and a consequent
decrease in density is noticed.
200
l = v × t ⇒ 20 = ×t Therefore, both A and R are true and R is the correct
0.125
explanation of A.
125 25 × 5
⇒ t = 20 × = 20 × 25. (a) In longitudinal waves, the constituents of the medium
2 × 10 5
2 × 10 5 oscillate about their mean position along the direction of
1 wave propagation. This means, it involve compressive
= 20 × 25 × stress (pressure).
0. 4 × 10 5
Since, solids as well as fluids have bulk modulus, i.e. they
1 20 × 5 can sustain compressive stress. So, longitudinal waves
= 20 × 5 =
4 ×10 4
2 × 10 2 can be propagated through solids and fluids both.
1 Therefore, both A and R are true and R is the correct
= = 0. 5 s explanation of A.
2
158 CBSE Term II Physics XI

B (v) (b) As, λ = 8 cm and d = 40 cm


26. (b) Speed of sound in a medium is given as v = …(i)
ρ ∴ v = f × λ = 2 × 8 = 16 cm/s
where, B is the bulk modulus and ρ is the density of the d 40
Time, t = = = 2. 5 s
medium. v 16
Since, solids and liquids are much more difficult to
compress than gases, so they have much higher values of
bulk modulus.
PART 2
Also, generally solids and liquids have higher mass 1. (i) Transverse and longitudinal
densities (ρ) than gases. (ii) Longitudinal
But corresponding increase in both the modulus B of (iii) Transverse and longitudinal
solids and liquids is much higher. So, in accordance to (iv) Longitudinal
Eq. (i), we can say that sound waves travel faster in solids 2. Given, A = 6cm = 0.06 m
and liquids.
ν = 200 Hz
Therefore, both A and R are true but R is not the correct
2π 2π  v
explanation of A. k= = Q λ = 
λ v/ ν  ν
27. (a) From the principle of superposition of waves, the net
displacement of two light travelling waves 2 πν 2 π × 200
k= = = π m −1
y1 ( x, t ) = A sin ( kx − ωt ) v 400
and y 2 ( x, t ) = A sin ( kx − ωt + φ ) is ω = 2 πν = 2 π × 200 = 400 π rad/s
φ  φ The standard equation of the progressive wave,
y ( x, t ) = 2 A cos sin  kx − ωt + 
2  2 y( x, t ) = A sin( kx − ωt )
φ Put the values to get the equation,
The initial phase angle =
2 y ( x, t ) = 0.06sin( πx − 400 π t ) m
The resultant amplitude of the given wave, T
3. We know that, speed, v =
φ µ
A( φ ) = 2 A cos
2 dv 1
⇒ =
As, from 0 ≤ φ ≤ π, cosine function is decreases. dT 2 µT
Thus, resultant amplitude also decreases for 0 ≤ φ ≤ π.
d v 1 dT  T
Therefore, both A and R are true and R is the correct ⇒ =  putting µ = 
explanation of A. v 2 T  v 
28. (c) The principle of superposition of valid only, if the dv
⇒ dT = (2 T )
frequencies of the oscillation are nearly equal. For beats v
to be heard, the condition is that difference in frequency  315 − 300 
of two oscillations should not be more than 10 times per = 2 × 400 ×  
 300 
second for a normal human ear to recognise it. Hence, we
cannot hear beats in the case of two tuning forks 2×4
= × 15
vibrating at frequencies 256 Hz and 512 Hz, respectively. 3
Therefore, A is true but R is false. = 2 × 4 × 5 = 40 N
29. (i) (b) Transverse waves are the types of waves Hence, tension should be increased by 40 N.
produced on water surface. 4. Given, mass of a suspended block, m = 3 kg
(ii) (a) From figure, amplitude = 2 mm Mass of the cord, m = 0.2 kg
(iii) (b) The displacement of B with time is Length of the cord, l = 6 m

+2 5m
0.50 1.00 1.25
Displacement 0 1m
0.25 0.75 time(s)
(mm)
–2 3 kg

Tension, T = weight of mass of 3 kg = m × g


So, the displacement of B are opposite to that of A
with equal magnitude. = 3 × 9.8 = 29.4 N
m 0.2
1
(iv) (c) Frequency, f = =
1
= 2 Hz µ= = = 0.033 kg/m
T 0. 50 l 6
CBSE Term II Physics XI 159

As we know that, the wave speed or speed of a pulse on 10. Given, ν1 = 125 Hz, ν 2 = 200 Hz and L 1 = 100 cm
the cord is given by Frequency of a string fixed at both ends,
T v
v= …(i) ν=
µ 2L
1 T  T
ν= Q v = 
µ 
Putting the values in Eq. (i) , we get
2L µ 
29. 4
v= • 29.85 m/s As, T and µ are fixed.
0. 033
ν1 L 2 ν
Thus, the time for this speed to travel from the wall to = ⇒ L 2 = 1 L1
ν2 L 1 ν2
pulley, i.e. a distance of 5 m.
125 125
Time =
5
= 0.168 s ⇒ L2 = × 100 = = 62.5 cm
29.85 200 2
5. We know that, volume of any gas at STP is 22.4 L. 11. As, there is piston at one end, it behaves as a closed
organ pipe.
Density of air at STP,
 Mass  Wavelength is given by
ρ0 =   λ
 Volume at STP  for one mole of air = 25.5 cm
4
29.0 × 10 −3
= = 1.29 kgm−3 ⇒ λ = 4 × 25.5 = 102 cm = 1. 02 m
22.4 × 10 −3
Speed of sound in air,
According to Newton’s formula,
1/ 2 v = νλ = 340 × (1. 02 ) = 346. 8 m/s
1.01 × 10 5Nm −2 
Speed of sound, v =   = 280 ms
−1 12. Given, L = 100 cm = 1 m,
−3
 1.29 kgm  ν = 2. 53 kHz = 2. 53 × 10 3 Hz
6. Given, l = 12 m, M = 2.10 kg, T = 2.06 × 10 4 N As the given rod is clamped at middle, hence there will
M 2.10 be a node at the middle.
µ= = kg / m
l 12
A A
T 2.06 × 10 4 N
v= = = 3.43 × 10 2 m / s
µ 2.10 12 λ λ λ
Hence, length, L = + =
2.5 4 4 2
7. Here, µ = kg m, T = 200 N
20 ⇒ λ = 2L = 2 m
T 200 200 × 20 Speed, v = νλ = 2. 53 × 10 3 × 2 = 5.06 ×10 3 ms−1
v= = =
µ 2.5 20 2.5 13. (i) All the points except the nodes on the string have
the same frequency and phase but not the same
4 × 10 4 2 × 10 2 amplitude.
= = = 40 m/s
25 5 2π
(ii) Given, y = 0. 06 sin x cos(120 π t )
l 20 3
Time, t = = = 0.50 s
v 40 Putting x = 0.375 m (QL = 1 m)
A1 3 I1 3 2π
8. Given, = ⇒ = Amplitude, y = 0. 06 sin × ( 0. 375 )
A2 5 I2 5 3
2 2 π 0. 06
I max  I1 + I 2   I /I + 1 = 0.06 sin = = 0. 042 m
Now, =   = 1 2


 I /I − 1 4 2
I min  I1 − I 2   1 2 
14. (i) A short pip by a whistle
2
 3 / 5 + 1 64 16 (a) will not have a fixed frequency.
=  = =
 3 / 5 − 1 4 1 (b) will not have fixed wavelength.
The ratio of I max / I min is 16 : 1. (c) will have definite speed that will be equal to
speed of sound in air.
9. We know that, speed, v ∝ T
(ii) 0.05 Hz will be the frequency of repetition of the
where, T is in kelvin. short pip.
vt 273 + t 15. Given, frequency of A , f A = 324 Hz
= =3
v0 273 + 0 Now, frequency of B , fB = f A ± beat frequency
273 + t = 324 ± 6
⇒ = 9 ⇒ t = 9 × 273 − 273 = 2184 ° C
273 or fB = 330 or 318 Hz
160 CBSE Term II Physics XI

Now, if tension in the string is slightly reduced, its t 2 = time taken by the splash’s sound to reach top of the
frequency will also reduce from 324 Hz. tower
Now, if fB = 330 and f A reduces, then beat frequency h 300  h
should increase which is not the case but if fB = 318 Hz t2 = = = 0.882 Q v = 
v 340  t 2
and f A decreases, the beat frequency should decrease,
which is the case and hence fB = 318 Hz. Total time, t = Time to hear splash of sound
16. Given, ν = 1000 kHz = 10 6 Hz = t 1 + t 2 = 7.82 + 0.882 = 8.702
v a = 340 m/s, v w = 1486 m/s 20. Givn, ν = 256 Hz, T = m × g = 90 × 9.8 = 882 N
m
Wavelength of reflected sound, µ= = 8.0 × 10 −3 kgm−1
v 340 L
λ a = a = 6 = 3. 4 × 10 −4 m
ν 10 Amplitude, a = 5 cm = 0. 05 m
Wavelength of transmitted sound, Velocity of the transverse wave,
v 1486 T 882
λw = w = = 1486 × 10 −6 v= = = 3.32 × 10 2 m/s
ν 10 6 µ 8 × 10 −3
λ w = 1.486 × 10 −3 m
ω = 2 πν = 2 × 3.14 × 256 = 1.61 × 10 3 rad/s
17. Given, y( x , t ) = 0.005 sin(80.0 x − 3.0 t )
v 3.32 × 10 2
Then, y( x, t ) = a sin( kx − ωt ) λ= =
ν 256
Now, compare the given equation with standard equation 2π 2 × 3.14 × 256
to find out all the physical quantities. k= = = 4.84 m −1
λ 3.32 × 10 2
a = 0.005 m, k = 80.0 rad/m As, wave propagating along positive X-axis.
ω = 3.0 rad/s i.e. y = a sin(ωt − kx)
The physical quantities by using the given fundamental = 0.05 sin (1.61 × 10 3 t − 4.84 x)
physical quantities.
Here, x, y are in metre and t is in second.
(i) Amplitude, a = 0.005 m = 5 mm
21. The transverse harmonic wave,
2π 2π
(ii) Wavelength, λ = = = 7 . 85 cm  π
k 80 y ( x, t ) = 3.0 sin  36 t + 0.018x + 
 4

(iii) Time period, T = 2 π ω = = 2.09 s
3 It is the equation of travelling wave along negative
1 1 direction of x.
and frequency, ν = = = 0.478 0.48 Hz
• For x = 0, …(i)
T 2.09
Now, the displacement y of the particle at a distance. y ( x, t ) = 3. 0 sin ( 36 t + π / 4 )
2π 2π π
x = 30.0 cm = 0.3 m and time t = 20 s Here, ω= = 36, T = = s
T 36 18
y (0.3, 20) = 0.005 sin (80 × 0.3 − 3.0 × 20)
For different values of t, we calculate y using Eq. (i).
y (0.3, 20) = 0.005 sin (24 − 60)
These values are tabulated below
= 0.00495 m • 5 mm
t 0 T/8 2T/8 3T/8 4T/8 5T/8 6T/8 7 T/8 T
18. Let v1, v 2 be the velocities of S wave and P wave and t 1, t 2
be the time taken by these waves to reach the y 3/ 2 3 3/ 2 0 –3/ 2 –3 –3/ 2 0 3/ 2
seismograph.
l = distance of occurrence of earthquake from the On plotting y versus t graph, we obtain a sinusoidal curve
seismograph as shown in figure below
v1t 1 = v 2t 2 3
3
⇒ v1 = 4 km s −1, v 2 = 8 km s −1 2
⇒ 4t1 = 8t 2 ⇒ t1 = 2t 2 ...(i) y (cm)
O t (s)
t 1 − t 2 = 4 min = 240 s ...(ii) T 2T 4T 6T T
8 8 8 8
On solving Eqs. (i) and (ii), t 2 = 240 s 3
2
⇒ t 1 = 2 t 2 = 2 × 240 = 480 s –3
⇒ l = v1t 1 = 4 × 480 = 1920 km
Similar graphs are obtained for x = 2 cm and x = 4 cm.
19. Given, h = 300 m, g = 9.8 m / s 2 , v = 340 ms −1
The oscillatory motion in travelling wave differs from one
t 1 = time taken by stone to strike the water surface point to another only in terms of phase. Amplitude and
2h 300  1  frequency of oscillatory motion remain the same in all the
t1 = = = 7 . 82 s  as, h = 0 + gt 12
g 4. 9  2  three cases.
CBSE Term II Physics XI 161

22. Given, ν 0 = 250 Hz, T0 = 273 K π


(iii) Initial phase, φ =
T1 = 273 + 40 = 313 K; λ 0 = 1. 32 m 4
∴ Speed of sound, v 0 = ν 0 ⋅ λ 0 = 250 × 1. 32 = 330 m/s 2π
(iv) ω = 36, k = = 0. 018
As we know that, speed of sound, v ∝ T λ
2π 2π
⇒ λ = least distance = = cm
v1 T1 k 0. 018
Thus, = = 349.1 cm
v0 T0
25. Given, frequency of the wave, ν = 256 Hz
T1 313 1 1
v1 = v 0 = 330 = 353. 34 m / s Time period, T = = s = 3.9 × 10 −3 s
T0 273 ν 256
Q v1 = v 0 λ 1 (a) Time taken to pass through mean position,
353.34 T 3.9 × 10 −3
λ1 = = 1.41 m t= = s = 9.8 × 10 −4s
250 4 4
∴ Change in the wavelength, (b) Nodes are A, B, C, D, E (i.e. zero displacement)
∆λ = λ 1 − λ 0 = 1. 41 − 1. 32 = 0. 09 m Anti-nodes are A ′ , C′ (i.e. maximum displacement)
23. Given, y1 = ( 5 cm) sin ( 4 x − t ) (c) It is clear from the diagram, A′ and C′ are
y 2 = ( 5 cm) sin ( 4 x + t ) consecutive anti-nodes, hence separation
The resulting wave y = (2 A sin kx) cos ωt v 360
= wavelength ( λ ) = = = 1. 41 m (∴ v = νλ)
ν 256
Now, compare the given equation,
26. The travelling harmonic wave,
y1 = ( 5 cm) sin ( 4 x − t ) with y1 = A sin ( kx − ωt ).
y ( x, t ) = 7 . 5 sin( 0. 0050 x + 12 t + π / 4 )
A = 5 cm, k = 4 and ω = 1 rad/s
At x = 1 cm and t = 1 s
y = (2 A sin kx) cos ωt
y(1 , 1 ) = 7 . 5 sin ( 0. 005 × 1 + 12 × 1 + π / 4 )
y = (10 sin 4 x) cos t
= 7 . 5 sin(12. 005 + π / 4 ) ...(i)
The maximum displacement of the motion at position,
Now, θ = (12. 005 + π / 4 ) rad
x = 2 ⋅ 0 cm
180
y max = 10 sin 4 x|x = 2.0 = 10 sin ( 4 × 2 ) = 10 sin ( 8 rad) = (12. 005 + π / 4 ) degree
π
y max = 9.89 cm
12.79 × 180
The wavelength by using the relation between = = 732.55 °
22 / 7
wavelength and wave number.
2π 2π π ∴ From Eq. (i), y(1 , 1 ) = 7 . 5 sin(732. 55 ° )
k= =4 ⇒ λ= = cm = 7 . 5 sin (720 + 12. 55 ° ) = 7 . 5 sin12. 55 ° cm
λ 4 2
The nodes and anti-nodes can be given as = 7 . 5 × 0.2173 = 1. 63 cm
nλ  π Velocity of oscillation,
Nodes at x = = n ×   cm,
2  4 d d   π
v= [ y (1 , 1 )]= 7 . 5 sin  0. 005 x + 12 t +  
where n = 0 , 1 , 2 , K . dt dt  4 
λ  π  π
Anti-nodes at x = (2 n + 1 ) = (2 n + 1 ) × = 7 . 5 × 12 cos 0. 005 x + 12 t + 
  cm,  4
4  8
where n = 0 , 1 , 2 , K . (i) At, x = 1 cm, t = 1 s
24. Given equation is y ( x , t ) = 3. 0 sin( 36 t + 0. 018 x + π / 4 ) v = 7 . 5 × 12 cos ( 0. 005 + 12 + π / 4 )
Comparing with standard equation = 90cos (732.55 ° ) = 90 cos (720 + 12. 55 )
y ( x, t ) = a sin(ω t + kx + φ ) v = 90cos (12.55 ° )
(i) The given equation represents a transverse = 90 × 0.9765 = 87.89 cm/s
harmonic wave travelling from right to left (i.e. along Comparing the given equations with the standard
negative X-axis). It is not a stationary wave. form,
By comparing, we get ω = 36 rad/s, k = 0. 018/cm
ω 36
∴ Speed of wave, v = = = 2000 cm/s
k 0. 018
(ii) By comparing amplitude, a = 3 cm ⇒ 2 πν = 36
36
⇒ Frequency, ν = = 5.73 Hz

162 CBSE Term II Physics XI

⇒ y( x, t ) = a sin( kx + ωt + φ ) Anti-node At this point, displacement is maximum, i.e.


6 amplitude of oscillation will be maximum and hence
We get, a = 7 . 5 cm, ω = 12 , 2 πν = 12 or ν = pressure will be minimum as it is inversely related.
π
2π (ii) Bats emit ultrasonic waves of large frequencies. These
= 0. 005 waves will be reflected by the obstacles in their path. The
λ
reflected rays received by the bat will give idea about the
2π 2 × 3.14
∴λ= = = 1256 cm = 12. 56 m obstacle, i.e. distance, direction, size and nature.
0. 005 0. 005
(iii) As, overtones produced and relative strengths of
Velocity of wave propagation, notes are different in two notes of violin and sitar.
6
v = νλ = × 12. 56 m/s = 24 m/s Although frequencies are same, we will distinguish
π by their strengths.
We find that velocity at x = 1 cm, t = 1 s is not equal (iv) The reason behind is that solids have both the
to velocity of wave propagation. elasticity of volume as well as shape, whereas gases
(ii) Now, all points which are at a distance of ± λ , ± 2 λ , ± 3 λ have only the volume elasticity.
from x = 1 cm will have same transverse displacement (v) As in the dispersive medium wavelengths are
and velocity. As λ = 12. 56 m, therefore all points at different, hence the velocities, therefore the shape of
distances ±12. 6 m, ± 25.2 m, ± 37 . 8 m from x = 1 cm will the pulse gets distorted.
have same displacement and velocity, as x = 1 cm point at 29. (i) Frequency of tuning fork A, ν A = 512 Hz
t = 2 s, 5 s and 11 s.
27. (i) Effect of Pressure Probable frequency of tuning fork B,
γp νB = ν A ± 5 = 512 ± 5 = 517 or 507Hz
v = speed of sound in a gas =
ρ When B is loaded, its frequency reduces.
M γ pV If it is 517 Hz, it might reduced to 507 Hz given
where, p = pressure, ρ = density = ⇒v =
V M again a beat of 5 Hz.
When T is constant, pV = constant If it is 507 Hz, reduction will always increase the
⇒ V = constant beat frequency, hence νB = 517 Hz
Hence, velocity of sound is independent of the change (ii) (a) Frequency of tuning fork is inversely
in pressure of the gas provided temperature remains proportional to the square root of its material
constant. density.
Therefore, frequency of tuning fork depends on
γp
(ii) Velocity, v = mass of its springs.
ρ
(iii) Wire of twice the length vibrates in its second
According to standard gas equation, pV = RT harmonic. Thus, if the tuning fork resonates at L , it
RT γ × RT γRT will resonate at 2L. This can be explained as below
⇒ p= ⇒ v= =
V ρV M The sonometer frequency is given by
n T
where, M = pV = molecular weight of the gas ν= ,
2L m
⇒ v∝ T
where, n = number of loops.
Hence, v increases with temperature.
Now, as it vibrates with length L, we assume ν = ν1
(iii) Due to presence of water vapours in air density
changes. Hence, velocity of sound changes with n = n1
humidity. n T
∴ ν1 = 1 …(i)
Let ρm = density of moist, ρd = density of dry air, 2L m
v m = velocity of sound in moist air n2 T
and v d = velocity of sound in dry air. When length is doubled, then ν 2 = …(ii)
2 × 2L m
γp γp
vm = , vd = Dividing Eq. (i) by Eq. (ii), we get
ρm ρd ν1 n1
= ×2
vm ρd ν2 n 2
= ( as, ρd > ρm ⇒ v m > v d)
vd ρm To keep the resonance,
28. (i) Node It is a point where the amplitude of oscillation is ν1 n
= 1 = 1 × 2 ⇒ n 2 = 2 n1
zero, i.e. displacement is minimum. As pressure is ν2 n2
inversely related with displacement, i.e. when
Hence, when the wire is doubled, the number of
displacement will be minimum, pressure will be
loops also get doubled to produce the resonance.
maximum.
Physics
Class 11th ( Term II )

Practice Paper 1 *

(Solved)
General Instructions T ime : 2 Hours
Max. Marks : 35
1. There are 9 questions in the question paper. All questions are compulsory.
2. Question no. 1 is a Case Based Question, which has five MCQs. Each question carries one mark.
3. Question no. 2-6 are Short Answer Type Questions. Each question carries 3 marks.
4. Question no. 7-9 are Long Answer Type Questions. Each question carries 5 marks.
5. There is no overall choice. However, internal choices have been provided in some questions. Students have to attempt
only one of the alternatives in such questions.
* As exact Blue-print and Pattern for CBSE Term II exams is not released yet. So the pattern of this
paper is designed by the author on the basis of trend of past CBSE Papers. Students are advised
not to consider the pattern of this paper as official, it is just for practice purpose.

1. Direction Read the following passage and answer the questions that follows
A class XI student took two different strings made up of different materials. He applied some load on the
strings by fixing one end at the rigid support. He noticed some changes in the length of strings.
Later he applied muscular force on some blocks made up of different materials. He noticed that some
materials were deformed and some materials regained their shape. The deformation in the object was more
when more force was applied.
He concluded that some materials are harder to press while some materials are squishy. This property of
materials is known as elasticity.
(i) The maximum load a wire can withstand without breaking, when its length is reduced to half of its
original length, will
(a) be double (b) be half
(c) be four times (d) remain same
(ii) A spring is stretched by applying a load to its free end. The strain produced in the spring is
(a) volumetric (b) shear
(c) longitudinal and shear (d) longitudinal
(iii) Consider two cylindrical rods of identical dimensions, one of rubber and the other of steel. Both the rods
are fixed rigidly at one end to the roof. A mass M is attached to each of the free ends at the centre of the
rods.
(a) Both the rods will elongate but there shall be no perceptible change in shape
(b) The steel rod will elongate and change shape but the rubber rod will only elongate
(c) The steel rod will elongate without any perceptible change in shape, but the rubber rod will elongate and the
shape of the bottom edge will change to an ellipse
(d) The steel rod will elongate, without any perceptible change in shape, but the rubber rod will elongate with the
shape of the bottom edge tapered to a tip at the centre
166 CBSE Term II Physics XI

(iv) A wire is suspended from the ceiling and stretched under the action of a weight F suspended from its other
end. The force exerted by the ceiling on its is equal and opposite to the weight.
(a) Tensile stress at any cross-section A of the wire is F / A
(b) Tensile stress at any cross-section is zero
(c) Tensile stress at any cross-section A of the wire is 2F / A
(d) Tension at any cross-section A of the wire is 2F
(v) A copper and a steel wire of the same diameter are connected end to end. A deforming force F is applied to
this composite wire which causes a total elongation of 1 cm. The two wires will have
(a) same stress, different strains
(b) same stress, same strain
(c) different stress, different strain
(d) different stress, same strain
2. An ideal refrigerator is working between the temperature of ice and temperature of atmosphere at 300 K. Find
the energy which has been supplied to it to freeze 2 kg of water at 0°C. Given that latent heat of ice
3. 33 × 10 5 J / kg.
3. A particle performs SHM on a rectilinear path. Starting from rest, it travels x 1 distance in first second and in the
next second, it travels x 2 distance. Find out the amplitude of this SHM.
4. A uniform heavy rod of weight W, cross-sectional area A and length l is hanging from a fixed support. Young’s
modulus of the material of the rod is Y. Neglecting the lateral contraction, find the elongation produced in the
rod.
b
Or Find the molar specific heat of the process p = for a diatomic gas, b being constant.
T
5. Write the difference between ideal gas and real gas.
Or Two mercury droplets of radii 0.1 cm and 0.2 cm collapse into one single drop. What amount of energy is
released? The surface tension of mercury S = 435.5 × 10 − 3 N / m .
When two or more droplets collapse to form a bigger drop, then its surface area decreases and energy is
released equal to S∆A. [NCERT]

6. What is capillarity? Water rises in a capillary tube to a height of 4.0 cm. In another capillary tube whose radius is
one-third of it, how much the water will rise?
Or Show that, if n equal rain droplets falling through air with equal steady velocity of 10 cms − 1 coalesce, the
resultant drop attains a new terminal velocity of 10 n 2/ 3 cms − 1 .
7. Derive an expression for C p and C V for a mixture of gases. Also, find the expression for γ of the mixture.
Or In a refrigerator, one removes heat from a lower temperature and deposits to the surroundings at a higher
temperature. In this process, mechanical work has to be done, which is provided by an electric motor. If the
motor is of 1 kW power and heat is transferred from –3°C to 27°C, find the heat taken out of the refrigerator per
second assuming its efficiency is 50% of a perfect engine.
8. Show that in an open organ pipe all harmonics are present. What is the nature of the wave formed in the pipe?
Or For the harmonic travelling wave y = 2 cos 2 π{10 t − 0. 008 x + 3. 5}
where, x and y are in cm and t in second. What is the phase difference between the oscillatory motion at two
points separated by a distance of
(i) 4 m (ii) 0.5 m
λ 3λ
(iii) (iv) (at a given instant of time)
2 4
(v) What is the phase difference between the oscillation of a particle located at x = 100 cm, at t = T s and t = 5 s?
[1]
CBSE Term II Physics XI 167

9. Consider the earth as a uniform sphere of mass M and radius R. Imagine a straight smooth tunnel made through
the earth which connects any two points on its surface. Show that the motion of the particle is SHM. Find the
time period.
Or A steel wire of length 2 l and cross-sectional area A is stretched within elastic limit as shown in figure. Calculate
the strain and stress in the wire.
2l

T
m

Explanations
1. (i) (d) We know that, Tension F
Now applying formula, Stress = =
Breaking force Area A
Breaking stress = ... (i)
Area of cross- section Tension = Applied force = F
When length of the wire changes, area of (v) (a) Consider the diagram where a deforming force F
cross-section remains same. is applied to the combination.
Stress F/A
Hence, breaking force will be same when length For steel wire, Ysteel = =
changes. Strain Strain
(ii) (c) Consider the diagram where a spring is stretched
by applying a load to its free end. Clearly the length
and shape of the spring changes. F
The change in length corresponds to longitudinal strain Copper Steel
and change in shape corresponds to shearing strain. wire wire

where, F is tension in each wire and A is


cross-section area of each wires.
Load As F and A are same for both the wire, hence, stress
will be same for both the wire.
Stress Stress
(iii) (d) Consider the diagram, A mass M is attached at the (Strain) steel = , (Strain) copper =
Ysteel Ycopper
centre. As the mass is attached to both the rods, both
rod will be elongated, but due to different elastic As, Ysteel ≠ Ycopper
properties of material rubber changes shape also. Hence, the two wires will have differents strain.
2. Here, T1 = 300 K, T2 = 0 ° C = 273 K
Heat extracted,
N M Q2 = mL 1 = 2 kg × 3.33 × 10 5 J / kg
= 6.66 × 10 5 J
Q T2
Steel As, β= 2 =
W T1 − T2
Rubber
Q2 ( T1 − T2 )
(iv) (a) As shown in the diagram ∴ W=
Clearly, forces at each cross-section is F. T2
6.66 × 10 5 × (300 − 273)
=
273
= 65868 J• 6.5 × 10 4 J
A 3. Because the particle starts from rest, so its starting point
will be extreme position.
Thus, the displacement of the particle from the mean
F position after one second
168 CBSE Term II Physics XI

A − x1 = A cosωt = A cosω ...(i) [putting t = 1 s] d V 2RT


=
where, A is amplitude of the SHM and for next second dT b
A − ( x1 + x2 ) = A cos ωt  2RT 
⇒ C = CV + p  
= A cos2 ω [putting t = 2 s]  b 
= A[2 cos 2 ω − 1 ] ...(ii) 5 9
= CV + 2R = R + 2R = R
2 2
[Q cos 2 ω = 2 cos 2 ω − 1]
5.
From Eq. (i) and Eq. (ii), we have
 A− x 2  Ideal Gas Real Gas
A − ( x1 + x2 ) = A 2 ⋅  1
 − 1 It obeys ideal gas equation, It does not obey, pV = µRT
  A   pV = µRT at all temperatures at all values of temperature
1 and pressures. and pressure.
= [2 A 2 + 2 x12 − 4 Ax1 − A 2 ]
A The volume of the molecules The volume of the molecules
of an ideal gas is zero. of a real gas is non-zero.
⇒ A 2 − A( x1 + x2 ) = A 2 + 2 x12 − 4 Ax1
There is no intermolecular There is intermolecular force
⇒ A [ 3 x1 − x2 ] = 2 x12 force between the molecules. of attraction or repulsion
2 x12 depending on whether
∴ A= intermolecular separation is
3 x1 − x2 larger or small.
4. As shown in figure, consider a small element of thickness There is no intermolecular Potential energy ( U) does not
dx at distance x from the fixed support. Force acting on potential energy ( U) because equal to zero as
the element dx is intermolecular force (F ) is intermolecular force (F ) is not
W zero. zero.
F = Weight of length ( l − x) of the rod = ( l − x)
l It has only kinetic energy. It has both kinetic and
potential energy.
At absolute zero, the volume, All real gases get liquefied
pressure and internal energy before reaching absolute
x become zero. zero. The internal energy of
the liquefied gas is not zero.
Or
dx
Radii of mercury droplets, r1 = 0.1 cm = 1 × 10 − 3 m
l–x
r2 = 0.2 cm = 2 × 10 − 3 m
Surface tension (S) = 435. 5 × 10 − 3 N / m
Elongation of the element
Let the radius of the big drop formed by collapsing be R.
Stress F/A W
= Original length × = dx × = ( l − x) dx ∴ Volume of big drop = Volume of small droplets
Y Y l Ay
4 4 4
Total elongation produced in the rod πR 3 = πr13 + πr23
3 3 3
l
W  x2 
l
W or R 3 = r13 + r23 = ( 0.1 )3 + ( 0.2 )3
l AY ∫0
= ( l − x ) dx =  lx − 
l Ay  2 0 = 0. 001 + 0. 008 = 0. 009
W  2 l Wl 2 or R = 0.21 cm = 2.1 × 10 − 3 m
= l −  =
l Ay  2  2 Ay ∴ Change in surface area
∆ A = 4 πR 2 − ( 4 πr12 + 4 πr22 ) = 4 π [R 2 − ( r12 + r22 )]
Or
We know that, dQ = dU + dW ∴ Energy released = S ⋅ ∆ A
d Q dU + d W dU d W = S × 4 π [R 2 − ( r12 + r22 )]
C= = = +
dT dT dT dT = 435. 5 × 10 − 3 × 4 × 3.14 [(2.1 × 10 − 3 )2 − (1 × 10 − 6
dU = CV d T + 4 × 10 − 6 )]
C dT p dV p dV = 435. 5 × 4 × 3.14 [ 4. 41 − 5 ] × 10 − 6 × 10 − 3 ‘
⇒ C= V + = CV +
dT dT dT
= − 32.27 × 10 − 7 = − 3.22 × 10 − 6 J
Ideal gas equation, pV = RT
RT RT 2  b (Negative sign shows absorption)
⇒ V= = Qp = 
p b 
 T Therefore, 3.22 × 10 − 6 J energy will be absorbed.
CBSE Term II Physics XI 169

6. Capillarity is the phenomenon of rise (or fall) of a liquid γ of the mixture


in a capillary tube as compared to the surroundings. We can write ( n1 + n 2 ) CV = n1 CV1 + n 2 CV2
2 T cos θ ( n1 + n 2 ) R nR nR
We know that, h = ⇒ = 1 + 2
rρg γ −1 γ1 − 1 γ 2 − 1
2 T cos θ ( n1 + n 2 ) n1 n2
⇒ hr = = constant ⇒ = +
ρg γ −1 γ1 − 1 γ 2 − 1
h 1 r1
⇒ h 1 r1 = h 2 r2 ⇒ h 2 = γ for the mixture can be calculated from above equation.
r2 Or
⇒ h 2 = ( 4 )( 3 ) = 12 cm Given, temperature of source, T1 = (27 + 273 ) K = 300 K
Or Temperature of sink, T2 = ( −3 + 273 ) K = 270 K
Volume of a bigger drop Efficiency of a perfect heat engine is given by
= n × Volume of a smaller droplet T 270 1
η=1 − 2 =1 − =
4 4 T1 300 10
or πR 3 = n × πr 3 or R 3 = nr 3
3 3 Efficiency of refrigerator is 50% of a perfect engine
or R = n1 / 3 r ∴
1
η′ = 0. 5 × η = η =
1
Terminal velocity of a small droplet is given by 2 20
2 r2 ∴ Coefficient of performance of the refrigerator
vs = (ρ − ρ′ ) g …(i) Q 1 − η′ 1 − (1 / 20 )
9 η β= 2 = =
W η′ (1 / 20 )
Terminal velocity of a bigger drop is given by
19 / 20
2 R2 = = 19
vb = (ρ − ρ′ ) g …(ii) 1 / 20
9 η
 Q2 
Dividing Eq. (ii) by Eq. (i), we get or Q2 = βW = 19 W Qβ = 
 W
vb R 2 = 19 × (1 kW) = 19 kW
= 2
vs r Therefore, heat is taken out of the refrigerator at a rate of
But R = n1 / 3 r 19 kW per second.
8. Consider an open organ pipe shown in the figure as
and vs = 10 cm/s below
R2  n 2 / 3r 2
∴ v b = vs ×  2  = 10 ×
r  r2
A A
N
v b = 10 n 2 / 3 cm/s
7. Consider mixture of two gases.
The given organ pipe is open at both ends. Let the length
CV of the mixture of the pipe is l.
We can write U = U1 + U 2 Let the wave in the forward direction is
⇒ dU = dU 1 + dU 2 y1 ( x, t ) = A sin ( kx − ωt )
⇒ nCV dT = n1 CV1dT + n 2 CV2 dT y 2 ( x, t ) = A sin ( kx + ωt )
⇒ ( n1 + n 2 ) CV dT = n1 CV1dT + n 2 CV2 dT Resultant wave y = y1 + y 2
= A sin ( kx − ω t ) + A sin ( kx + ωt )
⇒ ( n1 + n 2 ) CV = n1 CV1 + n 2 CV2
= 2A cos kx sin ωt
n1 CV1 + n 2 CV2 The amplitude of the resulting stationary wave
⇒ CV =
n1 + n 2 = 2A cos kx
Cp of the mixture Cp = CV + R Amplitude will be maximum when cos kx = ±1 or kx = 0
or nπ where, n = 1 , 2 , K
n1 CV1 + n 2 CV2
Cp = +R nπ nπ nλ
n1 + n 2 ⇒ x= = =
k 2π λ 2
n1 ( CV1 + R ) + n 2 ( CV2 + R )
= nλ
n1 + n 2 Hence, we will have l =
2
n1 Cp1 + n 2 Cp 2 2l
Cp = ⇒ λ=
n1 + n 2 n
170 CBSE Term II Physics XI

Frequency of standing waves so set-up will be 9. Consider the diagram shown as below
v v nv
ν= = =
λ 2l / n 2l
where, n = 1 , 2 , 3 , K
We can find frequency of various harmonics r x
v 2v 3v θ
ν1 = , ν 2 = , K , ν3 = O
2l 2l 2l
Thus, it is clear from above expression that in an open
organ pipe all harmonics are present. Clearly, standing
waves are formed.
Or
Let at some instant the particle is at ( r , θ ) as shown in the
Given, wave function diagram.
y = 2 cos 2 π{10 t − 0. 008 x + 3. 5} GMm
y = 2 cos{20 πt − 0. 016 πx + 7 π} The gravitational force on the particle = r
R3
On comparing the equation by standard equation Clearly, force will be towards the centre.
y = A cos(ω t − kx + φ )
Fx = − F sin θ = Force in vertically downward
We get, A = 2 cm
GMmr  x  GMm
⇒ ω = 20 π rad/s =−  =− x
R 3  r R3
k = 0. 016 π
Since, Fx ∝ −x
(i) Phase difference,
2π Hence, motion will be SHM.
∆φ = × Path difference
λ F
ax = x = − 3 x
GM
 2π  m R
∴ ∆φ = 0. 016 π × 4 × 100 Q λ = k  ∴ Time-period of oscillation,
= 6. 4 π rad x R3
T = 2π = 2π
2π ax GM
(ii) ∆φ = × ( 0. 5 × 100 )
λ R
= 0. 016 π × 0. 5 × 100 = 0. 8 π rad = 2π
g
2π  λ 
(iii) ∆φ = ×   = π rad Or
λ 2
Total length L = 2 l . Increase in length of the wire, when
2π 3λ 3π
(iv) ∆φ = × = rad it is stretched from its mid-point.
λ 4 2
2π 2π 1 l O l
(v) T = = = s A B
ω 20 π 10 2T1 cos θ
∴ At x = 100 cm, t = T θ θ
x
√l2 + x2
φ1 = 20 πT − 0. 016 π(100 ) + 7 π T1
C T
1
= 20 π   − 1. 6 π + 7 π T
10  M Mg

= 2 π − 1. 6 π + 7 π …(i)
From Pythagoras theorem, BC2 = l 2 + x2
Again at x = 100 cm, t = 5 s
φ2 = 20 π( 5 ) − 0. 016 π(100 ) + 7 π BC = l 2 + x2
= 100 π − ( 0. 016 × 100 )π + 7 π
Similarly, AC = l 2 + x2
= 100 π − 1. 6 π + 7 π …(ii)
Change in length, of the wire
∴ From Eqs. (i) and (ii), we get
∆φ = φ2 − φ1 ∆L = ( AC + CB ) − AB = ( l 2 + x2 + l 2 + x2 ) − 2 l
1/ 2
= (100 π − 1. 6 π + 7 π ) − (2 π − 1. 6 π + 7 π )  x2 
= 2 ( l 2 + x2 )1 / 2 − 2 l = 2 l 1 + 2  −2l
= 98π rad  l 
CBSE Term II Physics XI 171

Since x << l , so using Binomial expansion, we have 1/ 2


Mg Mgl  x2 
 2
1/ 2
 T1 = ( l 2 + x2 ) = 1 + 2 
x x  2
2x 2x  l 
1 + 2  = 1 + 2 
 l   2l  Mgl  x 2 
= 1 +  [using (1 + x)x = 1 + xx]
[neglecting terms containing higher powers of x] 2x  2l 2 

 x2
x2  x2 Q x << l ∴ →0
∴ ∆ L = 2 l 1 + 2  − 2 l = 2l 2
 2l  l
x2
Thus, 1 + =1
∆L x 2
x 2
2l 2
Hence, strain = = = Q T = 2 T1 cosθ
L l × 2 l 2l 2 Mgl
∴ T1 =
Mg 2x
∴ T1 = [Q T = Mg ] T1 Mgl
2 cosθ Stress in the wire = =
x A 2 xA
Putting cosθ =
l 2 + x2
Physics
Class 11th ( Term II )

Practice Paper 2 *

(Unsolved)
General Instructions T ime : 2 Hours
Max. Marks : 35
1. There are 9 questions in the question paper. All questions are compulsory.
2. Question no. 1 is a Case Based Question, which has five MCQs. Each question carries one mark.
3. Question no. 2-6 are Short Answer Type Questions. Each question carries 3 marks.
4. Question no. 7-9 are Long Answer Type Questions. Each question carries 5 marks.
5. There is no overall choice. However, internal choices have been provided in some questions. Students have to attempt
only one of the alternatives in such questions.
* As exact Blue-print and Pattern for CBSE Term II exams is not released yet. So the pattern of this
paper is designed by the author on the basis of trend of past CBSE Papers. Students are advised
not to consider the pattern of this paper as official, it is just for practice purpose.

1. Direction Read the following passage and answer the questions that follows
Entropy
Like pressure, volume, temperature, internal energy, etc., we have another thermodynamic variable of a
system, named entropy. It is related to the disorder of molecular motion of the system. Greater the
randomness or disorder, greater is the entropy.
The change in entropy,
Heat absorbed by the system ∆Q
i.e. ∆S = =
Absolute temperature T
This relation is also called mathematical form of second law of thermodynamics.
SI unit of ∆S is J/K and its dimensional formula is [ ML2T −2 K −1 ].
(i) Which of the following is true?
(a) For an isolated system, ds ≥ 0
(b) For a reversible process, ds = 0
(c) For an irreversible process, ds > 0
(d) All of the above
(ii) The entropy of an isolated system can never
(a) increase (b) decrease (c) be zero (d) unity
(iii) Clausius summarized the first and second laws of thermodynamics as
(a) the energy of the world is constant
(b) the entropy of the world tends towards a maximum
(c) Both (a) and (b)
(d) None of the above
CBSE Term II Physics XI 173

(iv) The final temperature of two bodies, initially at T1 and T2 can range from
T1 − T2 T1 + T2
(a) to T1 T2 (b) to T1 T2
2 2
T + T2 T − T1
(c) 1 to T1 T2 (d) 1 to T1 T2
2 2
(v) The degree of disorder of a mixture of two gases
(a) is always less than the degrees of disorder of inidividual gases.
(b) is always greater than the degrees of disorder of individual gases.
(c) is always equals to the degrees of disorder of individual gases.
(d) None of the above
2. A metal bar of length L and area of cross-section A, is rigidly clamped between two walls. The Young’s modulus
of its material is Y and the coefficient of linear expansion is α. The bar is heated so that its temperature is
increased from 0 to θ° C. Find the force exerted at the ends of the bar.
Or Can a gas be liquefied at any temperature by increase of pressure alone ?
10
3. A travelling wave pulse is given by y =
5 + (x + 2 t )2
Here, x and y are in metre and t in second. In which direction and with what velocity is the pulse propagating?
What is the amplitude of pulse?
4. Two soap bubbles have radii in the ratio 2 : 3. Compare the excess of pressure inside these bubbles.
5. A pan filled by hot food cools from 94°C to 86°C in 120 s, when the room temperature is 293 K. How long will it
take to cool from 71°C to 69°C?
Or From kinetic theory of gases, can we obtain ideal gas equation? If yes, how?
 5
6. A monoatomic ideal gas  γ =  initially at 17°C is suddenly compressed to one-eight of its original volume.
 3
Find the final temperature after compression.
Or Two identical samples of a gas are at a pressure p1 and volume V1 . The two samples are allowed to expand so as
to acquire a volume V2 . But one sample expands isothermally while the other expands adiabatically. In which
case, more work is done and why?
7. An electric bulb of volume 250 cm 3 was sealed off during manufacture at a pressure of 10 −3 mm of mercury at
27°C.
Compute the number of air molecules contained in the bulb.
Given that, R = 8.31 J/mol/K and N A = 6.02 × 10 23 mol −1 .
Or Explain the relation in phase between displacement, velocity and acceleration in SHM, graphically as well as
theoretically.
8. If a number of little droplets of water, all of the same radius r, coalesce to form a single drop of radius R , show
3T  1 1 
that the rise in temperature will be given by  −  , where T is the surface tension of water and J is the
ρJ  r R 
mechanical equivalent of heat.
Or
(i) Explain second law of thermodynamics.
(ii) How is heat engine different from refrigerator?
(iii) Write a paragraph explaining Carnot engine efficiency ( η) and its relation with temperature.
9. A wire of variable mass per unit length µ = µ 0 y is hanging from the ceiling as shown in figure. The length of
wire is l 0 . A small transverse disturbance is produced at its lower end. Find the time after which the disturbance
will reach to the other end.
174 CBSE Term II Physics XI

Or A slightly tapering wire of length l and end radii a and b on both sides is subjected to the stretching forces F on
both sides as shown in figure. If Y is the Young’s modulus of the wire, then calculate the extension produced in
the wire.

F F

r1 = a
r2 = b

Answers
4
1. (i) d, (ii) b, (iii) c, (iv) b, (v) b 3. Velocity = 2 m/s; amplitude = 2 m; along negative X-axis 4.
9
5. 42 s 8l0
6. 887°C 7. 8 × 1015 9.
g
Physics
Class 11th ( Term II )

Practice Paper 3 *

(Unsolved)
General Instructions T ime : 2 Hours
Max. Marks : 35
1. There are 9 questions in the question paper. All questions are compulsory.
2. Question no. 1 is a Case Based Question, which has five MCQs. Each question carries one mark.
3. Question no. 2-6 are Short Answer Type Questions. Each question carries 3 marks.
4. Question no. 7-9 are Long Answer Type Questions. Each question carries 5 marks.
5. There is no overall choice. However, internal choices have been provided in some questions. Students have to attempt
only one of the alternatives in such questions.
* As exact Blue-print and Pattern for CBSE Term II exams is not released yet. So the pattern of this
paper is designed by the author on the basis of trend of past CBSE Papers. Students are advised
not to consider the pattern of this paper as official, it is just for practice purpose.

1. Direction Read the following passage and answer the questions that follows
Wave Motion
A disturbance in the lake causes a ripple of water. They are said to be transverse in nature. Another example
of transverse wave is sound waves travelling in solids.
Sound waves travelling in the gases, fluids or plasma are said to be longitudinal waves.
In both cases the speed of the wave is given by
v = νλ
where, ν is frequency of the wave and λ is the wavelength of the wave.
 2π 
And general equation of wave is given by y ( x , t ) = A sin  ( x + vt ) + φ 
 λ 
which is travelling in + x-direction.
(i) Sound waves of wavelength λ travelling in a medium with a speed of v m /s enter into another medium
where its speed in 2v m /s. Wavelength of sound waves in the second medium is
λ
(a) λ (b)
2
(c) 2λ (d) 4λ
(ii) Speed of sound wave in air
(a) is independent of temperature
(b) increases with pressure
(c) increases with increase in humidity
(d) decreases with increase in humidity
176 CBSE Term II Physics XI

(iii) Change in temperature of the medium changes


(a) frequency of sound waves
(b) amplitude of sound waves
(c) wavelength of sound waves
(d) loudness of sound waves
(iv) With propagation of longitudinal waves through a medium, the quantity transmitted is
(a) matter (b) energy
(c) energy and matter (d) energy, matter and momentum
(v) A sound wave is passing through air column in the form of compression and rarefaction. In consecutive
compressions and rarefactions,
(a) density remains constant (b) Boyle’s law is obeyed
(c) bulk modulus of air oscillates (d) there is no transfer of heat
2. Show graphically the change of potential energy and kinetic energy of a block attached to a spring which obeys
Hooke’s law.
Or A liquid rises in a capillary such that the surface tension balances its weight of 5 × 10 −3 N of liquid, surface
tension of liquid is 5 × 10 − 2 Nm − 1 . Find the radius of the capillary.
3. How does thermal expansion of liquids differ from that of solid? Is real expansion always greater than the
apparent expansion?
Or A cup of tea cools from 81 ° C to 79° C in 1 min. The ambient temperature is 30° C. What time is needed for
cooling of same cup of tea in same ambience from 61 ° C to 59° C?
4. Explain, the limitations of the first law of thermodynamics.
Or If the ratio of molecular weights of two gases is 4. What will be ratio of the v rms values for the molecules of
those two gases?
5. Obtain an expression for the velocity of a particle executing SHM, when is this velocity
(i) maximum and (ii) minimum.
6. A standing wave is formed by two harmonic waves, Y1 = A sin(kx − ωt ) and Y2 = A sin(kx + ωt ) travelling on a
string in opposite directions. Mass density of the string is ρ and area of cross-section is s. Find the total
mechanical energy between two adjacent nodes on the string.
7. A whistle of frequency 540 Hz rotates in a circle of radius 2 m at a linear speed of 30 m/s. What is the lowest and
highest frequency heard by an observer along distance away at rest with respect to the centre of circle. Take
speed of sound in air as 330 m/s. Can the apparent frequency be ever equal to actual?
Or The stress-strain graph for a metal wire is shown in figure. Up to the point E, the wire returns to its original
state O along the curve EPO, when it is gradually unloaded. Point B corresponds to the fracture of the wire.

C
E A
P B
Stress

O O′
Strain

(i) Up to which point on the curve is Hooke’s law obeyed? This point is sometimes called proportionality limit.
(ii) Which point on the curve corresponds to elastic limit and yield point of the wire?
(iii) Indicate the elastic and plastic regions of the stress-strain graph.
CBSE Term II Physics XI 177

(iv) Describe what happens when the wire is loaded up to a stress corresponding to the point A on the graph and
then unloaded gradually. In particular, explain the dotted curve.
(v) What is peculiar about the portion of the stress-strain graph from C to B ? Up to what stress can the wire be
subjected without causing fracture?
8. What are the postulates of kinetic theory of gases? What will be expression for the pressure exerted by an ideal
gas?
Or If the terminal speed of a sphere of gold (density = 19. 5 kg/m 3 ) is 0.2 m/s in viscous liquid (density
= 19 . 5 kg/ m 3 ) is 0.2 m/s in viscous liquid (density = 1. 5 kg/ m 3 ).
Find out the terminal speed of a sphere of silver (density = 10. 5 kg/ m 3 ) of the same size in the same liquid.
9. The limbs of a manometer consist of uniform capillary tubes of radii 1.44 × 10 − 3 m and 7.2 × 10 −4 m. Find out the
correct pressure difference, if the level of the liquid (density 10 3 kgm − 3 , surface tension 72 × 10 − 3 Nm − 1 ) in the
narrower tube stands 0.2 m above that in the broader tube.
Or A gas is expanded twice in a way so that the volume becomes twice the initial value. In first case, gas is
expanded isothermally while in second case gas is expanded adiabatically.
(i) In which case, is the pressure is greater and why?
(ii) Which work done is more and why?

Answers
1. (i) c, (ii) c, (iii) c, (iv) b, (v) d 4. Or 0.5 7. 495 Hz, 594, Yes

You might also like